You are on page 1of 117

LABOR LAW REVIEW – 2ND EXAM organization.

Supervisory employees shall not be eligible for


membership in the collective bargaining unit of the rank-and-file
Xandredg Sumpt L. Latog employees but may join, assist or form separate collective bargaining
units and/or legitimate labor organizations of their own. The rank-and-
file union and the supervisors' union operating within the same
I establishment may join the same federation or national union.
RIGHT TO SELF-ORGANIZATION
Article 292(c) provides:
The Right to Self-Organization includes:
(c) Any employee, whether employed for a definite
a. The right to form, join, or assist labor organizations period or not, shall, beginning on his first day of service, be
of their own choosing for purposes of collective considered as an employee for purposes of membership in
bargaining; any labor union.
b. The right not to join any union;
c. The right to vote by 2/3 majority of its general An exception to Article 292(c) is when the position is not
membership to cancel registration of its union in a covered by the union and when there is qualification in the union
meeting called for the purpose; itself.
d. The right to file a petition for cancellation of union
registration and/or de-certification of the union on 2. Government Employees
grounds provided for by law.
a. Constitutional Bases
A. CONSTITUTIONAL PROVISIONS Article IX-B, Section 2(5) provides:

1. Right to Self-Organization in the Private Sector (5) The right to self-organization shall not be denied
to government employees.
a. Constitutional Bases
Article II, Section 18 provides: Article XI, Section 1 also provides:

SECTION 18. The State affirms labor as a primary SECTION 1. Public office is a public trust. Public
social economic force. It shall protect the rights of workers and officers and employees must at all times be accountable to the
promote their welfare. people, serve them with utmost responsibility, integrity, loyalty,
and efficiency, act with patriotism and justice, and lead modest
lives.
Article III, Section 8 also provides:
b. Labor Code Bases
SECTION 8. The right of the people, including those
employed in the public and private sectors, to form unions,
associations, or societies for purposes not contrary to law shall Article 254. [244] Right of Employees in the Public Service.
not be abridged. Employees of government corporations established under the
Corporation Code shall have the right to organize and to bargain
collectively with their respective employers. All other employees in the
Lastly, Article XIII, Section 3 provides:
civil service shall have the right to form associations for purposes not
contrary to law.
SECTION 3. The State shall afford full protection to
labor, local and overseas, organized and unorganized, and
promote full employment and equality of employment Article 291. [276] Government Employees. The terms and
opportunities for all. conditions of employment of all government employees, including
It shall guarantee the rights of all workers to self- employees of government-owned and controlled corporations, shall be
organization, collective bargaining and negotiations, and governed by the Civil Service Law, rules and regulations. Their
peaceful concerted activities, including the right to strike in salaries shall be standardized by the National Assembly as provided for
accordance with law. They shall be entitled to security of tenure, in the New Constitution. However, there shall be no reduction of
humane conditions of work, and a living wage. They shall also existing wages, benefits and other terms and conditions of employment
participate in policy and decision-making processes affecting being enjoyed by them at the time of the adoption of this Code.
their rights and benefits as may be provided by law.
XXX Right to Self-Organization vs. Freedom of Association

b. Labor Code Bases Right to Self- Freedom of


Organization Association
Article 253. [243] Coverage and Employees' Right to Self- Basis Statutory: Labor Constitutional: Bill of
Organization. All persons employed in commercial, industrial and Code Rights
agricultural enterprises and in religious, charitable, medical, or Purpose “in whole or in part” Any lawful purpose
educational institutions, whether operating for profit or not, shall have for the purpose of
the right to self-organization and to form, join, or assist labor collective bargaining
organizations of their own choosing for purposes of collective Right involved Economic right; Privileged right
bargaining. Ambulant, intermittent and itinerant workers, self- protected right
employed people, rural workers and those without any definite Juridical Carries with it the Does not carry with it
personality creation of a separate the right to have a
employers may form labor organizations for their mutual aid and
and distinct separate and distinct
protection.
personality by virtue personality
of registration
Article 255. [245] Ineligibility of Managerial Employees to E-Er Relationship Must exist Need not exist
Join any Labor Organization; Right of Supervisory Employees. Abridgement by Can be abridged and Cannot be abridged
Managerial employees are not eligible to join, assist or form any labor State regulated

Page 1 of 117
Who can exercise Private sector Private and organization. Supervisory employees shall not be eligible for
employees only government membership in the collective bargaining unit of the rank-and-file
employees employees but may join, assist or form separate collective bargaining
What it is called Labor Workers units and/or legitimate labor organizations of their own. The rank and
Organization/Union Association/Union in file union and the supervisors' union operating within the same
under the Labor Code General establishment may join the same federation or national union.

B. COVERAGE
The following are eligible to join, form or assist a labor
Rule II, Section 2 of DO 40-03 provides:
organization in the private sector:
Section 2. Who may join labor unions and workers’
associations. – All persons employed in commercial, industrial a. All persons employed in commercial,
and agricultural enterprises, including employees of government industrial and agricultural enterprises;
owned or controlled corporations without original charters
established under the Corporation Code, as well as employees of b. Employees of religious, charitable, medical
religious, charitable, medical or educational institutions whether or educational institutions, whether
operating for profit or not, shall have the right to self- operating for profit or not;
organization and to form, join or assist labor unions for purposes
of collective bargaining: provided, however, that supervisory
employees shall not be eligible for membership in a labor union
c. Front-line managers, commonly known as
of the rank-and-file employees but may form, join or assist supervisory employees;
separate labor unions of their own. Managerial employees shall There is no prohibition in the law or in the implementing
not be eligible to form, join or assist any labor unions for rules regarding the right of supervisory employees to organize a
purposes of collective bargaining. labor organization or workers’ association of their own. They are,
Alien employees with valid working permits issued however, not allowed to become members of a labor union
by the Department may exercise the right to self-organization composed of rank-and-file employees. This is clear under Article
and join or assist labor unions for purposes of collective
255 of the Labor Code. In case there is mixed membership of
bargaining if they are nationals of a country which grants the
same or similar rights to Filipino workers, as certified by the
supervisors and rank-and-file employees in one union, the new rule
Department of Foreign Affairs, or which has ratified either ILO enunciated in Article 256 of the Labor Code, unlike in the old law,
Convention No. 87 and ILO Convention No. 98. is that it cannot be invoked as a ground for the cancellation of the
For purposes of this section, any employee, whether registration of the union. The employees so improperly included
employed for a definite period or not, shall beginning on the are automatically deemed removed from the list of members of said
first day of his/her service, be eligible for membership in any union. In other words, their removal from the said list is by
labor organization. operation of law.
All other workers, including ambulant, intermittent
and other workers, the self-employed, rural workers and those
without any definite employers may form labor organizations
d. Alien employees;
for their mutual aid and protection and other legitimate purposes
except collective bargaining.” Article. 284. [269] Prohibition Against Aliens; Exceptions. -
All aliens, natural or juridical, as well as foreign organizations are
1. Labor Organization and Its Purpose strictly prohibited from engaging directly or indirectly in all forms of
trade union activities without prejudice to normal contacts between
Article 219(g) of the Labor Code provides:
Philippine labor unions and recognized international labor centers:
Provided, however, That aliens working in the country with valid
(g) "Labor organization" means any union or permits issued by the Department of Labor and Employment, may
association of employees which exists in whole or in part for the exercise the right to self-organization and join or assist labor
purpose of collective bargaining or of dealing with employers organizations of their own choosing for purposes of collective
concerning terms and conditions of employment. bargaining: Provided, further, That said aliens are nationals of a
country which grants the same or similar rights to Filipino workers.
2. In the Private Sector
As a general rule, all aliens, natural or juridical, as well
Article 253. [243] Coverage and Employees' Right to Self- as foreign organizations are strictly prohibited from engaging
Organization. All persons employed in commercial, industrial and
directly or indirectly in all forms of trade union activities without
agricultural enterprises and in religious, charitable, medical, or
educational institutions, whether operating for profit or not, shall have prejudice to normal contacts between Philippine labor unions and
the right to self-organization and to form, join, or assist labor recognized international labor centers.
organizations of their own choosing for purposes of collective For an alien employee to exercise his right to self-
bargaining. Ambulant, intermittent and itinerant workers, self- organize, the following requisites should be complied with:
employed people, rural workers and those without any definite
employers may form labor organizations for their mutual aid and (i) He should have a valid working permit issued
protection. by the DOLE; and
(ii) He is a national of a country which grants the
Article 254. [244] Right of Employees in the Public Service. same or similar rights to Filipino workers or
Employees of government corporations established under the which has ratified either ILO Convention No.
Corporation Code shall have the right to organize and to bargain
87 or ILO Convention No. 98, as certified by
collectively with their respective employers. All other employees in the
civil service shall have the right to form associations for purposes not
the Philippine Department of Foreign Affairs.
contrary to law.
e. Working children;
Working children have the same freedom as adults to
Article 255. [245] Ineligibility of Managerial Employees to
Join any Labor Organization; Right of Supervisory Employees. join the collective bargaining union of their own choosing in
Managerial employees are not eligible to join, assist or form any labor accordance with existing law. Under PD 603, it is clearly provided
that neither management nor any collective bargaining union shall
Page 2 of 117
threaten or coerce working children to join, continue or withdraw
as members of such union. Philippine Society for Prevention of Cruelty to Animals v.
However, children cannot be officers of a union or labor Commission on Audit
organization. Held: The petitioner is correct in stating that the charter test is
predicated, at best, on the legal regime established by the 1935
Constitution, Section 7, Article XIII, which states:
f. Homeworkers;
Sec. 7.  The National Assembly shall not, except by general law,
Homeworkers have the right to form, join or assist provide for the formation, organization, or regulation of private
organizations of their own choosing in accordance with law. The corporations, unless such corporations are owned or controlled by the
registration of homeworkers’ organizations or associations Government or any subdivision or instrumentality thereof.
following the requirements prescribed by law will vest legal The foregoing proscription has been carried over to the 1973
personality thereto. and the 1987 Constitutions.  Section 16 of Article XII of the present
Household workers or domestic workers are excluded. Constitution provides:
Sec. 16.  The Congress shall not, except by general law, provide
for the formation, organization, or regulation of private corporations. 
Homeworkers Household workers
Government-owned or controlled corporations may be created or
Place of work Home of employee Home of the established by special charters in the interest of the common good and
employer subject to the test of economic viability.
Classification Industrial worker Non-industrial Section 16 is essentially a re-enactment of Section 7 of Article
History Product of Industrial Antedates Industrial XVI of the 1935 Constitution and Section 4 of Article XIV of the 1973
Revolution Revolution
Constitution.
DOLE Inspection Subject to Not subject to
During the formulation of the 1935 Constitution, the Committee
and Visitorial
Powers on Franchises recommended the foregoing proscription to prevent the
Right to self- Yes No pressure of special interests upon the lawmaking body in the creation of
organization corporations or in the regulation of the same. To permit the lawmaking
body by special law to provide for the organization, formation, or
regulation of private corporations would be in effect to offer to it the
g. Employees of cooperatives;
temptation in many cases to favor certain groups, to the prejudice of others
Members of a cooperative have no right to form or join or to the prejudice of the interests of the country.
labor organizations for purposes of collective bargaining for being And since the underpinnings of the charter test had been
themselves co-owners of the cooperative. This prohibition covers introduced by the 1935 Constitution and not earlier, it follows that the test
employees of the cooperative who are at the same time members cannot apply to the petitioner, which was incorporated by virtue of Act No.
thereof [Cooperative Rural Bank v. Ferrer-Calleja]. However, 1285, enacted on January 19, 1905. Settled is the rule that laws in general
insofar as the cooperative’s employees who are not members or co- have no retroactive effect, unless the contrary is provided. All statutes are
owners thereof are concerned, they are entitled to exercise their to be construed as having only a prospective operation, unless the purpose
and intention of the legislature to give them a retrospective effect is
right to self-organization and collective bargaining as guaranteed in
expressly declared or is necessarily implied from the language used. In case
the Constitution and existing laws. It is the fact of ownership of the of doubt, the doubt must be resolved against the retrospective effect.
cooperative and not involvement in the management thereof which There are a few exceptions.  Statutes can be given retroactive
disqualifies a member from joining any labor organization within effect in the following cases: (1) when the law itself so expressly provides;
the cooperative. But employee-members of a cooperative may (2) in case of remedial statutes; (3) in case of curative statutes; (4) in case
withdraw as members of the cooperative for purposes of joining a of laws interpreting others; and (5) in case of laws creating new rights.
labor union [Central negros Electric Corporation v. Secretary of None of the exceptions is present in the instant case.
Labor]. The general principle of prospectivity of the law likewise
applies to Act No. 1459, otherwise known as the Corporation Law, which
had been enacted by virtue of the plenary powers of the Philippine
h. Employees of legitimate contractors – not Commission on March 1, 1906, a little over a year after January 19, 1905,
with the principals but with the the time the petitioner emerged as a juridical entity.  Even the Corporation
contractors. Law respects the rights and powers of juridical entities organized
An employee of a legitimate job contractor is entitled to beforehand, viz:
SEC. 75.  Any corporation or sociedad anonima formed,
all the rights and privileges due a regular employee as provided in
organized, and existing under the laws  of the  Philippine  Islands and  
the Labor Code, such as the right to self-organization, collective lawfully  transacting business   in   the Philippine Islands on the date of the
bargaining and peaceful concerted activities, including the right to passage of this Act, shall be subject to the provisions hereof so far as such
strike. But this right cannot be exercised and invoked against the provisions  may  be applicable  and  shall be entitled   at  its option either
principal but only against the independent contractor which to continue business as such corporation or to reform and organize under
employed them, except if there is labor-only contracting. and by virtue of the provisions of this Act, transferring all corporate
interests to the new corporation which, if a stock corporation, is authorized
i. Employees of GOCCs without original to issue its shares of stock at par to the stockholders or members of the old
corporation according to their interests.  (Emphasis supplied).
charters established under the Corporation
As pointed out by the OSG, both the 1935 and 1987
Code; Charter Test Constitutions contain transitory provisions maintaining all laws issued not
Under the present state of the law, the test in determining inconsistent therewith until amended, modified or repealed.
whether a government–owned or controlled corporation is subject In a legal regime where the charter test doctrine cannot be
to the Civil Service Law is the manner of its creation such that applied, the mere fact that a corporation has been created by virtue of a
government corporations created by special charter are subject to special law does not necessarily qualify it as a public corporation.
its provisions while those incorporated under the general What then is the nature of the petitioner as a corporate entity? 
Corporation Law are not within its coverage. What legal regime governs its rights, powers, and duties?
As stated, at the time the petitioner was formed, the applicable
However, the Philippine Society for Prevention of
law was the Philippine Bill of 1902, and, emphatically, as also stated above,
Cruelty to Animals, GOCCs with original charters severally liable no proscription similar to the charter test can be found therein.
under Article 106 and 107 of the Labor Code, and the Philippine The textual foundation of the charter test, which placed a
National Red Cross are within the coverage of the Labor Code, limitation on the power of the legislature, first appeared in the 1935
despite having original charters. Constitution.  However, the petitioner was incorporated in 1905 by virtue of

Page 3 of 117
Act No. 1258, a law antedating the Corporation Law (Act No. 1459) by a clear declaration in Article IX(B), Section 2(5) that "the right to
year, and the 1935 Constitution, by thirty years.  There being neither a self organization shall not be denied to government employees."
general law on the formation and organization of private corporations nor a Article III, Section 8 of the Bill of Rights likewise states, "[t]he
restriction on the legislature to create private corporations by direct right of the people, including those employed in the public and
legislation, the Philippine Commission at that moment in history was well
private sectors, to form unions, associations, or societies for
within its powers in 1905 to constitute the petitioner as a private juridical
entity.
purposes not contrary to law shall not be abridged."
Time and again the Court must caution even the most brilliant While the right to self-organization is absolute, the right
scholars of the law and all constitutional historians on the danger of of government employees to collective bargaining and negotiation
imposing legal concepts of a later date on facts of an earlier date. is subject to limitations. Collective bargaining is a series of
The amendments introduced by C.A. No. 148 made it clear that negotiations between an employer and a representative of the
the petitioner was a private corporation and not an agency of the employees to regulate the various aspects of the employer-
government.  This was evident in Executive Order No. 63, issued by then employee relationship such as working hours, working conditions,
President of the Philippines Manuel L. Quezon, declaring that the
benefits, economic provisions, and others [Ibid].
revocation of the powers of the petitioner to appoint agents with powers of
arrest "corrected a serious defect" in one of the laws existing in the statute
Social Security System Employees Association v. Court
books. of Appeals explains that instead of a collective bargaining
agreement or negotiation, government employees must course their
3. In the Public Sector petitions for a change in the terms and conditions of their
employment through the Congress for the issuance of new laws,
rules, or regulations to that effect: Government employees may,
Article 254. Right of Employees in the Public Service.
Employees of government corporations established under the therefore, through their unions or associations, either petition the
Corporation Code shall have the right to organize and to bargain Congress for the betterment of the terms and conditions of
collectively with their respective employers. All other employees in the employment which are within the ambit of legislation or negotiate
civil service shall have the right to form associations for purposes not with the appropriate government agencies for the improvement of
contrary to law, those which are not fixed by law.
In PCSO v. Chairperson Pulido-Tan, et al., the
In the public sector, all rank-and-file employees of all Commission on Audit disallowed the monthly cost of living
branches, subdivisions, instrumentalities, and agencies of allowance being received by Philippine Charity Sweepstakes
government, including government-owned and/or controlled Office's officials and employees. This Court held that the
corporations with original charters, can form, join or assist labor Philippine Charity Sweepstakes Office's charter does not allow its
organizations called “employees organizations” of their own Board complete liberty to set the salaries and benefits of its
choosing. officials and employees. This Court emphasized that as a
As to collective bargaining, it is the legislature or, in government-owned and controlled corporation, the Philippine
proper cases, the administrative heads of government and not the Charity Sweepstakes Office is covered by the compensation and
collective bargaining process nor the concessions wrung by labor position standards issued by the Department of Budget and
unions from management that determine how much the workers in Management and applicable laws. PCSO underscored that the
government-owned or controlled corporations may receive in terms power of a government-owned or controlled corporation to fix
of salaries, 13th-month pay, and other conditions or terms of salaries or allowances of its employees is subject to and must
employment. There are government institutions which can afford to conform to the compensation and classification standards laid
pay two weeks, three weeks, or even 13th-month salaries to their down by applicable law: Upon the effectivity of R.A. No. 6758,
personnel from their budgetary appropriations. However, these GOCCs like the PCSO are included in the Compensation and
payments must be pursuant to law or regulation [Alliance of Position Classification System because Section 16 of the law
Government Workers v. Minister of Labor]. repeals all laws, decrees, executive orders, corporate charters, and
The general rule in the past and up to the present is that other issuances or parts thereof, that exempt agencies from the
"the terms and conditions of employment in the Government, coverage of the System, or that authorize and fix position
including any political subdivision or instrumentality thereof are classification, salaries, pay rates or allowances of specified
governed by law" (Section 11, the Industrial Peace Act, R.A. No. positions, or groups of officials and employees or of agencies,
875, as amended and Article 277, the Labor Code, P.D. No. 442, as which are inconsistent with the System, including
amended). Since the terms and conditions of government the proviso under Section 2 and Section 16 of P.D. No. 985.
employment are fixed by law, government workers cannot use the
same weapons employed by workers in the private sector to secure 4. Labor Organization Only For Mutual Aid And
concessions from their employers. The principle behind labor Protection and Not For Collective Bargaining
unionism in private industry is that industrial peace cannot be Purposes
secured through compulsion by law. Relations between private
employers and their employees rest on an essentially voluntary Article 253. [243] Coverage and Employees' Right to Self-
basis. Subject to the minimum requirements of wage laws and Organization. All persons employed in commercial, industrial and
other labor and welfare legislation, the terms and conditions of agricultural enterprises and in religious, charitable, medical, or
employment in the unionized private sector are settled through the educational institutions, whether operating for profit or not, shall have
process of collective bargaining. In government employment, the right to self-organization and to form, join, or assist labor
organizations of their own choosing for purposes of collective
however, it is the legislature and, where properly given delegated
bargaining. Ambulant, intermittent and itinerant workers, self-
power, the administrative heads of government which fix the terms employed people, rural workers and those without any definite
and conditions of employment. And this is effected through statutes employers may form labor organizations for their mutual aid and
or administrative circulars, rules, and regulations, not through protection.
collective bargaining agreements [Ibid].
In GSIS v. Villanueva, it was held that the right to self- This refers to the right to engage in group action,
organization is not limited to private employees and encompasses provided it is peaceful, to support the organization's objective
all workers in both the public and private sectors, as shown by the which is not necessarily collective bargaining but, simply, to aid
Page 4 of 117
and protect its members. But this kind of group action must be
differentiated from strike which, because it is work stoppage, must In the first place, all these employees, with the exception
observe certain regulations; otherwise, the strike may be declared of the service engineers and the sales force personnel, are confidential
illegal and its leaders may be thrown out of their jobs. employees. Their classification as such is not seriously disputed by
PEO-FFW; the five (5) previous CBAs between PIDI and PEO-FFW
However, nothing in the foregoing implementing rules
explicitly considered them as confidential employees. By the very
which provides that workers, with definite employers, cannot form nature of their functions, they assist and act in a confidential capacity
or join a workers' association for mutual aid and protection. Section to, or have access to confidential matters of, persons who exercise
2 thereof even broadens the coverage of workers who can form or managerial functions in the field of labor relations. As such, the
join a workers'... association. Thus, the Court agrees with rationale behind the ineligibility of managerial employees to form,
Samahan's argument that the right to form a workers' association is assist or joint a labor union equally applies to them.
not exclusive to ambulant, intermittent and itinerant workers. The
option to form or join a union or a workers' association lies with In Bulletin Publishing
the workers themselves, and whether... they have definite Co., Inc. v. Hon. Augusto Sanchez, this Court elaborated on this
employers or not [Samahan v. BLR]. rationale, thus:

C. PERSONS WHO DON’T HAVE THE RIGHT TO SELF- . . . The rationale for this inhibition has been stated to
be, because if these managerial employees would belong to or
ORGANIZATION
be affiliated with a Union, the latter might not be assured of
their loyalty to the Union in view of evident conflict of interests.
1. Managerial Employees The Union can also become company-dominated with the
presence of managerial employees in Union membership.  
Article 255. [245] Ineligibility of Managerial Employees to To be sure, the Court in Philips Industrial was
Join any Labor Organization; Right of Supervisory Employees. dealing with the right of confidential employees to organize. But
Managerial employees are not eligible to join, assist or form any labor the same reason for denying them the right to organize justifies
organization. Supervisory employees shall not be eligible for even more the ban on managerial employees from forming
membership in the collective bargaining unit of the rank-and-file unions. After all, those who qualify as top or middle managers
employees but may join, assist or form separate collective bargaining are executives who receive from their employers information
units and/or legitimate labor organizations of their own. The rank-and- that not only is confidential but also is not generally available to
file union and the supervisors' union operating within the same the public, or to their competitors, or to other employees. It is
establishment may join the same federation or national union. hardly necessary to point out that to say that the first sentence of
Art. 245 is unconstitutional would be to contradict the decision
in that case.
Article 219(m) provides:

(m) "Managerial employee" is one who is vested a. Types of Managerial Employees


with the powers or prerogatives to lay down and execute There are three (3) types of managerial employees for
management policies and/or to hire, transfer, suspend, lay-off, purposes of determining whether they could exercise their right to
recall, discharge, assign or discipline employees. Supervisory self-organization.
employees are those who, in the interest of the employer,
effectively recommend such managerial actions if the exercise (i) Top Management
of such authority is not merely routinary or clerical in nature but Top management is composed of a comparatively small
requires the use of independent judgment. All employees not
group of executives. It is responsible for the overall management of
falling within any of the above definitions are considered rank-
and-file employees for purposes of this Book the organization. It establishes operating policies and guides the
organization’s interactions with its environment. Typical titles of
Article 255 is constitutional top managers are CEO, President, or Senior VP. Actual titles vary
Although the definition of "supervisory employees" from one organization to another and are not always a reliable
seems to have been unduly restricted to the last phrase of the guide to membership in the management classification.
definition in the Industrial Peace Act, the legal significance given
to the phrase "effectively recommends" remains the same. In fact, (ii) Middle Management
the distinction between top and middle managers, who set Middle management refers to more than one level in an
management policy, and front-line supervisors, who are merely organization. Middle managers direct the activities of other
responsible for ensuring that such policies are carried out by the managers and sometimes also those of operating employees. The
rank and file, is articulated in the present definition. When read in middle managers’ principal responsibilities are to direct the
relation to this definition in Art. 212(m), it will be seen that Art. activities that implement their organization’s policies and to
245 faithfully carries out the intent of the Constitutional balance the demands of their superiors with the capacities of their
Commission in framing Art. III, 8 of the fundamental law [United subordinates. A plant manager in an electronic firm is an example
Pepsi-Cola v. Laguesma]. of a middle manager.
Nor is the guarantee of organizational right in Art. III, 8
infringed by a ban against managerial employees forming a union.  (iii) First-Line Management
The right guaranteed in Art. III, 8 " is subject to the condition that First-line management is the lowest level in an
its exercise should be for purposes not contrary to law."  organization at which individuals are responsible for the work of
others. First-line managers direct operating employees only; they
Rational Basis of prohibiting managerial employees do not supervise other managers. Examples are the “foreman” or
In the case of Art. 245, there is a rational basis for production supervisor in a manufacturing plant, the technical
prohibiting managerial employees from forming or joining labor supervisor in a research department, and the clerical supervisor in a
organizations.  large office. First-level managers are often called supervisors.
As Justice Davide, Jr., himself a constitutional
commissioner, said in his ponencia in Philips Industrial b. Supervisory Employee Rule
Development, Inc. v. NLRC: 

Page 5 of 117
Supervisors may organize their own union but are of the job of route managers is given in a four-page pamphlet, prepared by
prohibited from joining the rank-and-file union. The reason for the the company, called "Route Manager Position Description.
segregation of supervisory and rank-and-file employees with The route managers as compared to the supervisors in this case
respect to the exercise of their right to self-organization is the Unlike supervisors who basically merely direct operating
employees in line with set tasks assigned to them, route managers are
difference in interests. Supervisory employees are more closely
responsible for the success of the company's main line of business through
identified with the employer than with the rank-and-file employees. management of their respective sales teams. Such management necessarily
If supervisory and rank-and-file employees in a company are involves the planning, direction, operation and evaluation of their
allowed to form a single union, the conflicting interests of these individual teams and areas which the work of supervisors does not entail.
groups impair their relationship and adversely affect discipline, The route managers cannot thus possibly be classified as mere
collective bargaining, and strikes. These consequences can obtain supervisors because their work does not only involve, but goes far beyond,
not only in cases where supervisory and rank-and-file employees in the simple direction or supervision of operating employees to accomplish
the same company belonging to a single union but also where objectives set by those above them. They are not mere functionaries with
simple oversight functions but business administrators in their own right.
unions formed independently by supervisory and rank-and-file
Thus, the route managers in this case are managerial employees.
employees of a company are allowed to affiliate with the same
national federation [La Salle University v. Laguesma].
c. Separation of Unions Doctrine
RA 9481 amended Article 255 by adding the phrase:
Managerial Employees vs. Supervisory Employees
“The rank-and-file union and the supervisors’ union operating
within the same establishment may join the same federation or
Managerial Supervisory
Have the power to decide and do Have the power only to national union.” By reason of this amendment, the so-called
managerial acts recommend managerial acts such “separation of unions doctrine” enunciated in Atlas Litographic
as laying down policy, hiring, and Services v. Laguesma, and in other related cases no longer applies.
dismissal of employees and the This doctrine prohibits the situation where the supervisory union
like and the rank-and-file union operating within the same
Not allowed to join any labor Allowed to join a supervisory
organization union but not the union of rank- establishment are both affiliated with one and the same federation
and-file employees and vice versa because of the possible conflict of interest which may arise in the
areas, inter alia, of discipline, collective bargaining and strike.
Under Labor Standards, a supervisory employee is a Thus, if the intent of the law is to avoid a situation where
managerial employee. In Labor Relations, however, a managerial supervisors would merge with the rank-and-file or where the
employee is not a supervisory employee in the sense that he cannot supervisors’ labor union would represent conflicting interests, then
join any labor organization because supervisory employees can join a local supervisors’ union should not be allowed to affiliate with
unions different from rank-and-file employees. the federation with which the rank-and-file union is also affiliated
and where the federation actively participates in the union activities
PMTI v. Ferrer-Calleja in the company. The intent of the law is clear especially where the
Held: Members of the union are rank and file and not supervisors will be co-mingling with the rank-and-file employees
managerial employees notwithstanding their title or nomenclature in whom they directly supervise in their own bargaining unit [Pepsi
position because: Cola v. Secretary of Labor].
They do not have the power to lay down and execute The law now explicitly allows the more extreme situation
management policies as they are given ready policies merely to execute and of a rank-and-file union and a supervisors’ union operating within
standard practices to observe;
the same establishment joining one and the same federation or
They do not have the power to hire, transfer, suspend, lay-off,
recall, discharge, assign or discipline employees but only to recommend for
national union as affiliates thereof.
such actions as the power rests upon the personnel manager; and
They do not have the power to effectively recommend any d. Effect of Inclusion as Members of Employees
managerial actions as their recommendations have to pass through the Outside Bargaining Unit
department manager for review, the personnel manager for attestation and
general manager/president for final actions. Article 256. [245-A] Effect of Inclusion as Members of
The recent amendments to the Labor Code contain separate Employees Outside the Bargaining Unit. - The inclusion as union
definitions for managerial and supervisory employees. Section 4 of members of employees outside the bargaining unit shall not be a
Republic Act No. 6715 states that: ground for the cancellation of the registration of the union. Said
"Managerial Employee"- is one who is vested with powers or employees are automatically deemed removed from the list of
prerogatives to lay down and execute management policies and/or to hire, membership of said union.
transfer, suspend, lay-off, recall, discharge, assign or discipline employees.
“Supervisory Employees”- are those who, in the interest of the
employer, effectively recommend such management actions if the exercise The inclusion in a union of disqualified employees is not
of such authority is not merely routinary or clerical in nature but requires among the grounds for cancellation, unless such inclusion is due to
the use of independent judgment. misrepresentation, false statement or fraud under the circumstances
All employees not falling within any of the above definitions are enumerated in Sections (a) and (c) of Article 247 of the Labor
considered rank and file employees for purposes of this Book. Section 18 Code [Air Philippines v. Bureau of Labor Relations].
retains the provision on the ineligibility of managerial employees to join Clearly then, for the purpose of de-certifying a union, it
any labor organization. Supervisory employees shall not be eligible for is not enough to establish that the rank-and-file union includes
membership in a labor organization of the rank-and-file employees but may
ineligible employees in its membership. Pursuant to Article 247 (a)
join, assist or form separate labor organizations of their own.
and (c) of the Labor Code, it must be shown that there was
misrepresentation, false statement or fraud in connection with the
United Pepsi Cola v. Laguesma adoption or ratification of the constitution and by-laws or
Held: The Court with the finding of the Secretary of Labor that
amendments thereto, the minutes of ratification, or in connection
the route managers are managerial employees as this is supported by
substantial evidence
with the election of officers, minutes of the election of officers, the
The Court now finds that the job evaluation made by the list of voters, or failure to submit these documents together with the
Secretary of Labor is indeed supported by substantial evidence. The nature

Page 6 of 117
list of the newly elected-appointed officers and their postal However, the mere access of an employee to confidential
addresses to the BLR [Ibid]. labor relations information which is merely incidental to his duties
and, therefore, knowledge thereof is not necessary in the
e. Managerial Employees in the Public Sector performance of said duties, does not make such employee a
confidential employee. If access to confidential labor relations
Executive Order No. 180 information is to be a factor in the determination of an employee’s
Sec. 1. This Executive Order applies to all employees of all confidential status, such information must relate to the employer’s
branches, subdivisions, instrumentalities, and agencies, of the Government, labor relations policies. Therefore, access to information which is
including government-owned or controlled corporations with original regarded by the employer to be confidential from the business
charters. For this purpose, employees, covered by this Executive Order standpoint, such as financial information or technical trade secrets,
shall be referred to as "government employees".
will not render an employee a confidential employee under this
Sec. 2. All government employees can form, join or assist
employees' organizations of their own choosing for the furtherance and
rule. An employee may not be excluded from an appropriate
protection of their interests. They can also form, in conjunction with bargaining unit merely because he has access to confidential
appropriate government authorities, labor-management committees, works information concerning the employer’s internal business operations
councils and other forms of workers' participation schemes to achieve the which is not related to the field of labor relations [NATU-Republic
same objectives. Planters Bank Supervisors v. Secretary].
Sec. 3. High-level employees whose functions are normally Confidential employees cannot be classified as rank and
considered as policy-making or managerial or whose duties are of a highly file. As previously discussed, the nature of employment of
confidential nature shall not be eligible to join the organization of rank-and-
confidential employees is quite distinct from the rank and file, thus,
file government employees.
Sec. 4. The Executive Order shall not apply to the members of
warranting a separate category. Excluding confidential employees
the Armed Forces of the Philippines, including police officers, policemen, from the rank and file bargaining unit, therefore, is not tantamount
firemen and jail guards. to discrimination [Metrolab v. Confesor].
Sec. 15. A Public Sector Labor Management Council,
hereinafter referred to as the Council, is hereby constituted to be composed Sugbuanon Rural Bank v. Laguesma
of the following: Held: Petitioner contends that it has only 5 officers running its
1) Chairman, Civil Service Commission Chairman day-to-day affairs. They assist in confidential capacities and have complete
2) Secretary, Department of Labor and Employment Vice access to the bank's confidential data. They form the core of the bank's
Chairman management team. Petitioner explains that:
3) Secretary, Department of Finance Member "...Specifically: (1) the Head or the Loans Department initially
4) Secretary, Department of Justice Member approves the loan applications before they are passed on to the Board for
5) Secretary, Department of Budget and Management Member confirmation. As such, no loan application is even considered by the Board
The Council shall implement and administer the provisions of and approved by petitioner without his stamp of approval based upon his
this Executive Order. For this purpose, the Council shall promulgate the interview of the applicant and determination of his (applicant's) credit
necessary rules and regulations to implement this Executive Order. standing and financial capacity. The same holds true with respect to
Sec. 16. The Civil Service and labor laws and procedures, renewals or restructuring of loan accounts. He himself determines what
whenever applicable, shall be followed in the resolution of complaints, account should be collected, whether extrajudicially or judicially, and
grievances and cases involving government employees. In case any dispute settles the problem or complaints of borrowers regarding their accounts;
remains unresolved after exhausting all the available remedies under "(2) the Cashier is one of the approving officers and authorized
existing laws and procedures, the parties may jointly refer the dispute to the signatories of petitioner. He approves the opening of accounts, withdrawals
Council, for appropriate action. and encashment, and acceptance of check deposits, He deals with other
banks and, in the absence of the regular Manager, manages the entire office
2. Confidential Employees or branch and approves disbursements of funds for expenses; and
Within the context of labor relations, confidential "(3) the Accountant, who heads the Accounting Department, is
employees are those who meet the following criteria: also one of the authorized signatories of petitioner and, in the absence of
the Manager or Cashier, acts as substitute approving officer and assumes
the management of the entire office. She handles the financial reports and
a. They assist or act in a confidential capacity; reviews the debit/credit tickets submitted by the other departments."
b. To persons or officers who formulate, determine, Petitioner's explanation, however, does not state who among the
and effectuate management policies specifically in employees has access to information specifically relating to its labor
the field of labor relations. relations policies. Even Cashier Patricia Maluya, who serves as the
secretary of the bank's Board of Directors may not be so classified. True,
The two (2) criteria are cumulative and both must be met the board of directors is responsible for corporate policies, the exercise of
if an employee is to be considered a confidential employee that corporate powers, and the general management of the business and affairs
of the corporation. As secretary of the bank's governing body, Patricia
would deprive him of his right to form, join, or assist a labor
Maluya serves the bank's management, but could not be deemed to have
organization [TPMAB v. Asia Brewery]. access to confidential information specifically relating to SRBI's labor
A confidential employee may be a rank-and-file or relations policies, absent a clear showing on this matter. Thus, while
supervisory employee but because in the normal course of his petitioner's explanation confirms the regular duties of the concerned
duties, he becomes aware of management policies relating to labor employees, it shows nothing about any duties specifically connected to
relations, he is not allowed to assist, form or join a rank-and-file labor relations.
union or supervisory union, as the case may be. To allow him to
join a union would give rise to a potential conflict of interest. Southern Philippines Federation of Labor v. Ferrer-
Management should not be required to handle labor relations Calleja
matters through employees who are represented by the union with Held: As regards the employees in the confidential payroll,
which the company is required to deal and who, in the normal the petitioner has not shown that the nature of their jobs is classified as
performance of their duties, may obtain advance information on the managerial except for its allegation that they are considered by management
company’sn position with regard to collective bargaining as occupying managerial positions and highly confidential.  Neither can
payment or non-payment of union dues be the determining factor of
negotiations, the disposition of grievances, or other labor relations
whether the challenged employees should be excluded from the bargaining
matter [San Miguel Supervisors and Exempt Employees Union v. unit since the union shop provision in the CBA applies only to newly hired
Laguesma]. employees but not to members of the bargaining unit who were not
Page 7 of 117
members of the union at the time of the signing of the CBA.  It is, therefore, embracing legislation may be inadequate to provide for the unfolding
not impossible for employees to be members of the bargaining unit even events of the future. So-called gaps in the law develop as the law is
though they are non-union members or not paying union dues. enforced. One of the rules of statutory construction used to fill in the gap is
the doctrine of necessary implication x x x x Every statute is understood, by
implication, to contain all such provisions as may be necessary to effectuate
Filoil Refinery v. Filoil Supervisory & Confidential
its object and purpose, or to make effective rights, powers, privileges or
Employees Association jurisdiction which it grants, including all such collateral and subsidiary
Held: The other principal ground of petitioner's appeal consequences as may be fairly and logically inferred from its
questioning the confidential employees' inclusion in the supervisors' terms. Ex  necessitate legis x x x x
bargaining unit is equally untenable.  Respondent court correctly held that In applying the doctrine of necessary implication, we took into
since the confidential employees are very few in number and are by practice consideration the rationale behind the disqualification of managerial
and tradition identified with the supervisors in their role as representatives employees expressed in Bulletin Publishing Corporation v. Sanchez thus:
of management vis-a-vis the rank and file employees, such identity of "x x x if these managerial employees would belong to or be affiliated with a
interest has allowed their inclusion in the bargaining unit of supervisors- Union, the latter might not be assured of their loyalty to the Union in view
managers for purposes of collective bargaining in turn as employees in of evident conflict of interests. The Union can also become company-
relation to the company as their employer. dominated with the presence of managerial employees in Union
No arbitrariness or grave abuse of discretion can be attributed membership." Stated differently, in the collective bargaining process,
against respondent court's allowing the inclusion of the confidential managerial employees are supposed to be on the side of the employer, to
employees in the supervisors' association for as admitted by petitioner act as its representatives, and to see to it that its interests are well protected.
itself, supra, the supervisors and confidential employees enjoy its trust and The employer is not assured of such protection if these employees
confidence.  This identity of interest logically calls for their inclusion in the themselves are union members. Collective bargaining in such a situation
same bargaining unit and at the same time fulfills the law's objective of can become one-sided. It is the same reason that impelled this Court to
insuring to them the full benefit of their right to self-organization and to consider the position of confidential employees as included in the
collective bargaining, which could hardly be accomplished if the disqualification found in Art. 245 as if the disqualification of confidential
respondent association's membership were to be broken up into five employees were written in the provision. If confidential employees could
separate ineffective tiny units, as urged by petitioner. unionize in order to bargain for advantages for themselves, then they could
be governed by their own motives rather than the interest of the employers.
Doctrine of Necessary Implication Moreover, unionization of confidential employees for the purpose of
The doctrine of necessary implication is the legal basis collective bargaining would mean the extension of the law to persons or
for the ineligibility of a confidential employee to join a union. The individuals who are supposed to act "in the interest of" the employers. It is
not farfetched that in the course of collective bargaining, they might
disqualification of managerial and confidential employees from
jeopardize that interest which they are duty-bound to protect. Along the
joining a bargaining unit of rank-and-file employees or supervisory same line of reasoning we held in Golden Farms, Inc. v. Ferrer-
employees is already well-entrenched in jurisprudence. While Calleja reiterated in Philips Industrial Development, Inc. v. NLRC, that
Article 255 of the Labor Code limits the ineligibility to join, assist, "confidential employees such as accounting personnel, radio and telegraph
or form a labor organization to managerial employees, operators who, having access to confidential information, may become the
jurisprudence has extended this prohibition to confidential source of undue advantage. Said employee(s) may act as spy or spies of
employees or those who, by reason of their positions or nature of either party to a collective bargaining agreement."
work, are required to assist or act in a fiduciary manner to In fine, only the Branch Managers/OICs, Cashiers and
Controllers of respondent Bank, being confidential employees, are
managerial employees, and, therefore, are likewise privy to
disqualified from joining or assisting petitioner Union, or joining, assisting
sensitive and highly confidential records [Standard Chartered or forming any other labor organization. But this ruling should be
Bank Employees Union v. Standard Chartered Bank]. understood to apply only to the present case based on the evidence of the
Article 255 of the Labor Code does not directly prohibit parties, as well as to those similarly situated. It should not be understood in
confidential employees from engaging in union activities. Their any way to apply to banks in general.
disqualification proceeds merely from the application of this
doctrine because what Article 255 singles out as ineligible to join, 3. Members-Employees of Cooperatives
assist or form any labor organization are managerial employees. By The right to collective bargaining is not available to an
necessary implication, confidential employees are similarly employee of a cooperative who at the same time is a member and
disqualified. This doctrine states that what is implied in a statute is co-owner thereof. With respect, however, to employees who are
as much a part thereof as that which is expressed [Chua v. Civil neither members nor co-owners of the cooperative they are entitled
Service Commission]. to exercise the rights to self-organization, collective bargaining and
negotiation as mandated by the 1987 Constitution and applicable
NATU-Republic Planters Bank Supervisors v. statutes. It is the fact of ownership of the cooperative, and not
Secretary involvement in the management thereof, which disqualifies a
Held: As regards the other claim of respondent Bank that member from joining any labor organization within the
Branch Managers/OICs, Cashiers and Controllers are confidential cooperative. Thus, irrespective of the degree of their participation
employees, having control, custody and/or access to confidential matters, in the actual management of the cooperative, all members thereof
e.g., the branch's cash position, statements of financial condition, vault
cannot form, assist or join a labor organization for the purpose of
combination, cash codes for telegraphic transfers, demand drafts and other
negotiable instruments, pursuant to Sec. 1166.4 of the Central Bank Manual collective bargaining [BENECO v. Calleja].
regarding joint custody, this claim is not even disputed by petitioner. A
confidential employee is one entrusted with confidence on delicate matters, Membership in the cooperative; distinguished from
or with the custody, handling, or care and protection of the employer's stockholders of ordinary corporations
property. While Art. 245 of the Labor Code singles out managerial While cooperatives may exercise some of the rights and
employees as ineligible to join, assist or form any labor organization, under privileges given to ordinary corporations provided under existing
the doctrine of necessary implication, confidential employees are similarly laws, such cooperatives enjoy other privileges not granted to the
disqualified. This doctrine states that what is implied in a statute is as much
latter [Cooperative Rural Bank of Davao City v. Ferrer-Calleja].
a part thereof as that which is expressed, as elucidated in several cases the
latest of which is Chua v. Civil Service Commission where we said: Similarly, members of cooperatives have rights and obligations
No statute can be enacted that can provide all the details different from those of stockholders of ordinary corporations. The
involved in its application. There is always an omission that may not meet a Court held that: A cooperative . . . is by its nature different from an
particular situation. What is thought, at the time of enactment, to be an all- ordinary business concern being run either by persons,
Page 8 of 117
partnerships, or corporations. Its owners and/or members are the An employee does not become a member of a bargaining
ones who run and operate the business while the others are its unit. He becomes a member of the union. His position falls under
employees. As above stated, irrespective of the number of shares the positions covered by the bargaining unit being represented by
owned by each member they are entitled to cast one vote each in the union. If the employee does not belong to that community of
deciding upon the affairs of the cooperative. Their share capital interest represented by the bargaining unit, the union has the
earn limited interest. They enjoy special privileges as — exemption freedom to reject him. The union may, however, accept him but he
from income tax and sales taxes, preferential light to supply their is not covered by the benefits of the CBA.
products to State agencies and even exemption from the minimum There is no hard and fast rule in how the Med-Arbiter
wage laws. An employee therefore of such a cooperative who is a should determine an appropriate ( not most appropriate) bargaining
member and co-owner thereof cannot invoke the right to collective unit. The test whether the designation of a bargaining unit is
bargaining for certainly an owner cannot bargain with himself or appropriate is whether it will best assure to all employees the
his co-owners [Ibid]. exercise of their collective bargaining rights. There should be a
community of interest which should be reflected in groups having
II substantial similarity of work and duties or similarity of
STEPS IN EXERCISING THE RIGHT TO SELF- compensation and working conditions, among other criteria
ORGANIZATION [Democratic Labor Association v. Cebu Stevedoring].
In International School Alliance of Educators v.
A. EMPLOYER-EMPLOYEE RELATIONSHIP Quisumbing, there are certain tests which may be used in
The question of whether employer-employee relationship determining the appropriate collective bargaining unit, to wit:
exists is a primordial consideration before extending labor benefits
under the workmen's compensation, social security, medicare, a. Community or mutuality of interest doctrine;
termination pay and labor relations law. It is important in the b. Globe doctrine;
determination of who shall be included in a proposed bargaining c. Collective bargaining history doctrine; and
unit because, it is the sine qua non, the fundamental and essential d. Employment status doctrine
condition that a bargaining unit be composed of employees. Failure
to establish this juridical relationship between the union members Inclusion of excluded employees
and the employer affects the legality of the union itself. It means In DLSU v. DLSU Employees Association, it was held:
the ineligibility of the union members to present a petition for
certification election as well as to vote therein [Singer Sewing v. The Court agrees with the Solicitor General that the
express exclusion of the computer operators and discipline officers
Drilon].
from the bargaining unit of rank-and-file employees in the 1986
If the union members are not employees, no right to collective bargaining agreement does not bar any re-negotiation for
organize for purposes of bargaining, nor to be certified as the future inclusion of the said employees in the bargaining unit.
bargaining agent can be recognized. Since the persons involved are During the freedom period, the parties may not only renew the
not employees of the company, we held that they are not entitled to existing collective bargaining agreement but may also propose and
the constitutional right to join or form a labor organization for discuss modifications or amendments thereto. With regard to the
purposes of collective bargaining [Republic v. Laguesma]. alleged confidential nature of the said employees’ functions, after a
careful consideration of the pleadings filed before this Court, we rule
In Hijo Resources v. Mejares, the Court is tasked to
that the said computer operators and discipline officers are not
resolve the issue of whether the Labor Arbiter, in the illegal
confidential employees. As carefully examined by the Solicitor
dismissal case, is bound by the ruling of the Med-Arbiter regarding General, the service record of a computer operator reveals that his
the existence or non-existence of employer-employee relationship duties are basically clerical and non-confidential in nature. As to the
between the parties in the certification election case.  To dismiss discipline officers, we agree with the voluntary arbitrator that based
the illegal dismissal case filed before the Labor Arbiter on the basis on the nature of their duties, they are not confidential employees and
of the pronouncement of the Med-Arbiter in the certification should therefore be included in the bargaining unit of rank-and-file
election case that there was no employer-employee relationship employees.
The Court also affirms the findings of the voluntary
between the parties, which the respondent union could not even
arbitrator that the employees of the College of St. Benilde should be
appeal to the DOLE Secretary because of the dismissal of its excluded from the bargaining unit of the rank-and-file employees of
members, would be tantamount to denying due process to the Dela Salle University, because the two educational institutions have
complainants in the illegal dismissal case. This, we cannot allow. their own separate juridical personality and no sufficient evidence was
shown to justify the piercing of the veil of corporate fiction.
B. DETERMINATION OF APPROPRIATE OR
COLLECTIVE BARGAINING UNIT (ABU/CBU 1. Substantial Mutual Interests Rule
As defined in Department Order No. 40-03 which is now Under this doctrine, the employees sought to be
the revised Book V of the Rules Implementing the Labor Code, represented by the collective bargaining agent must have
"bargaining unit" refers to a group of employees sharing mutual community or mutuality of interest in terms of employment and
interests within a given employer unit, comprised of all or less than working conditions as evinced by the type of work they perform. It
all of the entire body of employees in the employer unit or any is characterized by similarity of employment status, same duties
specific occupational or geographical grouping within such and responsibilities and substantially similar compensation and
employer unit. working conditions [San Miguel Corporation Employees Union v.
It may also refer to a group of employees of a given Confesor].
employer, comprise of all or less than all of the entire body of In San Miguel Corporation v. Laguesma, the Supreme
employees, which the collective interests of all the employees, Court applied this principle in the petition of the union which seeks
consistent with equity to the employer, indicated to be best suited to represent the sales personnel in the various Magnolia sales office
to serve reciprocal rights and duties of the parties under the in Northern Luzon. Petitioner took the position that each sales
collective bargaining provisions of the law [Belyca Corporation v. office should constitute one bargaining unit. In disagreeing this
Ferrer-Calleja]. proposition of petitioner, the Court said: “What greatly militates
against this position is the meager number of sales personnel in
Page 9 of 117
each of the Magnolia sales office in Northern Luzon. Even the We only have to look at the experience of Coca-Cola Bottlers
bargaining unit sought to be represented by respondent union in the Philippines, Inc., since this company was organized about ten years ago, to
entire Northern Luzon sales area consists only of approximately 55 see the benefits that arise from restructuring a division of San Miguel into a
employees. Surely, it would not be for the best interest of these more competitive organization.  As a stand-alone enterprise, CCBPI
engineered a dramatic turnaround and has sustained its sales and market
employees if they would further be fractionalized. The adage ‘there
share leadership ever since.
is strength in number’ is the very rationale underlying the We are confident that history will repeat itself, and the
formation of a labor union.” transformation of Magnolia and FLD will be successful as that of CCBPI.
In San Miguel Corporation Supervisors and Exempt [17]

Employees Union v. Laguesma, the fact that the 3 plants Undeniably, the transformation of the companies was a
comprising the bargaining unit are located in 3 different places was management prerogative and business judgment which the courts can not
declared immaterial. Geographical location can be completely look into unless it is contrary to law, public policy or morals.  Neither can
disregarded if the communal or mutual interests of the employees we impute any bad faith on the part of SMC so as to justify the application
of the doctrine of piercing the corporate veil. [18] Ever mindful of the
are not sacrificed. The distance among the 3 plants is not
employees’ interests, management has assured the concerned employees
productive of insurmountable difficulties in the administration of that they will be absorbed by the new corporations without loss of tenure
union affairs. Neither are there regional differences that are likely and retaining their present pay and benefits according to the existing CBAs.
to impede the operations of a single bargaining representative. [19]
 They were advised that upon the expiration of the CBAs, new
In University of the Philippines v. Ferrer-Calleja, all agreements will be negotiated between the management of the new
non-academic rank-and-file employees of UP Diliman, Quezon corporations and the bargaining representatives of the employees
City, Padre Faura, Manila, Los Banos, Laguna and the Visayas concerned.  As a result of the spin-offs:
were allowed to participate in a certification election as one 1. Each of the companies are run by, supervised and controlled
by different management teams including separate human
bargaining unit.
resource/personnel managers.
2. Each Company enforces its own administrative and
San Miguel Corporation v. San Miguel Supervisors operational rules and policies and are not dependent on each other in their
Held: Thus, applying the ruling to the present case, the Court operations.
affirms the finding of the CA that there should be only one bargaining unit 3. Each entity maintains separate financial statements and are
for audited separately from each other.
the employees in Cabuyao, San Fernando, and Otis 25 of Indubitably, therefore, Magnolia and SMFI became distinct
Magnolia Poultry Products Plant involved in "dressed" chicken processing entities with separate juridical personalities.  Thus, they can not belong to a
and Magnolia Poultry Farms engaged in "live" chicken operations. Certain single bargaining unit as held in the case of Diatagon Labor Federation
factors, such as specific line of work, working conditions, location of work, Local 110 of the ULGWP v. Ople. We elucidate:
mode of compensation, and other relevant conditions do not affect or The fact that their businesses are related and that the 236
impede their commonality of interest. Although they seem separate and employees of Georgia Pacific International Corporation were originally
distinct from each other, the specific tasks of each division are actually employees of Lianga Bay Logging Co., Inc. is not a justification for
interrelated and there exists mutuality of interests which warrants the disregarding their separate personalities.  Hence, the 236 employees, who
formation of a single bargaining unit. are now attached to Georgia Pacific International Corporation, should not
Petitioner asserts that the CA erred in not excluding the position be allowed to vote in the certification election at the Lianga Bay Logging
of Payroll Master in the definition of a confidential employee and, thus, Co., Inc.  They should vote at a separate certification election to determine
prays that the said position and all other positions with access to salary and the collective bargaining representative of the employees of Georgia Pacific
compensation data be excluded from the bargaining unit. International Corporation.
This argument must fail. Confidential employees are defined as Petitioner-union’s attempt to include the employees of Magnolia
those who (1) assist or act in a confidential capacity, in regard (2) to and SMFI in the SMC bargaining unit so as to have a bigger mass base of
persons who formulate, determine, and effectuate management policies in employees has, therefore, no more valid ground.
the field of labor relations.26 The two criteria are cumulative, and both must Moreover, in determining an appropriate bargaining unit, the
be met if an employee is to be considered a confidential employee - that is, test of grouping is mutuality or commonality of interests.  The employees
the confidential relationship must exist between the employee and his sought to be represented by the collective bargaining agent must have
supervisor, and the supervisor must handle the prescribed responsibilities substantial mutual interests in terms of employment and working conditions
relating to labor relations. The exclusion from bargaining units of as evinced by the type of work they performed. [22] Considering the spin-
employees who, in the normal course of their duties, become aware of offs, the companies would consequently have their respective and
management policies relating to labor relations is a principal objective distinctive concerns in terms of the nature of work, wages, hours of work
sought to be accomplished by the "confidential employee rule." and other conditions of employment.  Interests of employees in the different
companies perforce differ.  SMC is engaged in the business of beer
San Miguel Corporation Employees Union v. Confesor manufacturing.  Magnolia is involved in the manufacturing and processing
Held: With respect to the second issue, there is, likewise, no of dairy products[23] while SMFI is involved in the production of feeds and
merit in petitioner-union’s assertion that the employees of Magnolia and the processing of chicken.[24] The nature of their products and scales of
SMFI should still be considered part of the bargaining unit of SMC. business may require different skills which must necessarily be
Magnolia and SMFI were spun-off to operate as distinct commensurated by different compensation packages.  The different
companies on October 1, 1991.  Management saw the need for these companies may have different volumes of work and different working
transformations in keeping with its vision and long term strategy as it conditions.  For such reason, the employees of the different companies see
explained in its letter addressed to the employees dated August 13, 1991: the need to group themselves together and organize themselves into
x x x As early as 1986, we announced the decentralization distinctive and different groups.  It would then be best to have separate
program and spoke of the need for structures that can react fast to bargaining units for the different companies where the employees can
competition, a changing environment, shorter product life cycles and shifts bargain separately according to their needs and according to their own
in consumer preference.  We further stated in the 1987 Annual Report to working conditions.
Stockholders that San Miguel’s businesses will be more autonomous and
self sufficient so as to better acquire and master new technologies, cope We reiterate what we have explained in the case of University of the
with a labor force with different expertises and expectations, and master Philippines v. Ferrer-Calleja that:
and satisfy the changing needs of our customers and end-consumers.  As [T]here are various factors which must be satisfied and
subsidiaries, Magnolia and FLD will gain better industry focus and considered in determining the proper constituency of a bargaining unit.  No
flexibility, greater awareness of operating results, and speedier, more one particular factor is itself decisive of the determination.  The weight
responsive decision making. accorded to any particular factor varies in accordance with the particular
xxx question or questions that may arise in a given case.  What are these

Page 10 of 117
factors?  Rothenberg mentions a good number, but the most pertinent to our " * * In said report, it is likewise emphasized that the basic test
case are: (1) will of the employees (Globe Doctrine); (2) affinity and unit of in determining the appropriate bargaining unit is that a unit, to be
employees’ interest, such as substantial similarity of work and duties, or appropriate, must affect a grouping of employees who have substantial,
similarity of compensation and working conditions; (3) prior collective mutual interests in wages, hours, working conditions and other subjects of
bargaining history; and (4) employment status, such as temporary, seasonal collective bargaining (citing Smith on Labor Laws, 316-317; Francisco,
and probationary employees x x. Labor Laws, 162). * *. "
xxx The Court further explained that "(t)he test of the grouping is
An enlightening appraisal of the problem of defining an community or mutuality of interests. And this is so because ‘the basic test
appropriate bargaining unit is given in the 10th Annual Report of the of an asserted bargaining unit's acceptability is whether or not it is
National Labor Relations Board wherein it is emphasized that the factors fundamentally the combination which will best assure to all employees the
which said board may consider and weigh in fixing appropriate units are: exercise of their collective bargaining rights’ (Rothenberg on Labor
the history, extent and type of organization of employees; the history of Relations, 490)." Hence, in that case, the Court upheld the trial court's
their collective bargaining; the history, extent and type of organization of conclusion that two separate bargaining units should be formed, one
employees in other plants of the same employer, or other employers in the consisting of regular and permanent employees and another consisting of
same industry; the skill wages, work, and working conditions of the casual laborers or stevedores.
employees; the desires of the employees; the eligibility of the employees Since then, the "community or mutuality of interests" test has
for membership in the union or unions involved; and the relationship provided the standard in determining the proper constituency of a collective
between the unit or units proposed and the employer’s organization, bargaining unit. In Alhambra Cigar & Cigarette Manufacturing Company,
management, and operation x x. et al. vs. Alhambra Employees' Association (PAFLU), 107 Phil. 23, the
x x In said report, it is likewise emphasized that the basic test in Court, noting that the employees in the administrative, sales and dispensary
determining the appropriate bargaining unit is that a unit, to be appropriate, departments of a cigar and cigarette manufacturing firm perform work
must affect a grouping of employees who have substantial, mutual interests which have nothing to do with production and maintenance, unlike those in
in wages, hours, working conditions and other subjects of collective the raw lead (malalasi), cigar, cigarette, packing (precintera) and
bargaining (citing Smith on Labor Laws, 316-317; Francisco, Labor Laws, engineering and garage departments, authorized the formation of the former
162) x x. set of employees into a separate collective bargaining unit. The ruling in
Finally, we take note of the fact that the separate interests of the the Democratic Labor Association case, supra, was reiterated in Philippine
employees of Magnolia and SMFI from those of SMC has been recognized Land-Air-Sea Labor Union vs. Court of Industrial Relations, 110 Phil. 176,
in the case of Daniel Borbon v. Laguesma.[26] We quote: where casual employees were barred from joining the union of the
Even assuming in gratia argumenti that at the time of the permanent and regular employees.
election they were regular employees of San Miguel, nonetheless, these Applying the same "community or mutuality of interests" test,
workers are no longer connected with San Miguel Corporation in any but resulting in the formation of only one collective bargaining unit is the
manner because Magnolia has ceased to be a division of San Miguel case of National Association of Free Trade Unions vs. Mainit Lumber
Corporation and has been formed into a separate corporation with a Development Company Workers Union-United Lumber and General
personality of its own (p. 305, Rollo).  This development, which was Workers of the Phils., G.R. No. 79526, December 21, 1990, 192 SCRA
brought to our attention by private respondents, necessarily renders moot 598. In said case, the Court ordered the formation of a single bargaining
and academic any further discourse on the propriety of the elections which unit consisting of the Sawmill Division in Butuan City and the Logging
petitioners impugn via the present recourse (p. 319, Rollo). Division in Zapanta Valley, Kitcharao, Agusan Norte of the Mainit Lumber
Development Company. The Court reasoned:
"Certainly, there is a mutuality of interest among the employees
University of the Philippines v. Ferrer-Calleja
of the Sawmill Division and the Logging Division. Their functions mesh
Held: When first confronted with the task of determining the
with one another. One group needs the other in the same way that the
proper collective bargaining unit in a particular controversy, the Court had
company needs them both. There may be difference as to the nature of their
perforce to rely on American jurisprudence. In Democratic Labor
individual assignments but the distinctions are not enough to warrant the
Association vs. Cebu Stevedoring Company, Inc., decided on February 28,
formation of a separate bargaining unit."
1958, the Court observed that "the issue of how to determine the proper
In the case at bar, the University employees may, as already
collective bargaining unit and what unit would be appropriate to be the
suggested, quite easily be categorized into two general classes: one, the
collective bargaining agency" * * * "is novel in this jurisdiction; however,
group composed of employees whose functions are non-academic, i.e.,
American precedents on the matter abound ** (to which resort may be had)
janitors, messengers, typists, clerks, receptionists, carpenters, electricians,
considering that our present Magna Carta has been patterned after the
grounds-keepers, chauffeurs, mechanics, plumbers; and two, the group
American law on the subject." Said the Court:
made up of those performing academic functions, i.e., full professors,
" * * Under these precedents, there are various factors which
associate professors, assistant professors, instructors -- who may be judges
must be satisfied and considered in determining the proper constituency of
or government executives -- and research, extension and professorial
a bargaining unit. No one particular factor is itself decisive of the
staff. Not much reflection is needed to perceive that the community or
determination. The weight accorded to any particular factor varies in
mutuality of interests which justifies the formation of a single collective
accordance with the particular question or questions that may arise in a
bargaining unit is wanting between the academic and non-academic
given case. What are these factors? Rothenberg mentions a good number,
personnel of the university. It would seem obvious that teachers would find
but the most pertinent to our case are: (1) will of the employees (Globe
very little in common with the University clerks and other non-academic
Doctrine); (2) affinity and unit of employees' interest, such as substantial
employees as regards responsibilities and functions, working conditions,
similarity of work and duties, or similarity of compensation and working
compensation rates, social life and interests, skills and intellectual pursuits,
conditions; (3) prior collective bargaining history; and (4) employment
cultural activities, etc. On the contrary, the dichotomy of interests, the
status, such as temporary, seasonal and probationary employees. * *.
dissimilarity in the nature of the work and duties as well as in the
" * * * * * *
compensation and working conditions of the academic and non-academic
"An enlightening appraisal of the problem of defining an
personnel dictate the separation of these two categories of employees for
appropriate bargaining unit is given in the 10th Annual Report of the
purposes of collective bargaining. The formation of two separate bargaining
National Labor Relations Board wherein it is emphasized that the factors
units, the first consisting of the rank-and-file non-academic personnel, and
which said board may consider and weigh in fixing appropriate units are:
the second, of the rank-and-file academic employees, is the set-up that will
the history, extent and type of organization of employees; the history of
best assure to all the employees the exercise of their collective bargaining
their collective bargaining; the history, extent and type of organization of
rights. These special circumstances, i.e., the dichotomy of interests and
employees in other plants of the same employer, or other employers in the
concerns as well as the dissimilarity in the nature and conditions of work,
same industry; the skill, wages, work, and working conditions of the
wages and compensation between the academic and non-academic
employees; the desires of the employees; the eligibility of the employees
personnel, bring the case at bar within the exception contemplated in
for membership in the union or unions involved; and the relationship
Section 9 of Executive Order No. 180. It was grave abuse of discretion on
between the unit or units proposed and the employer's organization,
the part of the Labor Relations Director to have ruled otherwise, ignoring
management, and operation. * *.
plain and patent realities.

Page 11 of 117
Belyca Corporation v. Ferrer-Calleja 4. Employment Status Doctrine
Held: Hence, still later following the substantial-mutual interest The determination of the appropriate bargaining unit
test, the Court ruled that there is a substantial difference between the work based on the employment status of the employees is considered an
performed by musicians and that of other persons who participate in the acceptable mode. For instance, casual employees and those
production of a film which suffice to show that they constitute a proper employed on a day-to-day basis do not have the mutuality or
bargaining unit. (LVN Pictures, Inc. v. Philippine Musicians Guild 1 SCRA
community of interest with regular and permanent employees.
132 (1961]).
Coming back to the case at bar, it is beyond question that the
Hence, their inclusion in the bargaining unit composed of the latter
employees of the livestock and agro division of petitioner corporation is not justified [Philippine Land-Air-Sea Labor Union v. CIR].
perform work entirely different from those performed by employees in the
supermarts and cinema. Among others, the noted difference are: their International School Alliance of Educators v. Quisumbing
working conditions, hours of work, rates of pay, including the categories of Held: A bargaining unit  is "a group of employees of a given
their positions and employment status. As stated by petitioner corporation employer, comprised of all or less than all of the entire body of employees,
in its position paper, due to the nature of the business in which its livestock- consistent with equity to the employer indicate to be the best suited to serve
agro division is engaged very few of its employees in the division are the reciprocal rights and duties of the parties under the collective bargaining
permanent, the overwhelming majority of which are seasonal and casual provisions of the law." The factors in determining the appropriate collective
and not regular employees (Rollo, p. 26). Definitely, they have very little in bargaining unit are (1) the will of the employees (Globe Doctrine); (2)
common with the employees of the supermarts and cinemas. To lump all affinity and unity of the employees' interest, such as substantial similarity
the employees of petitioner in its integrated business concerns cannot result of work and duties, or similarity of compensation and working conditions
in an efficacious bargaining unit comprised of constituents enjoying a (Substantial Mutual Interests Rule); (3) prior collective bargaining history;
community or mutuality of interest. Undeniably, the rank and file and (4) similarity of employment status. The basic test of an asserted
employees of the livestock-agro division fully constitute a bargaining unit bargaining unit's acceptability is whether or not it is fundamentally the
that satisfies both requirements of classification according to employment combination which will best assure to all employees the exercise of their
status and of the substantial, similarity of work and duties which will collective bargaining rights.
ultimately assure its members the exercise of their collective bargaining It does not appear that foreign-hires have indicated their
rights. intention to be grouped together with local-hires for purposes of collective
bargaining. The collective bargaining history in the School also shows that
2. Globe Doctrine these groups were always treated separately. Foreign-hires have limited
tenure; local-hires enjoy security of tenure. Although foreign-hires perform
This principle is based on the will of the employees. It is
similar functions under the same working conditions as the local-hires,
called Globe doctrine because this principle was first enunciated in foreign-hires are accorded certain benefits not granted to local-hires. These
the United States case of Globe Machine and Stamping Co., where benefits, such as housing, transportation, shipping costs, taxes, and home
it was ruled, in defining the appropriate bargaining unit, that in a leave travel allowance, are reasonably related to their status as foreign-
case where the company’s production workers can be considered hires, and justify the exclusion of the former from the latter. To include
either as a single bargaining unit appropriate for purposes of foreign-hires in a bargaining unit with local-hires would not assure either
collective bargaining or as three (3) separate and distinct group the exercise of their respective collective bargaining rights.
bargaining units, the determining factor is the desire of the
workers themselves consequently, a certification election should be C. FORMATION OF LABOR ORGANIZATIONS
held separately to choose which representative union will be
chosen by the workers [Mechanical Department Labor Union v. 1. Definition of Terms
CIR].
In International School Alliance of Educators v. a. Union
Quisumbing, the High Court ruled that foreign-hired teachers do "Company union" means any labor organization whose
not belong to the bargaining unit of the local hires because the formation, function or administration has been assisted by any act
former have not indicated their intention to be grouped with the defined as unfair labor practice by this Code [Article 219].
latter for purposes of collective bargaining. Moreover, the “Union” refers to any labor organization in the private
collective bargaining history of the school also shows that these sector organized for collective bargaining and for other legitimate
groups were always treated separately. purposes [IRR].

3. Collective Bargaining History Doctrine b. Labor Organization


This principle puts premium to the prior collective "Labor organization" means any union or association of
bargaining history and affinity of the employees in determining the employees which exists in whole or in part for the purpose of
appropriate bargaining unit. However, the existence of a prior collective bargaining or of dealing with employers concerning
collective bargaining history has been held as neither decisive nor terms and conditions of employment [Article 219].
conclusive in the determination of what constitutes an appropriate "Labor organization" means any union or association of
bargaining unit [San Miguel Corporation v. Laguesma]. employees in the private sector which exists in whole or in part for
It was ruled in National Association of Free Trade the purpose of collective bargaining, mutual aid, interest,
Unions v. Mainit Lumber Development Company Workers Union cooperation, protection, or other lawful purposes [IRR].
that there is mutuality of interest among the workers in the sawmill
division and logging division as to justify their formation of a c. Legitimate Labor Organization (LLO)
single bargaining unit. This holds true despite the history of said "Legitimate labor organization" means any labor
two divisions being treated as separate units and notwithstanding organization duly registered with the Department of Labor and
their geographical distance from each other. Employment, and includes any branch or local thereof. [Article
In San Miguel v. Laguesma, despite the collective 219].
bargaining history of having a separate bargaining unit for each "Legitimate labor organization" means any labor
sales office, the Supreme Court applied the principle of mutuality organization in the private sector registered or reported with the
or commonality of interests in holding that the appropriate Department of Labor and Employment [IRR].
bargaining unit is comprised of all the sales force in the whole of
Northern Luzon. d. Workers’ Association
Page 12 of 117
“Workers’ Association” refers to an association of
workers organized for the mutual aid and protection of its members (iii) Trade Union Center
or for any other purpose other than collective bargaining [IRR]. Any group registered national unions or federations
organized for mutual aid and protection of its members, assisting in
e. Legitimate Workers’ Association such collective bargaining or participating in the formulation of
“Legitimate Workers’ Association” refers to an social and employment policies, standards and programs, duly
association of workers organized for the mutual aid and protection registered with the DOLE.
of its members or for any other purpose other than collective
bargaining registered with the Department [IRR]. Electromat v. Lagunzad
Held: We resolve the core issue of whether D.O. 40-03 is a
Labor Organization vs. Workers’ Association valid exercise of the rule-making power of the DOLE.
We rule in the affirmative. Earlier in Progressive
As to Labor Workers’ Development Corporation v. Secretary, Department of Labor and
Employment, the Court encountered a similar question on the validity of the
Organization Association
old Section 3, Rule II, Book  V of the Rules Implementing the Labor
Existence of Er-E Necessary Not necessary
Relationship Code which stated:
Purpose In whole or in part, For mutual aid and Union affiliation; direct membership with a national union. -
for collective protection The affiliate of a labor federation or national union may be a local or
bargaining chapter thereof or an independently registered union.
Formation or Can be registered as Registration is a) The labor federation or national union concerned shall issue a
registration an independent union provided under the charter certificate indicating the creation or establishment of a local or
or become a IRR chapter, copy of which shall be submitted to the Bureau of Labor Relations
chartered local under within thirty (30) days from issuance of such charter certificate.
the Labor Code xxxx
Right to file a Can file Cannot file e)  The local or chapter of a labor federation or national union
Petition for shall have and maintain a constitution and by-laws, set of officers and
Certification books of accounts. For reporting purposes, the procedure governing the
Election reporting of independently registered unions, federations or national unions
Basis Right to Self- Constitutional right shall be observed.
Organization of Freedom of
Interpreting these provisions of the old rules, the Court said that
Association
by force of law,  the local or chapter of a labor federation or national union
becomes a legitimate labor organization upon compliance with Section 3,
2. Classifications Rule II, Book V of the Rules Implementing the Labor Code, the only
requirement being the submission of the charter certificate to the BLR.
a. As to How Formed Further, the Court noted that Section 3 omitted several requirements which
are otherwise required for union registration, as follows:
(i) Independent Union 1) The requirement that the application for registration
This refers to a labor organization operating at the must be signed by at least 20% of the employees in the appropriate
bargaining unit;
enterprise level that acquired legal personality through independent
2) The submission of officers' addresses, principal
registration via issuance of Certificate of Registration [Article address of the labor organization, the minutes of organization meetings and
240]. the list of the workers who participated in such meetings;
3) The submission of the minutes of the adoption or
(ii) Chartered Local ratification of the constitution and by-laws and the list of the members who
This refers to a labor organization in the private sector participated in it.
operating at the enterprise level that acquired legal personality Notwithstanding these omissions, the Court upheld the
through the issuance of a charter certificate by a duly registered government's implementing policy expressed in the old rules when it
declared in Progressive Development -
federation or national union, and reported to the national office
Undoubtedly, the intent of the law in imposing lesser
[IRR]. requirements in the case of a branch or local of a registered federation or
national union is to encourage the affiliation of a local union with a
(iii) Affiliate federation or national union in order to increase the local union's bargaining
This refers to (1) an independent union affiliated with a powers respecting terms and conditions of labor.
federation, national union or (2) a chartered local which was It was this same Section 3 of the old rules that D.O. 40-03 fine-
subsequently granted independent registration but did not tuned when the DOLE amended the rules on Book V of the Labor Code,
disaffiliate from its federation, reported to the Regional Office and thereby modifying the government's implementing policy on the
registration of locals or chapters of labor federations or national unions.
the Bureau [IRR].
The company now assails this particular amendment as an invalid exercise
of the DOLE's rule-making power.
b. As to Components We disagree. As in the case of D.O. 9 (which introduced the
above-cited Section 3 of the old rules) in Progressive Development, D.O.
(i) Federation 40-03 represents an expression of the government's implementing policy on
A group of legitimate labor unions in a private trade unionism.  It builds upon the old rules by further simplifying the
establishment organized for collective bargaining or for dealing requirements for the establishment of locals or chapters. As in D.O. 9, we
with employers concerning terms and conditions of employment see nothing contrary to the law or the Constitution in the adoption by the
Secretary of Labor and Employment of D.O. 40-03 as this department order
for their member unions registered with the BLR.
is consistent with the intent of the government to encourage the affiliation
of a local union with a federation or national union to enhance the local's
(ii) National Union (Labor Center) bargaining power. If changes were made at all, these were those made to
A group of legitimate labor unions in a private recognize the distinctions made in the law itself between federations and
establishment organized not for collective bargaining but for their local chapters, and independent unions; local chapters seemingly have
participating in the formulation of social and employment policies, lesser requirements because they and their members are deemed to be direct
standards and programs, registered with the BLR.

Page 13 of 117
members of the federation to which they are affiliated, which federations A labor organization may be organized under the
are the ones subject to the strict registration requirements of the law. Corporation Law as a nonstock corporation and issued a certificate
In any case, the local union in the present case has more than of incorporation by the Securities and Exchange Commission. But
satisfied the requirements the petitioner complains about; specifically, the such incorporation has only the effect of giving to it juridical
union has submitted: (1) copies of the ratified CBL; (2) the minutes of the
personality before regular courts of justice. Such incorporation
CBL's adoption and ratification; (3) the minutes of the organizational
meetings; (4) the names and addresses of the union officers; (5) the list of
does not grant the rights and privileges of a legitimate labor
union members; (6) the list of rank-and-file employees in the company; (7) organization.
a certification of non-existence of a CBA in the company; (8) the resolution
of affiliation with WASTO and the latter's acceptance; and (9) their Charter Cebu Seamen’s Association, Inc. v. Ferrer-Calleja
Certificate. These submissions were properly verified as required by the Held: The determinative issue in this case is who is entitled to
rules. In sum, the petitioner has no factual basis for questioning the union's the collection and custody of the union dues? Cebu Seamen's Association
registration, as even the requirements for registration as an independent headed by Gabayoyo or Seamen's Association of the Philippines headed by
local have been substantially complied with. Nacua.
We, thus, find no compelling justification to nullify D.O. 40-03. As stated in the findings of fact in the questioned resolution of
Director Pura Ferrer-Calleja on October 23, 1950, a group of deck officers
3. Registration organized the Cebu Seamen's Association, Inc., (CSAI), a non-stock
corporation and registered it with the Securities and Exchange Commission
(SEC). The same group registered the organization with the Bureau of
a. Requirements
Labor Relations (BLR) as Seamen's Association of the Philippines (SAPI).
It is the registration of the organization with the BLR and not with the SEC
Article 240. [234] Requirements of Registration. - A which made it a legitimate labor organization with rights and privileges
federation, national union or industry or trade union center or an granted under the Labor Code.
independent union shall acquire legal personality and shall be entitled We gathered from the records that CSAI, the corporation was
to the rights and privileges granted by law to legitimate labor already inoperational before the controversy in this case arose. In fact, on
organizations upon issuance of the certificate of registration based on August 24, 1984, the SEC ordered the CSAI to show cause why its
the following requirements: certificate of registration should not be revoked for continuous inoperation
(a) Fifty pesos (P50.00) registration fee; (p. 343, Rollo). There is nothing in the records which would show that
(b) The names of its officers, their addresses, the principal CSAI answered said show-cause order.
address of the labor organization, the minutes of the organizational Also, before the controversy, private respondent Dominica
meetings and the list of the workers who participated in such meetings; Nacua was elected president of the labor union, SAPI. It had an existing
(c) In case the applicant is an independent union, the names CBA with Aboitiz Shipping Corporation. Before the end of the term of
of all its members comprising at least twenty percent (20%) of all the private respondent Nacua, some members of the union which included
employees in the bargaining unit where it seeks to operate; Domingo Machacon and petitioner Manuel Gabayoyo showed signs of
(d) If the applicant union has been in existence for one or discontentment with the leadership of Nacua. This break-away group
more years, copies of its annual financial reports; and revived the moribund corporation and issued an undated resolution
(e) Four copies of the constitution and by-laws of the expelling Nacua from the association (pp. 58-59, Rollo). Sometime in
applicant union, minutes of its adoption or ratification, and the list of February, 1987, it held its own election of officers supervised by the
the members who participated in it. Securities and Exchange Commission. It also filed a case of estafa against
Nacua sometime in May, 1986 (p. 52, Rollo).
The certification and attestation requirements are The expulsion of Nacua from the corporation, of which she
preventive measures against the commission of fraud. They denied being a member, has however, not affected her membership with the
labor union. In fact, in the elections of officers for 1987-1989, she was re-
likewise afford a measure of protection to unsuspecting employees
elected as the president of the labor union. In this connection, We cannot
who may be lured into joining unscrupulous or fly-by-night unions agree with the contention of Gabayoyo that Nacua was already expelled
whose sole purpose is to control union funds or to use the union for from the union. Whatever acts their group had done in the corporation do
dubious ends [Progressive Development Corporation v. Secretary]. not bind the labor union. Moreover, Gabayoyo cannot claim leadership of
the labor group by virtue of his having been elected as a president of the
b. Effect of Registration dormant corporation CSAI.
Rule IV, Section 8 of the IRR provides: Under the principles of administrative law in force in this
jurisdiction, decisions of administrative officers shall not be disturbed by
Section 8. Effect of Registration. – The labor union courts, except when the former acted without or in excess of their
or workers’ association shall be deemed registered and vested jurisdiction or with grave abuse of discretion.
with legal personality on the date of the issuance of its Public respondent Bureau of Labor Relations correctly ruled on
certificate of registration or certificate of creation of chartered the basis of the evidence presented by the parties that SAPI, the legitimate
local. labor union, registered with its office, is not the same association as CSAI,
Such legal personality may be questioned only the corporation, insofar as their rights under the Labor Code are concerned.
through an independent petition for cancellation of union Hence, the former and not the latter association is entitled to the release and
registration in accordance with Rule XIV of these Rules, and not custody of union fees with Aboitiz Shipping and other shipping companies
by way of collateral attack in petition for certification election with whom it had an existing CBA. As correctly held by public respondent:
proceedings under Rule VIII. "It is undisputed from the records that the election of the so
called set of officers headed by Manuel Gabayoyo was conducted under the
supervision of the SEC, presumably in accordance with its constitution and
After a certificate of registration is issued to a union, its by-laws as well as the articles of incorporation of respondent CSAI, and the
legal personality cannot be subject to collateral attack. It may be Corporation Code. That had been so precisely on the honest belief of the
questioned only in an independent petition for cancellation The participants therein that they were acting in their capacity as members of
inclusion in a union of disqualified employees is not among the the said corporation. That being the case, the aforementioned set of officers
grounds for cancellation, unless such inclusion is due to is of the respondent corporation and not of the complainant union.
misrepresentation, false statement or fraud under the It follows, then, that any proceedings and actions taken by said set of
circumstances enumerated [Tagaytay Highlands v. Tagaytay officers can not, in any manner, affect the union and its members.
"On the other hand, we rule and so hold that the other set of
Highlands Employees Union].
officers headed by Dominica C. Nacua is the lawful set of officers of SAPI
and therefore, is entitled to the release and custody of the union dues as
Registration under the Corporation Code well as the agency fees, if any, there be. A record check with the Labor

Page 14 of 117
Organizations (LOD), this Bureau, shows that SAPI has submitted to it for the majority of the 119 union members would already constitute a
file the list of this new set of officers, in compliance with the second quorum for the meeting to proceed and to validly ratify the
paragraph of Article 242 (c) of the Labor Code. This list sufficiently Constitution and By-laws of the union. There is, therefore, no basis
sustains the view that said officers were lawfully elected, in the absence of for petitioner to contend that grounds exist for the cancellation of
clear and convincing proof to the contrary." (pp. 9-10, Rollo)
respondent's union registration. For fraud and misrepresentation to
be grounds for cancellation of union registration under Article 240
c. Where to File Registration of the Labor Code, the nature of the fraud and misrepresentation
Applications for registration of independent labor unions, must be grave and compelling enough to vitiate the consent of a
chartered locals, workers’ associations shall be filed with the majority of union members.
Regional Office where the applicant principally operates. It shall be
processed by the Labor Relations Division at the Regional Office. e. Requirements for Federations and National
Applications for registration of federations, national Unions
unions, or workers’ associations operating in more than one region The application or registration of federations and national
shall be filed with the Bureau or the Regional offices, but shall be unions shall be accompanied by the following documents:
processed by the Bureau.
(i) A statement indicating the name of the applicant
d. Requirements for Registration for labor union, its principal address, the name of its
Independent Labor Unions officers and their respective addresses;
The application for registration of an independent labor (ii) The minutes of the organizational meeting(s) and
union shall be accompanied by the following documents: the list of employees who participated in the said
meeting(s);
(iii) The annual financial reports if the applicant has
(i) The name of the applicant labor union, the
been in existence for one or more years, unless it has
principal address, the name of its officers and
not collected any amount from the members, in
their respective addresses, approximate number of
which case a statement to this effect shall be
employees in the bargaining unit where it seeks to
included in the application;
operate, with a statement that it is not reported as
(iv) The applicant’s constitution and by-laws, minutes of
a chartered local of any federation or national
its adoption or ratification, and the list of the
union;
members who participated in it. The list of ratifying
(ii) The minutes of the organizational meeting(s) and
members shall be dispensed with where the
the list of employees who participated in the said
constitution and by-laws was ratified or adopted
meeting(s);
during the organizational meeting. In such a case,
(iii) The name of all its members comprising at least
the factual circumstances of the ratification shall be
20% of the employees in the bargaining unit;
recorded in the minutes of the organizational
(iv) The annual financial reports if the applicant has
meeting(s);
been in existence for one or more years, unless it
(v) The resolution of affiliation at least ten (10)
has not collected any amount from the members,
legitimate labor organizations, whether independent
in which case a statement to this effect shall be
unions or chartered locals, each of which must be a
included in the application;
duly certified or recognized bargaining agent in the
(v) The applicant’s constitution and by-laws, minutes
establishment where it seeks to operate; and
of its adoption or ratification, and the list of the
(vi) The name and addresses of the companies where the
members who participated in it. The list of
affiliates operate and the list of all the members in
ratifying members shall be dispensed with where
each company involved.
the constitution and by-laws was ratified or
adopted during the organizational meeting. In
such a case, the factual circumstances of the f. Approval
ratification shall be recorded in the minutes of the It cannot be over-emphasized that the registration or the
organizational meeting(s). recognition of a labor union after it has submitted the
corresponding papers is not ministerial on the part of the BLR. Far
It does not appear in the Labor Code that the attendees in from it. After a labor organization has filed the necessary
the organizational meeting must comprise 20% of the employees in registration documents, it becomes mandatory for the BLR to
the bargaining unit. In fact, even the Implementing Rules and check if the requirements under Article 240 of the Labor Code
Regulations of the Labor Code does not so provide. It only requires have been sedulously complied with. If the union's application is
the names of all its members comprising at least twenty percent infected by falsification and like serious irregularities, especial
(20%) of all the employees in the bargaining unit where it seeks to those appearing on the face of the application and its attachments, a
operate. Clearly, the 20% minimum requirement pertains to the union should be denied recognition as a legitimate labor
employees’ membership in the union and not to the list of workers organization [S.S. Ventures International, Inc. v. S.S. Ventures
who participated in the organizational meeting. In Takata v. BLR, Labor Union].
the total number of employees in the bargaining unit was 396, and
20% of which was about 79. Respondent submitted a document g. Registration of Chartered Local
entitled "Pangalan ng Mga Kasapi ng Unyon" showing the names
of 119 employees as union members, thus respondent sufficiently Article 241. [234-A] Chartering and Creation of a Local
complied even beyond the 20% minimum membership Chapter. - A duly registered federation or national union may directly
requirement. Respondent also submitted the attendance sheet of the create a local chapter by issuing a charter certificate indicating the
organizational meeting which contained the names and signatures establishment of the local chapter. The chapter shall acquire legal
personality only for purposes of filing a petition for certification
of the 68 union members who attended the meeting. Considering
election from the date it was issued a charter certificate.
that there are 119 union members which are more than 20% of all The chapter shall be entitled to all other rights and
the employees of the bargaining unit, and since the law does not privileges of a legitimate labor organization only upon the submission
provide for the required number of members to attend the of the following documents in addition to its charter certificate:
organizational meeting, the 68 attendees which comprised at least

Page 15 of 117
(a) The names of the chapter's officers, their addresses, and An affiliate is (a) an independent union affiliated with a
the principal office of the chapter; and federation or national union; or (b) a chartered local which was
(b) The chapter's constitution and by-laws: Provided, That subsequently granted independent registration but did not
where the chapter's constitution and by-laws are the same as that of disaffiliate from its federation.
the federation or the national union, this fact shall be indicated
In relation to an affiliate, the federation or national union
accordingly.
The additional supporting requirements shall be certified
is commonly known as the mother union. The mother union, acting
under oath by the secretary or treasurer of the chapter and attested by for and in behalf of its affiliate, has the status of an agent while the
its president. affiliate or local chapter or chartered local remains the principal –
the basic unit of the association free to serve the common interest
Chartered locals have two types of personalities. First, it of all its members, subject only to the restraints imposed by the
has provisional personality if it only has a charter certificate. The constitution and by-laws of the association [Progressive
chartered local is considered an LLO only for the purpose of filing Development Corporation v. Secretary].
a Petition for Certification Election. Second, it has full personality
if it submits all other documents for registration. It will be a. Requirements of Affiliation
considered as an LLO for all purposes (i.e., to file cases for its The report of affiliation of an independently registered
members, to purchase property in its name, etc.) and possesses full labor union with a federation or national union shall be filed with
powers of a LLO. the Regional Office that issued the certificate of registration [IRR].
A local or chapter need not be independently registered Moreover, the report of affiliation of independently
to acquire legal personality [Laguna Autoparts v. Office of the registered labor unions with a federation or national union shall be
Secretary]. A chapter acquires legal personality on "the date it was accompanied by the following documents:
issued a charter certificate" by its mother federation or national
union. The acquisition of legal personality seems to happen (i) Resolution of the labor union’s board of directors
approving the affiliation;
automatically, but "only for purposes of filing a petition for a
(ii) Minutes of the general membership meeting
certification election." approving the affiliation;
(iii) The total number of members comprising the labor
Not required to acquire independent registration in union and the names of members who approved the
order to have legal personality affiliation;
(iv) The certificate of affiliation issued by the federation
Ordinarily, a labor organization attains the status of
in favor of the independently registered labor union;
legitimacy only upon the issuance in its name of a Certificate of and
Registration by the Bureau of Labor Relations pursuant to Articles (v) Written notice to the employer concerned if the
241 and 242 of the Labor Code. However, the procedure is not the affiliating union is the incumbent bargaining agent.
only way by which a labor union may become legitimate,
however.  When an unregistered union becomes a branch, local or b. Effects of Affiliation
chapter of a federation, some of the aforementioned requirements The following are the effects of affiliation:
for registration are no longer require [San Miguel Foods v.
Laguesma]. (i) The mother union becomes the agent, and the
affiliate union, the principal.
Distinguished from other unions (ii) The affiliate union becomes subject to the
rules of the mother union
Federation, national union, Chartered local (iii) The appendage of the acronym of the mother
industry or trade union or union after the name of the affiliate union does
independent union
not mean that the affiliate cannot
Acquisition of legal personality Acquisition of legal personality
under Article 240 in relation to under Article 241 in relation to independently stand on its own [THFEU-
IRR IRR CGW vs. Tropical Hut].
Acquires legal personality upon Acquires legal personality in 2 (iv) Affiliation does not give the mother union the
the issuance of a certificate of stages (provisional/partial and license to act independently of the affiliate
registration full) union. However, if the mother union
More stringent because it requires Easier because it only requires
Certificate of Registration charter certificate negotiated the CBA, the affiliate union cannot
(notarized) negotiate the renewal of the CBA without the
consent and participation of the mother union
h. Action on and Denial of Application for [Abaria v. NLRC].
Registration (v) The affiliate union and not the mother union
has the power to administer and enforce the
Article 242. [235] Action on Application. - The Bureau shall CBA with the employer [NAFLU v. Noriel].
act on all applications for registration within thirty (30) days from (vi) In case of illegal strike, the affiliate union, not
filing. the mother union, is liable for damages
All requisite documents and papers shall be certified under [Filipino Pipe v. NLRC].
oath by the secretary or the treasurer of the organization, as the case
may be, and attested to by its president.
c. Significance of Affiliation
Legal significance of affiliation is that the mother union,
Article 243. [236] Denial of Registration; Appeal. - The acting for and in behalf of its affiliate, has the status of an agent
decision of the Labor Relations Division in the regional office denying while the local union remains the basic unit of association free to
registration may be appealed by the applicant union to the Bureau
serve the common interest of all its members subject only to the
within ten (10) days from receipt of notice thereof.
restraints imposed by the constitution and by-laws of the
association [Pambansang Kapatiran v. Secretary].
4. Affiliation
Page 16 of 117
party to the illegal dismissal of the petitioner workers.  Under such a
5. Disaffiliation situation, the respondent company should be jointly and severally liable
The right to disaffiliate by the local union from its with the respondent PAFLU for the payment of backwages to the petitioner
mother federation or national union is a constitutionally-guaranteed workers.
right which may be invoked by the former at any time. It is
axiomatic that an affiliate union is a separate and voluntary Philippine Skylanders v. NLRC
association free to serve the interest of all its members – consistent Held: Upon an application of the aforecited principle to the issue
with the freedom of association guaranteed in the Constitution at hand, the impropriety of the questioned Decisions becomes clearly
[Volkschel Labor Union v. BLR]. apparent.  There is nothing shown in the records nor is it claimed by
PAFLU that the local union was expressly forbidden to disaffiliate from the
It is not an act of disloyalty on the part of the local union
federation nor were there any conditions imposed for a valid breakaway. 
nor is it a violation of the union security clause in the CBA. As such, the pendency of an election protest involving both the mother
Disaffiliation should always carry the will of the federation and the local union did not constitute a bar to a valid
majority. It cannot be effected by a mere minority group of union disaffiliation.  Neither was it disputed by PAFLU that 111 signatories out of
members [Villar v. Inciong]. the 120 members of the local union, or an equivalent of 92.5% of the total
The right of a local union to disaffiliate from its mother union membership supported the claim of disaffiliation and had in fact
federation is not a novel thesis unillumined by case law.  In the disauthorized PAFLU from instituting any complaint in their behalf. 
Surely, this is not a case where one (1) or two (2) members of the local
landmark case of Liberty Cotton Mills Workers Union vs. Liberty
union decided to disaffiliate from the mother federation, but it is a case
Cotton Mills, Inc. we upheld the right of local unions to separate
where almost all local union members decided to disaffiliate.
from their mother federation on the ground that as separate and It was entirely reasonable then for PSI to enter into a collective
voluntary associations, local unions do not owe their creation and bargaining agreement with PSEA-NCW.  As PSEA had validly severed
existence to the national federation to which they are affiliated but, itself from PAFLU, there would be no restrictions which could validly
instead, to the will of their members.  The sole essence of hinder it from subsequently affiliating with NCW and entering into a
affiliation is to increase, by collective action, the common collective bargaining agreement in behalf of its members.
bargaining power of local unions for the effective enhancement and There is a further consideration that likewise argues for the
granting of the petitions.   It stands unchallenged that PAFLU instituted the
protection of their interests.   Admittedly, there are times when
complaint for unfair labor practice against the wishes of workers whose
without succor and support local unions may find it hard, unaided interests it was supposedly protecting.  The mere act of disaffiliation did not
by other support groups, to secure justice for themselves. Yet the divest PSEA of its own personality; neither did it give PAFLU the license
local unions remain the basic units of association, free to serve to act independently of the local union.  Recreant to its mission, PAFLU
their own interests subject to the restraints imposed by the cannot simply ignore the demands of the local chapter and decide for its
constitution and by-laws of the national federation, and free also to welfare.  PAFLU might have forgotten that as an agent it could only act in
renounce the affiliation upon the terms laid down in the agreement representation of and in accordance with the interests of the local union. 
which brought such affiliation into existence [Philippine The complaint then for unfair labor practice lodged by PAFLU against PSI,
PSEA and their respective officers, having been filed by a party which has
Skylanders v. NLRC].
no legal personality to institute the complaint, should have been dismissed
at the first instance for failure to state a cause of action.
Liberty Cotton Mills Workers’ Union v. Liberty Policy considerations dictate that in weighing the claims of a
Cotton Mills local union as against those of a national federation, those of the former
Held: Likewise, the records show that the disaffiliation of the must be preferred.  Parenthetically though, the desires of the mother
local union members from the PAFLU was caused by the alleged federation to protect its locals are not altogether to be shunned.  It will
negligence of PAFLU and its lack of concern over the problems of the local however be to err greatly against the Constitution if the desires of the
union and its members, particularly its neglect in not providing the local federation would be favored over those of its members.  That, at any rate, is
union with a good lawyer who would attend to their ULP case against the the policy of the law.  For if it were otherwise, instead of protection, there
company.  This apparent laxity or negligence of PAFLU invites suspicion. would be disregard and neglect of the lowly workingmen.
The records also show that the local union members were
dissatisfied with the way PAFLU negotiated the Collective Bargaining
Tropical Hut Employees Union v. Tropical Hut
Agreement with the company because it did not fight for their demands and
Held: This brings Us to the question of the legality of the
instead accepted the proposals of the company.  
dismissal meted to petitioner employees. In the celebrated case of Liberty
And furthermore, PAFLU expelled only six (6) union members,
Cotton Mills Workers Union v. Liberty Cotton Mills, L-33187, September
because PAFLU erroneously contends that their disaffiliation and their
4, 1975, 66 SCRA 512, We held that the validity of the dismissals pursuant
refusal to retract amounted to disloyalty.  It was not disloyalty; it was their
to the union security clause in the collective bargaining agreement hinges
dissatisfaction with PAFLU that compelled them to disaffiliate.  The
on the validity of the disaffiliation of the local union from the federation.
constitutional guarantee of security of tenure of the worker and his freedom
The right of a local union to disaffiliate from its mother
of association - to join or not to join a union - are paramount and should
federation is well-settled. A local union, being a separate and voluntary
prevail over a contractual condition for continued union membership and
association, is free to serve the interest of all its members including the
over whimsical or arbitrary termination of his employment.
freedom to disaffiliate when circumstances warrant. This right is consistent
Respondent PAFLU also overlooked the fact that only sixteen
with the constitutional guarantee of freedom of association (Volkschel
(16) out of the original thirty-two (32) signatories retracted their
Labor Union v. Bureau of Labor Relations,  No. L-45824, June 19,1985,
disaffiliation.  PAFLU should have also expelled the remaining sixteen
137 SCRA 42).
members who did not retract, instead of only the six members, if indeed
All employees enjoy the right to self organization and to form
their unretracted disaffiliation were an act of disloyalty instead of
and join labor organizations of their own choosing for the purpose of
dissatisfaction with the PAFLU's failure to promote and defend their
collective bargaining and to engage in concerted activities for their mutual
interests.
aid or protection. This is a fundamental right of labor that derives its
It is also worth considering here the fact that of the six members
existence from the Constitution. In interpreting the protection to labor and
expelled by PAFLU and subsequently dismissed by the company, four (4)
social justice provisions of the Constitution and the labor laws or rules or
were officers of the local union and two were ordinary members.
regulations, We have always adopted the liberal approach which favors the
The company also failed to notice this fact and proceeded
exercise of labor rights.
immediately to grant the request of PAFLU by dismissing the petitioner
Relevant on this point is the basic principle We have repeatedly
workers without giving them the opportunity to be heard.
affirmed in many rulings: 
These facts and circumstances on record further underscore the
"x x x The locals are separate and distinct units primarily
existence of conspiracy or connivance between the company and PAFLU in
designed to secure and maintain an equality of bargaining power between
the dismissal of the petitioner workers.  Respondent company is therefore a

Page 17 of 117
the employer and their employee-members in the economic struggle for the discretion in reversing the decision of the labor arbiter (San Miguel
fruits of the joint productive effort of labor and capital; and the association Corporation v. NLRC, L-50321, March 13, 1984, 128 SCRA 180). In the
of the locals into the national union (PAFLU) was in furtherance of the instant case, the factual findings of the arbitrator were correct against that
same end. These associations are consensual entities capable of entering of public respondents.
into such legal relations with their members. The essential purpose was the Further, there is no merit in the contention of the respondents
affiliation of the local unions into a common enterprise to increase by that the act of disaffiliation violated the union security clause of the CBA
collective action the common bargaining power in respect of the terms and and that their dismissal as a consequence thereof is valid. A perusal of the
conditions of labor. Yet the locals remained the basic units of association, collective bargaining agreements shows that the THEU-NATU, and not the
free to serve their own and the common interest of all, subject to the NATU federation, was recognized as the sole and exclusive collective
restraints imposed by the Constitution and By-Laws of the Association, and bargaining agent for all its workers and employees in all matters concerning
free also to renounce the affiliation for mutual welfare upon the terms laid wages, hours of work and other terms and conditions of employment (pp.
down in the agreement which brought it into existence.'" (Adamson & 667-706, Rollo). Although NATU was designated as the sole bargaining
Adamson, Inc. v. CIR, No. L-35120, January 31, 1984, 127 SCRA agent in the check-off authorization form attached to the CBA, this simply
268; Elisco-Elirol Labor Union (NAFLU) v. Noriel, No. L-41955, means it was acting only for and in behalf of its affiliate. The NATU
December 29, 1977, 80 SCRA 681; Liberty Cotton Mills Workers Union v. possessed the status of an agent while the local union remained the basic
Liberty Cotton Mills, Inc., supra). principal union which entered into contract with the respondent company.
The inclusion of the word NATU after the name of the local When the THEU disaffiliated from its mother federation, the former did not
union THEU in the registration with the Department of Labor is merely to lose its legal personality as the bargaining union under the CBA. Moreover,
stress that the THEU is NATU's affiliate at the time of the registration. It the union security clause embodied in the agreements cannot be used to
does not mean that the said local union cannot stand on its own. Neither can justify the dismissals meted to petitioners since it is not applicable to the
it be interpreted to mean that it cannot pursue its own interests circumstances obtaining in this case. The CBA imposes dismissal only in
independently of the federation. A local union owes its creation and case an employee is expelled from the union for joining another federation
continued existence to the will of its members and not to the federation to or for forming another union or who fails or refuses to maintain
which it belongs. membership therein. The case at bar does not involve the withdrawal of
When the local union withdrew from the old federation to join a merely some employees from the union but of the whole THEU itself from
new federation, it was merely exercising its primary right to labor its federation. Clearly, since there is no violation of the union security
organization for the effective enhancement and protection of common provision in the CBA, there was no sufficient ground to terminate the
interests. In the absence of enforceable provisions in the federation's employment of petitioners.
constitution preventing disaffiliation of a local union, a local may sever its Public respondents considered the existence of Arturo Dilag's
relationship with its parent (People's Industrial and Commercial Employees group as the remaining true and valid union. We, however, are inclined to
and Workers Organization (FFW) v. People's Industrial and Commercial agree instead with the Arbitrator's findings when he declared: 
Corporation, No-37687, March 15, 1982, 112 SCRA 440). "x x x. Much more, the so called THEU-NATU under Dilag's
There is nothing in the constitution of the NATU or in the group which assumes to be the original THEU-NATU has a very doubtful
constitution of the THEU-NATU that the THEU was expressly forbidden to and questionable existence not to mention that the alleged president is
disaffiliate from the federation (pp. 62, 281, Rollo). The alleged non- performing supervisory functions and not qualified to be a bona fide 
compliance of the local union with the provision in the NATU Constitution member of the rank and file union." (p. 146, Rollo)
requiring the service of three months notice of intention to withdraw did not Records show that Arturo Dilag had resigned in the past as
produce the effect of nullifying the disaffiliation for the following grounds: President of THEU-NATU because of his promotion to a managerial or
firstly, NATU was not even a legitimate labor organization, it appearing supervisory position as Assistant Unit Manager of respondent Company.
that it was not registered at that time with the Department of Labor, and Petitioner Jose Encinas replaced Dilag as President and continued to hold
therefore did not possess and acquire, in the first place, the legal personality such position at the time of the disaffiliation of the union from the
to enforce its constitution and laws, much less the right and privilege under federation. It is therefore improper and contrary to law for Dilag to
the Labor Code to organize and affiliate chapters or locals within its group, reassume the leadership of the remaining group which was alleged to be the
and secondly, the act of non-compliance with the procedure on withdrawal true union since he belonged to the managerial personnel who could not be
is premised on purely technical grounds which cannot rise above the expected to work for the betterment of the rank and file employees.
fundamental right of self-organization. Besides, managers and supervisors are prohibited from joining a rank and
Respondent Secretary of Labor, in affirming the decision of the file union (Binalbagan Isabela Sugar Co., Inc. (BISCOM) v. Philippine
respondent Commission, concluded that the supposed decision to Association of Free Labor Unions (PAFLU), et al., L-18782, August 29,
disaffiliate was not the subject of a free and open discussion and decision 1963, 8 SCRA 700). Correspondingly, if a manager or supervisor organizes
on the part f the THEU-NATU general membership (p. 305, Rollo). This, or joins a rank and file union, he will be required to resign
however, is contradicted by the evidence on record. Moreover, We are therefrom (Magalit, et al. v. Court of Industrial Relations, et al., L-20448,
inclined to believe Arbitrator Villatuya's findings to the contrary, as May 25, 1965, 14 SCRA 72).
follows:  Public respondents further submit that several employees who
"x x x. However, the complainants refute this allegation by disaffiliated their union from the NATU subsequently retracted and
submitting the following: a) Letter dated December 20, 1973 signed by 142 reaffirmed their membership with the THEU-NATU. In the decision which
members (Exhs. 'B to B-5'); b) resolution dated January 12, 1974, signed by was affirmed by respondent Secretary of Labor, the respondent
140 members (Exhs. 'H to H-6'); c) letter dated February 26, 1974 to the Commission stated that: 
Department of Labor signed by 165 members (Exhs. 'I to 1-10'); d) letter "x x x out of the alleged one hundred and seventy-one (171)
dated January 30, 1974 to the Secretary of the National Defense signed by members of the THEU-CGW whose signatures appeared in the ‘Analysis of
144 members (Exhs. '0 to 0-5') and; e) letter dated March 6,1974 signed by Various Documents Signed by Majority Members of the THEU-CGW,
146 members addressed to the President of the Philippines (Exhs. 'HH to (Annex 'T', Complainants), which incidentally was relied upon by
HH-5'), to show that in several instances, the members of the THEU-NATU Arbitrator Villatuya in holding that complainant THEU-CGW commanded
have acknowledged their disaffiliation from NATU. The letters of the the majority of employees in respondent company, ninety?three (93) of the
complainants also indicate that an overwhelming majority have freely and alleged signatories reaffirmed their membership with the THEU-NATU and
voluntarily signed their union's disaffiliation from NATU, otherwise, if reanounced whatever connection they may have had with other labor
there was really deception employed in securing their signatures as claimed unions, (meaning the complainant THEU-CGW) either through resolution
by NATU/Dilag, it could not be possible to get their signatures in five or membership application forms they have unwittingly signed.’ ” (p.
different documents." (p. 144, Rollo) 306, Rollo)
We are aware of the time-honored doctrine that the findings of Granting arguendo, that the fact of retraction is true, the
the NLRC and the Secretary of Labor are binding on this Court if supported evidence on record shows that the letters of retraction were executed on
by substantial evidence. However, in the same way that the findings of facts various dates beginning January 11, 1974 to March 8, 1974 (pp. 278-
unsupported by sustantial and credible evidence do not bind this Court, 280, Rollo). This shows that the retractions were made more or less after
neither will We uphold erroneous conclusions of the NLRC and the the suspension pending dismissal on January 11, 1974 of Jose Encinas,
Secretary of Labor when We find that the latter committed grave abuse of formerly THEU-NATU President, who became THEU-CGW President,

Page 18 of 117
and the suspension pending their dismissal of the other elected officers and application for registration and the supporting documents, such as
members of the THEU-CGW on January 15, 1974. It is also clear that some the adoption or ratification of the constitution and by-laws or
of the retractions occurred after the suspension of the first set of workers amendments thereto and the minutes of ratification of the
numbering about twenty-four (24) on March 5, 1974. There is no use in constitution or by-laws, among other documents [SS Ventures v. SS
saying that the retractions obliterated the act of disaffiliation as there are
Ventures Labor Union]. 
doubts that they were freely and voluntarily done especially during such
time when their own union officers and co-workers were already suspended
In De Ocampo v. Bigkis, it was held that for fraud and
pending their dismissal. misrepresentation to constitute grounds for cancellation of union
registration under the Labor Code, the nature of the fraud and
Disaffiliation of independently-registered union and misrepresentation must be grave and compelling enough to vitiate
local chapter, distinguished the consent of a majority of union members. De Ocampo insists
The disaffiliation of an independently-registered union that "by conveniently disregarding" BMDOMMC's existence
does nto affect its legitimate status as a labor organization. during the filing of its application, despite having the same set of
However, the same thing may not be said of a union which is not officers and members,37 BMDOMSI "had misrepresented facts,
independently-registered (Local chapter). Once a local chapter made false statements and committed fraud in its application for
disaffiliates from the federation, it ceases to be entitled to the rights union registration for alleging facts therein which they [know] or
and privileges granted to a legitimate labor organization. Thus, it ought to have known to be false." BMDOMSI did not commit
can no longer file a petition for certification election [Villar v. fraud or misrepresentation in its application for registration. In the
Incion]. form "Report of Creation of Local Chapter ” filed by BMDOMSI,
the applicant indicated in the portion "Description of the
a. Effects of Disaffiliation Bargaining Unit" that it is composed of "Rank and File" and under
The following are the effects of disaffiliation: the "Occupational Classification," it marked "Technical" and
"Faculty." Further, the members appearing in the Minutes of the
(i) It terminates the right to check-off federation dues; General Membership and the List of Workers or Members who
(ii) It does not affect the CBA, it does not amend or attended the organizational meeting and adopted/ratified the
change the administration of the contract; Constitution and By-Laws are, as represented, employees of the
(iii) Once the fact of disaffiliation has been manifested school and the General Services Division, though some of the latter
beyond doubt, a certification election is the most employees service the hospital. Moreover, there is nothing in the
expeditious way of determining which labor form "Report of Creation of Local Chapter" that requires the
organization is to be treated as the exclusive
applicant to disclose the existence of another union, much less the
bargaining agent.
names of the officers of such other union. Thus, we cannot see how
BMDOMSI made the alleged misrepresentation or false statements
b. When Allowed to Disaffiliate
in its application.
A member can only disaffiliate during the 60-day
freedom period.
b. Where to File
However, the local can disaffiliate at any time even if the
Subject to the requirements of notice and due process, the
constitution and by-laws of the CBA constitutes that the local can
registration of any legitimate independent labor union, local
disaffiliate during the 60-day freedom period because the
chapter and workers’ association may be cancelled by the Regional
federation is merely an agent of the local.
Director upon the filing of a petition for cancellation for union
registration, or application by the organization itself for voluntary
6. Cancellation of Registration
dissolution.
The petition for cancellation or application for voluntary
a. Grounds for Cancellation
dissolution shall be filed in the Regional Office which issued its
certificate of registration or creation.
Article 247. [239] Grounds for Cancellation of Union
In the case of federation, national or industry unions and
Registration. - The following may constitute grounds for cancellation of
union registration: trade union centers, the Bureau Director may cancel the registration
(a) Misrepresentation, false statement or fraud in connection upon the filing of a petition for cancellation or application for
with the adoption or ratification of the constitution and by-laws or voluntary dissolution in the Bureau of Labor Relations [IRR].
amendments thereto, the minutes of ratification, and the list of
members who took part in the ratification; c. Who may File
(b) Misrepresentation, false statements or fraud in Any party-in-interest may commence a petition for
connection with the election of officers, minutes of the election of
cancellation of registration, except in actions involving violations
officers, and the list of voters;
of Article 250 which can only be commenced by members in the
(c) Voluntary dissolution by the members.
labor organization concerned [IRR].
 
While a certificate of registration confers a union with
d. Effect of a Petition for Cancellation
legitimacy with the concomitant right to participate in or ask for
certification election in a bargaining unit, the registration may be
Article 246. [238-A] Effect of a Petition for Cancellation of
canceled or the union may be decertified as the bargaining unit, in
Registration. - A petition for cancellation of union registration shall not
which case the union is divested of the status of a legitimate labor suspend the proceedings for certification election nor shall it prevent
organization. Among the grounds for cancellation is the the filing of a petition for certification election.
commission of any of the acts enumerated in Art. 247(a) of the In case of cancellation, nothing herein shall restrict the right
Labor Code, such as fraud and misrepresentation in connection of the union to seek just and equitable remedies in the appropriate
with the adoption or ratification of the union's constitution and like courts.
documents. The Court, has in previous cases, said that to decertify
a union, it is not enough to show that the union includes ineligible Cancellation of registration does not affect the litigation.
employees in its membership. It must also be shown that there was The litigation continues because the union is a mere representative
misrepresentation, false statement, or fraud in connection with the
Page 19 of 117
party. The real parties in interest are the individual employees Although the union has every right to represent its
concerned. members in the negotiation regarding the terms and conditions of
The labor organization may still be a party to the case but their employment, it cannot negate their wishes on matters which
the decision is binding only to those who did not withdraw from are purely personal and individual to them. In this case, the forty
the case before its trial and decision on the merits. employees freely opted to be covered by the Old Plan; their
If some members in the meantime execute a quit-claim or decision should be respected. The company gave them every
waiver because they have been paid in advance, such members are opportunity to choose, and they voluntarily exercised their choice.
no longer covered by the decision. The union cannot pretend to know better; it cannot impose its will
The non-renewal of registration permit does not dismiss on them [Caltex Refinery Employees Association v. Brillantes].
the cause because the court already acquired jurisdiction thereof.
b. Right to be Certified as Exclusive
e. Voluntary Cancellation Bargaining Agent
In order to represent its members in collective
Article 248. [239-A] Voluntary Cancellation of Registration. - bargaining, it is an LLO’s inherent right that it be allowed to file a
The registration of a legitimate labor organization may be cancelled by Petition for Certification Election.
the organization itself: Provided, That at least two-thirds of its general
membership votes, in a meeting duly called for that purpose to dissolve c. Right to Request for Audited Financial
the organization: Provided, further, That an application to cancel
Statement
registration is thereafter submitted by the board of the organization,
attested to by the president thereof.
In order to request for the Employer’s Audited Financial
Statement, the LLO must generally be an Exclusive Bargaining
Agent. This right includes the right to the notes and other
7. Rights of Legitimate Labor Organizations
documents in support thereof.
To better equip the union in preparing for or in
Article 251. [242] Rights of Legitimate Labor Organizations. -
negotiating with the employer, the law gives it the right to be
A legitimate labor organization shall have the right:
(a) To act as the representative of its members for the furnished with the employer's audited financial statements. There
purpose of collective bargaining; are four points in time when the union may ask in writing for these
(b) To be certified as the exclusive representative of all the statements:
employees in an appropriate bargaining unit for purposes of collective
bargaining; (i) after the union has been recognized by the
(c) To be furnished by the employer, upon written request, employer as sole bargaining representative of
with its annual audited financial statements, including the balance the employees in the bargaining unit; or
sheet and the profit and loss statement, within thirty (30) calendar days
(ii) after the union is certified by DOLE as such
from the date of receipt of the request, after the union has been duly
recognized by the employer or certified as the sole and exclusive sole bargaining representative; or
bargaining representative of the employees in the bargaining unit, or (iii) within the last 60 days of the life of a CBA; or
within sixty (60) calendar days before the expiration of the existing (iv) during the collective bargaining negotiation.
collective bargaining agreement, or during the collective bargaining
negotiation; The audited financial statements, including the balance
(d) To own property, real or personal, for the use and sheet and the profit and loss statement, should be provided by the
benefit of the labor organization and its members;
employer within 30 calendar days after receipt of the union's
(e) To sue and be sued in its registered name; and
request.
(f) To undertake all other activities designed to benefit the
organization and its members, including cooperative, housing, welfare
and other projects not contrary to law. Standard Chartered Bank Employees Union v. Confesor
Notwithstanding any provision of a general or special law to Held: We, likewise, find that the Union failed to substantiate its
the contrary, the income and the properties of legitimate labor claim that the Bank refused to furnish the information it needed.
organizations, including grants, endowments, gifts, donations and While the refusal to furnish requested information is in itself an
contributions they may receive from fraternal and similar unfair labor practice, and also supports the inference of surface
organizations, local or foreign, which are actually, directly and bargaining, in the case at bar, Umali, in a meeting dated May 18, 1993,
exclusively used for their lawful purposes, shall be free from taxes, requested the Bank to validate its guestimates on the data of the rank and
duties and other assessments. The exemptions provided herein may be file. However, Umali failed to put his request in writing as provided for in
withdrawn only by a special law expressly repealing this provision. Article 242(c) of the Labor Code:
Article 242. Rights of Legitimate Labor Organization…
(c) To be furnished by the employer, upon written request, with
The first three rights mentioned in this article do not
the annual audited financial statements, including the balance sheet and the
pertain to just about any union but only to the union that has been profit and loss statement, within thirty (30) calendar days from the date of
selected as the bargaining representative of the employees in the receipt of the request, after the union has been duly recognized by the
bargaining unit. employer or certified as the sole and exclusive bargaining representatives of
the employees in the bargaining unit, or within sixty (60) calendar days
a. Right to Act as Representative before the expiration of the existing collective bargaining agreement, or
In order to act as a representative for purposes of during the collective negotiation;
collective bargaining, the LLO must generally be the exclusive The Union, did not, as the Labor Code requires, send a written
request for the issuance of a copy of the data about the Bank’s rank and file
bargaining agent.
employees. Moreover, as alleged by the Union, the fact that the Bank made
The union whose demand for collective bargaining was use of the aforesaid guestimates, amounts to a validation of the data it had
rebuffed by the employer, because the union was not the certified used in its presentation.
bargaining agent, has no right to stage a strike. The strike is illegal.
Such illegality is reason enough for the NLRC to declare that the d. Right to Own Property
union officers have lost their employment status [Philippine As a consequence of having a separate personality, LLOs
Diamond Hotel v. Manila Diamond Hotel Employees Union]. can acquire personal and real property under its name.

Page 20 of 117
motion for intervention, will have to be denied upon a finding that
e. Right to Sue and be Sued in its Registered those members are already well represented by their union. The
Name intervention may be allowed, however, when there is a suggestion
It is the function of a labor union to represent its of fraud or collusion or that the representative will not act in good
members against the employer's unfair labor practices. It can file an faith for the protection of all interests represented by [the union]. In
action in their behalf without the cumbersome procedure of joining this case the members who desire to intervene have not shown
each and every member as a separate party [Davao Free Workers fraud, collusion, or lack of good faith on the part of their union
v. CIR]. [Acedera v. International Container Terminal Services].
The union and its attorney should be allowed to
participate in making compromise settlements with employees who Negotiation Phase vs. Administration Phase
are union members. In one case, the company was adjudged to While in the negotiation phase, the union is the EBA for
have acted with evident bad faith and malice when it secured the 53 the purpose of collective bargaining. Thus, the employee can
quitclaim agreements individually with the complainant workers compel representation.
without the intervention of court and without involving the union. However, during the administration proceedings (i.e.,
This subterfuge is tantamount to a sabotage of the interest of the filing complaints arising from the CBA), the union is no longer the
association. Needless to say, the means employed by the employer EBA because any group of employees or a single employee can
in dealing with the workers individually, instead of collectively bring a grievance to management. Hence, the employee cannot
through the union and its counsel, violates good morals as they compel representation. He can do so by himself.
undermine the unity of the union and fuels industrial disputes,
contrary to the declared policy in the Industrial Peace Act f. Right to Undertake all other Activities not
[Pampanga Sugar v. CIR]. Contrary to Law
This is the right to undertake all other activities designed
LAKAS v. Marcelo Enterprises to benefit the organization and its members, including cooperative,
Held: In NARIC Workers' Union vs. CIR, We ruled that, "(a) housing, welfare and other projects not contrary to law.
labor union would go beyond the limits of its legitimate purposes if it is
given the unrestrained liberty to prosecute any case even for employees g. Right to Tax Exemption
who are not members of any union at all. A suit brought by another in Notwithstanding any provision of a general or special
representation of a real party in interest is defective." Under the
law to the contrary, the income and the properties of legitimate
uncontroverted facts obtaining herein, the aforestated ruling is applicable,
the only difference being that, here, a labor federation seeks to represent
labor organizations, including grants, endowments, gifts, donations
members of a registered local union never affiliated with it and members of and contributions they may receive from fraternal and similar
registered local unions which, in the course of the proceedings before the organizations, local or foreign, which are actually, directly and
industrial court, disaffiliated from it. exclusively used for their lawful purposes, shall be free from
This is not to say that the complaining employees were without taxes, duties and other assessments. The exemptions provided
any venue for redress. Under the aforestated considerations, the respondent herein may be withdrawn only by a special law expressly repealing
court should have directed the amendment of the complaint by dropping this provision.
LAKAS as the complainant and allowing the suit to be further prosecuted
in the individual names of those who had grievances. A class suit under
Rule 3, Section 12 of the Rules of Court is authorized and should suffice for
8. Rights and Conditions of Membership
the purpose.
Article 250. [241] Rights and Conditions of Membership in a
Labor Organization. - The following are the rights and conditions of
Money Claims
membership in a labor organization:
The rule in this jurisdiction is that money claims due to (a) No arbitrary or excessive initiation fees shall be required
laborers cannot be the object of settlement or compromise effected of the members of a legitimate labor organization nor shall arbitrary,
by the union, union officers or counsel without the specific excessive or oppressive fine and forfeiture be imposed;
individual consent of each laborer concerned. This is so because (b) The members shall be entitled to full and detailed reports
the aggrieved parties are the individual complainants themselves. from their officers and representatives of all financial transactions as
Their representative can only assist but not decide for them. In the provided for in the constitution and by-laws of the organization;
light of the categorical denial by the employees that Peran was (c) The members shall directly elect their officers in the local
union, as well as their national officers in the national union or
authorized to enter into an amicable settlement as regards their
federation to which they or their local union is affiliated, by secret
claims, the Court holds that public respondent Secretary of ballot at intervals of five (5) years. No qualification requirement for
labor ruled correctly in upholding the Regional Director's rejection candidacy to any position shall be imposed other than membership in
of the agreement [Marquez v. Secretary of Labor]. good standing in subject labor organization. The secretary or any other
For a waiver thereof to be legally effective, the individual responsible union officer shall furnish the Secretary of Labor and
consent or ratification of the workers or employees involved must Employment with a list of the newly-elected officers, together with the
be shown.  Neither the officers nor the majority of the union had appointive officers or agents who are entrusted with the handling of
any authority to waive the accrued rights pertaining to the funds within thirty (30) calendar days after the election of officers or
from the occurrence of any change in the list of officers of the labor
dissenting minority members, even under a collective bargaining
organization;
agreement which provided for a "union shop." [General Rubber v. (d) The members shall determine by secret ballot, after due
Drilon] deliberation, any question of major policy affecting the entire
The authority of a union under Article 251 to act as membership of the organization, unless the nature of the organization
representative of its members for the purposes of collective or force majeure renders such secret ballot impractical, in which case,
bargaining includes the power to represent its members for the the board of directors of the organization may make the decision in
purpose of enforcing the provisions of the CBA. When a union behalf of the general membership;
files a case "for and in behalf of its members," a member or several (e) No labor organization shall knowingly admit as members
or continue in membership any individual who belongs to a subversive
members of that union will not be permitted to file in the same case
organization or who is engaged directly or indirectly in any subversive
a complaint-in-intervention even if it alleges that the union was not activity;
pursuing the case diligently. Such complaint, together with the
Page 21 of 117
(f) No person who has been convicted of a crime involving authorization should specifically state the amount, purpose and
moral turpitude shall be eligible for election as a union officer or for beneficiary of the deduction; and
appointment to any position in the union; (p) It shall be the duty of any labor organization and its
(g) No officer, agent or member of a labor organization shall officers to inform its members on the provisions of its constitution and
collect any fees, dues, or other contributions in its behalf or make any by-laws, collective bargaining agreement, the prevailing labor relations
disbursement of its money or funds unless he is duly authorized system and all their rights and obligations under existing labor laws.
pursuant to its constitution and by-laws; For this purpose, registered labor organizations may assess
(h) Every payment of fees, dues or other contributions by a reasonable dues to finance labor relations seminars and other labor
member shall be evidenced by a receipt signed by the officer or agent education activities.
making the collection and entered into the record of the organization to Any violation of the above rights and conditions of
be kept and maintained for the purpose; membership shall be a ground for cancellation of union registration or
(i) The funds of the organization shall not be applied for any expulsion of officers from office, whichever is appropriate. At least
purpose or object other than those expressly provided by its thirty percent (30%) of the members of a union or any member or
constitution and by-laws or those expressly authorized by written members specially concerned may report such violation to the Bureau.
resolution adopted by the majority of the members at a general The Bureau shall have the power to hear and decide any reported
meeting duly called for the purpose; violation to mete the appropriate penalty.
(j) Every income or revenue of the organization shall be Criminal and civil liabilities arising from violations of above
evidenced by a record showing its source, and every expenditure of its rights and conditions of membership shall continue to be under the
funds shall be evidenced by a receipt from the person to whom the jurisdiction of ordinary courts.
payment is made, which shall state the date, place and purpose of such
payment. Such record or receipt shall form part of the financial a. Rights of Membership
records of the organization.
Any action involving the funds of the organization shall
prescribe after three (3) years from the date of submission of the (i) Right against arbitrary or excessive
annual financial report to the Department of Labor and Employment initiation fees, fines, and forfeiture
or from the date the same should have been submitted as required by No arbitrary or excessive initiation fees shall be required
law, whichever comes earlier: Provided, That this provision shall apply of the members of a legitimate labor organization nor shall
only to a legitimate labor organization which has submitted the arbitrary, excessive or oppressive fine and forfeiture be imposed.
financial report requirements under this Code: Provided, further, That Article 292 (a) provides:
failure of any labor organization to comply with the periodic financial
reports required by law and such rules and regulations promulgated
(a) All unions are authorized to collect reasonable
thereunder six (6) months after the effectivity of this Act shall
membership fees, union dues, assessments and fines and other
automatically result in the cancellation of union registration of such
contributions for labor education and research, mutual death and
labor organization;
hospitalization benefits, welfare fund, strike fund and credit and
(k) The officers of any labor organization shall not be paid
cooperative undertakings
any compensation other than the salaries and expenses due to their
positions as specifically provided for in its constitution and by-laws, or
in a written resolution duly authorized by a majority of all the (ii) Right to full and detailed reports
members at a general membership meeting duly called for the purpose. The members shall be entitled to full and detailed reports
The minutes of the meeting and the list of participants and ballots cast from their officers and representatives of all financial transactions
shall be subject to inspection by the Secretary of Labor or his duly as provided for in the constitution and by-laws of the organization
authorized representatives. Any irregularities in the approval of the such as financial statements, balanced sheet, and income
resolutions shall be a ground for impeachment or expulsion from the statements.
organization;
(l) The treasurer of any labor organization and every officer
(iii) Right to election of officers by secret
thereof who is responsible for the account of such organization or for
the collection, management, disbursement, custody or control of the ballot
funds, moneys and other properties of the organization, shall render to The members shall directly elect their officers in the
the organization and to its members a true and correct account of all local union, as well as their national officers in the national union
moneys received and paid by him since he assumed office or since the or federation to which they or their local union is affiliated at
last day on which he rendered such account, and of all bonds, securities intervals of five (5) years.
and other properties of the organization entrusted to his custody or No qualification requirement for candidacy to any
under his control. The rendering of such account shall be made: position shall be imposed other than membership in good standing
(1) At least once a year within thirty (30) days after the close
in subject labor organization.
of its fiscal year;
(2) At such other times as may be required by a resolution of The secretary or any other responsible union officer shall
the majority of the members of the organization; and furnish the Secretary of Labor and Employment with a list of the
(3) Upon vacating his office. newly-elected officers, together with the appointive officers or
The account shall be duly audited and verified by affidavit agents who are entrusted with the handling of funds within thirty
and a copy thereof shall be furnished the Secretary of Labor. (30) calendar days after the election officers or form the occurrence
(m) The books of accounts and other records of the financial of any change in the list of officers of the labor organization.
activities of any labor organization shall be open to inspection by any
Only members of the union can participate in the election
officer or member thereof during office hours;
of union officers. The question of eligibility to vote may be
(n) No special assessment or other extraordinary fees may be
levied upon the members of a labor organization unless authorized by a determined through the use of the applicable payroll period and
written resolution of a majority of all the members in a general employee's status during the applicable payroll period. The payroll
membership meeting duly called for the purpose. The secretary of the of the month next preceding the labor dispute in case of regular
organization shall record the minutes of the meeting including the list employees and the payroll period at or near the peak of operations
of all members present, the votes cast, the purpose of the special in case of employees in seasonal industries [Tancinco v. Ferrer-
assessment or fees and the recipient of such assessment or fees. The Calleja].
record shall be attested to by the president.
(o) Other than for mandatory activities under the Code, no
(iv) Right to vote in determination of major
special assessments, from any amount due to an employee without an
individual written authorization duly signed by the employee. The policies by secret ballot

Page 22 of 117
The members shall determine by secr3et ballot, after due employee without an individual written authorization duly signed
deliberation, any question of major policy affecting the entire by the employee. The authorization should specifically state the
membership of the organization, unless the nature of the amount, purpose and beneficiary of the deduction.
organization or force majeure renders such secret ballot It is very clear from the above-quoted provision that
impractical, in which case, the Board of Directors of the attorney's fees may not be deducted or checked off from any
organization may make the decision on behalf of the general amount due to an employee without his written consent except for
membership. mandatory activities under the Code.  A mandatory activity has
been defined as a judicial process of settling dispute laid down by
(v) Right to a receipt of any payment to the the law.  In the instant case, the amicable settlement entered into by
union the management and the union cannot be considered as a
Every payment of fees, dues, or other contributions by a mandatory activity under the Code.  It is true that the union filed a
member shall be evidenced by a receipt signed by the officer or claim for emergency cost of living allowance and other benefits
agent making the collection and entered into the record of the before the Ministry of Labor.  But this case never reached its
organization to be kept and maintained for that purpose. conclusion in view of the parties’ agreement.  It is not also shown
from the records that Atty. Benjamin Sebastian was instrumental in
(vi) Right against unauthorized forging the said agreement on behalf of the union members
expenditures [Vengco v. Trajano].
The funds of the organization shall not be applied for any For special assessment, attorney’s fees, negotiation fees,
purpose or object other than (a) those expressly provided by its and other extraordinary fees, automatic check-offs require (1)
constitution and by-laws or (b) those expressly authorized by written resolution of the majority of all the members at a general
written resolution adopted by the majority of the members at a membership meeting called for the purpose; and (2) individual
general meeting duly called for the purpose. written authorization for check-off duly signed by the employee
For this purpose, registered labor organizations may concerned.
assess reasonable dues to finance labor relations seminars and other Article 228 provides:
labor education activities.
Article 228. [222] Appearances and Fees. - (a) Non-
(vii) Right to report from the treasurer or lawyers may appear before the Commission or any Labor
other responsible officer to render an Arbiter only:
1. If they represent themselves; or
account
2. If they represent their organization or members
The treasurer of any labor organization and every officer thereof.
thereof who is responsible for the account of such organization or (b) No attorney’s fees, negotiation fees or similar
for the collection, management, disbursement, custody or control charges of any kind arising from any collective bargaining
of the funds, monies, and other properties of the organization, shall agreement shall be imposed on any individual charged against
render to the organization and to its members a true and correct union funds in an amount to be agreed upon by the parties. Any
account of all moneys received and paid by him since he assumed contract, agreement or arrangement of any sort to the contrary
office or since the last day on which he rendered such account, and shall be null and void.
of all bonds, securities and other properties of the organization
entrusted to his custody or under his control. The rendering of such Article 222(b) of the Labor Code, as amended, prohibits
account shall be made: the payment of attorney's fees only when it is effected through
(1) At least once a year within thirty (30) days after the forced contributions from the employees from their own funds as
close of its fiscal year; distinguished from union funds. Hence, the general rule is that
(2) At such other times as may be required by a attorney's fees, negotiation fees, and other similar charges may
resolution of the majority of the members of the organization; and only be collected from union funds, not from the amounts that
(3) Upon vacating his office. pertain to individual union members. As an exception to the
The account shall be duly audited and verified by general rule, special assessments or other extraordinary fees may
affidavit and a copy thereof shall be furnished the Secretary of be levied upon or checked off from any amount due an employee
Labor. for as long as there is proper authorization by the employee
[Marino v. Gamilla].
(viii) Right to inspect financial records
during office hours When individual authorization not required
The books of accounts and other records of the financial i. Assessment from non-members of the
activities of any labor organization shall be open to inspection by bargaining agent of “Agency Fees;”
any officer or member thereof during office hours. ii. Deductions for fees for mandatory activities;
iii. Check-off for union service fees authorized by
(ix) Right against unauthorized assessment law;
or fees iv. Deductions for withholding tax (NIRC);
No special assessment or other extraordinary fees may be v. Deductions for withholding of wages because
levied upon the members of a labor organization unless authorized of employee’s deb to the employer which is
by a written resolution of a majority of all the members in a general already due;
membership meeting duly called for the purpose. vi. Deductions made pursuant to a judgment
against the worker under circumstances where
(x) Right against unauthorized deductions the wages may be subject to attachment or
from salary execution but only for debts incurred for food,
Other than mandatory activities under the Code, no clothing, shelter, and medical attendance;
special assessments, attorney’s fees, negotiation fees or any other vii. Deductions from wages ordered by the court;
extraordinary fees may be checked off from any amount due to an

Page 23 of 117
viii. Deductions authorized by law such as the deduction; but the second kind does not require any such
premiums for PhilHealth, SSS, Pag-IBIG, ECC authorization since the law itself recognizes and allows it upon the
and the like. non-SEBA member’s acceptance of benefits resulting from the
CBA [Holy Cross v. Joaquin].
Agency Fee To effect the check-off of agency fees, no individual
Article 259(e) provides: authorization is necessary unlike members under Article 250(o).
In ABS-CBN Supervisors-Employees Union Members v.
(e) x x x. Employees of an appropriate bargaining ABS-CBN Broadcasting Corporation, the Supreme Court declared
unit who are not members of the recognized collective that Article 250 of the Labor Code, as amended, speaks of three (3)
bargaining agent may be assessed a reasonable fee equivalent to requisites, to wit: (1) authorization by a written resolution of the
the dues and other fees paid by members of the recognized
majority of all members at the general membership meeting called
collective bargaining agent, if such non-union members accept
the benefits under the collective bargaining agreement: for the purpose; (2) secretary's record of the minutes of the
Provided, That the individual authorization required under meeting; and (3) individual written authorization for check-off duly
Article 250, paragraph (o) of this Code shall not apply to the signed by the employee concerned.
non-members of the recognized collective bargaining agent;
(xi) Right to information
Agency Fee means a fee deducted by an employer from It shall be the duty of any labor organization and its
the salary or wages of an employee who is not a member of an officers to inform its members on the provisions of its constitution
employee organization, which is paid to the employee organization and by-laws, collective bargaining agreement, the prevailing labor
that is the exclusive bargaining agent for the bargaining unit of the relations system and all their rights and obligations under existing
employee [IRR]. labor laws.
This fee is collected when the bargaining agent
successfully negotiates a CBA with the employer. It is imposed on b. Conditions of Membership
non-members who are employees covered by the bargaining unit
being represented by the bargaining agent – in case they accept the (i) Individuals belonging to subversive
benefits under the CBA. organization or engaged in subversive
The fact that the non-member is also paying union dues activity
to their own unions does not free them from their obligations to pay No labor organization shall knowingly admit as members
the agency fee, and vice versa. Thus they are required to pay (a) or continue in membership any individual who belongs to a
union dues and special assessments of their own union; and (b) subversive organization or who is engaged directly or indirectly in
agency fee to the bargaining agent. any subversive activity.
It is neither contractual nor statutory but quasi-
contractual. Payment of agency fee to the bargaining union/agent (ii) Persons convicted of a crime involving
which negotiated the CBA is but a reasonable requirement moral turpitude
recognized by law, to prevent non-union members from enriching No person who has been convicted of a crime involving
themselves at the expense of union members [Holy Cross of Davao moral turpitude shall be eligible for election as a union officer or
v. Joaquin]. The non-member does not become a member when he for appointment to any position in the union.
pays the agency fee.
(iii) Collection or disbursement of funds
Check-Off No officer, agent or member of a labor organization shall
Check-off is a method of deducting by the employer collect any fees, dues, or other contributions in its behalf or make
from the employee’s pay at prescribed periods, any amount due for any disbursement of its money or funds unless he is duly
fees, fines or assessments [AL Ammen Transportation v. Bicol authorized pursuant to its constitution and by-laws.
Transportation Employees]. Strictly speaking, it is a process or
device whereby the employer, on agreement with the union (iv) Action involving funds
certified as the SEBA, or on prior authorization from its Any action involving the funds of the organization shall
employees, deducts union dues or agency fees from the latter’s prescribe after three (3) years from the date of submission of the
wages and remits them directly to the union [Gabriel v. Secretary annual financial report to the Department of Labor and
of Labor]. Employment or from the date the same should have been submitted
This only applies to the SEBA and not to the minority as required by law, whichever comes earlier: Provided, That this
union. provision shall apply only to a legitimate labor organization which
Check-off may refer to two (2) things: has submitted the financial report requirements under this Code:
Provided, further, That failure of any labor organization to comply
i. Collection of union dues, special assessments with the periodic financial reports required by law and such rules
and fees (such as attorney’s fees, negotiation and regulations promulgated thereunder six (6) months after the
fees or any other extraordinary fees by the effectivity of this Act shall automatically result in the cancellation
SEBA from its members; and of union registration of such labor organization;
ii. Collection of agency fees from non-members
of the SEBA but covered by and included in Who can file a complaint
the collective bargaining unit (CBU) who If it is a general complaint, the complaint must be under
accept the benefits provided in the Collective oath with written consent of at least 20% of the total membership
Bargaining Agreement (CBA). of the labor organization, or it may be exercised by the Secretary of
DOLE motu proprio.
The first kind mentioned requires for its validity, the The Regional or Bureau Director may inquire into the
execution by the employees of individual written authorization financial activities of any legitimate labor organization and
which should specifically state the amount, purpose, beneficiary of examine their books of accounts and other records to determine

Page 24 of 117
compliance with the law and the organization’s constitution and
by-laws. Such examination shall be made upon the filing of a (a) Request for SEBA certification (which repealed and
request or complaint for the conduct of an accounts examination by replaced “Voluntary Recognition” as a mode of
any member of the labor organization, supported by the written securing SEBA status);
consent of at least 20% of its total membership [IRR]. (b) Certification election;
If it is a specific complaint and it involves the member (c) Consent election;
only, there is no need for the written consent of 20% of the (d) Run-off election.
members. If it involves the entire membership, it must be supported
by the written consent of 30% of the total membership. 1. SEBA Certification
Any complaint or petition with allegations of Before, it was voluntary recognition, which refers to the
mishandling, misappropriation, or non-accounting of funds in free and voluntary act of the employer of extending and conferring
violation of Article 250 shall be treated as an intra-union dispute. It full recognition to a union as the sole and exclusive bargaining
shall be heard and resolved by the Mediator-Arbiter pursuant to the representative of the employees in an appropriate bargaining unit,
provisions or Rule XI [IRR]. for purposes of collective bargaining.
Where the issue involves the entire membership of the Department Order No. 40-I-15, Series of 2015 has
labor organization, the complaint or petition shall be supported by expressly repealed the entire set of Rules applicable to Voluntary
at least 30% of its members [IRR]. Recognition in the Labor Code’s Implementing Rules and replaced
it with the freshly-minted mode of securing the status of a SEBA
(v) Compensation of officers through a Request for SEBA Certification or simply “Request.”
Any action involving the funds of the organization shall Any legitimate labor organization may file a Request in
prescribe after three (3) years from the date of submission of the the DOLE Regional Office which issued to it its Certificate of
annual financial report to the Department of Labor and Registration or Certificate of Creation of Chartered Local, as the
Employment or from the date the same should have been submitted case may be [IRR].
as required by law, whichever comes earlier: Provided, That this
provision shall apply only to a legitimate labor organization which 2. Consent Election
has submitted the financial report requirements under this Code: Consent election refers to the process, voluntarily and
Provided, further, That failure of any labor organization to comply mutually agreed upon by the contending unions, of determining
with the periodic financial reports required by law and such rules through secret ballot the SEBA of the employees in an appropriate
and regulations promulgated thereunder six (6) months after the CBU for purposes of collective bargaining with the employer. It is
effectivity of this Act shall automatically result in the cancellation conducted with or without the intervention of the DOLE [IRR].
of union registration of such labor organization;
Consent Election vs. Certification Election
c. Effect of Violation of Rights and Conditions Consent election is but a form of certification election.
of Membership They may be distinguished from each other in the following
Any violation of the above rights and conditions of manner:
membership shall be a ground for cancellation of union registration
or expulsion of officers from office, whichever is appropriate. At a. The former is held upon the mutual agreement of
least thirty percent (30%) of the members of a union or any the contending unions; while the latter does not
member or members specially concerned may report such violation require the mutual consent of the parties as it is
to the Bureau. The Bureau shall have the power to hear and decide conducted upon the order of the Med-Arbiter [IRR].
any reported violation to mete the appropriate penalty. Criminal b. The former may be conducted with or without the
and civil liabilities arising from violations of above rights and control and supervision of the DOLE; while the
conditions of membership shall continue to be under the latter is always conducted under the control and
jurisdiction of ordinary courts. supervision of the DOLE [IRR].
c. The former is being conducted as a voluntary mode
D. DETERMINATION OF REPRESENTATION STATUS of resolving labor dispute; while the latter, although
The term sole and exclusive bargaining agent (SEBA) non-adversarial, is a compulsory method of
refers to a legitimate labor union duly certified as the sole and adjudicating a labor dispute.
exclusive bargaining representative or agent of all the employees in d. The former is given the highest priority; while the
a collective bargaining unit (CBU). A labor union certified as a latter is resorted only when the contending unions
SEBA means that it shall remain as such during the existence of the fail or refuse to submit their representation dispute
CBA, to the exclusion of all other labor organizations existing and through the former [IRR]. This is so because under
operating in the same CBU, and no petition for certification the Implementing Rules, as amended, even in cases
election (PCE) questioning its majority status shall be entertained where a PCE is filed, the Med-Arbiter, during the
nor shall certification election be conducted outside the 60-day preliminary conference and hearing thereon, is
freedom period immediately before the expiry date of the 5-year tasked to determine the possibility of a consent
term of the CBA. election. It is only when the contending unions fail
Once certified, what is represented by the SEBA are not to agree to the conduct of a consent election during
only its members but also those who are members of other unions, the preliminary conference that the Med-Arbiter will
called “minority” unions, who are included in the CBU [National proceed with the process of certification election by
Brewery & Allied Industries Labor Union v. San Miguel Brewery]. conducting as many hearings as he may deem
An unorganized establishment becomes an “Organized necessary up to its actual holding. But in no case
Establishment” where there exists a recognized or certified sole shall the conduct of the certification election exceed
and exclusive bargaining agent. 15 days from the date of the scheduled preliminary
The SEBA of the employees in a CBU may be conference/hearing after which time, the PCE is
determined through any of the following modes: considered submitted for decision.

Page 25 of 117
e. The former necessarily involves at least 2 or more PCE should be resolved in favor of the holding of a certification
contending unions; while the latter may only election [National Federation of Labor v. Secretary of Labor].
involve 1 petitioner union. The facts sought to be determined in a certification
f. The former may be conducted in the course of the election are: first, whether the Bargaining Unit wants
proceeding in the latter or during its pendency. representation; and second, who will represent them if they want
g. The purpose of the former is only to determine (i) representation.
who has the majority representation of all the A certification election proceeding directly involves two
workers in the appropriate collective bargaining (2) issues, namely: (a) the proper composition and constituency of
unit; where as the purpose of the latter is to the bargaining unit; and (b) the validity of majority representation
determine (i) whether the bargaining unit wants claims. It is therefore incumbent upon the Med-Arbiter to rule on
representation; and (ii) who will represent them if the appropriateness of the bargaining unit once its composition and
they want representation. constituency is questioned [Holy Child v. Sto. Tomas].

Warren v. BLR b. Where to File


Held: The records show that petitioner admitted that what was A petition for certification election shall be filed with the
held on August 25, 1985 at the Company’s premises and which became the Regional Office which has jurisdiction over the principal office of
root of this controversy, was a consent election and not a certification the petitioner. The petition shall be in writing and under oath
election (underscoring supplied). The election held on August 25, 1985 was [IRR].
not for the purpose of determining which labor union should be the
Under the omnibus rules implementing the Labor Code
bargaining representative in the negotiation for a collective contract, there
being an existing collective bargaining agreement yet to expire on July 31,
as amended by D.O. No. 9, it is supposed to be filed in the
1986; but only to determine which labor union shall administer the said Regional Office which has jurisdiction over the principal office of
existing contract. the employer or where the bargaining unit is principally
 As correctly distinguished by private respondent, a consent situated. The rules further provide that where two or more petitions
election is an agreed one, its purpose being merely to determine the issue of involving the same bargaining unit are filed in one Regional
majority representation of all the workers in the appropriate collective Office, the same shall be automatically consolidated. Hence, the
bargaining unit while a certification election is aimed at determining the filing of multiple suits and the possibility of conflicting decisions
sole and exclusive bargaining agent of all the employees in an appropriate
will rarely happen in this proceeding and, if it does, will be easy to
bargaining unit for the purpose of collective bargaining. From the very
nature of consent election, it is a separate and distinct process and has
discover. Notably, under the Labor Code and the rules pertaining to
nothing to do with the import and effect of a certification election. Neither the form of the petition for certification election, there is no
does it shorten the terms of an existing CBA nor entitle the participants requirement for a certificate of non-forum shopping either in D.O.
thereof to immediately renegotiate an existing CBA although it does not No. 9, series of 1997 or in D.O. No. 40-03, series of 2003 which
preclude the workers from exercising their right to choose their sole and replaced the former. Considering the nature of a petition for
exclusive bargaining representative after the expiration of the sixty (60) day certification election and the rules governing it, we therefore hold
freedom period. that the requirement for a certificate of non-forum shopping is
inapplicable to such a petition [SAMMA-LIKHA v. SAMMA].
3. Certification Election
Certification election refers to the process of determining c. Who may File
through secret ballot the SEBA of the employees in an appropriate
CBU for purposes of collective bargaining with the employer. A (i) Organized Establishments
certification election is conducted only upon the order of the Med-
Arbiter of the BLR [IRR]. Article 268. [256] Representation Issue in Organized
Establishments. In organized establishments, when a verified petition
Pendency of a petition for cancellation questioning the majority status of the incumbent bargaining agent is
The pendency of a petition for cancellation of union filed by any legitimate labor organization including a national union or
registration does not preclude certification election or collective federation which has already issued a charter certificate to its local
bargaining. chapter participating in the certification election or a local chapter
which has been issued a charter certificate by the national union or
federation before the Department of Labor and Employment within
a. Nature of Certification Election the sixty (60)-day period before the expiration of the collective
Certification election is the most democratic method of bargaining agreement, the Med-Arbiter shall automatically order an
determining the choice of the employees of their bargaining election by secret ballot when the verified petition is supported by the
representative [PAL Employees’ Association v. Ferrer-Calleja]. It written consent of at least twenty-five percent (25%) of all the
is held to ensure that the employees are properly represented in the employees in the bargaining unit to ascertain the will of the employees
exercise of their right to collective bargaining with their employer in the appropriate bargaining unit. To have a valid election, at least a
[FOITAF v. Noriel]. No obstacle must be placed to the holding of a majority of all eligible voters in the unit must have cast their votes. The
labor union receiving the majority of the valid votes cast shall be
certification election for it is the statutory policy that it should not
certified as the exclusive bargaining agent of all the workers in the unit.
be circumvented [Trade Unions of the Philippines v. Laguesma]. It When an election which provides for three or more choices results in
is not a litigation proceeding in a sense in which this term is no choice receiving a majority of the valid votes cast, a run-off election
commonly understood. It is a mere investigation of a non-adversary shall be conducted between the labor unions receiving the two highest
fact-finding character in which the DOLE plays the part of a number of votes: Provided, That the total number of votes for all
disinterested investigator seeking merely to ascertain the desires of contending unions is at least fifty percent (50%) of the number of votes
the employees as to the matter of determining which labor cast. In cases where the petition was filed by a national union or
organization will represent the employees in their collective federation, it shall not be required to disclose the names of the local
chapter’s officers and members.
bargaining with the employer [The Heritage Hotel v. Secretary of
At the expiration of the freedom period, the employer shall
Labor]. It is not therefore bound by the technical rules of evidence continue to recognize the majority status of the incumbent bargaining
[Associated Labor Unions v. Ferrer-Calleja]. In case of doubt, the agent where no petition for certification election is filed.

Page 26 of 117
(ii) Unorganized Establishments However, no certification lection may be held under the
following rules:
Article 269. [257] Petitions in Unorganized Establishments. In
any establishment where there is no certified bargaining agent, a (a) Certification year-bar rule;
certification election shall automatically be conducted by the Med- (b) Negotiations-bar rule;
Arbiter upon the filing of a petition by any legitimate labor (c) Bargaining deadlock-bar rule; or
organization, including a national union or federation which has
(d) Contract-bar rule.
already issued a charter certificate to its local/chapter participating in
the certification election or a local/chapter which has been issued a
charter certificate by the national union or federation. In cases where (i) Certification Year Bar Rule
the petition was filed by a national union or federation, it shall not be Although the Labor Code does not contain any provision
required to disclose the names of the local chapter’s officers and on when the CBA negotiation process should start after a union is
members. duly certified as the SEBA of the employees it seeks to represent in
a given bargaining unit, there is, however, this provision in the
 When a duly organized union files a petition for Rules when the Med-Arbiter may dismiss the PCE if the same is
certification election, the Med-Arbiter has the duty to automatically filed within one (1) year reckoned and counted:
conduct an election. He has no discretion on the matter [Philippine
Scout Veterans v. Secretary of Labor]. (i) From the date the SEBA certification is issued
by the DOLE Regional Director in cases of
(iii) When an Employer May File Request for SEBA Certification filed in an
unorganized establishment with only one (1)
Article 270. [258] When an Employer May File Petition. legitimate labor organization; or
When requested to bargain collectively, an employer may petition the (ii) From the date of issuance of certification as
Bureau for an election. If there is no existing certified collective SEBA and not from the date of the conduct of
bargaining agreement in the unit, the Bureau shall, after hearing, valid certification, consent, run-off or re-run
order a certification election. election [IRR].
All certification cases shall be decided within twenty (20)
working days.
The Bureau shall conduct a certification election within In the first instance, the period is counted from the
twenty (20) days in accordance with the rules and regulations issuance of the SEBA certification which shall have the effect of
prescribed by the Secretary of Labor . barring the filing of a PCE by any labor organization. It is only
upon the expiration of this 1-year period that any legitimate labor
A petition for certification election may be filed by: organization may file a PCE in the same collective bargaining unit
(CBU) represented by the certified SEBA, unless a CBA between
a. A legitimate labor organization, including: (i) a the employer and the certified SEBA was executed and registered
national union or federation that has issued a charter with the DOLE Regional Office [IRR].
certificate to its local chapter/chartered local. The In the second, there must be a distinction made between
former is filing the PCE for and on behalf of the the reckoning of the 1-year statutory bar and the 1-year
latter; or (ii) the local chapter/chartered local itself certification year bar. The 1-year period in the former should be
which has been issued a charter certificate by the counted from the date of election; while the 1-year period in the
national union or federation; or (iii) an latter should be from the date of certification of the SEBA. The
independently registered union; union will be deprived of its entitlement to the critical one (1) year
b. An employer, when requested to bargain as a certified SEBA if this period is reckoned from the actual date
collectively in a bargaining unit where no registered when the certification, consent, run-off or re-run election was
CBA exists and the status of the union is in doubt. conducted. The union is certainly entitled to a full 12 months as
SEBA. Until certification is made in its favor, its status as SEBA is
Role of Employer in Certification Election not definite.
Where an appeal has been filed from the order of the
Article 271. [258-A] Employer as Bystander. - In all cases, Med-Arbiter certifying the results of the election, the running of
whether the petition for certification election is filed by an employer or the one (1) year period is deemed suspended until the decision on
a legitimate labor organization, the employer shall not be considered a the appeal has become final and executory [IRR].
party thereto with a concomitant right to oppose a in such proceedings If the SEBA fails to commence the collective bargaining
shall be limited to: (1) being notified or informed of petitions of such process within said period, its being the SEBA may be questioned
nature; and (2) submitting the list of employees during the pre-election by another union through the filing of a new PCE. This is best
conference should the Med-Arbiter act favorably on the petition. illustrated by the case of KAMPIL-KATIPUNAN v. Trajano, where
the SEBA failed to initiate the collective bargaining process within
In issues as to the appropriateness of employees in a a period of more than 4 years thereby enabling another union to file
bargaining unit, the employer has a proper role. He can object to a new petition for certification election.
the appropriateness of the bargaining unit through a motion or a
manifestation. Once the bargaining unit is already established, (ii) Negotiations Bar Rule
neither the employer nor the employees can change its Under the negotiations bar rule, a certification election
configuration. petition may not be filed while the SEBA and employer have
commenced and sustained negotiations in good faith in accordance
d. When to File the Petition for Certification with Article 261 within one (1) year from the date of entry of
Election SEBA certification, or from the date of a valid certification,
In the absence of a CBA duly registered in accordance consent, or run-off election or from the date of issuance of the
with Article 237, a petition for certification lection may be filed at SEBA certification by the DOLE Regional Director in cases of
any time. Request for SEBA Certification.

Page 27 of 117
Once the CBA negotiations have commenced and while unjustified premium on the failure of the respondent hospital to perform its
the parties are engaged in this process, no challenging union is duty to bargain collectively as mandated in Article 252 of the Labor Code,
allowed file a PCE that would disturb the negotiation process and as amended, which states."
unduly delay, preempt or forestall the prompt and timely "Article 252. Meaning of duty to bargain collectively - the duty
to bargain collectively means the performance of a mutual obligation to
conclusion thereof.
meet and convene promptly and expeditiously in good faith for the purpose
It must be noted that there is no law or rule that imposes of negotiating an agreement with respect to wages, hours of work and all
a time limitation or caps as to when the parties should negotiate other terms and conditions of employment including proposals for adjusting
and conclude a CBA. The parties have all the freedom and leeway any grievance or questions arising under such agreement and executing a
to negotiate the CBA’s terms and conditions without being contract incorporating such agreements if requested by either party but such
constrained by any time restriction. In other words, the negotiation duty does not compel any party to agree to a proposal or to make any
process may last for days, weeks, months, even years, and during concession."
the entire duration thereof, no PCE may be filed by any challenging The duly certified bargaining agent, CMCEA - AFW, should not
be made to further bear the brunt flowing from the respondent hospital's
union.
reluctance and thinly disguised refusal to bargain."
If the law proscribes the conduct of a certification election when
(iii) Bargaining Deadlock Bar Rule there is a bargaining deadlock submitted to conciliation or arbitration, with
Collective bargaining deadlock refers to a situation more reason should it not be conducted if, despite attempts to bring an
where there is failure in the collective bargaining negotiations employer to the negotiation table by the certified bargaining agent, there
between the SEBA and the employer resulting in an impasse or was "no reasonable effort in good faith" on the employer to bargain
stalemate [San Miguel Corporation v. NLRC]. This happens when, collectively.
despite their efforts at bargaining in good faith, the parties have In the case of Kaisahan ng Manggagawang Pilipino vs. Trajano,
201 SCRA 453 (1991), penned by Chief Justice Andres R. Narvasa, the
failed to resolve the issues and it appears that there are no other
factual milieu of which is similar to this case, this Court allowed the
definite options or plans in sight to break the standoff. Simply holding of a certification election and ruled that the one year period known
stated, there is a deadlock when there is a complete blocking or as the "certification year" has long since expired. We also ruled, that:
stoppage in the negotiation resulting from the action of equal and   "xxx prior to the filing of the petition for election in this case,
opposing forces [Capitol Medical Center Alliance of Concerned there was no such 'bargaining deadlock xx (which) had been submitted to
Employees v. Laguesma]. conciliation or arbitration or had become the subject of a valid notice of
Under this rule, a certification election petition may not strike or lockout.' To be sure, there are in the record assertions by NAFLU
be filed when a bargaining deadlock to which an incumbent SEBA that its attempts to bring VIRON to the negotiation table had been
unsuccessful because of the latter's recalcitrance, and unfulfilled promises
is a party has been submitted to conciliation, compulsory or
to bargain collectively; but there is no proof that it had taken any action to
voluntary arbitration or has become the subject of a valid notice of legally coerce VIRON to comply with its statutory duty to bargain
strike or lockout [IRR]. collectively. It could have charged VIRON with unfair labor practice; but it
The Deadlock Bar Rule simply provides that a petition did not. It could have gone on a legitimate strike in protest against VIRON's
for certification election can only be entertained if there is no refusal to bargain collectively and compel it to do so; but it did not. There
pending bargaining deadlock submitted to conciliation or are assertions by NAFLU, too, that its attempts to bargain collectively had
arbitration or had become the subject of a valid notice of strike or been delayed by continuing challenges to the resolution pronouncing it the
lockout. The principal purpose is to ensure stability in the sole bargaining representative in VIRON; but there is no adequate
substantiation thereof , or of how it did in fact prevent initiation of the
relationship of the workers and the management [NACUSIP v.
bargaining process between it and VIRON."
Trajano]. Although the statements pertinent to this case are merely obiter,
still the fact remains that in the Kaisahan case, NAFLU was counselled by
CMCEA-AFW v. Laguesma this Court on the steps that it should have undertaken to protect its interest,
Held: Was there a bargaining deadlock between CMC and but which it failed to do so.
respondent union, before the filing of petitioner of a petition for This is what is strikingly different between the Kaisahan case
certification election, which had been submitted to conciliation or had and the case at bench for in the latter case, there was proof that the certified
become the subject of a valid notice of strike or lockout? bargaining agent, respondent union, had taken an action to legally coerce
In the case of Divine Word University of Tacloban v. Secretary the employer to comply with its statutory duty to bargain collectively, i. e.,
of Labor and Employment, we had the occasion to define what a deadlock charging the employer with unfair labor practice and conducting a strike in
is, viz: protest against the employer's refusal to bargain. [25] It is only just and
"A 'deadlock' is xxx the counteraction of things producing entire equitable that the circumstances in this case should be considered as similar
stoppage; xxx There is a deadlock when there is a complete blocking or in nature to a "bargaining deadlock" when no certification election could be
stoppage resulting from the action of equal and opposed forces xxx. The held. This is also to make sure that no floodgates will be opened for the
word is synonymous with the word impasse, which xxx 'presupposes circumvention of the law by unscrupulous employers to prevent any
reasonable effort at good faith bargaining which, despite noble intentions, certified bargaining agent from negotiating a CBA. Thus, Section 3, Rule
does not conclude in agreement between the parties. ' " V, Book V of the Implement Rules should be interpreted liberally so as to
Although there is no "deadlock" in its strict sense as there is no include a circumstance, e. g. where a CBA could not be concluded due to
"counteraction" of forces present in this case nor "reasonable effort at good the failure of one party to willingly perform its duty to bargain collectively.
faith bargaining, "such can be attributed to CMC's fault as the bargaining
proposals of respondent union were never answered by CMC. In fact, what (iv) Contract Bar Rule
happened in this case is worse than a bargaining deadlock for CMC
Under this rule, the existence of the CBA, the contract
employed all legal means to block the certification of respondent union as
the bargaining agent of the rank-and-file; and use it as its leverage for its
referred to therein, bars the filing of a PCE. Once a CBA is duly
failure to bargain with respondent union. Thus, we can only conclude that registered and validly subsisting, no PCE or any other action
CMC was unwilling to negotiate and reach an agreement with respondent should be entertained that may disturb the administration of the
union. CMC has not at any instance shown willingness to discuss the duly registered existing CBA [IRR]. Neither party should terminate
economic proposals given by respondent union. nor modify such agreement during its lifetime. Inter-union electoral
As correctly ratiocinated by public respondent, to wit: contests are therefore not allowed [Foamtex Labor Union v.
"For herein petitioner to capitalize on the ensuing delay which Noriel].
was caused by the hospital and which resulted in the non-conclusion of a
For the entire 5-year lifetime of the CBA, no PCE
CBA within the certification year, would be to negate and render a mockery
of the proceedings undertaken before this Department and to put an
questioning the majority status of the incumbent SEBA shall be
entertained and no certification election shall be conducted by the
Page 28 of 117
DOLE outside of the 60-day freedom period immediately before present between petitioner and Triumph International. The CBA
the date of expiry of such five-year term of the CBA. was effective up to September 24, 1989. There is no doubt that the
respondent union’s CBA constituted a bar to the holding of the
Exceptions certification election as petitioned by the respondent union with
The contract bar rule admits of several exceptions where public Respondent. The members of the respondent union should
a PCE may be validly filed: wait for the proper time.

(i) During the 60-day freedom period immediately Oriental Tin v. Secretary of Labor
prior to the expiry date of a CBA. Held: It is uncontroverted that the petition for certification
(ii) When the CBA is not registered with the BLR election in this case was filed on March 18, 1994, twenty-eight days before
or any of the DOLE Regional Offices. the expiration of the existing CBA on April 15, 1994, and well within the
(iii) When the CBA, although registered, contains 60-day period provided for by the Code. The OTCLU, however, is
concerned with the effect of the employees' ratification of the new CBA on
provisions lower than the standards fixed by
the timely filing of the petition for certification election. Would such
law or illegal per se clauses. ratification nullify the petition?
(iv) When the documents supporting the CBA’s The law dictates a negative reply. The filing of a petition for
registration are falsified, fraudulent or tainted certification election during the 60-day freedom period gives rise to a
with misrepresentation. representation case that must be resolved even though a new CBA has been
(v) When the CBA is not complete as it does not entered into within that period. This is clearly provided for in the
contain any of the mandatory provisions which aforequoted Section 4, Rule V, Book V of the Omnibus Rules
the law requires. Such kind of agreement Implementing the Labor Code. The reason behind this rule is obvious. A
petition for certification election is not necessary where the employees are
cannot promote industrial peace as it leaves out
one in their choice of a representative in the bargaining process. Moreover,
matters which the parties should have said provision of the Omnibus Rules manifests the intent of the legislative
stipulated [Buklod ng Saulog Transit v. authority to allow, if not encourage, the contending unions in a bargaining
Casalla]. unit to hold a certification election during the freedom period. Hence, the
(vi) When the CBA was extended during its term as Court held in the case of Warren Manufacturing Workers Union (WMWU)
when it was negotiated and entered into prior to v. Bureau of Labor Relations, that the agreement prematurely signed by the
the 60-day freedom period. The agreement in union and the company during the freedom period does not affect the
this case is deemed hastily entered into in order petition for certification election filed by another union.
to frustrate the will of the employees in
choosing their bargaining representative Schism
[Associated Trade Union v. Noriel]. Excepted from the contract bar rule are certain types of
(vii) When there is a schism in the union resulting in contracts which do not foster industrial stability, such as contracts
an industrial dispute wherein the CBA can no where the identity of the representative is in doubt. Any stability
longer foster industrial peace. The conduct of a derived from such contracts must be subordinated to the
certification election in such a situation employees’ freedom of choice because it does not establish the
becomes imperative to clear any doubt as to the type of industrial peace contemplated by law. Where, therefore, the
real and legitimate representative of the fact of disaffiliation has been demonstrated beyond doubt, a
employees [Firestone Tire and Rubber certification election is the most expeditious way of determining
Company Employees Union v. Estrella]. which labor organization is to be the exclusive bargaining
(viii) When there is an automatic renewal provision representative [Firestone Tire and Rubber Company Employees
in the CBA but prior to the date when such Union v. Estrella].
automatic renewal became effective, the
employer seasonably filed a manifestation with Associated Workers Union-PTGWO v. NLRC
the BLR of its intention to terminate the said Held: While it is true that AWUM as a local union, being an
agreement if and when it is established that the entity separate and distinct from AWU, is free to serve the interest of all its
members and enjoys the freedom to disaffiliate, such right to disaffiliate
SEBA does not represent anymore the majority
may be exercised, and is thus considered a protected labor activity, only
of the workers in the bargaining unit [PLDT
when warranted by circumstances.  Generally, a labor union may
Employees’ Union v. PLDT]. disaffiliate from the mother union to form a local or independent union only
during the 60-day freedom period immediately preceding the expiration of
60-Day Freedom Period the CBA. Even before the onset of the freedom period (and despite the
The 60-day freedom period is superior to all other rules. closed-shop provision in the CBA between the mother union and
During this period, a petition for certification election and a motion management) disaffiliation may still be carried out, but such disaffiliation
or intervention can be filed. Outside of this, the petition should be must be effected by a majority of the members in the bargaining unit. This
happens when there is a substantial shift in allegiance on the part of the
dismissed outright.
majority of the members of the union.  In such a case, however, the CBA
During this period, the union security clause is not continues to bind the members of the new or disaffiliated and independent
effective and the employees are free to form or join another union union up to the CBA's expiration date.
without fear of getting fired. However, they are not free from
paying union dues and agency fees because the EBA representation
ALU v. Calleja
status is still conclusive up to the expiration of the CBA term.
Held: The Court has long since declared that:
Upon the expiration of the period, the employer should ... Basic to the contract bar rule is the proposition that the delay
continue to recognize the majority status of the incumbent EBA of the right to select represen tatives can be justified only where stability is
where no petition for certification election challenging such deemed paramount. Excepted from the contract which do not foster
majority status is filed by any other union. industrial stability, such as contracts where the identity of the representative
In PMTI-ULGP v. Calleja, respondent union’s petition is in doubt. Any stability derived from such contracts must be subordinated
for certification election was filed on November 25, 1987. At the to the employees' freedom of choice because it does nto establish the type
time of the filing of the said petition, a valid and existing CBA was of industrial peace contemplated by the law. 11

Page 29 of 117
At this juncture, petitioner should be reminded that the technical In formulating the “substitutionary” doctrine, the only
rules of rpocedure do not strictly apply in the adjudication of labor consideration involved was the employees’ interest in the existing
disputes. 12 Consequently, its objection that the evidence with respect to the bargaining agreement. The agent’s interest never entered the picture. In
aforesaid repudiiation of the supposed collective bargaining agreement fact, the justification for said doctrine was:
cannot be considered for the first time on appeal on the Bureau of Labor ". . . that the majority of the employees, as an entity under the
Relations should be disregarded, especially considering the weighty statute, is the true party in interest to the contract, holding rights through the
significance thereof. agency of the union representative. Thus, any exclusive interest claimed by
Both petitioner and private respondent GAW Trading, Inc. the agent is defeasible at the will of the principal.. .” (Italics supplied)
allege that the employees of the latter are now enjoying the benefits of the Stated otherwise, the “substitutionary” doctrine only provides
collective bargaining agreement that both parties had forged. However, We that the employees cannot revoke the validly executed collective bargaining
cannot find sufficient evidence of record to support this contention. The contract with their employer by the simple expedient of changing their
only evidence cited by petitioner is supposed payment of union fees by said bargaining agent. And it is in the light of this that the phrase “said new
employees, a premise too tenuous to sustain the desired conclusion. Even agent would have to respect said contract” must be understood. It only
the actual number of workers in the respondent company is not clear from means that the employees, thru their new bargaining agent, cannot renege
the records. Said private respondent claims that it is two hundred eighty- on their collective bargaining contract, except of course to negotiate with
one (281)13 but petitioner suggests that it is more than that number. The said management for the shortening thereof.
parties should be aware that this Court is not an adjudicator of facts. Worse, The “substitutionary” doctrine, therefore, cannot be invoked to
to borrow a trite but apt phrase, they would heap the Ossa of confusion support the contention that a newly certified collective bargaining agent
upon the Pelion of uncertainty and still expect a definitive ruling on the automatically assumes all the personal undertakings — like the no-strike
matter thus confounded. stipulation here — in the collective bargaining agreement made by the
Additionally, the inapplicability of the contract bar rule is deposed union. When BBWU bound itself and its officers not to strike, it
further underscored by the fact that when the disputed agreement was filed could not have validly bound also all the other rival unions existing in the
before the Labor Regional Office on May 27, 1986, a petition for bargaining units in question. BBWU was the agent of the employees, not of
certification election had already been filed on May 19, 1986. Although the the other unions which possess distinct personalities. To consider UNION
petition was not supported by the signatures of thirty percent (30%) of the contractually bound to the no-strike stipulation would therefore violate the
workers in the bargaining unit, the same was enough to initiate said legal maxim that res inter alios acta alios nec prodest nec nocet.
certification election. Of course, UNION, as the newly certified bargaining agent,
could always voluntarily assume all the personal undertakings made by the
Substitutionary Doctrine displaced agent. But as the lower court found, there was no showing at all
that, prior to the strike,[11] UNION formally adopted the existing
Under this doctrine, employees are allowed to change
CONTRACT as its own and assumed all the liabilities imposed by the same
their SEBA, but the CBA continues to bind them up to its upon BBWU.
expiration date. They may bargain however for the shortening of
said expriation date.
e. Appeal
Thus, the new SEBA cannot negotiate a new CBA, it can
only administer the old CBA, but the new SEBA is not bound by
Article 272. [259] Appeal from Certification Election Orders.
the personal obligations imposed by the old CBA such as the “no Any party to an election may appeal the order or results of the election
strike, no lockout rule.” as determined by the Med-Arbiter directly to the Secretary of Labor
and Employment on the ground that the rules and regulations or parts
Benguet Consolidated v. BCI Employees thereof established by the Secretary of Labor and Employment for the
Held: In support of an affirmative answer to the first question, conduct of the election have been violated. Such appeal shall be decided
BENGUET first invokes the so-called “Doctrine of Substitution" referred to within fifteen (15) calendar days.
in General Maritime Stevedore's Union v. South Sea Shipping Lines, L-
14689, July 26, 1960. There it was remarked: f. Conduct of Certification Election
“We also hold that where the bargaining contract is to run for The process of certification election requires the
more than two years, the principle of substitution may well be adopted and
application of the double majority rule for the following twin
enforced by the CIR to the effect that after two years of the life of a
bargaining agreement, a certification election may be allowed by the purposes:
CIR; that if a bargaining agent other than the union or organization that
executed the contract, is elected, said new agent would have to respect said (i) To have a valid certification election; and
contract, but that it may bargain with the management for the shortening of (ii) To declare the winning union that will be certified
the life of the contract if it considers it too long, or refuse to renew the as a SEBA.
contract pursuant to an automatic renewal clause.” (Italics supplied)
The submission utterly fails to persuade Us. The above-quoted (i)First Majority: To be a valid
pronouncement was obiter dictum. The only issue in the General Maritime
certification election, at least a majority
Stevedores’ Union case was whether a collective bargaining agreement
which had practically run for 5 years constituted a bar to certification of all eligible voters in the bargaining
proceedings. We held it did not and accordingly directed the court a quo to unit should have cast their votes
order certification elections. With that, nothing more was necessary for the The first majority is essential to validate the certification
disposition of the case. Moreover, the pronouncement adverted to was election process itself. According to Article 268, in order to have a
rather premature. The possible certification of a union different from that valid certification election, it is required that at least a majority of
which signed the bargaining contract was a mere contingency then since the all eligible voters in the bargaining unit must have cast their votes
elections were still to be held. Clearly, the Court was not called upon to rule [Samahan ng Manggagawa v. Laguesma]. If less than such
on the possible effects of such proceedings on the bargaining agreement
majority have cast their votes, the certification election process
But worse, BENGUET’s reliance upon the Principle of
Substitution is totally misplaced. This principle, formulated by the itself is not valid and, therefore, not one of the contending unions
NLRB[7] as its initial compromise solution to the problem facing it when therein, even if chosen by the majority of the votes cast, can be
there occurs a shift in employees’ union allegiance after the execution of a certified as the SEBA to represent the CBU.
bargaining contract with their employer, merely states that even during the
effectivity of a collective bargaining agreement executed between employer (ii) Second Majority: Majority vote of the
and employees thru their agent, the employees can change said agent but valid votes cast in order to be chosen as
the contract continues to bind them up to its expiration date. They may the SEBA
bargain however for the shortening of said expiration date.

Page 30 of 117
After establishing the validity of the certification election committed during the conduct thereof, such as, inter alia,
process itself, the next point to ascertain and establish is whether disenfranchisement of the voters, lack of secrecy in the voting,
the petitioning union, in a single-union contest, or any of the fraud or bribery or acts of terrorism, force, threat and intimidation
unions, in a multi-union election, has garnered the majority of the employed by any of the contending unions or the employer. Such
valid votes cast. invalidation would necessitate the conduct of a re-run election
Under the same Article 268, it is required that only “the among the contending unions to determine the true will and desire
labor union receiving the majority of the valid votes cast shall be of the employees-electorate [Confederation of Citizens Labor
certified as the exclusive bargaining agent of all the workers in the Unions v. Noriel].
unit” and under Article 267, it is likewise provided that the labor
organization designated or selected by the majority of the Re-Run Election vs. Failure of Election
employees in an appropriate CBU is the exclusive representative of
the employees in such unit for purposes of collective bargaining Re-Run Election Failure of Election
[Peral v. United Employees Welfare Association]. There is a valid certification There is no valid certification
election but because of certain election because the number of
circumstances, the election is votes cast is less than the majority
4. Run-Off Election
nullified and another one is of the number of eligible voters
A run-off election refers to an election between the labor ordered to truly reflect the will and there are no challenged votes
unions receiving the two (2) highest number of votes when a and sentiment of the electorate- that could materially change the
certification election which provides for three (3) or more choices employees. results.
result in no choice receiving a majority of the valid votes cast;
provided, that the total number of votes for all contending unions is III
at least fifty percent (50%) of the number of votes cast. COLLECTIVE BARGAINING
Thus, a run-off election may only be conducted under the
following elements: A. DUTY TO BARGAIN COLLECTIVELY

a. There are three (3) or more unions competing in a Article 263. [252] Meaning of Duty to Bargain Collectively. -
certification or consent election; The duty to bargain collectively means the performance of a mutual
b. None of the contending unions garnered the obligation to meet and convene promptly and expeditiously in good
majority of the valid votes cast; faith for the purpose of negotiating an agreement with respect to
c. But the total number of votes for all contending wages, hours of work and all other terms and conditions of
employment including proposals for adjusting any grievances or
unions, if added, is at least fifty (50%) of the
questions arising under such agreement and executing a contract
number of valid votes cast; incorporating such agreements if requested by either party but such
d. If the above three (3) elements are present, a run-off duty does not compel any party to agree to a proposal or to make any
election will be conducted between the labor unions concession
receiving the two (2) highest number of votes in
such certification election or consent election. The Noteworthy in the above definition is the requirement on
third union and the others, if any, will no longer be both parties of the performance of the mutual obligation to meet
allowed to participate in such election. And for and convene promptly and expeditiously in good faith for the
obvious reason, the choice of “No Union” should no purpose of negotiating an agreement [Colegio de San Juan de
longer be included in the run-off election [IRR]. Letran v. Association of Employees and Faculties of Letran].
The duty does not compel any party to agree blindly to a
If the above conditions that justify the conduct of a run- proposal nor to make concession. While the law imposes on both
off election are present and there are no objections or challenges the employer and the bargaining union the mutual duty to bargain
which, if sustained, can materially alter the election results, the collectively, the employer is not under any legal obligation to
Election Officer should motu proprio conduct a run-off election initiate collective bargaining negotiations [Kiok Loy v. NLRC].
within 10 days form the close of the election proceeding between The duty to bargain collectively does not exist when the
the labor unions receiving the two highest number of votes [IRR]. majority status of the employees’ representative is not established.
The employer has no such duty to bargain with the individual
5. Re-Run Election workers or with the minority union [Lakas ng Manggagawang
This mode of choosing the SEBA is not expressly Makabayan v. Marcelo Enterprises]. In Philippine Diamond Hotel
provided in the Labor Code nor in the rendering of its v. Manila Diamond Hotel Employees Union, it was held that since
implementing rules. It was only in 2015 that an issuance of the the respondent union is not the exclusive representative of the
DOLE Secretary has introduced this term for the first time as an majority of the employees of petitioner, it could not demand from
amendment to the Rules to Implement the Labor Code and defines petitioner the right to bargain collectively in their behalf.
it as follows: Petitioner’s refusal, therefore, to bargain collectively with
respondent union cannot be considered ULP.
“‘Re-run election’ refers to an election doncuted to Obviously, the ultimate purpose of collective bargaining
break a tie between contending unions, including between ‘no
is to reach an agreement resulting in a contract binding on the
union’ and one of the unions. It shall likewise refer to an
election doncuted after a failure of election has been declared by parties; but the failure to reach an agreement after negotiations
the Election Officer and/or affirmed by the Mediator-Arbiter.” continued for a reasonable period does not establish a lack of good
faith. The statutes invite and contemplate a collective bargaining
A re-run election is obviously in the nature of a contract, but they do not compel one. The duty to bargain does not
corrective action meant to cure a seriously defective and distorted include the obligation to reach an agreement [Union of Filipino
certification election. Consequently, a third ground that may be Employees v. Nestle Philippines].
cited as would justify the conduct of a fair re-run election is when
the certification, consent or run-off election has been invalidated or 1. Duty to Bargain Collectively in the Absence of a
nullified due to certain serious irregularities that have been CBA

Page 31 of 117
Article 262. [251] Duty to Bargain Collectively in the Absence B. COLLECTIVE BARGAINING AGREEMENT
of Collective Bargaining Agreements. - In the absence of an agreement A Collective Bargaining Agreement is “a contract
or other voluntary arrangement providing for a more expeditious executed upon request of either the employer or the exclusive
manner of collective bargaining, it shall be the duty of employer and bargaining representative incorporating the agreement reached after
the representatives of the employees to bargain collectively in
negotiations with respect to wages, hours of work and all other
accordance with the provisions of this Code
terms and conditions of employment, including proposals for
adjusting any grievances or questions arising under such
Clearly, the law gives utmost premium and extends due agreement.” The primary purpose of a CBA is the stabilization of
respect to the voluntary arrangement between the parties on how labor-management relations in order to create a climate of a sound
they will discharge their respective duties to bargain collectively and stable industrial peace. In construing a CBA, the courts must
before resort to the procedure laid down in the Labor Code may be be practical and realistic and give due consideration to the context
made. In other words, it is only when there is no such voluntary in which it is negotiated and the purpose which it is intended to
arrangement that the procedure laid down in Article 261 of the serve [Rivera v. Espiritu].
Labor Code should be followed. The CBA embodies all the agreements reached after
negotiations between the employer and the SEBA with respect to
2. Duty to Bargain Collectively When There Exists the terms and conditions of their employment relationship
a CBA [Pantranco North Express v. NLRC]. Consequently, from the
moment it is perfected and during its lifetime, it is considered the
Article 264. [253] Duty to Bargain Collectively When There law between the parties [SMTFM-UWP v. NLRC] and as such, they
Exists a Collective Bargaining Agreement. - When there is a collective
are bound not only to the fulfillment of what has been expressly
bargaining agreement, the duty to bargain collectively shall also mean
that neither party shall terminate nor modify such agreement during stipulated but also to all consequences which, according to their
its lifetime. However, either party can serve a written notice to nature, may be in keeping with good faith [Goya v. Goya
terminate or modify the agreement at least sixty (60) days prior to its Employees Union] and the mandate of the law [Supreme Steel v.
expiration date. It shall be the duty of both parties to keep the status NMS-IND-APL]. Being the law between the parties, any violation
quo and to continue in full force and effect the terms and conditions of thereof can be subject of redress in the court [Faculty Association
the existing agreement during the 60-day period and/or until a new of Mapua v. Court of Appeals].
agreement is reached by the parties.  In United Kimberly-Clark Employees Union Philippine
Transport General Workers Organization (UKCEU-PTGWO) v.
This last 60-day period is the freedom period. It is Kimberly-Clark Philippines, Inc., this Court emphasized that:
denominated as such because it is the only time when the law
allows the parties to freely serve a notice to terminate, alter or As a general proposition, an arbitrator is confined to
modify the existing CBA. It is also the time when the majority the interpretation and application of the collective bargaining
status of the SEBA may be challenged by another union by filing agreement. He does not sit to dispense his own brand of
the appropriate petition for certification election [MRR Yard Crew industrial justice: his award is legitimate only in so far as it
v. PNR]. draws its essence from the CBA, i.e., when there is a rational
nexus between the award and the CBA under consideration. It is
Another exception is the so-called Fractual Bargaining.
said that an arbitral award does not draw its essence from the
This is when the change that gradually occurs because of the actual CBA; hence, there is an unauthorized amendment or alteration
practice of the employer which in fact increases the benefits thereof, if:
provided in the CBA. 1. It is so unfounded in reason and fact;
2. It is so unconnected with the working and purpose
Automatic Renewal Clause of the agreement;
The last sentence of Article 253 refers to the so-called 3. It is without factual support in view of its
"automatic renewal" of a CBA. The parties shall continue the CBA language, its context, and any other indicia of the parties'
intention;
in "full force and effect" until they reach a new agreement.
4. It ignores or abandons the plain language of the
It is clear from the above provision of law that until a contract;
new Collective Bargaining Agreement has been executed by and 5. It is mistakenly based on a crucial assumption
between the parties, they are duty-bound to keep the status quo and which concededly is a nonfact;
to continue in full force and effect the terms and conditions of the 6. It is unlawful, arbitrary or capricious; and
existing agreement. The law does not provide for any exception nor 7. It is contrary to public policy.
qualification as to which of the economic provisions of the existing xx xx
agreement are to retain force and effect, therefore, it must be If the terms of a CBA are clear and [leave] no doubt
upon the intention of the contracting parties, the literal meaning
understood as encompassing all the terms and conditions in the said
of its stipulation shall prevail. However, if, in a CBA, the parties
agreement [New Pacific Timber v. NLRC]. stipulate that the hirees must be presumed of employment
qualification standards but fail to state such qualification
Effect of CBA Renewal or Registration Before or standards in said CBA, the VA may resort to evidence extrinsic
During the 60-Day Freedom Period of the CBA to determine the full agreement intended by the
It is well settled that the 60-day freedom period based on parties. When a CBA may be expected to speak on a matter, but
the original CBA shall not be affected by any amendment, does not, its sentence imports .ambiguity on that subject. The
extension or renewal of the CBA for purposes of certification VA is not merely to rely on the cold and cryptic words on the
face of the CBA but is mandated to discover the intention of the
election.
parties. Recognizing the inability of the parties to anticipate or
In the case of Warren Manufacturing Workers Union v. address all future problems, gaps may be left to be filled in by
Bureau of Labor Relations, it was held that an agreement reference to the practices of the industry, and the step which is
prematurely signed by the union and the company during the equally a part of the CBA although not expressed in it. In order
freedom period does not affect the petition for certification election to ascertain the intention of the contracting parties, their
filed by another union. contemporaneous and subsequent acts shall be principally
considered The VA may also consider and rely upon negotiating
Page 32 of 117
and contractual history of the parties, evidence of past practices The rule is that unless expressly assumed, labor contracts
interpreting ambiguous provisions. The VA has to examine such such as employment contracts and collective bargaining
practices to determine the scope of their agreement, as where the agreements are not enforceable against a transferee of an
provision of the CBA has been loosely formulated. Moreover,
enterprise, labor contracts being in personam, thus binding only
the CBA must be construed liberally rather than narrowly and
technically and the Court must place a practical and realistic between the parties. A labor contract merely creates an action in
construction upon it personally and does not create any real right which should be
respected by third parties. This conclusion draws its force from the
1. Law Deemed Written or Incorporated right of an employer to select his employees and to decide when to
As in all contracts, the parties in a CBA may establish engage them as protected under our Constitution, and the same can
such stipulations, clauses, terms and conditions as they may deem only be restricted by law through the exercise of the police power.
convenient provided these are not contrary to law, morals, good As a general rule, there is no law requiring a bona fide purchaser of
customs, public order or public policy. Thus, where the CBA is assets of an on-going concern to absorb in its employ the
clear and unambiguous, it becomes the law between the parties and employees of the latter. However, although the purchaser of the
compliance therewith is mandated by the express policy of the law assets or enterprise is not legally bound to absorb in its employ the
[Hongkong Union v. HSBC]. employers of the seller of such assets or enterprise, the parties are
Parties may validly agree in the CBA to reduce wages liable to the employees if the transaction between the parties is
and benefits provided such reduction does not go below the colored or clothed with bad faith [Sundowner v. Drilon]. 
minimum standards [Manila Fashions v. NLRC].
If the CBA provides for just the minimum wage rates, the 4. Effectivity in Case of Merger/Consolidation;
CBA is void for being a Sweetheart contract. Wiley Doctrine
The lack of a provision in the plan of merger regarding
2. Entitlement to Benefits the transfer of employment contracts to the surviving corporation
The following are entitled to the benefits stipulated in the could have very well been deliberate on the part of the parties to
CBA: the merger, in order to grant the surviving corporation the freedom
to choose who among the dissolved corporation's employees to
a. Members of the Bargaining Union; retain, in accordance with the surviving corporation's business
b. Non-members of the bargaining union but are needs.  If terminations, for instance due to redundancy or labor-
members of the bargaining unit; saving devices or to prevent losses, are done in good faith, they
c. Members of the minority union/s who paid agency would be valid.  The surviving corporation too is duty-bound to
fees to the bargaining union; and protect the rights of its own employees who may be affected by the
d. New employees hired after the conclusion of the merger in terms of seniority and other conditions of their
CBA and during its effectivity or even after its employment due to the merger.  Thus, we are not convinced that in
expiration. the absence of a stipulation in the merger plan the surviving
corporation was compelled, or may be judicially compelled, to
Employees excluded from the bargaining unit, like absorb all employees under the same terms and conditions
confidential employees or managerial employees or supervisory obtaining in the dissolved corporation as the surviving corporation
employees, in the case of rank-and-file CBU, or vice versa, are not should also take into consideration the state of its business and its
entitled to the benefits flowing from the CBA. But two exceptions obligations to its own employees, and to their certified collective
may be cited, to wit: bargaining agent or labor union [BPI v. BPI Employees].

a. When the CBA benefits are granted to managerial 5. Substitutionary Doctrine


employees by reason of company policy or Under this doctrine, employees are allowed to change
company practice; their SEBA, but the CBA continues to bind them up to its
b. When adjustments are made to avoid distortion in expiration date. They may bargain however for the shortening of
the levels of wages or benefits. said expriation date.
Thus, the new SEBA cannot negotiate a new CBA, it can
In the first, in order to make it an enforceable and only administer the old CBA, but the new SEBA is not bound by
demandable right, there should be evidence of existence of such the personal obligations imposed by the old CBA such as the “no
policy or practice; otherwise, the absence thereof would not justify strike, no lockout rule.”
any claim or demand therefor.
In the second, certain economic benefits may be Benguet Consolidated v. BCI Employees
voluntarily extended to excluded employees such as increases in Held: In support of an affirmative answer to the first question,
wages and other monetary benefits because a CBA was concluded BENGUET first invokes the so-called “Doctrine of Substitution" referred to
in General Maritime Stevedore's Union v. South Sea Shipping Lines, L-
with the rank-and-file employees. But if so given, the same cannot,
14689, July 26, 1960. There it was remarked:
strictly speaking, be considered as having been based on the CBA “We also hold that where the bargaining contract is to run for
but simply as adjustments to prevent distortion in the levels of more than two years, the principle of substitution may well be adopted and
wages and benefits among the employees included in the CBU, in enforced by the CIR to the effect that after two years of the life of a
particular, and all the employees of the establishment, in general. bargaining agreement, a certification election may be allowed by the
There is nothing to prevent the employer from granting CIR; that if a bargaining agent other than the union or organization that
benefits to managerial employees equal to or higher than those executed the contract, is elected, said new agent would have to respect said
afforded to union members. There can be no conflict of interest contract, but that it may bargain with the management for the shortening of
the life of the contract if it considers it too long, or refuse to renew the
where the employer himself voluntarily agrees to grant such
contract pursuant to an automatic renewal clause.” (Italics supplied)
benefits to managerial employees [Martinez v. NLRC]. The submission utterly fails to persuade Us. The above-quoted
pronouncement was obiter dictum. The only issue in the General Maritime
3. In case of change of ownership of establishment Stevedores’ Union case was whether a collective bargaining agreement
which had practically run for 5 years constituted a bar to certification

Page 33 of 117
proceedings. We held it did not and accordingly directed the court a quo to C. PROVISIONS SUBJECT TO COLLECTIVE
order certification elections. With that, nothing more was necessary for the BARGAINING
disposition of the case. Moreover, the pronouncement adverted to was
rather premature. The possible certification of a union different from that Article 263. [252] Meaning of Duty to Bargain Collectively. -
which signed the bargaining contract was a mere contingency then since the The duty to bargain collectively means the performance of a mutual
elections were still to be held. Clearly, the Court was not called upon to rule obligation to meet and convene promptly and expeditiously in good
on the possible effects of such proceedings on the bargaining agreement faith for the purpose of negotiating an agreement with respect to
But worse, BENGUET’s reliance upon the Principle of wages, hours of work and all other terms and conditions of
Substitution is totally misplaced. This principle, formulated by the employment including proposals for adjusting any grievances or
NLRB[7] as its initial compromise solution to the problem facing it when questions arising under such agreement and executing a contract
there occurs a shift in employees’ union allegiance after the execution of a incorporating such agreements if requested by either party but such
bargaining contract with their employer, merely states that even during the duty does not compel any party to agree to a proposal or to make any
effectivity of a collective bargaining agreement executed between employer concession.
and employees thru their agent, the employees can change said agent but
the contract continues to bind them up to its expiration date. They may
bargain however for the shortening of said expiration date. 1. Non-Economic/Non-Monetary/Political
In formulating the “substitutionary” doctrine, the only This covers the following provisions that the parties to a
consideration involved was the employees’ interest in the existing CBA usually stipulate: (a) Coverage or Scope of the Agreement;
bargaining agreement. The agent’s interest never entered the picture. In (b) Exclusions; (c) Rights and Responsibilities of Parties; (d)
fact, the justification for said doctrine was: Union Security Arrangement; (e) Job Security (Security of
". . . that the majority of the employees, as an entity under the Tenure); (f) Management Rights and Prerogatives; (g) Company
statute, is the true party in interest to the contract, holding rights through the
Rules and Regulations; (h) Discipline of Employees; (i) Union
agency of the union representative. Thus, any exclusive interest claimed by
the agent is defeasible at the will of the principal.. .” (Italics supplied) Dues and Special Assessments; (j) Agency Fee; (k) Check-Off; (l)
Stated otherwise, the “substitutionary” doctrine only provides Grievance Machinery; (m) Voluntary Arbitration; (n) Labor-
that the employees cannot revoke the validly executed collective bargaining Management Council (LMC); (o) No-Strike, No-Lockout; (v)
contract with their employer by the simple expedient of changing their Waiver and Completeness of Agreement; and (q) Duration and
bargaining agent. And it is in the light of this that the phrase “said new Effectivity of Agreement.
agent would have to respect said contract” must be understood. It only
means that the employees, thru their new bargaining agent, cannot renege 2. Economic/Monetary/Non-Political
on their collective bargaining contract, except of course to negotiate with
This includes: (a) Wage Increases; (b) Allowances; (c)
management for the shortening thereof.
The “substitutionary” doctrine, therefore, cannot be invoked to Premiums for Work on Rest Days, Holidays, etc.; (d) Meal, Rice
support the contention that a newly certified collective bargaining agent and other Subsidies; (e) Leave Benefits; (f) Union Leave; (g)
automatically assumes all the personal undertakings — like the no-strike Uniforms; (h) Union Office; (i) Promotions; (j) Bonuses; (k)
stipulation here — in the collective bargaining agreement made by the Insurance; (l) Hospitalization; (m) Retirement; (n) Excursion; and
deposed union. When BBWU bound itself and its officers not to strike, it (o) Others which have monetary values.
could not have validly bound also all the other rival unions existing in the
bargaining units in question. BBWU was the agent of the employees, not of 3. Personal
the other unions which possess distinct personalities. To consider UNION
These are provisions solely for the benefit of the union
contractually bound to the no-strike stipulation would therefore violate the
legal maxim that res inter alios acta alios nec prodest nec nocet. and which the new EBA are not bound to.
Of course, UNION, as the newly certified bargaining agent,
could always voluntarily assume all the personal undertakings made by the D. PROCEDURE IN COLLECTIVE BARGAINING
displaced agent. But as the lower court found, there was no showing at all
that, prior to the strike,[11] UNION formally adopted the existing Article 261. [250] Procedure in Collective Bargaining. - The
CONTRACT as its own and assumed all the liabilities imposed by the same following procedures shall be observed in collective bargaining:
upon BBWU. (a) When a party desires to negotiate an agreement, it shall
serve a written notice upon the other party with a statement of its
6. Principles proposals. The other party shall make a reply thereto not later than ten
(10) calendar days from receipt of such notice;
(b) Should differences arise on the basis of such notice and
 The CBA is executed not only upon the request of
reply, either party may request for a conference which shall begin not
the EBA but also upon request of the employer. later than ten (10) calendar days from the date of request.
 Unilateral changes in the CBA are not allowed (c) If the dispute is not settled, the Board shall intervene
 CBA should be construed liberally upon request of either or both parties or at its own initiative and
 Workers are allowed to negotiate wage increases immediately call the parties to conciliation meetings. The Board shall
separately and distinctly from legislated wage have the power to issue subpoenas requiring the attendance of the
parties to such meetings. It shall be the duty of the parties to
increase.
participate fully and promptly in the conciliation meetings the Board
 The literal meaning of the stipulations of the CBA, may call;
as with every other contract, control if they are clear (d) During the conciliation proceedings in the Board, the
and leave no doubt upon the intention of the parties are prohibited from doing any act which may disrupt or impede
contracting parties. Thus, where the CBA is clear the early settlement of the disputes; and
and unambiguous, it, becomes the law between the (e) The Board shall exert all efforts to settle disputes
parties and compliance therewith is mandated by the amicably and encourage the parties to submit their case to a voluntary
arbitrator.
express policy of the law. Moreover, it is a familiar
rule in interpretation of contracts that the various
stipulations of a contract shall be interpreted 1. Phases of the Collective Bargaining Process
together, attributing to the doubtful ones that sense
which may result from all of them taken jointly a. Negotiation Phase
[Coca-Cola v. Ilo-Ilo Coca Cola Plant Employees]. Collective bargaining formally takes the fort of
negotiations when major conditions of employment to be written

Page 34 of 117
into an agreement are under consideration and of grievance adjusting any grievances or question arising under such
committee meetings and arbitration when questions arising in the agreement and a violation of this obligation is an unfair labor
administration of an agreement are at stake [Republic Savings practice [Ibid].
Bank v. Court of Industrial Relations].
c. Renegotiation Phase
Some Principles This is the phase where the duties in phase 1 and 2 are
i. A proposal not embodied in the CBA is not part carried over.
thereof.
ii. Minutes of CBA negotiation has no effect if its 2. Procedure is Mandatory
contents are not incorporated in the CBA. The procedure in collective bargaining prescribed by the
iii. Making a promise during the CBA negotiation is not Labor Code is mandatory because of the basic interest of the State
considered bad faith. in ensuring lasting industrial peace. As held in the case of Kiok Loy
iv. Adamant stance resulting in impasse is not bad vs. NLRC, the company's refusal to make counter-proposal to the
faith. union's proposed CBA is an indication of its bad faith. Where the
v. The DOLE Secretary cannot order inclusion of employer did not even bother to submit an answer to the bargaining
terms and conditions in CBA which the law and the proposals of the union, there is a clear evasion of the duty to
parties did not intend to reflect therein. bargain collectively. In the case at bar, petitioner's actuation show a
vi. Signing bonus is not demandable under the law. lack of sincere desire to negotiate rendering it guilty of unfair labor
vii. Allegations of bad faith are wiped out with the practice [Colegio de San Juan de Letran v. Association of
signing of the CBA. Employees and Faculty of Letran].

Effect of the Refusal of a Party to Sign the CBA 3. Kiok Loy Doctrine
A party to a fully-concluded CBA may be compelled to The doctrine is based on the ruling in Kiok Loy v. NLRC,
sign it, especially if said refusal to sign is the only remaining hitch where the petitioner, Sweden Ice Cream Plant, refused to submit
to its being implemented. Such refusal is considered unfair labor any counter-proposal to the CBA proposed by its employees’
practice [Roadway Express v. General Teamster]. certified SEBA. The Court rules that the employer had thereby lost
its right to bargain the terms and conditions of the CBA.
CBA negotiated and concluded during suspension of Consequently, all the terms and conditions of the CBA as proposed
operations by the SEBA are deemed approved and accepted lock, stock and
In San Pedro Hospital of Digos v. Secretary of Labor, it barrel (LSB) by the erring employer.
was held: This case epitomizes the classic case of negotiating a
CBA in bad faith consisting of the employer’s refusal to bargain
Temporary suspension of operations is reorganized with the SEBA by ignoring all notices for negotiations and requests
as a valid exercise of management prerogative provided it is not for counter-proposals made to the former by the latter. Such refusal
carried  out in order to circumvent provisions of the Labor Code to send its counter-proposals to the SEBA’s proposals and to
or to defeat the rights of the employees under the Code. The
bargain on the economic terms of the CBA constitutes an unfair
determination to cease or suspend operations is a prerogative of
management that the State usually does not interfere with, as no labor practice [General Milling Corporation v, Court of Appeals].
business can be required to continue operating at a loss simply In Divine Word University of Tacloban v. Secretary of
to maintain the workers in employment.  Such an act would be Labor and Employment, the university refused to perform its duty
tantamount to a taking of property without due process of law, to bargain collectively. Hence, the Court upheld the unilateral
which the employer has a right to resist.  But where it is shown imposition on the university of the CBA proposed by the Divine
that the closure is motivated not by a desire to prevent further Word University Employees Union.
losses, but to discourage the workers from organizing In General Milling Corporation v. Court of Appeals, the
themselves into a union for more effective negotiations with
Supreme Court imposed on the employer the draft CBA proposed
management, the State is bound to intervene.
If a legitimate, valid and legal suspension of by the SEBA for the last 2 years commencing from the expiration
operations does not terminate but merely suspends the of the 3-year term of the original CBA. This was because of the
employee-employer relationship, with more reason will an employer’s refusal to counter-propose to the SEBA’s proposals
invalid and illegal suspension of operations, as in this case, not which was declared as an ULP.
affect the employment relationship.
The foregoing premises considered, it is clear that 4. CBA Deadlock
there is no basis for petitioner to claim that a new CBA should In case of a deadlock in the initial negotiation or re-
not be entered into or that collective bargaining should not be
negotiation or renewal of the CBA, the law provides that the parties
conducted during the effectivity of a temporary suspension of
operations.  In this instance, petitioner expressly represented that may exercise their respective rights under the Labor Code which
the suspension was to be for six months only.  In the absence of include the following:
any other information, the plain and natural presumption will be
that petitioner would resume operations after six months, and a. Submission of the deadlocked issue to conciliation
therefore, it follows that a new CBA will be needed to govern and mediation by the NCMB. This means that the
the employment relations of the parties, the old one having deadlock may be taken cognizance of motu proprio
already expired.  Clearly then, under the circumstances, the by the NCMB or through the filing of a notice of
respondent Secretary cannot be faulted nor considered to have
strike by the union or notice of lockout by the
gravely abused his discretion for ordering the parties to enter
into a new CBA. employer or notice of preventive mediation by any
of the parties,
b. Administration Phase b. Declaration and actual staging of a strike by the
The duty to bargain imposes on the parties during the union or lockout by the employer.
term of their agreement the mutual obligation "to meet and confer c. Referral of case to compulsory or voluntary
promptly and expeditiously and to good faith for the purpose off arbitration.

Page 35 of 117
d. In case of industries indispensable to the national of expiry of the term of such other provisions as fixed in such Collective
interest, filing of petition for assumption of Bargaining Agreement, shall retroact to the day immediately following
jurisdiction over the labor dispute or certification such date. If any such agreement is entered into beyond six months, the
thereof to the NLRC for compulsory arbitration. parties shall agree on the duration of retroactivity thereof. In case of a
deadlock in the renegotiation of the Collective Bargaining Agreement,
the parties may exercise their rights under this Code.
E. PUBLICATION, RATIFICATION, AND
REGISTRATION OF THE CBA
Under this provision, insofar as representation is
Article 237, Paragraph 2 of the Labor Code provides;
concerned, a CBA has a term of five years, while the other
Within thirty (30) days from the execution of a provisions, except for representation, may be negotiated not later
Collective Bargaining Agreement, the parties shall submit than three years after the execution [Rivera v. Espiritu].
copies of the same directly to the Bureau or the Regional Offices The duty to renegotiate not later than 3 years after its
of the Department of Labor and Employment for registration execution is more serious than the duty to negotiate the CBA
accompanied with verified proofs of its posting in two during the 60-day freedom period. During the 3-year period, the
conspicuous places in the place of work and ratification by the duty to bargain is mandatory. However, during the 60-day period,
majority of all the workers in the bargaining unit. The Bureau or the duty to bargain is permissive.
Regional Offices shall act upon the application for registration
The "representation aspect" refers to the identity and
of such Collective Bargaining Agreement within five (5)
calendar days from receipt thereof. The Regional Offices shall majority status of the union that negotiated the CBA as the
furnish the Bureau with a copy of the Collective Bargaining exclusive bargaining representative of the appropriate bargaining
Agreement within five (5) days from its submission. unit concerned. "All other provisions" simply refers to the rest of
the CBA, economic as well as non-economic provisions, except
If the CBA is not registered, will it bar a petition for representation [San Miguel Employees v. Confesor].
certification election?
No. In Trade Unions of the Philippines v. Laguesma, it 1. Waiver of Right to Renegotiation
was held: As a general rule, no. The basic rights of labor are not
waivable because it is contrary to public policy. However, if there
It appears that the procedural requirement of filing is already adequate consideration such as higher benefit of some
the CBA within 30 days from date of execution under Article sort, the union can waive such right to renegotiate in certain
231 was not met. The subject CBA was executed on November circumstances.
28, 1989. It was ratified on December 8, 1989, and then filed
with DOLE for registration purposes on March 14, 1990. Be that
2. Suspension of a CBA
as it may, the delay in the filing of the CBA was sufficiently
explained, i.e., there was an inter-union conflict on who would In Rivera v. Espiritu, it was held:
succeed to the presidency of ILO?PHILS. The
CBA was registered by the DOLE only on May 4, 1990. It The assailed PAL-PALEA agreement was the result of
would be injudicious for us to assume, as what petitioner did, voluntary collective bargaining negotiations undertaken in the light of
that the said CBA was filed only on April 30, 1990, or five (5) the severe financial situation faced by the employer, with the peculiar
days before its registration, on the unsupported surmise that it and unique intention of not merely promoting industrial peace at PAL,
was done to suit the law that enjoins Regional Offices of DOLE but preventing the latter’s closure.  We find no conflict between said
to act upon an application for registration of a CBA within five agreement and Article 253-A of the Labor Code.  Article 253-A has a
(5) days from its receipt thereof. In the absence of any two-fold purpose.  One is to promote industrial stability and
substantial evidence that DOLE officials or personnel, in predictability.  Inasmuch as the agreement sought to promote
collusion with private respondent, had antedated the filing date industrial peace at PAL during its rehabilitation, said agreement
of the CBA, the presumption on regularity in the performance of satisfies the first purpose of Article 253-A.  The other is to assign
official functions holds. specific timetables wherein negotiations become a matter of right and
More importantly, non-compliance with the cited requirement. Nothing in Article 253-A, prohibits the parties from
procedural requirement should not adversely affect the waiving or suspending the mandatory timetables and agreeing on the
substantive validity of the CBA between ILO-PHILS and the remedies to enforce the same.
Transunion Corporation-Glassware Division covering the In the instant case, it was PALEA, as the exclusive
company's rank and file employees. A collective bargaining bargaining agent of PAL’s ground employees, that voluntarily entered
agreement is more than a contract. It is highly impressed with into the CBA with PAL.  It was also PALEA that voluntarily opted
public interest for it is an essential instrument to promote for the 10-year suspension of the CBA.  Either case was the union’s
industrial peace. Hence, it bears the blessings not only of the exercise of its right to collective bargaining.  The right to free
employer and employees concerned but even the Department of collective bargaining, after all, includes the right to suspend it.
Labor and Employment. To set it aside on technical grounds is The acts of public respondents in sanctioning the 10-year
not conducive to the public good. suspension of the PAL-PALEA CBA did not contravene the
“protection to labor” policy of the Constitution.  The agreement
F. DURATION OF THE CBA; RETROACTIVITY afforded full protection to labor; promoted the shared responsibility
between workers and employers; and the exercised voluntary modes
in settling disputes, including conciliation to foster industrial peace."
Article 265. [253-A] Terms of a Collective Bargaining
Agreement. Any Collective Bargaining Agreement that the parties may
3. Extension of CBA
enter into shall, insofar as the representation aspect is concerned, be
for a term of five (5) years. No petition questioning the majority status While the parties may agree to extend the CBA's original
of the incumbent bargaining agent shall be entertained and no five-year term together with all other CBA provisions, any such
certification election shall be conducted by the Department of Labor amendment or term in excess of five years will not carry with it a
and Employment outside of the sixty-day period immediately before change in the union's exclusive collective bargaining status. By
the date of expiry of such five-year term of the Collective Bargaining express provision of the above-quoted Article 253-A, the exclusive
Agreement. All other provisions of the Collective Bargaining bargaining status cannot go beyond five years and the
Agreement shall be renegotiated not later than three (3) years after its representation status is a legal matter not for the workplace parties
execution. Any agreement on such other provisions of the Collective
to agree upon. In other words, despite an agreement for a CBA
Bargaining Agreement entered into within six (6) months from the date

Page 36 of 117
with a life of more than five years, either as an original provision or Personnel policies are guiding principles stated in broad,
by amendment, the bargaining union's exclusive bargaining status long-range terms that express the philosophy or beliefs of an
is effective only for five years and can be challenged within sixty organization’s top authority regarding personnel matter. They deal
(60) days prior to the expiration of the CBA's first five years [FVC with matters affecting efficiency and well-being of employees and
Labor Union v. SANAMA-FVC-SIGLO]. include, among others, the procedures in the administration of
wages, benefits, promotions, transfers and other personnel
4. Retroactivity movements which are usually not spelled out in the collective
In general, a CBA negotiated within six months after the agreement. The usual sources of grievances, however, are the rules
expiration of the existing CBA retroacts to the day immediately and regulations governing disciplinary actions [Maneja v. NLRC].
following such date and if agreed thereafter, the effectivity depends
on the agreement of the parties. 18 On the other hand, the law is Sanyo Philippines Workers’ Union v. Canizares
silent as to the retroactivity of a CBA arbitral award or that granted Held: In the instant case, however, We hold that the Labor
not by virtue of the mutual agreement of the parties but by Arbiter and not the Grievance Machinery provided for in the CBA has the
intervention of the government. Despite the silence of the law, the jurisdiction to hear and decide the complaints of the private respondents.
Court rules herein that CBA arbitral awards granted after six While it appears that the dismissal of the private respondents was made
upon the recommendation of PSSLU pursuant to the union security clause
months from the expiration of the last CBA shall retroact to such
provided in the CBA, We are of the opinion that these facts do not come
time agreed upon by both employer and the employees or their within the phrase "grievances arising from the interpretation or
union. Absent such an agreement as to retroactivity, the award shall implementation of (their) Collective Bargaining Agreement and those
retroact to the first day after the six-month period following the arising from the interpretation or enforcement of company personnel
expiration of the last day of the CBA should there be one policies," the jurisdiction of which pertains to the Grievance Machinery or
[MERALCO v. Quisimbing]. thereafter, to a voluntary arbitrator or panel of voluntary arbitrators. Article
260 of the Labor Code on grievance machinery and voluntary arbitrator
states that "(t)he parties to a Collective Bargaining Agreement shall include
IV
therein provisions that will ensure the mutual observance of its terms and
DISPUTE RESOLUTION conditions. They shall establish a machinery for the adjustment and
resolution of grievances arising from the interpretation or implementation
A. GRIEVANCE PROCEDURE of their Collective Bargaining Agreement and those arising from the
interpretation or enforcement of company personnel policies". It is further
Article 273. [260] Grievance Machinery and Voluntary provided in said article that the parties to a CBA shall name or designate
Arbitration. - The parties to a Collective Bargaining Agreement shall their respective representatives to the grievance machinery and if the
include therein provisions that will ensure the mutual observance of its grievance is not settled in that level, it shall automatically be referred to
terms and conditions. They shall establish a machinery for the voluntary arbitrators (or panel of voluntary arbitrators designated in
adjustment and resolution of grievances arising from the interpretation advance by the parties. It need not be mentioned that the parties to a CBA
or implementation of their Collective Bargaining Agreement and those are the union and the company. Hence, only disputes involving the union
arising from the interpretation or enforcement of company personnel and the company shall be referred to the grievance machinery or voluntary
policies. arbitrators.
All grievances submitted to the grievance machinery which In the instant case, both the union and the company are united or
are not settled within seven (7) calendar days from the date of its have come to an agreement regarding the dismissal of private respondents.
submission shall automatically be referred to voluntary arbitration No grievance between them exists which could be brought to grievance
prescribed in the Collective Bargaining Agreement. machinery. The problem or dispute in the present case is between the union
For this purpose, parties to a Collective Bargaining and the company on the one hand and some union and non-union members
Agreement shall name and designate in advance a Voluntary who were dismissed, on the other hand. The dispute has to be settled before
Arbitrator or panel of Voluntary Arbitrators, or include in the an impartial body. The grievance machinery with members designated by
agreement a procedure for the selection of such Voluntary Arbitrator the union and the company cannot be expected to be impartial against the
or panel of Voluntary Arbitrators, preferably from the listing of dismissed employees. Due process demands that the dismissed workers
qualified Voluntary Arbitrators duly accredited by the Board. In case grievances be ventilated before an impartial body. Since there has already
the parties fail to select a Voluntary Arbitrator or panel of Voluntary been an actual termination, the matter falls within the jurisdiction of the
Arbitrators, the Board shall designate the Voluntary Arbitrator or Labor Arbiter.
panel of Voluntary Arbitrators, as may be necessary, pursuant to the
selection procedure agreed upon in the Collective Bargaining 2. Grievance Machinery and Grievance Procedure
Agreement, which shall act with the same force and effect as if the Grievance machinery refers to the mechanism for the
Arbitrator or panel of Arbitrators have been selected by the parties as adjustment and resolution of grievances arising from the
described above.
interpretation or implementation of a CBA and those arising from
the interpretation or enforcement of company personnel policies.
1. Grievance Additionally, it is also mandated to process, adjust and resolve
A grievance or grievable issue is any question raised by violations of the CBA which are not gross in character. It is part of
either the employer or the union regarding any of the following the continuing process of collective bargaining [Republic Savings
issues or controversies: Bank v. CIR].
Grievance procedure is the series of formal steps that
a. The interpretation or application of the CBA; parties to a CBA agreed to take for the adjustment of grievances or
b. The interpretation or enforcement of company questions arising out of the interpretation or implementation of the
personnel policies; CBA or company personnel policies including voluntary arbitration
c. Any claim by either party that the other party is as the terminal step. The grievance procedure provides the parties a
violating any provisions of the CBA or company first crack in addressing problems in the CBA administration and
personnel policies [IRR]; or its use is an essential requisite before a Voluntary Arbitrator can
d. Wage Distortion (Art. 124, RA 6727). take cognizance of the unresolved grievance. It usually consists of
a multi-step procedure starting from the discussion of the grievance
Personnel Policies between the employee and/or the union steward, on the one hand,
and the foreman and supervisor, on the other hand, and ending with

Page 37 of 117
the highest decision-making officials of the company, reflecting the consent of the representatives of the employer and the labor union
hierarchy of command or responsibility [NCMB Manual]. involved in a labor union dispute to submit their case for
The reference to a Grievance Machinery and Voluntary arbitration.
Arbitrators for the adjustment or resolution of grievances arising
from the interpretation or implementation of their CBA and those 1. Voluntary Arbitrator
arising from the interpretation or enforcement of company A Voluntary Arbitrator refers to:
personnel policies is mandatory. The law grants to voluntary
arbitrators original and exclusive jurisdiction to hear and decide all a. Any person who has been accredited by the National
unresolved grievances arising from the interpretation or Conciliation and Mediation Board as such; or
implementation of the Collective Bargaining Agreement and those b. Any person named or designated in the CBA by the
arising from the interpretation or enforcement of company parties as their Voluntary Arbitrator; or
personnel policies [Sanyo Philippines Workers’ Union v. c. One chosen by the parties with or without the
Canizares]. assistance of the NCMB, pursuant to a selection
procedure agreed upon in the CBA; or
3. Establishment of Grievance Machinery d. One appointed by the NCMB in case either of the
Section 1, Rule XIX of the IRR provides: parties to the CBA refuses to submit to voluntary
arbitration.
Section 1. Establishment of grievance machinery. –
The parties to a collective bargaining agreement shall establish a Based on the above definition, Voluntary Arbitrators may
machinery for the expeditious resolution of grievances arising be classified into two (2) kinds, namely:
from the interpretation or implementation of the collective
bargaining agreement and those arising from the interpretation
or enforcement of company personnel policies. Unresolved a. Permanent Arbitrator referring to the VA
grievances will be referred to voluntary arbitration and for this specifically named or designated in the CBA by the
purpose, parties to a collective bargaining agreement shall name parties as their Val and
and designate in advance a voluntary arbitrator or panel of b. Ad Hoc Arbitrator referring to the VA chosen by the
voluntary arbitrators, or include in the agreement a procedure parties in accordance with the established procedure
for the selection of such voluntary arbitrator or panel of in the CBA or the one appointed by the NCMB in
voluntary arbitrators, preferably from the listing of qualified case there is failure in the selection or in case either
voluntary arbitrators duly accredited by the Board.
of the parties to the CBA refuses to submit to
In the absence of applicable provision in the
collective bargaining agreement, a grievance committee shall be voluntary arbitration.
created within ten (10) days from signing of the collective
bargaining agreement. The committee shall be composed of at A Voluntary Arbitrator need not be an employee of the
least two (2) representatives each from the members of the government. He may be a private individual but authorized to
bargaining unit and the employer, unless otherwise agreed upon render arbitration services provided under labor laws. He is not
by the parties. The representatives from among the members of required to be a lawyer. Though he is not a part of the government,
the bargaining unit shall be designated by the union. he is a quasi-judicial officer by nature of his functions, whether
acting solely or by a panel.
4. Procedure in Handling Grievances
Section 2, Rule XIX of the IRR provides: 2. Submission for Voluntary Arbitration
Section 3, Rule XIX provides:
Section 2. Procedure in handling grievances. – In the
absence of a specific provision in the collective bargaining
Section 3. Submission to voluntary
agreement or existing company practice prescribing for the
arbitration. – Where grievance remains unresolved,
procedures in handling grievance, the following shall apply:
either party may serve notice upon the other of its
(a) An employee shall present this grievance or
decision to submit the issue to voluntary arbitration.
complaint orally or in writing to the shop steward. Upon receipt
The notice shall state the issue or issues to be
thereof, the shop steward shall verify the facts and determine
arbitrated, copy thereof furnished the board or the
whether or not the grievance is valid.
voluntary arbitrator or panel of voluntary arbitrators
(b) If the grievance is valid, the shop steward shall
named or designated in the collective bargaining
immediately bring the complaint to the employee’s immediate
agreement.
supervisor. The shop steward, the employee and his immediate
If the party upon whom the notice is
supervisor shall exert efforts to settle the grievance at their level.
served fails or refuses to respond favorably within
(c) If no settlement is reached, the grievance shall be
seven (7) days from receipt thereof, the voluntary
referred to the grievance committee which shall have ten (10)
arbitrator or panel of voluntary arbitrators designated
days to decide the case.
in the collective bargaining agreement shall
Where the issue involves or arises from the
commence voluntary arbitration proceedings. Where
interpretation or implementation of a provision in the collective
the collective bargaining agreement does not so
bargaining agreement, or from any order, memorandum, circular
designate, the board shall call the parties and appoint
or assignment issued by the appropriate authority in the
a voluntary arbitrator or panel of voluntary
establishment, and such issue cannot be resolved at the level of
arbitrators, who shall thereafter commence
the shop steward or the supervisor, the same may be referred
arbitration proceedings in accordance with the
immediately to the grievance committee.
proceeding paragraph.
In instances where parties fail to select a
B. VOLUNTARY ARBITRATION voluntary arbitrator or panel of voluntary arbitrators,
Voluntary arbitration refers to the mode of settling labor- the regional branch of the Board shall designate the
management disputes in which the parties select a competent, voluntary arbitrator or panel of voluntary arbitrators,
trained and impartial third person who is tasked to decide on the as may be necessary, which shall have the same force
merits of the case and whose decision is final and executory. It is a and effect as if the parties have selected the
third-party settlement of a labor dispute involving the mutual arbitrator.

Page 38 of 117
Section 5. Powers of voluntary arbitrator
3. Parties to a Proceeding with the Voluntary or panel of voluntary arbitrators. – The voluntary
Arbitrator arbitrator or panel of voluntary arbitrators shall have
the power to hold hearings, receive evidence and take
Well-entrenched is the rule that when a case does not
whatever action is necessary to resolve the issue/s
involve the parties to a CBA – the employer and the bargaining subject of the dispute.
union – it is not subject to voluntary arbitration. The voluntary arbitrator or panel of
While an individual employee or group of employees, voluntary arbitrators may conciliate or mediate to aid
without participation of the union, are granted the right to bring a the parties in reaching a voluntary settlement of the
grievance directly to the employer, they cannot submit the same dispute.
grievance, if unresolved by the employer, for voluntary arbitration
without the union’s approval and participation. Moreover, the VA has the power to issue a writ of
execution to enforce final decisions and in connection therwith, it
4. Jurisdiction of the Voluntary Arbitrator shall be his duty too see that his decision is fully satisfied, inquire
Section 4, Rule XIX of the IRR provides: into the correctness of the execution, consider supervening events
during execution, and determine every question of fact and law
Section 4. Jurisdiction of voluntary arbitrator or which may be involved in the execution.
panel of voluntary arbitrators. - The voluntary arbitrator or panel As a general proposition, an arbitrator is confined to the
of voluntary arbitrators shall have exclusive and original interpretation and application of the collective bargaining
jurisdiction to hear and decide all grievances arising from the agreement. He does not sit to dispense his own brand of industrial
implementation or interpretation of the collective bargaining
justice: his award is legitimate only in so far as it draws its essence
agreements and those arising from the interpretation or
enforcement of company personnel policies which remain
from the CBA,  i.e., when there is a rational nexus between the
unresolved after exhaustion of the grievance procedure. award and the CBA under consideration. It is said that an arbitral
They shall also have exclusive and original award does not draw its essence from the CBA; hence, there is an
jurisdiction, to hear and decide wage distortion issues arising unauthorized amendment or alteration thereof, if:
from the application of any wage orders in organized
establishments, as well as unresolved grievances arising from a. It is so unfounded in reason and fact;
the interpretation and implementation of the productivity b. It is so unconnected with the working and purpose
incentive programs under RA 6971.
of the agreement;
The National Labor Relations Commission, its
regional branches and Regional Directors of the Department of c. It is without factual support in view of its
Labor and Employment shall not entertain disputes, grievances language, its context, and any other indicia of the
or matters under the exclusive and original jurisdiction of the parties' intention;
voluntary arbitrator or panel of voluntary arbitrators and shall d. It ignores or abandons the plain language of the
immediately dispose and refer the same to the appropriate contract; 
grievance machinery or voluntary arbitration provided in the e. It is mistakenly based on a crucial assumption
collective bargaining agreement. which concededly is a nonfact; 
Upon agreement of the parties, any other labor
f. It is unlawful, arbitrary or capricious; and
dispute may be submitted to a voluntary arbitrator or panel of
voluntary arbitrators. Before or at any stage of the compulsory g. It is contrary to public policy. 
arbitration process, the parties may opt to submit their dispute to
voluntary arbitration. A CBA is more than a contract; it is a generalized code to
govern a myriad of cases which the draftsmen cannot wholly
The Voluntary Arbitrator or panel of Voluntary anticipate. It covers the whole employment relationship and
Arbitrators shall have exclusive and original jurisdiction over the prescribes the rights and duties of the parties. It is a system of
following cases: industrial self-government with the grievance machinery at the
very heart of the system. [41] The parties solve their problems by
a. Unresolved grievances arising from the molding a system of private law for all the problems which may
interpretation or implementation of the CBA except arise and to provide for their solution in a way which will generally
if grievance is a gross violation of the CBA (ULP) accord with the variant needs and desires of the parties [United
or if the grievance results in termination. Kimberly Clark Employees Union v. Kimberly Clark].
b. Unresolved grievances arising from the If the terms of a CBA are clear and have no doubt upon
interpretation or enforcement of personnel policies. the intention of the contracting parties, the literal meaning of its
c. Violations of the CBA which are not gross in stipulation shall prevail.  However, if, in a CBA, the parties
character. stipulate that the hirees must be presumed of employment
d. Other labor disputes, including unfair labor qualification standards but fail to state such qualification standards
practices and bargaining deadlocks, upon agreement in said CBA, the VA may resort to evidence extrinsic of the CBA
of the parties. to determine the full agreement intended by the parties. When a
e. National Interest cases. CBA may be expected to speak on a matter, but does not, its
f. Wage distortion issues arising from the application sentence imports ambiguity on that subject. The VA is not merely
of any wage orders in organized establishments. to rely on the cold and cryptic words on the face of the CBA but is
g. Unresolved grievances arising from the mandated to discover the intention of the parties. Recognizing the
interpretation and implementation of the inability of the parties to anticipate or address all future problems,
Productivity Incentive Programs under RA 6971. gaps may be left to be filled in by reference to the practices of the
industry, and the step which is equally a part of the CBA although
5. Powers of the Voluntary Arbitrator not expressed in it. In order to ascertain the intention of the
Section 5, Rule XIX of the IRR provides: contracting parties, their contemporaneous and subsequent acts
shall be principally considered. The VA may also consider and rely
upon negotiating and contractual history of the parties, evidence of

Page 39 of 117
past practices interpreting ambiguous provisions. The VA has to the voluntary arbitrator to refuse or fail to turn over to the board,
examine such practices to determine the scope of their for its further disposition, the records of the case within ten (10)
agreement, as where the provision of the CBA has been loosely calendar days from demand thereof.
formulated. Moreover, the CBA must be construed liberally rather
than narrowly and technically and the Court must place a practical 7. Costs of Voluntary Arbitrator
and realistic construction upon it [Ibid]. Section 9, Rule XIX of the IRR provides:

Section 9. Cost of voluntary arbitration and voluntary


Goya, Inc. v. Goya Inc. Employees Union arbitrator’s fee. – The parties to a collective bargaining
Held: Generally, the arbitrator is expected to decide only those agreement shall provide therein a proportionate sharing scheme
questions expressly delineated by the submission agreement. Nevertheless, on the cost of voluntary arbitration including the voluntary
the arbitrator can assume that he has the necessary power to make a final arbitrator’s fee. The fixing of fee of voluntary arbitrators or
settlement since arbitration is the final resort for the adjudication of panel of voluntary arbitrators, whether shouldered wholly by the
disputes. The succinct reasoning enunciated by the CA in support of its parties or subsidized by the Special Voluntary Arbitration Fund,
holding, that the Voluntary Arbitrator in a labor controversy has jurisdiction shall take into account the following factors:
to render the questioned arbitral awards, deserves our concurrence, thus: (a) Nature of the case;
In general, the arbitrator is expected to decide those questions (b) Time consumed in hearing the case;
expressly stated and limited in the submission agreement. However, since (c) Professional standing of the voluntary arbitrator;
arbitration is the final resort for the adjudication of disputes, the arbitrator (d) Capacity to pay of the parties; and
can assume that he has the power to make a final settlement. Thus, (e) Fees provided for in the Revised Rules of Court.
assuming that the submission empowers the arbitrator to decide whether an Unless the parties agree otherwise, the cost of
employee was discharged for just cause, the arbitrator in this instance can voluntary arbitration proceedings and voluntary arbitrator’s fee
reasonably assume that his powers extended beyond giving a yes-or-no shall be shared equally by the parties
answer and included the power to reinstate him with or without back pay. Parties are encouraged to set aside funds to answer
In one case, the Supreme Court stressed that “xxx the Voluntary for the cost of voluntary arbitration proceedings including
Arbitrator had plenary jurisdiction and authority to interpret the agreement voluntary arbitrator’s fee. In the event the said funds are not
to arbitrate and to determine the scope of his own authority subject only, in sufficient to cover such expenses, an amount by way of subsidy
a proper case, to the certiorari jurisdiction of this Court. The Arbitrator, as taken out of the Special Voluntary Arbitration fund may be
already indicated, viewed his authority as embracing not merely the availed of by either or both parties subject to the guidelines on
determination of the abstract question of whether or not a performance voluntary arbitration to be issued by the Secretary.
bonus was to be granted but also, in the affirmative case, the amount
thereof.
By the same token, the issue of regularization should be viewed
8. Finality of the Award or Decision
as two-tiered issue. While the submission agreement mentioned only the Section 7, Rule XIX of the IRR provides:
determination of the date or regularization, law and jurisprudence give the
voluntary arbitrator enough leeway of authority as well as adequate Section 7. Finality of Award/Decision. –
prerogative to accomplish the reason for which the law on voluntary The decision, order, resolution or award of the
arbitration was created – speedy labor justice. It bears stressing that the voluntary arbitrator or panel of voluntary arbitrators
underlying reason why this case arose is to settle, once and for all, the shall be final and executory after ten (10) calendar
ultimate question of whether respondent employees are entitled to higher days from receipt of the copy of the award or
benefits. To require them to file another action for payment of such benefits decision by the parties and it shall not be subject of a
would certainly undermine labor proceedings and contravene the motion for reconsideration.
constitutional mandate providing full protection to labor.
Indubitably, Ludo fortifies, not diminishes, the soundness of the The 10-day period stated in Article 276 should be
questioned VA Decision. Said case reaffirms the plenary jurisdiction and understood as the period within which the party adversely affected
authority of the voluntary arbitrator to interpret the CBA and to determine by the ruling of the Voluntary Arbitrators or Panel of Arbitrators
the scope of his/her own authority. Subject to judicial review, the leeway of
may file a motion for reconsideration. Only after the resolution of
authority as well as adequate prerogative is aimed at accomplishing the
rationale of the law on voluntary arbitration – speedy labor justice. In this
the motion for reconsideration may the aggrieved party appeal to
case, a complete and final adjudication of the dispute between the parties the CA by filing the petition for review under Rule 43 of the Rules
necessarily called for the resolution of the related and incidental issue of of Court within 15 days from notice pursuant to Section 4 of Rule
whether the Company still violated the CBA but without being guilty of 43 [Guagua National Colleges v. Court of Appeals].
ULP as, needless to state, ULP is committed only if there is gross violation
of the agreement. Chin v. Maersk-Filipinas Crewing Inc.
Held: In the 2018 case of Guagua National Colleges vs.
6. Procedure in Voluntary Arbitration CA (Guagua), the Court acknowledged the variance in its rulings and
Section 6, Rule XIX of the IRR provides: categorically declared that the correct period to appeal the decision or
award of the Voluntary Arbitrator or Panel of Arbitrators to the CA via a
Section 6. Procedure. – All parties to the dispute petition for review under Rule 43 of the Rules of Court is the fifteen (15)-
shall be entitled to attend the arbitration proceedings. The day period set forth in Section 4 thereof reckoned from notice or receipt of
attendance of any third party or the exclusion of any witness the VA's resolution on the motion for reconsideration, and that the ten (10)-
from the proceedings shall be determined by the voluntary day period provided in Article 276 of the Labor Code refers to the period
arbitrator or panel of voluntary arbitrators. Hearing may be within which an aggrieved party may file said motion for reconsideration,
adjourned for cause or upon agreement by the parties. to wit:
Unless the parties agree otherwise, it shall be Given the variable rulings of the Court, what should now be the
mandatory for the voluntary arbitrator or panel of voluntary period to be followed in appealing the decisions or awards of the Voluntary
arbitrators to render an award or decision within twenty (20) Arbitrators or Panel of Arbitrators?
calendar days from the date of submission for resolution. In the 2010 ruling in Teng v. Pagahac, the Court clarified that
Failure on the part of the voluntary arbitrator to the 10-day period set in Article 276 of the Labor Code gave the aggrieved
render a decision, resolution, order or award within the parties the opportunity to file their motion for reconsideration, which was
prescribed period, shall upon complaint of a party, be sufficient more in keeping with the principle of exhaustion of administrative
ground for the Board to discipline said voluntary arbitrator, remedies, holding thusly:
pursuant to the guidelines issued by the Secretary. In cases that In the exercise of its power to promulgate implementing rules
the recommended sanction is de-listing, it shall be unlawful for and regulations, an implementing agency, such as the Department of Labor,

Page 40 of 117
is restricted from going beyond the terms of the law it seeks to implement; UNFAIR LABOR PRACTICE
it should neither modify nor improve the law. The agency formulating the
rules and guidelines cannot exceed the statutory authority granted to it by
the legislature. V
By allowing a 10-day period, the obvious intent of Congress UNFAIR LABOR PRACTICE
in amending Article 263 to Article 262-A is to provide an opportunity
for the party adversely affected by the VA's decision to seek recourse A. CONCEPT OF UNFAIR LABOR PRACTICE
via a motion for reconsideration or a petition for review under Rule 43
of the Rules of Court filed with the CA. Indeed, a motion for
Article 258. [247] Concept of Unfair Labor Practice and
reconsideration is the more appropriate remedy in line with the
Procedure for Prosecution Thereof. - Unfair labor practices violate the
doctrine of exhaustion of administrative remedies. For this reason, an
constitutional right of workers and employees to self-organization, are
appeal from administrative agencies to the CA via Rule 43 of the Rules
inimical to the legitimate interests of both labor and management,
of Court requires exhaustion of available remedies as a condition
including their right to bargain collectively and otherwise deal with
precedent to a petition under that Rule.
each other in an atmosphere of freedom and mutual respect, disrupt
The requirement that administrative remedies be exhausted is
industrial peace and hinder the promotion of healthy and stable labor-
based on the doctrine that in providing for a remedy before an
management relations.
administrative agency, every opportunity must be given to the agency to
Consequently, unfair labor practices are not only violations
resolve the matter and to exhaust all opportunities for a resolution under the
of the civil rights of both labor and management but are also criminal
given remedy before bringing an action in, or resorting to, the courts of
offenses against the State which shall be subject to prosecution and
justice. Where Congress has not clearly required exhaustion, sound judicial
punishment as herein provided.
discretion governs, guided by congressional intent.
Subject to the exercise by the President or by the Secretary
By disallowing reconsideration of the VA's decision, Section
of Labor and Employment of the powers vested in them by Articles 263
7, Rule XIX of DO 40-03 and Section 7 of the 2005 Procedural
and 264 of this Code,201 the civil aspects of all cases involving unfair
Guidelines went directly against the legislative intent behind Article
labor practices, which may include claims affirmative relief, shall be
262-A of the Labor Code. These rules deny the VA the chance to
under the jurisdiction of the Labor Arbiters. The Labor Arbiters shall
correct himself and compel the courts of justice to prematurely
give utmost priority to the hearing and resolution of all cases involving
intervene with the action of an administrative agency entrusted with
unfair labor practices. They shall resolve such cases within thirty (30)
the adjudication of controversies coming under its special knowledge,
calendar days from the time they are submitted for decision.
training and specific field of expertise. In this era of clogged court
Recovery of civil liability in the administrative proceedings
dockets, the need for specialized administrative agencies with the special
shall bar recovery under the Civil Code.
knowledge, experience and capability to hear and determine promptly
No criminal prosecution under this Title may be instituted
disputes on technical matters or intricate questions of facts, subject to
without a final judgment finding that an unfair labor practice was
judicial review, is indispensable. In Industrial Enterprises, Inc. v. Court of
committed, having been first obtained in the preceding paragraph.
Appeals, we ruled that relief must first be obtained in an administrative
During the pendency of such administrative proceeding, the running of
proceeding before a remedy will be supplied by the courts even though the
the period of prescription of the criminal offense herein penalized shall
matter is within the proper jurisdiction of a court. (Emphasis supplied)
be considered interrupted: Provided, however, That the final judgment
Hence, the 10-day period stated in Article 276 should be
in the administrative proceedings shall not be binding in the criminal
understood as the period within which the party adversely affected by the
case nor be considered as evidence of guilt but merely as proof of
ruling of the Voluntary Arbitrators or Panel of Arbitrators may file a
compliance of the requirements therein set forth.
motion for reconsideration. Only after the resolution of the motion for
reconsideration may the aggrieved party appeal to the CA by filing the
petition for review under Rule 43 of the Rules of Court within 15 days  ULP relates to the commission of acts that transgress the
from notice pursuant to Section 4 of Rule 43. (Citations omitted; workers’ right to organize [T&H v. T&H Union]. 
emphasis and underscoring supplied) At the outset, it must be clarified that not all unfair acts
The Court further noted in Guagua that despite the clarification constitute ULPs. While an act or decision of an employer or a
made in Teng v. Pagahac in 2010, the Department of Labor and union may be unfair, certainly not every unfair act or decision
Employment (DOLE) and NCMB have yet to revise or amend Section
thereof may constitute ULP as defined and enumerated under
7, Rule VII of the Revised Procedural Guidelines in the Conduct of
Voluntary Arbitration Proceedings and that such inaction has caused Articles 259 and 260 of the Labor Code [Galaxie Steel Workers
confusion, particularly with respect to the filing of the motion for Union v. NLRC].
reconsideration as a condition precedent to the filing of the petition for The act complained of as ULP must have a proximate
review in the CA. Thus, the Court expressly directed the DOLE and the and casual connection with the following:
NCMB to cause the revision or amendment of the aforesaid section in order
to allow the filing of motions for reconsideration in line with Article 276 of a. Exercise of the right to self-organization;
the Labor Code. Unfortunately, no revision has yet been made in this b. Exercise of the right to collective bargaining; or
regard. Consequently, the DOLE and the NCMB are again reminded to
c. Compliance with the CBA.
cause the revision or amendment of Section 7, Rule VII of the Revised
Procedural Guidelines in the Conduct of Voluntary Arbitration Proceedings
insofar as it prohibits the filing of a motion for reconsideration, if they have Sans this connection, the unfair acts do not fall within the
not done so. technical signification of the term unfair labor practice [Allied
In view of the foregoing, petitioner in this case had fifteen (15) Banking v. Court of Appeals].
days from receipt of the Resolution denying his motion for reconsideration The only ULP which is the exception as it may or may
to file his petition for review with the CA. Having received a copy of the not relate to the exercise of the right to self-organization and
VA's October 29, 2018 Resolution on November 22, 2018, petitioner collective bargaining is the act described under paragraph (f) of
therefore had until December 7, 2018 to file his petition. As the records
Article 259, i.e., to dismiss, discharge or otherwise prejudice or
show that the petition was filed on December 4, 2018, albeit through a
private courier, it was therefore timely filed and the CA erred in dismissing discriminate against an employee for having given or being about
it outright. To rule otherwise would be clearly antithetical to the tenets of to give testimony under the Labor Code [Philcom Employees
fair play, not to mention the undue prejudice to petitioner's rights. [30] Thus, Union v. Philippine Global Communications].
in light of the fact that the CA dismissed the petition for review outright
based solely on procedural grounds, a remand of the case for a resolution 1. Aspects of ULP
on the merits is warranted. Under Article 258, an Unfair Labor Practice has two (2)
aspects, namely:
V
Page 41 of 117
a. Civil aspect; and Code.” If an act is not covered by any of the ULPs expressly
b. Criminal aspect. mentioned in the law, it cannot be so deemed a ULP act.

The civil aspect of ULP includes claims for actual, moral 5. Closure of Business
and exemplary damages, attorney’s fees and other affirmative  In the case of Chronicle Securities Corp. v. NLRC, we
reliefs. Generally, these civil claims should be asserted in the labor ruled that even an employer who is “found guilty of unfair labor
case before the Labor Arbiters who have original and exclusive practice in dismissing his employee may not be ordered so to pay
jurisdiction over ULP cases. the criminal aspect, on the other hand, backwages beyond the date of closure of business where such
can only be asserted before the regular courts. closure was due to legitimate business reasons and not merely an
attempt to defeat the order of reinstatement.”
2. Labor Code Provisions
There are only five (5) articles in the Labor Code related 6. Prescription
to ULP, to wit:
Article 305. [290] Offenses. Offenses penalized under this
a. Article 258 which describes the concept of ULPs Code and the rules and regulations issued pursuant thereto shall
and prescribes the procedure for their prosecution; prescribe in three (3) years.
b. Article 259 which enumerates the ULPs that may be All unfair labor practice arising from Book V shall be filed
with the appropriate agency within one (1) year from accrual of such
committed by employers;
unfair labor practice; otherwise, they shall be forever barred.
c. Article 260 which enumerates the ULPs that may be
committed by labor organizations;
B. ULP IN COLLECTIVE BARGAINING
d. Article 274 which considers violations of the CBA
as no longer ULPs unless the same are gross in
1. Bad Faith Bargaining
character which means flagrant and/or malicious
It is essential that the employer and employees should
refusal to comply with the economic provisions
both act in good faith when it comes to collective bargaining. Good
thereof;
faith bargaining requires that claims made by either bargainer
e. Article 278(c) which refers to union-busting, a form
should be honest.
of ULP, involving the dismissal from employment
In Union of Filipro Employees v. Nestle Philippines, it
of union officers duly elected in accordance with the
was held:
union constitution and by-laws, where the existence
of the union is threatened thereby.
Obviously, the purpose of collective bargaining is the
reaching of an agreement resulting in a contract binding on the
3. Parties Who May Commit ULP parties; but the failure to reach an agreement after negotiations
An Unfair Labor Practice may be committed by an have continued for a reasonable period does not establish a lack
employer or by a labor organization. Article 259 describes the of good faith. The statutes invite and contemplate a collective
ULPs that may be committed by the employer; while Article 260 bargaining contract, but they do not compel one. The duty to
enumerates those which may be committed by the Labor bargain does not include the obligation to reach an agreement.
Organization. The crucial question, therefore, of whether or not a
party has met his statutory duty to bargain in good faith typically
On the part of the employer, only the officers and agents
turns on the facts of the individual case. As we have said, there
of corporations, associations or partnerships who have actually is no per se test of good faith in bargaining. Good faith or bad
participated in or authorized or ratified the ULPs are criminally faith is an inference to be drawn from the facts. To some degree,
liable. the question of good faith may be a question of credibility.
On the part of the union, only the officers, members of The effect of an employer’s or a union’s individual actions is not
governing boards, representatives or agents or members of the the test of good-faith bargaining, but the impact of all such
labor associations or organizations who have actually participated occasions or actions, considered as a whole, and the inferences
in or authorized or ratified the ULPs are criminally liable. fairly drawn therefrom collectively may offer a basis for the
finding of the NLRC.
For a charge of unfair labor practice to prosper, it
4. Elements of ULP must be shown that Nestlé was motivated by ill will, “bad faith,
Before an employer or labor organization may be said to or fraud, or was oppressive to labor, or done in a manner
have committed ULP, the following elements must concur: contrary to morals, good customs, or public policy, and, of
course, that social humiliation, wounded feelings, or grave
a. There should exist an employer-employee anxiety resulted x x x” in disclaiming unilateral grants as proper
relationship between the offended party and the subjects in their collective bargaining negotiations. While the
offender; and law makes it an obligation for the employer and the employees
to bargain collectively with each other, such compulsion does
b. The act complained of must be expressly mentioned
not include the commitment to precipitately accept or agree to
and defined in the Labor Code as a ULP. the proposals of the other. All it contemplates is that both parties
should approach the negotiation with an open mind and make
Absent one of the elements aforementioned will not reasonable effort to reach a common ground of agreement.
make the act an unfair labor practice. Herein, the union merely bases its claim of refusal to
The first requisite is necessary because ULP may only be bargain on a letter dated 29 May 2001 written by Nestlé where
committed in connection with the right to self-organization and the latter laid down its position that “unilateral grants, one-time
collective bargaining by employees. Necessarily, there must be an company grants, company-initiated policies and programs,
which include, but are not limited to the Retirement Plan,
employment relationship in order for the organizational right to be
Incidental Straight Duty Pay and Calling Pay Premium, are by
validly and lawfully invoked. their very nature not proper subjects of CBA negotiations and
The second requisite should be present since the Labor therefore shall be excluded therefrom.” But as we have stated in
Code itself requires that the ULP be “expressly defined by this this Court’s Decision, said letter is not tantamount to refusal to
bargain. In thinking to exclude the issue of Retirement Plan

Page 42 of 117
from the CBA negotiations, Nestlé, cannot be faulted for 4. Blue-Sky Bargaining
considering the same benefit as unilaterally granted, considering Blue-Sky Bargaining means making exaggerated or
that eight out of nine bargaining units have allegedly agreed to unreasonable proposals. This is a kind of ULP which can only be
treat the Retirement Plan as a unilaterally granted benefit. This
committed by a SEBA.
is not a case where the employer exhibited an indifferent attitude
towards collective bargaining, because the negotiations were not In Standard Chartered Bank Employees Union v.
the unilateral activity of the bargaining representative. Nestlé’s Confesor, the minutes of the meeting show that the SEBA based its
desire to settle the dispute and proceed with the negotiation economic proposals on data of rank-and-file employees and the
being evident in its cry for compulsory arbitration is proof prevailing economic benefits received by bank employees from
enough of its exertion of reasonable effort at good-faith other foreign banks doing business in the Philippines and other
bargaining. branches of the bank in the Asian region, hence, it cannot be said
In the case at bar, Nestle never refused to bargain that the union was guilty of ULP for blue-sky bargaining.
collectively with UFE-DFA-KMU. The corporation simply
wanted to exclude the Retirement Plan from the issues to be
taken up during CBA negotiations, on the postulation that such 5. Surface Bargaining
was in the nature of a unilaterally granted benefit. An Surface bargaining is defined as going through the
employer’s steadfast insistence to exclude a particular motions of negotiating without any legal intent to reach an
substantive provision is no different from a bargaining agreement. It is a form of ULP that may only be committed by the
representative’s perseverance to include one that they deem of employer. In the same case of Standard Chartered Bank Employees
absolute necessity. Indeed, an adamant insistence on a Union, it involves the question of whether an employer’s conduct
bargaining position to the point where the negotiations reach an demonstrates an unwillingness to bargain in good faith or is merely
impasse does not establish bad faith.[fn24 p.10] It is but natural
hard bargaining. There can be no surface bargaining, absent any
that at negotiations, management and labor adopt positions or
make demands and offer proposals and counter-proposals. On evidence that management had done acts, both at and away from
account of the importance of the economic issue proposed by the bargaining table, which tend to show that it did not want to
UFE-DFA-KMU, Nestle could have refused to bargain with the reach an agreement with the union or to settle the differences
former – but it did not. And the management’s firm stand between it and the union. Here, admittedly, the parties were not
against the issue of the Retirement Plan did not mean that it was able to agree and thus reached a deadlock. However, it must be
bargaining in bad faith. It had a right to insist on its position to emphasized that the duty to bargain does not compel either party to
the point of stalemate. agree to a proposal or require the making of a concession. Hence,
the parties’ failure to agree does not amount to ULP under Article
Boulwarism 259(g) [Standard v. Confesor].
Boulwarism is a negotiation tactic named after the former
vice president of General Electric, Lemuel Boulware, who C. ULP OF EMPLOYERS
pioneered this strategy. It is a labor law principle in which
management opens the negotiation with a generous effort that is
Article 259. [248] Unfair Labor Practices of Employers. - It
not meant to be negotiated. In other words, it is an offer which is shall be unlawful for an employer to commit any of the following
ultimate and to which no further revisions will be made. This “take unfair labor practices:
it or leave it” offer or counter-offer does not constitute proper (a) To interfere with, restrain or coerce employees in the
collective bargaining not only within the contemplation of United exercise of their right to self-organization;
States laws but also under Philippine law [Philippine National (b) To require as a condition of employment that a person or
Railways v. Union de Maquinistas]. an employee shall not join a labor organization or shall withdraw from
one to which he belongs;
(c) To contract out services or functions being performed by
2. Refusal to Bargain union members when such will interfere with, restrain or coerce
The failure of the employer to submit its counter- employees in the exercise of their right to self-organization;
proposals to the demands of the SEBA does not, by itself, (d) To initiate, dominate, assist or otherwise interfere with
constitute refusal to bargain as would amount to ULP [Philippine the formation or administration of any labor organization, including
Marine Radio Officers Association v. CIR]. However, it is different the giving of financial or other support to it or its organizers or
if the employer refuses to submit an answer or reply to the written supporters;
bargaining proposals of the SEBA. In this case, unfair labor (e) To discriminate in regard to wages, hours of work and
other terms and conditions of employment in order to encourage or
practice is committed. While the law does not compel the parties to
discourage membership in any labor organization. Nothing in this
reach an agreement, it does not contemplate that both parties will Code or in any other law shall stop the parties from requiring
approach the negotiation with an open mind and make a reasonable membership in a recognized collective bargaining agent as a condition
effort to reach a common ground of agreement [Kiok Loy v. for employment, except those employees who are already members of
NLRC]. another union at the time of the signing of the collective bargaining
Other examples of ULP are: (a) stonewalling, or the agreement. Employees of an appropriate bargaining unit who are not
refusal to communicate or cooperate; and (b) giving flimsy excuses members of the recognized collective bargaining agent may be assessed
for the delay in the negotiation. a reasonable fee equivalent to the dues and other fees paid by members
of the recognized collective bargaining agent, if such non-union
members accept the benefits under the collective bargaining
3. Individual Bargaining agreement: Provided, That the individual authorization required
To negotiate or attempt to negotiate with individual under Article 250, paragraph (o) of this Code shall not apply to the
workers rather than with the SEBA is ULP. For instance, the act of non-members of the recognized collective bargaining agent;
the employer in notifying absent employees individually during a (f) To dismiss, discharge or otherwise prejudice or
strike following unproductive efforts at collective bargaining that discriminate against an employee for having given or being about to
the plant would be operated the next day and that their jobs were give testimony under this Code;
open for them should they want to come in has been held to be a (g) To violate the duty to bargain collectively as prescribed
by this Code;
ULP, an active inference with the right of collective bargaining
(h) To pay negotiation or attorney’s fees to the union or its
through dealing with the employees individually instead of through officers or agents as part of the settlement of any issue in collective
their collective bargaining representative. bargaining or any other dispute; or

Page 43 of 117
(i) To violate a collective bargaining agreement. b. The act of the employer in presenting the letters by
The provisions of the preceding paragraph notwithstanding, 13 union members signifying resignation from the
only the officers and agents of corporations, associations or union clearly indicative of the employer’s pressure
partnerships who have actually participated in, authorized or ratified on its employees to prove that the union no longer
unfair labor practices shall be held criminally liable.
enjoyed the support of the workers [General Milling
v. Court of Appeals].
In summarized form, the nine U.L.P. acts of an employer c. The employer’s refusal to bargain to its acts of
under Article 248 are: (1) interference, (2) fellow dog" condition, economic inducements resulting in the promotion of
(3) contracting out, (4) company unionism, (5) discrimination for those who withdrew from the union, the use of
or against union membership, (6) discrimination because of armed guards to prevent the organizers to come in,
testimony, (7) violation of duty to bargain, (8) paid negotiation, and the dismissal of union officials and members
and (9) violation of CBA. [Hacienda Fatima v. National Federation of
It is the union which has the burden of proof to present Sugarcane Workers].
substantial evidence to support its allegations of unfair labor d. Termination of employee in order to strip the union
practices committed by the employer. of a leader who would fight for the right of her co-
By the very nature of an unfair labor practice, it is not workers at the bargaining table [Colegio de San
only a violation of the civil rights of both labor and management Juan de Letran v. Association of Employees and
but is also a criminal offense against the State which is subject to Faculty of Letran].
prosecution and punishment [Hongkong and Shang Hai Banking v. e. Dismissing a supervisory employee on account of
NLRC]. his union activities related to the formation of the
supervisory union [Cathay Pacific Steel Corp. v.
1. Interference Court of Appeals].
The employer commits interference by way of speech f. Employer assigns active union members to do
when it contains a promise of a reward or a threat of reprisal. This overtime work on the day of the union meeting to
kind of speech is no longer protected by the Constitutional right to prevent them from attending.
free speech except if done in the Conciliation Proceedings in the g. The employer unilaterally closing its establishment
NCMB under Article 239. on the pretext that the demands of its employees are
According to the case of Insular Life Co., Ltd., excessive [St John’s Colleges v. St. Johns Academy
Employees Association v. Insular Life, the test of the employer’s Faculty and Employees Union]. 
interference with, restraint or coercion of employees within the h. Offer of reinstatement and attempt to bribe the
meaning of the law is whether the employer has engaged in strikers with comfortable cots, free coffee and
conduct which may reasonably tend to interfere with the free occasional movies, overtime pay for work
exercise of the employees’ twin rights to self-organization and performed in excess of 8 hours and offer of
collective bargaining. It is not necessary that there be direct arrangements for their families so they would
evidence that any employee was in fact restrained, intimidated or abandon the strike and return to work, which
coerced by the statements or threats of the employer; what matters constitute strike-breaking which is a ULP [Insular
is that there is a reasonable inference that the anti-union conduct of Life Assurance Co., Ltd., Employees Association v.
the employer does have an adverse effect on the exercise of said Insular Life Assurance].
rights. i. Dismissal of employees after they have organized
their union and about to start with the effort at
Totality of Conduct Doctrine having it certified as their SEBA [Samahan ng
The totality of conduct doctrine means that expressions Manggagawa sa Bandolino v. NLRC].
of opinion by an employer, though innocent by themselves, may be j. Transferring, laying off or assigning employees
held to constitute ULP because of the circumstances under which more difficult work or tasks, or otherwise punishing
they were uttered, the history of the particular employer’s labor them because they engaged in organizing and
relations or anti-union bias or because of their connection with an forming a union [T&H Shopfitters Corporation/Gin
established collateral plan of coercion or interference. An Queen Corp. v. T&H Shopfitters/Gin Queen
expression which may be permissibly uttered by one employer Workers Union].
might, in the mouth of a more hostile employer, be deemed k. Establishing a savings account for the union where
improper and consequently actionable as ULP [Samahan ng all collected union dues and agency fees will be
Manggagawa sa Bandolino v. NLRC]. The past conduct of the deposited and held in trust and discontinuing normal
employer and like considerations, coupled with an intimate relations with any group within the union including
connection between the employer’s action and the union affiliation the incumbent sets of officers at the height of an
or activities of the particular employee or employees taken as a intra-union dispute involving the election of officers
whole, may raise a suspicion as to the motivation for the of the union [De La Salle University v. De La Salle
employer’s conduct. The failure of the employer to ascribe a valid University Employees Association].
reason therefor may justify an inference that his unexplained l. In RMC Faculty Union v. NLRC, it was established
conduct in respect of the particular employee or employees was that said teachers were permanent employees who
inspired by the latter’s union membership and activities [Royal had rendered six (6) to twenty (20) years of service.
Undergarment Corporation v. CIR]. Their permanent status notwithstanding, they were
Examples are: dismissed because Abellera feared that if their
contracts were renewed, there would be a strike in
a. Interference in the choice of union’s bargaining the school the following semester. This is
panel [Standard Chartered Bank Employees Union indisputably an unwarranted interference with the
v. Confesor]. right of workers to self-organization and to engage
in concerted activities. An apprehension that there

Page 44 of 117
might be a future strike in the school is not a ground Held: It is to be emphasized that contracting out of services is
for dismissal of the workers. While a strike may not illegal perse. It is an exercise of business judgment or management
result in hardships or prejudice to the school and the prerogative. Absent proof that the management acted in a malicious or
studentry, the employer is not without recourse. If arbitrary manner, the Court will not interfere with the exercise of judgment
by an employer. In this case, bad faith cannot be attributed to BPI because
the employer feels that the action is tainted with
its actions were authorized by CBP Circular No. 1388, Series of
illegality, the law provides the employer with ample 1993 issued by the Monetary Board of the then Central Bank of the
remedies to protect his interests. Decidedly, Philippines (now Bangko Sentral ng Pilipinas). The circular covered
dismissal of employees in anticipation of an amendments in Book I of the Manual of Regulations for Banks and Other
exercise of a constitutionally protected right is not Financial Intermediaries, particularly on the matter of bank service
one of them. contracts. A finding of ULP necessarily requires the alleging party to prove
it with substantial evidence. Unfortunately, the Union failed to discharge
2. Yellow Dog Contract this burden.
Much has been said about the applicability of D.O. No. 10. Both
Paragraph [b] of Article 259 describes what is commonly
the NLRC and the CA agreed with BPI that the said order does not apply.
known as yellow dog contract. It is one which exacts from workers With BPI, as a commercial bank, its transactions are subject to the rules and
as a condition of employment that they shall not join or belong to a regulations of the governing agency which is the Bangko Sentral ng
labor organization, or attempt to organize one during their period Pilipinas. The Union insists that D.O. No. 10 should prevail.
of employment or that they shall withdraw therefrom in case they The Court is of the view, however, that there is no conflict
are already members of a labor organization. between D.O. No. 10 and CBP Circular No. 1388. In fact, they complement
A typical yellow dog contract embodies the following each other.
stipulations: Consistent with the maxim, interpretare et concordare leges
legibus est optimus interpretandi modus, a statute should be construed not
only to be consistent with itself but also to harmonize with other laws on
(a) A representation by the employee that he is not a the same subject matter, as to form a complete, coherent and intelligible
member of a labor organization; system of jurisprudence. The seemingly conflicting provisions of a law or
(b) A promise by the employee that he will not join a of two laws must be harmonized to render each effective. It is only when
union; and harmonization is impossible that resort must be made to choosing which
(c) A promise by the employee that upon joining a law to apply.
labor organization, he will quit his employment. In the case at bench, the Union submits that while the Central
Bank regulates banking, the Labor Code and its implementing rules
regulate the employment relationship. To this, the Court agrees. The fact
The act of the employer in imposing such a condition
that banks are of a specialized industry must, however, be taken into
constitutes ULP. Such stipulation in the contract is null and void. account. The competence in determining which banking functions may or
may not be outsourced lies with the BSP. This does not mean that banks
3. Contracting Out of Services and Functions can simply outsource banking functions allowed by the BSP through its
This is also called “ULP Contracting.” Paragraph [c] of circulars, without giving regard to the guidelines set forth under D.O. No.
Article 259 describes when the act of the employer of contracting 10 issued by the DOLE.
out of services or functions being performed by SEBA members is While D.O. No. 10, Series of 1997, enumerates the permissible
considered ULP. contracting or subcontracting activities, it is to be observed that,
particularly in Sec. 6(d) invoked by the Union, the provision is general in
As a general rule, the act of an employer in having work
character – "x x x Works or services not directly related or not integral to
or certain services or functions being performed by SEBA the main business or operation of the principal… x x x." This does not limit
members contracted out is not per se ULP. This is so because or prohibit the appropriate government agency, such as the BSP, to issue
contracting-out of a job, work or service is clearly an exercise by rules, regulations or circulars to further and specifically determine the
the employer of its business judgment and its inherent management permissible services to be contracted out. CBP Circular No.
rights and prerogatives. Hiring of workers is within the employer’s 1388 enumerated functions which are ancillary to the business of banks,
inherent freedom to regulate its business and is a valid exercise of hence, allowed to be outsourced. Thus, sanctioned by said circular, BPI
its management prerogative subject only to special laws and outsourced the cashiering (i.e., cash-delivery and deposit pick-up) and
accounting requirements of its Davao City branches. The Union even
agreements on the matter and the fair standards of justice. The
described the extent of BPI’s actual and intended contracting out to BOMC
employer cannot be denied the faculty of promoting efficiency and as follows:
attaining economy by a study of what units are essential for its "As an initiatory move, the functions of the Cashiering Unit of
operation. It has the ultimate right to determine whether services the Processing Center of BPI, handled by its regular rank and file
should be performed by its personnel or contracted to outside employees who are members of the Union, xxx [were] transferred to
agencies [Manila Electric Company v. Quisumbing]. BOMC with the Accounting Department as next in line. The Distributing,
It is only when the contracting out of a job, work or Clearing and Bookkeeping functions of the Processing Center of the former
service being performed by the SEBA members will interfere with, FEBTC were likewise contracted out to BOMC."
Thus, the subject functions appear to be not in any way directly
restrain or coerce employees in the exercise of their right to self-
related to the core activities of banks. They are functions in a processing
organization that it shall constitute ULP. center of BPI which does not handle or manage deposit transactions.
In Shell Oil Workers Union v. Shell Oil Company, the Clearly, the functions outsourced are not inherent banking functions, and,
Court ruled that the contracting out of security services to an thus, are well within the permissible services under the circular.
outside private security agency to undertake the work of the The Court agrees with BPI that D.O. No. 10 is but a guide to
company security guards who were re-assigned to other sections of determine what functions may be contracted out, subject to the rules and
the company, is violative of the existing CBA. It could have been established jurisprudence on legitimate job contracting and prohibited
purely an exercise of management prerogative on the part of the labor-only contracting. Even if the Court considers D.O. No. 10 only, BPI
would still be within the bounds of D.O. No. 10 when it contracted out the
company if it were not bound by what was stipulated in the CBA to
subject functions. This is because the subject functions were not related or
continue to maintain a security guard section at least during the not integral to the main business or operation of the principal which is the
lifetime of the agreement. lending of funds obtained in the form of deposits. From the very definition
of "banks" as provided under the General Banking Law, it can easily be
BPI Employees Union v. BPI discerned that banks perform only two (2) main or basic functions – deposit
and loan functions. Thus, cashiering, distribution and bookkeeping are but

Page 45 of 117
ancillary functions whose outsourcing is sanctioned under CBP Circular work premises to perform the warehousing activities on its own account.
No. 1388 as well as D.O. No. 10. Even BPI itself recognizes that deposit Similarly, the job contracted out is directly related to CEPALCO's electric
and loan functions cannot be legally contracted out as they are directly distribution business, which involves logistics, inventories, accounting,
related or integral to the main business or operation of banks. The CBP's billing services, and other related operations. Lastly, same as above, no
Manual of Regulations has even categorically stated and emphasized on the evidence has been offered to establish that CESCO exercised control with
prohibition against outsourcing inherent banking functions, which refer to respect to the manner and methods of achieving the warehousing works, or
any contract between the bank and a service provider for the latter to that it supervised the workers assigned to perform the same.
supply, or any act whereby the latter supplies, the manpower to service the At this juncture, it should be made clear that the disposition of
deposit transactions of the former. these cases should be limited only to the foregoing declaration. Again, the
complaints filed by respondent were only for ULP. While there is nothing
infirm in passing upon the matter of labor-only contracting since it was
CEPALCO v. CEPALCO Union
vigorously litigated in these proceedings, the resolution of the same must
Held: The need to determine whether or not the contracting out
only be read in relation to the charges of ULP. As earlier stated, labor-only
of services (or any particular activity or scheme devised by the employer
contracting was invoked by respondent as a prohibited act under Article
for that matter) was intended to defeat the workers' right to self-
259 (c) of the Labor Code, as amended. As it turned out, however,
organization is impelled by the underlying concept of ULP. This is stated in
respondent failed to relate the arrangement to the defining element of
Article 258 of the Labor Code, as amended, to wit:
ULP, i.e., that it violated the workers' right to self-organization. Hence,
Article 258. Concept of Unfair Labor Practice and Procedure
being a preliminary matter actively argued by respondent to prove the
for Prosecution Thereof. - Unfair labor practices violate the
charges of ULP, the same was not rendered moot and academic by the
constitutional right of workers and employees to self-organization, are
eventual dismissal of the complaints as an issue only becomes moot and
inimical to the legitimate interests of both labor and
academic if it becomes a "dead" issue, devoid of any practical value or use
management, including their right to bargain collectively and otherwise
to be passed upon. In Pormento v. Estrada:
deal with each other in an atmosphere of freedom and mutual respect,
An action is considered "moot" when it no longer presents a
disrupt industrial peace and hinder the promotion of healthy and stable
justiciable controversy because the issues involved have become academic
labor-management relations.
or dead or when the matter in dispute has already been resolved and hence,
Thus, in Great Pacific Employees Union v. Great Pacific Life
one is not entitled to judicial intervention unless the issue is likely to be
Assurance Corporation,69 the Court observed:
raised again between the parties. There is nothing for the court to resolve as
There should be no dispute that all the prohibited acts
the determination thereof has been overtaken by subsequent
constituting unfair labor practice in essence relate to the workers' right to
events.83chanroblesvirtuallawlibrary
self-organization. Thus, an employer may be held liable under this
For another, the Court also observes that while respondent did
provision if his conduct affects in whatever manner the right of an
ask for the nullification of the subject contracts between petitioners, and
employee to self-organize.
even sought that the employees provided by CESCO to CEPALCO be
Similarly, in Bankard, Inc. v. NLRC:
declared as the latter's own employees, petitioners correctly argue that
The Court has ruled that the prohibited acts considered as ULP
respondent is not a real party-in-interest and hence, had no legal standing
relate to the workers' right to self-organization and to the observance of a
insofar as these matters are concerned. This is because respondent failed to
CBA. It refers to "acts that violate the workers' right to organize."
demonstrate how it stands to be benefited or injured by a judgment on the
Without that element, the acts, even if unfair, are not ULP. Thus, an
same, or that any personal or direct injury would be sustained by it if these
employer may only be held liable for unfair labor practice if it can be
reliefs were not granted. In Joya v. Presidential Commission on Good
shown that his acts affect in whatever manner the right of his employees to
Government,84 the Court explained:
self-organize.
"Legal standing" means a personal and substantial interest in the
In these cases, the Court agrees with the CA that CEPALCO
case such that the party has sustained or will sustain direct injury as a result
was engaged in labor-only contracting as its Contract for Meter-Reading
of the x x x act being challenged. The term "interest" is material interest, an
Work dated February 19, 2007 and Contract of Service To Perform
interest in issue and to be affected by the decree, as distinguished from
Warehousing Works dated January 5, 2010 (subject contracts) with CESCO
mere interest in the question involved, or a mere incidental interest.
fit the criteria provided for in Section 5 of DO 18-02, as above-highlighted.
Moreover, the interest of the party plaintiff must be personal and not one
To be specific, petitioners failed to show that CESCO has
based on a desire to vindicate the constitutional right of some third and
substantial capital or investment which relates to the job, work or service to
unrelated party.85chanroblesvirtuallawlibrary
be performed. While it is true that: (a) CESCO's Amended Articles of
If at all, it would be the employees of CESCO who are entitled
Incorporation73 as of November 26, 2008 shows that CESCO's authorized
to seek the foregoing reliefs since in cases of labor-only contracting, "the
capital stock is P200,000,000.00 as of September 26, 2008, 74 which was
person or intermediary shall be considered merely as an agent of the
increased from P100,000,000.0075 on May 30, 2007; and (b) its financial
employer who shall be responsible to the workers in the same manner and
statement76 as of 2010 and 2011 shows that its paid-up capital stock is in the
extent as if the latter were directly employed by him."86 However, they have
sum of P81,063,000.00,77 there is no available document to show CESCO's
not been impleaded in these cases. Thus, as prayed for by petitioners, the
authorized capital stock at the time of the contracting out of
Court must set aside the portions of the assailed CA Decisions declaring:
CEPALCO's meter-reading activities to CESCO on February 19, 2007.
(a) the workers hired by CESCO, pursuant to the contracts subject of these
As it is, the increases in its authorized capital stock and paid-up capital
cases, as regular employees of CEPALCO; and (b) the latter responsible to
were only made after November 26, 2008, hence, are only relevant with
said workers in the same manner and extent as if they were directly
regard to the time CEPALCO contracted out its warehousing works to
employed by it. This pronouncement not only squares with the rules on real
CESCO on January 5, 2010. Since the amount of CESCO's authorized
party-in-interest and legal standing, but also with the precept that no one
capital stock at the time CEPALCO contracted out its meter-reading
shall be affected by any proceeding to which he is a stranger, and that
activities was not shown, the Court has no means of determining whether it
strangers to a case are not bound by any judgment rendered by the court.
had substantial capital at the time the contract therefor was entered into.
Furthermore, the list78 of CESCO's office equipment, furniture and fixtures,
and vehicles offered in evidence by petitioners does not satisfy the 4. Company Union
requirement that they could have been used in the performance of the "Company union" means any labor organization whose
specific work contracted out, i.e., meter-reading service. As the CA aptly formation, function or administration has been assisted by any act
pointed out,79 the tools and equipment utilized by CESCO in the meter- defined as unfair labor practice by this Code.
reading activities are owned by CEPALCO, emphasizing the fact that Paragraph [d] of Article 259 considers it a ULP to
CESCO has no basic equipment to carry out the service contracted out by
initiate, dominate, assist or otherwise interfere with the formation
CEPALCO.
On the other hand, although it may be said that CESCO had
or administration of any labor organization, including the giving of
substantial capital when CEPALCO contracted out financial or other support to it or its organizers or supports. Such
its warehousing works on January 5, 2010, there is, however, lack of union is called “company union” as its formation, function or
credible evidence to show that CESCO had the aforesaid substantial administration has been assisted by any act of the employer defined
investment in the form of equipment, tools, implements, machineries, and as ULP under the Labor Code [IRR].
Page 46 of 117
While generally, the pendency of a ULP case filed In Manila Pencil Co. v. CIR, this Court had occasion to
against a labor organization participating in the certification observe that even where business conditions justified a lay-off of
election does not stay the holding thereof [Barrera v. CIR], employees, unfair labor practices were committed in the form of
however, the pendency of a formal charge of company domination discriminatory dismissal where only unionists were permanently
against one of the unions which is participating in the certification dismissed. This was despite the valid excuse given by the Manila
election is a prejudicial question that bars the holding thereof until Pencil Company that the dismissal of the employees was due to the
its final resolution [United CMC Workers Union v. BLR]. reduction of the company's dollar allocations for importation and
that both union members and non-union members were laid-off.
Examples: The Court, thru Justice Makalintal, rebuffed the petitioner
Company and said:
a. Management files a joint motion with the defeated
union to nullify the result of the certification . . . The explanation, however, does not by any
election. means account for the permanent dismissal of five of the
b. No member of the union is dismissed despite a unionists, where it does not appear that non-unionists were
similarly dismissed.
retrenchment policy which resulted in the dismissal
xxx xxx xxx
of other employees who are officers and members of And the discrimination shown by the Company
another union [Oceanic Arc Products v. CIR]. strongly is confirmed by the fact that during the period from
c. Dismissing employees for not joining the union October 1958 to August 17, 1959 it hired from fifteen to twenty
being supported by the company [Progressive new employees and ten apprentices. It says these employees
Development Corporation v. CIR]. were for its new lead factory, but is (sic) not shown that the five
who had been permanently dismissed were not suitable for work
5. Discrimination in that new factory.
It is ULP to discriminate in regard to wages, hours of
work and other terms and conditions of employment in order to In Bataan Shipyard and Engineering Co., Inc. v. NLRC,
encourage or discourage membership in any labor organization. under the circumstances obtaining in this case, We are inclined to
Discrimination has been defined as the failure to treat all believe that the company had indeed been discriminatory in
persons equally when no reasonable distinction can be found selecting the employees who were to be retrenched. All of the
between those favored and those not favored [Sugue v. Triumph retrenched employees are officers and members of the NAFLU.
International]. There is discrimination only when one is denied The record of the case is bereft of any satisfactory explanation
privileges which are granted to others under similar conditions and from the Company regarding this situation. As such, the action
circumstances [Caltex v. Philippine Labor Organization]. Thus, taken by the firm becomes highly suspect. It leads Us to conclude
before a claim for discrimination can prosper, it must be that the firm had been discriminating against membership in the
established that first, there is no reasonable distinction or NAFLU, an act which amounts to interference in the employees'
classification that can be obtained between persons belonging to exercise of their right of self-organization. Under Art. 249 (now
the same class; and, second, persons belonging to the same class Art. 248) of the Labor Code of the Philippines, such interference is
have not been treated alike. It must be stressed, however, that considered an act of unfair labor practice on the part of the
discrimination per se is not unlawful. Further, there can be no Company
discrimination where the employees concerned are not similarly
situated [Wise and Co. v. Wise and Co., Inc. Employees Union]. a. Closed-Shop Agreement
In Manila Railroad v. Kapisanan ng Manggagawa sa A closed-shop arrangement may be defined as a scheme
Manila Railroad, the non-regularization of long-time employees in which, by agreement between the employer and its employees or
because of their affiliation with the union while new employees their representatives, no person is allowed to be employed in any
were immediately regularized was declared an act of departments of the enterprise unless he/she is, becomes and, for the
discrimination. duration of the agreement, remains a member in good standing of a
In Philippine Blooming Mills Employees Organization v. SEBA entirely comprised of or of which the employees in interest
Philippine Blooming Mills, the employer reserved its right under are a part [Del Monte Philippines v. Saldivar].
the CBA to grant better bonus to those who are exceptionally good Basically, this kind of agreement stipulates the
or efficient, it was held that it is not discriminatory or ULP for said undertaking by the employer not to hire or employ any person who
employer to give such bonus to non-union members, it being clear is not a member of the SEBA. Once employed, it is required that
that many union members were also given the bonus and it was the said person should remain a member of the SEBA in good
purely a valid exercise of management prerogative. standing as a condition for his/her continued employment, at least
But in Manila Hotel v. Pines Hotel Employees during the whole duration of the CBA. This requirement for
Association, it was enunciated that there was unjust discrimination employees to become members of the SEBA as a condition for
when management departed from its previous practice of dividing their continued employment redounds to their benefit and
equally to all employees certain percentage of its net profits as advantage because by holding out to loyal members a promise of
Christmas bonus – giving only to its employees in the operation employment in the closed shop, the union wields group solidarity.
where there was no union and not giving any to its unionized In fact, it is said that “the closed shop contract is the most prized
departments. achievement of unionism.” It adds membership and compulsory
In Davao Free Workers Front v. CIR, it was held that dues [Manila Mandarin Employees Union v. NLRC].
requiring medical examination as a pre-condition to re-admission
to work after a strike is improper. They may be subjected to b. Maintenance of Membership Agreement
periodic physical exam as old reinstated workers but not as a There is a maintenance of membership arrangement
precondition to their reinstatement. And if they are ill or suffering when employees who are SEBA members as of the effective date
from disability, they are entitled to all the benefits that the laws and of the agreement, or who thereafter become its members, must
company practices provide. maintain their union membership as a condition for their continued
employment until they are promoted or transferred out of the
bargaining unit, or the agreement is terminated [Pico Resources v.
Page 47 of 117
Dequilla]. Its role is to protect the SEBA’s current membership. By b. The SEBA is requesting for the enforcement of such
its express terms, it covers and renders continued membership clause;
compulsory for: (1) those who were already SEBA members at the c. There is sufficient evidence to support the SEBA’s
time the CBA was signed; and (2) the newly-hired employees who decision to expel the employee from membership.
will become regular during the lifetime of the CBA.
This form of union security clause is considered the In the case of Cariño vs. National Labor Relations
mildest because it does not require non-members of the SEBA to Commission, the Court pronounced that while the company, under
moin the latter but simply stipulates that those who are its members a maintenance of membership provision of the collective
at the time of the execution of the CBA and those who may, after bargaining agreement, is bound to dismiss any employee expelled
its execution, on their own, voluntarily join it, should maintain by the union for disloyalty upon its written request, this
their membership in good standing therein for the whole duration undertaking should not be done hastily and summarily. The
of the CBA as a condition for their continued employment until company acts in bad faith in dismissing a worker without giving
they are promoted or transferred out of the bargaining unit or the him the benefit of a hearing.
agreement is terminated. Simply put, employees who are not
members of the SEBA at the time of the execution of the CBA are "The power to dismiss is a normal prerogative of the
not, in any manner, required to become its members. Employees employer. However, this is not without limitation. The employer
hired after the execution of the CBA are likewise not duty-bound to is bound to exercise caution in terminating the services of his
join it. They may or may not join it. employees especially so when it is made upon the request of a
labor union pursuant to the Collective Bargaining Agreement,
c. Union Shop Agreement xxx. Dismissals must not be arbitrary and capricious. Due
process must be observed in dismissing an employee because it
There is union shop arrangement when all new regular
affects not only his position but also his means of livelihood.
employees are required to join the SEBA within a certain period as Employers should respect and protect the rights of their
a condition for their continued employment [Picop Resources v. employees, which include the right to labor."
Taneca]. Its role is to compel membership of those who are not yet
SEBA members. Under this scheme, the employer is given the MSMG-UWP v. Ramos
freedom to hire and employ any person who is not a member of the Held: In the case under scrutiny, petitioner union officers were
SEBA. Once such person becomes an employee, he is required to expelled by the federation for allegedly commiting acts of disloyalty and/or
become a member of the SEBA and to remain as such member in inimical to the interest of ULGWP and in violation of its Constitution and
good standing for the whole duration of the effectivity of the CBA By-laws. Upon demand of the federation, the company terminated the
as a condition for his continued employment. petitioners without conducting a separate and independent investigation.
Respondent company did not inquire into the cause of the expulsion and
whether or not the federation had sufficient grounds to effect the same.
d. Modified Union Shop Agreement
Relying merely upon the federation’s allegations, respondent company
Employees under this arrangement who are not SEBA terminated petitioners from employment when a separate inquiry could
members at the time of the signing or execution of the CBA are not have revealed if the federation had acted arbitrarily and capriciously in
required to join it. However, any and all workers hired or employed expelling the union officers. Respondent company’s allegation that
after the signing or the execution of the CBA are required to join petitioners were accorded due process is belied by the termination letters
the SEBA. received by the petitioners which state that the dismissal shall
be immediately effective.
e. Agency Shop Agreement As held in the aforecited case of Cariño, "the right of an
employee to be informed of the charges against him and to reasonable
Under this scheme, there is no requirement for non-
opportunity to present his side in a controversy with either the company or
members of the SEBA to become its members. However, it is his own union is not wiped away by a union security clause or a union shop
required that such non-SEBA members should pay the SEBA an clause in a collective bargaining agreement. An employee is entitled to be
agency fee as a condition or their continued employment. The third protected not only from a company which disregards his rights but also
sentence of Article 259[e] of the Labor Code validates this from his own union the leadership of which could yield to the temptation of
arrangement. swift and arbitrary expulsion from membership and mere dismissal from his
job.
f. Termination for Violation of Union While respondent company may validly dismiss the employees
expelled by the union for disloyalty under the union security clause of the
Security Clause
collective bargaining agreement upon the recommendation by the union,
It cannot be said that the stipulation providing that the this dismissal should not be done hastily and summarily thereby eroding the
employer may dismiss an employee whenever the union employees’ right to due process, self-organization and security of tenure.
recommends his expulsion either for disloyalty or for any violation The enforcement of union security clauses is authorized by law provided
of its by-laws and constitution is illegal or constitute of unfair labor such enforcement is not characterized by arbitrariness, and always with due
practice, for such is one of the matters on which management and process. Even on the assumption that the federation had valid grounds to
labor can agree in order to bring about harmonious relations expell the union officers, due process requires that these union officers be
between them and the union, and cohesion and integrity of their accorded a separate hearing by respondent company.
organization. And as an act of loyalty a union may certainly require
its members not to affiliate with any other labor union and to Picop Resources v. Taneca
consider its infringement as a reasonable cause for separation Held: However, in terminating the employment of an employee
[Tanduay v. NLRC].  by enforcing the union security clause, the employer needs to determine
and prove that: (1) the union security clause is applicable; (2) the union is
Alabang Country Club v. NLRC has enunciated the
requesting for the enforcement of the union security provision in the CBA;
following requisites that the employer should determine, prove and and (3) there is sufficient evidence to support the decision of the union to
comply with prior to terminating the employment of an employee expel the employee from the union. These requisites constitute just cause
by virtue of the enforcement of the union security clause: for terminating an employee based on the union security provision of the
CBA.
a. The union security clause is applicable; As to the first requisite, there is no question that the CBA
between PRI and respondents included a union security clause, specifically,

Page 48 of 117
a maintenance of membership as stipulated in Sections 6 of Article II,
Union Security and Check-Off.  Following the same provision, PRI, upon 7. Payment of Negotiation Fees or Attorney’s Fees
written request from the Union, can indeed terminate the employment of Article 259[h] considers as ULP the act of the employer
the employee who failed to maintain its good standing as a union member. in paying negotiation fees or attorney’s fees to the SEBA or its
Secondly, it is likewise undisputed that NAMAPRI-SPFL, in
officers or agents as part of the settlement of any issue in collective
two (2) occasions demanded from PRI, in their letters dated May 16 and 23,
2000, to terminate the employment of respondents due to their acts of
bargaining or any other dispute.
disloyalty to the Union. Article 228[b] requires that such attorney’s fees,
However, as to the third requisite, we find that there is no negotiations fees, or similar charges should be paid from the union
sufficient evidence to support the decision of PRI to terminate the funds. These fees cannot be collected from the employees
employment of the respondents. individually [Pacific Banking Corporation v. Clave].
PRI alleged that respondents were terminated from employment
based on the alleged acts of disloyalty they committed when they signed an 8. Violation of the Collective Bargaining Agreement
authorization for the Federation of Free Workers (FFW) to file a Petition
Article 259[i] should be read in relation to Article 274.
for Certification Election among all rank-and-file employees of PRI.  It
contends that the acts of respondents are a violation of the Union Security
Under the latter article, as amended, violations of the CBA, except
Clause, as provided in their Collective Bargaining Agreement. those which are gross in character, are no longer considered as
We are unconvinced. ULPs but merely as grievances that must be resolved through the
We are in consonance with the Court of Appeals when it held grievance machinery provided in the CBA. Gross violation of CBA
that the mere signing of the authorization in support of the Petition for means flagrant and/or malicious refusal to comply with its
Certification Election of FFW on March 19, 20 and 21, or before the economic provisions [Flight Attendant and Stewards Association
"freedom period," is not sufficient ground to terminate the employment of of the Philippines v. Philippine Airlines].
respondents inasmuch as the petition itself was actually filed during the
In other words, (a) ordinary violations of a CBA which
freedom period.  Nothing in the records would show that respondents failed
to maintain their membership in good standing in the Union. Respondents involves non-economic provisions thereof; (b) violations of its non-
did not resign or withdraw their membership from the Union to which they economic provisions, even if gross in nature; or (c) violations of its
belong. Respondents continued to pay their union dues and never joined the economic provisions which are not gross in character, are no longer
FFW. treated as ULP. Consequently, they should be resolved as ordinary
Significantly, petitioner's act of dismissing respondents stemmed grievances properly cognizable under the grievance machinery and
from the latter's act of signing an authorization letter to file a petition for voluntary arbitration clause of a CBA.
certification election as they signed it outside the freedom period. However, The act of the employer in refusing to implement the
we are constrained to believe that an "authorization letter to file a petition
negotiated wage increase stipulated in the CBA is ULP [Philippine
for certification election" is different from an actual "Petition for
Certification Election."  Likewise, as per records, it was clear that the actual Apparel Workers Union v. NLRC].
Petition for Certification Election of FFW was filed only on May 18, Refusal for a considerable number of years to give salary
2000. Thus, it was within the ambit of the freedom period which adjustments according to the improved salary scaled in the CBAs is
commenced from March 21, 2000 until May 21, 2000. Strictly speaking, unfair labor practice [Benguet Consolidated v. BCI Employees and
what is prohibited is the filing of a petition for certification election outside Workers Union].
the 60-day freedom period. This is not the situation in this case. If at all, the
signing of the authorization to file a certification election was merely Runaway Shop
preparatory to the filing of the petition for certification election, or an
A "runaway shop" is defined as an industrial plant moved
exercise of respondents' right to self-organization.
by its owners from one location to another to escape union labor
regulations or state laws, but the term is also used to describe a
6. Retaliatory Measures
plant removed to a new location in order to discriminate against
Under paragraph [f] of Article 259, it is an unfair labor
employees at the old plant because of their union activities. It is
practice for an employer to dismiss, discharge or otherwise
one wherein the employer moves its business to another location or
prejudice or discriminate against an employee for having given or
it temporarily closes its business for anti-union purposes. A
being about to give testimony under the Labor Code pertaining to
"runaway shop" in this sense, is a relocation motivated by anti-
labor relations.
union animus rather than for business reasons. [Complex
In Mabesa v. NLRC, the acts of the employer in forcing
Electronics Employees v. Complex]. 
the employees to sign an instrument indicating that the employer
observed labor standards provisions when he might not have not,
AC Ransom Labor Union v. NLRC
together with terminating or coercing those who refuse to
Held: Aggravating RANSOM's clear evasion of payment of its
cooperate with the employer’s schemes constitute ULP. The act financial obligations is the organization of a "run-away corporation",
clearly preempts the right of the hotel workers to seek better terms ROSARIO, in 1969 at the time the unfair labor practice case was pending
and conditions of employment through concerted action. before the CIR by the same persons who were the officers and stockholders
of RANSOM, engaged in the same line of business as RANSOM,
Ordinary Illegal Dismissal vs. ULP Dismissal producing the same line of products, occupying the same compound, using
the same machineries, buildings, laboratory, bodega and sales and accounts
ULP Dismissal (259) Ordinary Illegal Dismissal departments used by RANSOM, and which is still in existence.  Both
(118/294) corporations were closed corporations owned and managed by members of
Ground for strike Not a ground for strike the same family.  Its organization proved to be a convenience instrument to
Motivated to prevent an employee Arises if the dismissal is without avoid payment of backwages and the reinstatement of the 22 workers.  This
from exercising his right to self- just cause is another instance where the fiction of separate and distinct corporate
organization entities should be disregarded.
In labor relations, the action for In labor standards, the action for “It is very obvious that the second corporation seeks the
ULP dismissal prescribes within 1 illegal dismissal prescribes within protective shield of a corporate fiction whose veil in the present case could,
year 4 years and should, be pierced as it was deliberately and maliciously designed to
A ULP dismissal can be An ordinary dismissal cannot be evade its financial obligation to its employees.
transformed into an ordinary transformed into a ULP dismissal "x x x x When a notion of legal entity is used to defeat public
illegal dismissal case case convenience, justify wrong, protect fraud, or defend crime, the law will
Only Article 259 ripens into a The dismissal does not ripen into
criminal action a criminal action
Page 49 of 117
regard the corporation as an association or persons, or, in the case of two labor organization can be formed ads the act of recruiting and
corporations, will merge them into one." convincing the employees to join it is definitely an act of
"The corporation will be treated merely as an aggregation of interference. It becomes unlawful within the context of paragraph
individuals or, where there are two corporations, they will be merged as [a] only when it amounts to restrain or coercion which is expressly
one, the one being merely regarded as part of the instrumentality of the
prohibited thereunder.
other."

Mendoza v. Officers
Complex Electronics Employees v. Complex Held: The right of self-organization includes the right to
The mere fact that one or more corporations are owned or organize or affiliate with a labor union or determine which of two or more
controlled by the same or single stockholder is not a sufficient ground for unions in an establishment to join, and to engage in concerted activities
disregarding separate corporate personalities. Thus, in Indophil Textile Mill with co-workers for purposes of collective bargaining through
Workers Union v. Calica, we ruled that:chanrob1es virtual 1aw library representatives of their own choosing, or for their mutual aid and
[I]n the case at bar, petitioner seeks to pierce the veil of protection, i.e., the protection, promotion, or enhancement of their rights
corporate entity of Acrylic, alleging that the creation of the corporation is a and interests.
devise to evade the application of the CBA between petitioner Union and As members of the governing board of MWEU, respondents are
private respondent company. While we do not discount the possibility of presumed to know, observe, and apply the union’s constitution and by-laws.
the similarities of the businesses of private respondent and Acrylic, neither Thus, their repeated violations thereof and their disregard of petitioner’s
are we inclined to apply the doctrine invoked by petitioner in granting the rights as a union member – their inaction on his two appeals which resulted
relief sought. The fact that the businesses of private respondent and Acrylic in his suspension, disqualification from running as MWEU officer, and
are related, that some of the employees of the private respondent are the subsequent expulsion without being accorded the full benefits of due
same persons manning and providing for auxiliary services to the units of process – connote willfulness and bad faith, a gross disregard of his rights
Acrylic, and that the physical plants, offices and facilities are situated in the thus causing untold suffering, oppression and, ultimately, ostracism from
same compound, it is our considered opinion that these facts are not MWEU. "Bad faith implies breach of faith and willful failure to respond to
sufficient to justify the piercing of the corporate veil of Acrylic. plain and well understood obligation."
Likewise, in Del Rosario v. National Labor Relations
Commission, the Court stated that substantial identity of the incorporators
of two corporations does not necessarily imply that there was fraud 2. Discrimination
committed to justify piercing the veil of corporate fiction. There are three (3) kinds of discrimination that a union
may commit under this article, thus:
D. ULP OF LABOR ORGANIZATIONS
a. The act of the union to cause or attempt to cause an
Article 260. [249] Unfair Labor Practices of Labor employer to discriminate against an employee, in
Organizations. - It shall be unfair labor practice for a labor general, irrespective of whether he or she is a
organization, its officers, agents or representatives: member or non-member of the union.
(a) To restrain or coerce employees in the exercise of their b. The discriminatory act of the union against an
right to self-organization. However, a labor organization shall have the employee “with respect to whom membership in
right to prescribe its own rules with respect to the acquisition or such organization has been denied.”
retention of membership; c. The discriminatory act of the union against an
(b) To cause or attempt to cause an employer to discriminate
employee whose membership therein has been
against an employee, including discrimination against an employee
with respect to whom membership in such organization has been terminated based “on any ground other than the
denied or to terminate an employee on any ground other than the usual usual terms and conditions under which membership
terms and conditions under which membership or continuation of or continuation of membership is made available to
membership is made available to other members; other members.”
(c) To violate the duty, or refuse to bargain collectively with
the employer, provided it is the representative of the employees; It is the basic characteristic of the first kind of union
(d) To cause or attempt to cause an employer to pay or
discrimination mentioned above that the union’s act which
deliver or agree to pay or deliver any money or other things of value, in
constitutes ULP consists in inducing or instigating the employer to
the nature of an exaction, for services which are not performed or not
to be performed, including the demand for fee for union negotiations; commit discrimination against an employee who may or may not
(e) To ask for or accept negotiation or attorney’s fees from be its member. The purpose is to encourage or discourage union
employers as part of the settlement of any issue in collective bargaining membership. An example is the act of the union in convincing an
or any other dispute; or employer to penalize employees who engage in anti-union
(f) To violate a collective bargaining agreement. The activities.
provisions of the preceding paragraph notwithstanding, only the The discriminatory ULP act under the second and third
officers, members of governing boards, representatives or agents or
kinds of discrimination mentioned above is perpetrated by the
members of labor associations or organizations who have actually
union itself against an employee whose membership therewith has
participated in, authorized or ratified unfair labor practices shall be
held criminally liable been either: (1) denied by the union; or (2) terminated by the union.
The first involves an employee who has not become a
1. Restraint and Coercion member of the union because his membership therein has been
Under paragraph [a], it is ULP for a labor organization, discriminatorily denied by the union; while the second refers to the
its officers, agents, or representatives, to restrain or coerce case of an employee who is already a member of the union but
employees in the exercise of their right to self-organization. whose membership therein is discriminatorily terminated based on
Compared to similar provision of paragraph [a] of Article 259, any ground other than the usual terms and conditions under which
notably lacking is the use of the word “interfere” in the exercise of membership or continuation of membership is made available to
the employees’ right to self-organize. The significance in the other members.
omission of this term lies in the grant of unrestricted license to the Excepted from this coverage is the union security
labor organization, its officers, agents or representatives to interfere provision of the CBA. The law allows a union which has been
with the exercise by the employees of their right to self- designated as SEBA, to bargain collectively for a contract that
organization. Such interference is not lawful since without it, no permits it to cause an employer to discharge employees who fail to
join or maintain membership in good standing therein as a
Page 50 of 117
condition for continued employment. This certainly is an exception Paragraph [e] is the counterpart provision of Article
to the general rule that unions may not cause or attempt to cause an 259(h) regarding the payment, on the part of the employer, of
employer to discriminate against (or in favor of) employees on negotiation fees or attorney’s fees to the union or its officers or
union-related grounds. However, if the employee discharged on the agents as part of the settlement of any issue in collective bargaining
basis of the recommendation of a union is a religious objector, or any other dispute.
hence exempted from the coverage of any form of union security The reason for this policy of the law is to prevent undue
clause, the same may be considered ULP of the labor organization influence by the employer on the independence of the union in its
so recommending [Lakas ng Manggagawang Makabayan v. decision-making over any issues it may have with the former.
Abiera]. Moreover, it is possible that the matter of fixing the amount of
The broad rule is that the union has the right to determine negotiation fees or attorney’s fees alone would present a problem
its membership and to prescribe the conditions for the acquisition much complicated than the more substantive issues involving the
and retention thereof. Consequently, admission to membership may terms and conditions of employment and the rights, benefits or
not be compelled. This rule, however, is qualified in the case of welfare of the workers.
labor unions holding a monopoly in the supply of labor, either in a
given locality, or as regards a particular employer by reason of a 6. Gross Violation of Economic Provisions of the
closed-shop or similar agreements. In such case, qualified CBA
applicants may not be arbitrarily excluded from membership and Paragraph [f] is the counterpart provision of Article
their admission may not be barred by unreasonable rules [Salunga 259(i) regarding the employer’s act of violating a CBA. But it must
v. CIR]. be noted that under Article 274 of the Labor Code, simple violating
Just as the Court has stricken down unjust exploitation of of the CBA is generally considered no longer a ULP but merely a
laborers by oppressive employers, so will it strike down their unfair grievable issue. It becomes ULP only if the violation is gross in
treatment by their own unworthy leaders. The Constitution enjoins character which means that there is flagrant and/or malicious
the State to afford protection to labor. Fair dealing is equally refusal to comply with the economic stipulations in the CBA. This
demanded of unions as well as of employers in their dealings with principle applies not only to the employer but with equal force to
employees [Heirs of Cruz v. CIR]. the labor organization as well.

3. Violation of Union to Bargain Collectively VI


The purpose of the law in imposing it as a duty on the PEACEFUL AND CONCERTED ACTIVITIES
part of the SEBA to bargain collectively is to ensure that it will
negotiate with management in good faith in order for them to Article 278. [263] Strikes, Picketing, and Lockouts. (a) It is
conclude a mutually beneficial agreement on the terms and the policy of the State to encourage free trade unionism and free
conditions of their employment relationship. collective bargaining. (b) Workers shall have the right to engage in
The requisites before a union may be held liable for ULP concerted activities for purposes of collective bargaining or for their
are as follows: mutual benefit and protection. The right of legitimate labor
organizations to strike and picket and of employers to lockout,
a. The union is a duly certified SEBA; and consistent with the national interest, shall continue to be recognized
and respected. However, no labor union may strike and no employer
b. It commits any of the following:
may declare a lockout on grounds involving inter-union and intra-
i. it violates the duty to bargain collectively; union disputes. (c) In cases of bargaining deadlocks, the duly certified
or or recognized bargaining agent may file a notice of strike or the
ii. it refuses to bargain collectively with the employer may file a notice of lockout with the Ministry at least 30 days
employer. before the intended date thereof. In cases of unfair labor practice, the
period of notice shall be 15 days and in the absence of a duly certified
4. Featherbedding Law or recognized bargaining agent, the notice of strike may be filed by any
legitimate labor organization in behalf of its members. However, in
Paragraph [d] is the featherbedding provision.
case of dismissal from employment of union officers duly elected in
Featherbedding or “make-work” refers to the practice, cause and
accordance with the union constitution and by-laws, which may
induced by a union, of hiring more workers than are needed to constitute union busting where the existence of the union is threatened,
perform a given work, job or task or to adopt work procedures the 15-day cooling-off period shall not apply and the union may take
which is evidently senseless, wasteful, inefficient and without action immediately.
legitimate justifications since it is meant purely for the purpose of (d) The notice must be in accordance with such
employing additional workers than necessary. This is resorted to by implementing rules and regulations as the Minister of Labor and
the union as a response to laying-off workers occasioned by their Employment may promulgate.
(e) During the cooling-off period, it shall be the duty of the
obsolescence because of the introduction of machines, robots or
Ministry to exert all efforts at mediation and conciliation to effect a
new innovative technological changes and improvements in the
voluntary settlement. Should the dispute remain unsettled until the
workplace or as required by minimum health and safety standards, lapse of the requisite number of days from the mandatory filing of the
among other reasons. Its purpose is to unduly secure the jobs of the notice, the labor union may strike or the employer may declare a
workers. Because of these lay-offs, the unions are constrained to lockout.
resort to some featherbedding practices. Accordingly, they usually (f) A decision to declare a strike must be approved by a
request that the technological changes be introduced gradually, or majority of the total union membership in the bargaining unit
not at all, or that a minimum number of retaining workers even concerned, obtained by secret ballot in meetings or referenda called for
that purpose. A decision to declare a lockout must be approved by a
though there may be little work left for them to do and perform. It
majority of the board of directors of the corporation or association or
therefore unnecessarily maintains or increases the number of of the partners in a partnership, obtained by secret ballot in a meeting
employees used or the amount of time consumed to work on a called for that purpose. The decision shall be valid for the duration of
specific job, work, or undertaking. By so increasing the demand for the dispute based on substantially the same grounds considered when
workers, featherbedding obviously keeps the wages higher. the strike or lockout vote was taken. The Ministry may, at its own
initiative or upon the request of any affected party, supervise the
5. Kickbacks conduct of the secret balloting. In every case, the union or the employer

Page 51 of 117
shall furnish the Ministry the results of the voting at least seven days The second applies solely to picketing which, under
before the intended strike or lockout, subject to the cooling-off period Section 4, Article III of the Constitution, is considered part of the
herein provided. twin freedoms of speech and of expression.
(g) When, in his opinion, there exists a labor dispute causing The third is the State’s guarantee under Section 8, Article
or likely to cause a strike or lockout in an industry indispensable to the
III of the Constitution to employees in both the public and private
national interest, the Secretary of Labor and Employment may assume
jurisdiction over the dispute and decide it or certify the same to the
sectors that their right to form unions shall not be abridged.
Commission for compulsory arbitration. Such assumption or Towards this end, Section 3, Article XIII guarantees the rights of
certification shall have the effect of automatically enjoining the all workers to self-organization, then to engage in collective
intended or impending strike or lockout as specified in the assumption bargaining and negotiations and should there be conflict, to
or certification order. If one has already taken place at the time of conduct peaceful concerted activities, including the right to strike
assumption or certification, all striking or locked out employees shall in accordance with law.
immediately return to work and the employer shall immediately
resume operations and readmit all workers under the same terms and
A. PICKETING
conditions prevailing before the strike or lockout. The Secretary of
Labor and Employment or the Commission may seek the assistance of
Picketing is a concerted activity of workers consisting in
law enforcement agencies to ensure compliance with this provision as peacefully marching to and fro before an establishment involved in
well as with such orders as he may issue to enforce the same. a labor dispute generally accompanied by the carrying and display
In line with the national concern for and the highest respect of signs, placards and banners intended to inform the public about
accorded to the right of patients to life and health, strikes and lockouts the dispute [Ilaw at Buklod ng Manggagawa v. NLRC].
in hospitals, clinics and similar medical institutions shall, to every Unlike the Right to Strike which is guaranteed under the
extent possible, be avoided, and all serious efforts, not only by labor Constitutional provision on the right of workers to conduct
and management but government as well, be exhausted to substantially
peaceful concerted activities under Section 3, Article XIII, the
minimize, if not prevent, their adverse effects on such life and health,
through the exercise, however legitimate, by labor of its right to strike Right to Picket is guaranteed under the Freedom of Speech Clause
and by management to lockout. In labor disputes adversely affecting under the Bill of Rights.
the continued operation of such hospitals, clinics or medical Picketing, if peacefully carried out, cannot be prohibited
institutions, it shall be the duty of the striking union or locking-out even in the absence of employer-employee relationship between
employer to provide and maintain an effective skeletal workforce of the picketers and the employer being picketed [Philippine
medical and other health personnel, whose movement and services Association of Free Labor Unions v. CFI].
shall be unhampered and unrestricted, as are necessary to insure the
proper and adequate protection of the life and health of its patients,
1. Limitation on the Right to Picket
most especially emergency cases, for the duration of the strike or
lockout. In such cases, therefore, the Secretary of Labor and As a general rule, injunction cannot be issued against the
Employment may immediately assume, within twenty four (24) hours conduct of picketing by the workers., under our constitutional set
from knowledge of the occurrence of such a strike or lockout, up, picketing is considered part of the freedom of speech duly
jurisdiction over the same or certify it to the Commission for guaranteed by the Constitution.
compulsory arbitration. For this purpose, the contending parties are However, excepted from this legal proscription are the
strictly enjoined to comply with such orders, prohibitions and/or following situations:
injunctions as are issued by the Secretary of Labor and Employment or
the Commission, under pain of immediate disciplinary action,
a. Where picketing is carried out through the use of
including dismissal or loss of employment status or payment by the
locking-out employer of backwages, damages and other affirmative illegal means;
relief, even criminal prosecution against either or both of them. b. Where picketing involves the use of violence and
The foregoing notwithstanding, the President of the other illegal acts; or
Philippines shall not be precluded from determining the industries c. Where injunction becomes necessary to protect the
that, in his opinion, are indispensable to the national interest, and from rights of third parties
intervening at any time and assuming jurisdiction over any such labor
dispute in order to settle or terminate the same. It is important to stress that the right to peaceful
(h) Before or at any stage of the compulsory arbitration
picketing should be exercised by the workers with due respect for
process, the parties may opt to submit their dispute to voluntary
arbitration. the rights of others. Hence, commission by any picketing employee
(i) The Secretary of Labor and Employment, the of any act of violence, coercion or intimidation is prohibited.
Commission or the voluntary arbitrator or panel of voluntary Similarly, stationary picket and the use of means like placing of
arbitrators shall decide or resolve the dispute within thirty (30) objects to constitute permanent blockade or to effectively close
calendar days from the date of the assumption of jurisdiction or the points of entry or exit in company premises are likewise not
certification or submission of the dispute, as the case may be. The allowed by law [IRR]. The strikers staging the picket cannot also
decision of the President, the Secretary of Labor and Employment, the rightfully prevent employees of another company which is not their
Commission or the voluntary arbitrator shall be final and executory
employer, from getting in and out of its rented premises since this
ten (10) calendar days after receipt thereof by the parties.
will violate the right of innocent bystanders [Liwayway
Publications v. Permanent Concrete Workers Union]. In fact,
One of the most fundamental elements of a strike or
under the so-called “Innocent Bystander Rule,” such employer is
picketing is its being a “concerted activity” of the employees. If not
considered an innocent bystander who has no employer-employee
concerted, an activity cannot be characterized as a strike or picket.
relationship with the picketing strikers and thus may file for an
First and foremost, among the most significant guarantees in the
injunctive relief with the regular courts to enjoin the conduct of the
Constitution is the assurance that is given by the State to workers
picket. The NLRC has no jurisdiction to issue an injunction in
that, under Section 18, Article II, “the State shall affirm labor as a
favor of the innocent bystander because of the absence of such
primary social economic force and protect the rights of works and
relationship.
promote their welfare.” The twin rights to strike and to picket
According to Phimco Industries v. Phimco Industries
certainly fall under the ambit of protection of this provision.
Labor Association, while the right of employees to publicize their
dispute falls within the protection of freedom of expression and the
right to peacably assemble to air grievances, these rights are by no

Page 52 of 117
means absolute. Protected picketing does not extend to blocking of whether the action staged by petitioners is a strike and not
ingress to and egress from the company premises. That the picket merely a picket is the totality of circumstances surrounding the
was moving, was peaceful and was not attended by actual violence situation.
may not free it from taints of illegality if the picket effectively
blocked entry to and exit from the company premises. Picketing Strike
Marching to and for before the Temporary stoppage of work by
2. Use of Foul Language premises of an establishment the concerted action of the
involved in a labor dispute employees as a result of an
In the event the picketers employ discourteous and
industrial or labor dispute
impolite language in their picket, such may not result in, or give Purpose is to publicize a labor Purpose is stoppage of work
rise to, libel or action for damages [Philippine Commercial and dispute
Industrial Bank v. Philnabank Employees Association]. Emanates from freedom of Emanates from the right to self-
Speech organization
B. BOYCOTT Privilege right, thus cannot be Protected right, can be enjoined
enjoined
A boycott is an act of voluntary and intentional Government employees can picket Government employees can’t
abstention from using, buying, or dealing with a person, strike
organization, or country as an expression of protest, usually for Can be done by 1 person Can’t be done by only 1 person,
moral, social, political or environmental reasons. must be a concerted action
As applied to labor unions, a "boycott" has been defined Does not require formalities Requires notice of strike, cooling
off period, strike vote, waiting
as an attempt, by arousing a fear of loss, to coerce others, against
period
their will to withhold from one denominated "unfriendly to labor" No need ER-EE relationship Needs ER-EE relationship.
their beneficial business intercourse.
C. SLOWDOWN
1. Primary Boycott Slowdown is a "strike on the installment plan;" as a
A primary boycott is an organized effort of a labor union willful reduction in the rate of work by concerted action of workers
and its members to discourage consumers from buying the products for the purpose of restricting the output of the employer, in relation
of a particular employer. It is applied directly and alone to the to a labor dispute; as an activity by which workers, without a
offending person by withdrawing from him all business relations complete stoppage of work, retard production or their performance
on the part of the organization that initiated the boycott. This is of duties and functions to compel management to grant their
legal. demands. The Court also agrees that such a slowdown is generally
condemned as inherently illicit and unjustifiable, because while
2. Secondary Boycott the employees "continue to work and remain at their positions and
A Secondary Boycott is a boycott of a secondary accept the wages paid to them," they at the same time "select what
employer with which a union does not have a dispute that is part of their allotted tasks they care to perform of their own volition
intended to induce the secondary employer to cease doing business or refuse openly or secretly, to the employer's damage, to do other
with the primary employer with which the union does have a work;" in other words, they "work on their own terms [Ramirez v.
dispute. A secondary boycott is an attempt to influence the actions Polyson].
of one business by exerting pressure on another business. This is A slowdown is an inherently illegal activity essentially
illegal. illegal even in the absence of a no-strike clause in a collective
bargaining contract, or statute or rule [IBM v. NLRC].
Distinction Between Strike and Picketing  A slowdown is generally condemned as inherently illicit
In distinguishing between a picket and a strike, the and unjustifiable, because while the employees "continue to work
totality of the circumstances obtaining in a case should be taken and remain at their positions and accept the wages paid to them,"
into account. For instance, the petitioners in Santa Rosa Coca-Cola they at the same time "select what part of their allotted tasks they
Plant Employees Union v. Coca-Cola bottlers, contend that what care to perform of their own volition or refuse openly or secretly,
they conducted was a mere picketing and not a strike. In to the employer’s damage, to do other work;" in other words, they
disagreeing to this contention, the Court emphasized that it is not "work on their own terms [Interphil Union v. Interphil].
an issue in this case that there was a labor dispute between the
parties as petitioners had notified the respondent of their intention D. STRIKES
to stage a strike, and not merely to picket. Petitioner’s insistence to Strike means any temporary stoppage of work by the
stage a strike is evident in the fact that an amended notice of strike concerted action of the employees as a result of an industrial or
was filed even as respondent moved to dismiss the first notice. The labor dispute [Article 219(o)].
basic elements of a strike are present in this case: 106 members of The term “strike” is a very broad and comprehensive
petitioner Union, whose respective applications for leave of term. It encompasses not only concerted work stoppages but also
absence on September 21, 1999 were disapproved, opted not to slowdowns, mass leaves, overtime boycott, sitdowns, attempts to
report for work on said date, and gathered in front of the company damage, destroy or sabotage plant equipment and facilities, and
premises to hold a mass protest action. Petitioners deliberately similar activities [Solidbank Corporation v. Garrier]. It is
absented themselves and instead wore ribbons and carried placards axiomatic, therefore, that the fact that the conventional term
with slogans such as: “YES KAMI SA STRIKE,” “PROTESTA “strike” was not used by the striking employees to describe their
KAMI,” “SAHOD KARAPATAN NG MANGGAGAWA common course of action is inconsequential since the substance of
IPAGLABAN,” “CBA-WAG BABOYIN,” “STOP UNION the situation and not its appearance, is deemed controlling [Gesite
BUSTING.” They marched to and fro in front of the company’s v. Court of Appeals].
premises during working hours. Thus, petitioners engaged in a Strike is the most preeminent of the economic weapons
concerted activity which already affected the company’s of workers which they unsheathe to force management to agree to
operations. The mass concerted activity obviously constitutes a an equitable sharing of the joint product of labor and capital.
strike. The Mayor’s description of what activities petitioners were Undeniably, strikes exert some disquieting effects not only on the
allowed to conduct is inconsequential. To repeat, what is definitive relationship between labor and management but also on the general

Page 53 of 117
peace and progress of society. Our laws thus regulate their exercise Marsman]. It is a strike which arose out
within reason by balancing the interests of labor and management of a bargaining deadlock in the CBA
together with the overarching public interest [Lapanday Workers negotiations [NUHWHRAIN-APL-IUF
Union v. NLRC]. v. Court of Appeals].
Strikes are construed strictly against labor because strikes (ii) Unfair Labor Practice or Political
are inimical to the General Welfare Clause of the Constitution. Strike – one called to protest against
the employer’s ULPs enumerated in
Entitlement to Pay Article 259 of the Labor Code,
Strikers are not entitled to pay during strikes. Even if the including gross violation of the CBA
strike is legal, strikers may not collect their wages during the days under Article 274 and union-busting
they did not go to work, for the same reasons that laborers who under Article 278(c).
voluntarily absent themselves from work are not entitled to pay
during the period of such absence.  The age-old rule governing the c. As to Nature of the Strikers’ Action
relation between labor and capital or management and employee is
that of a "fair day's wage for a fair day labor."  If there is no work (i) Slowdown Strike – one staged where
performed by the employee there can be no wage or pay, unless of the workers do not quit their work but
course, the laborer was able, willing and ready to work but was merely slacked or reduced their normal
illegally locked out, dismissed or suspended.  It is hardly fair or work output [Fadriquelan v. Monterey
just for an employee or laborer to fight or litigate against his Foods].
employer on the employer's time [JP Helibronn Co. v. National (ii) Sit-Down Strike- one where the
Labor Union]. workers stop working but do not leave
In a case where a laborer absents himself from work their place of work [G&S Transport v.
because of a strike or to attend a conference or hearing in a case or Infante].
incident between him and his employer, he might seek
reimbursement of his wages from his union which had declared the d. As to Coverage
strike or filed the case in the industrial court.  Or, in the present
case, he might have his absence from his work charged against his (i) General Strike – one which covers and
vacation leave [Ibid]. extends over a whole province or
However, if both parties have acted in pari delicto in that country. In this kind of strike, the
the employer is guilty of illegal lockout and the union is culpable employees of various companies and
for illegal strike, the dismissal of the striking employees is industries cease to work in sympathy
unwarranted and their reinstatement should be ordered as a matter with striking workers of another
of courses. They should be restored to their respective positions company. It is also resorted to for the
prior to the illegal strike and illegal lockout. Nonetheless, if purpose of putting pressure on the
reinstatement is no longer feasible, the concerned employees government to enact certain labor-
should be given separation pay up to the date set for the return of related measures such as mandated
the complaining employees in lieu of reinstatement [AER v. wage increases or to cease from
Progresibong Unyon]. implementing a law which workers
If the strikers are not guilty of illegal strike, the general consider inimical to their interest. It is
rule that strikers are not entitled to backwages is not applicable also mounted for purposes of
where the employer is guilty of oppression and union-busting paralyzing or crippling the entire
activities and strikers ordered reinstated are denied such economic dispensation.
reinstatement and therefore are declared entitled to backwages (ii) Particular Strike – one which covers a
from the date of such denial [Philippines Inter-Fashion v. NLRC]. particular establishment or employer or
one industry involving one union or
1. Forms of Strike federation.

a. As to Nature e. As to the Extent of the Interest of the


Employees
(i) Legal Strike – one called for a valid
purpose and conducted through means (i) Primary strike – refers to a strike
allowed by law. conducted by the workers against their
(ii) Illegal Strike – one staged for a employer, involving a labor dispute
purpose not recognized by law or, if for directly affecting them.
a valid purpose, it is conducted through (ii) Secondary Strike – refers to a strike
means not sanctioned by law. staged by the workers of an employer
involving an issue which does not
b. As to Purpose directly concern or affect their
relationship but rather, by some other
(i) Economic Strike – one declared to circumstances, affect the workers, such
demand higher wages, overtime pay, as when the employer persists to deal
holiday pay, vacation pay, etc. it is with a third person against whom the
declared for the purpose of forcing workers have an existing grievance.
wage or other concessions from the Workers stage this kind of strike to
employer which he is not required by secure the economic assistance of their
law to grant [Consolidated Labor v. employer to force the third person to

Page 54 of 117
yield to the union on the issues out that in Philippine Blooming Mills Employees Organization, the mass
involving it and said third person. action staged in Malacañang to petition the Chief Executive against the
(iii) Sympathy Strike – refers to a strike abusive behavior of some police officers was a proper exercise of the
where the strikers have no demands or employees’ right to speak out and to peaceably gather and ask government
for redress of their grievances.
grievances or labor dispute of their own
The Union’s position fails to convince us.
against their employer but nonetheless While the facts in Philippine Blooming Mills Employees
stage the strike for the purpose of Organization are similar in some respects to that of the present case, the
aiding, directly or indirectly, other Union fails to realize one major difference: there was no labor dispute
strikers in other establishments or in Philippine Blooming Mills Employees Organization. In the present case,
companies, without necessarily having there was an on-going labor dispute arising from Toyota’s refusal to
any direct relation to the advancement recognize and negotiate with the Union, which was the subject of the notice
of the strikers’ interest. This is patently of strike filed by the Union on January 16, 2001. Thus, the Union’s reliance
on Phililippine Blooming Mills Employees Organization is misplaced, as it
an illegal strike [Dee C. Chuan & Sons
cannot be considered a precedent to the case at bar.
v. Kaisahan ng Manggagawa]. An A strike means any temporary stoppage of work by the
example is the “welga ng bayan” where concerted action of employees as a result of an industrial or labor dispute.
workers refuse to render work to join a A labor dispute, in turn, includes any controversy or matter concerning
general strike which does not involve a terms or conditions of employment or the association or representation of
labor or industrial dispute between the persons in negotiating, fixing, maintaining, changing, or arranging the
strikers and the employer struck against terms and conditions of employment, regardless of whether the disputants
but it is staged in pursuit of certain stand in the proximate relation of the employer and the employee.
In Bangalisan v. Court of Appeals, it was explained that “[t]he
ends, such as reduction in the electric
fact that the conventional term ‘strike’ was not used by the striking
power rates, increase in the legislated employees to describe their common course of action is inconsequential,
wages, etc. since the substance of the situation and not its appearance, will be deemed
controlling.” The term “strike” has been elucidated to encompass not only
f. As to Approval of Union concerted work stoppages, but also slowdowns, mass leaves, sit-downs,
attempts to damage, destroy, or sabotage plant equipment and facilities, and
(i) Wildcat Strike – one declared and similar activities.
staged without the majority approval of Applying pertinent legal provisions and jurisprudence, we rule
that the protest actions undertaken by the Union officials and members on
the certified bargaining agent. Is it a
February 21 to 23, 2001 are not valid and proper exercises of their right to
spontaneous or unannounced illegal assemble and ask government for redress of their complaints, but are illegal
concerted action by a section or group strikes in breach of the Labor Code. The Union’s position is weakened by
of employees without the sanction or the lack of permit from the City of Manila to hold “rallies.” Shrouded as
authorization of the union or in demonstrations, they were in reality temporary stoppages of work
violation of the union’s constitution perpetrated through the concerted action of the employees who deliberately
and by-laws, or without following the failed to report for work on the convenient excuse that they will hold a rally
proper procedure for striking such as at the BLR and DOLE offices in Intramuros, Manila, on February 21 to 23,
2001. The purported reason for these protest actions was to safeguard their
majority approval of the union
rights against any abuse which the med-arbiter may commit against their
members through appropriate union cause. However, the Union failed to advance convincing proof that the
balloting. It is also called an “outlaw med-arbiter was biased against them. The acts of the med-arbiter in the
strike” or “quickie strike.” performance of his duties are presumed regular. Sans ample evidence to the
contrary, the Union was unable to justify the February 2001 mass actions.
2. Elements of a Strike What comes to the fore is that the decision not to work for two days was
The following are the elements of strike: designed and calculated to cripple the manufacturing arm of Toyota. It
becomes obvious that the real and ultimate goal of the Union is to coerce
Toyota to finally acknowledge the Union as the sole bargaining agent of the
a. Temporary stoppage of work by the employees;
company. This is not a legal and valid exercise of the right of assembly and
b. Through their concerted action; to demand redress of grievance.
c. Occasioned by an industrial or labor dispute; and We sustain the CA’s affirmance of the NLRC’s finding that the
d. Work stoppage must be directly intended. protest rallies staged on February 21 to 23, 2001 were actually illegal
strikes. The illegality of the Union’s mass actions was succinctly elaborated
A labor dispute, in turn, includes any controversy or by the labor tribunal, thus:
matter concerning terms or conditions of employment or the We have stated in our questioned decision that such mass
association or representation of persons in negotiating, fixing, actions staged before the Bureau of Labor Relations on February 21-23,
2001 by the union officers and members fall squarely within the definition
maintaining, changing, or arranging the terms and conditions of
of a strike (Article 212 (o), Labor Code). These concerted actions resulted
employment, regardless of whether the disputants stand in the in the temporary stoppage of work causing the latter substantial losses.
proximate relation of the employer and the employee [TMPCWA v. Thus, without the requirements for a valid strike having been complied
NLRC]. with, we were constrained to consider the strike staged on such dates as
illegal and all employees who participated in the concerted actions to have
TMPCWA v. NLRC consequently lost their employment status.
Held: Petitioner Union contends that the protests or rallies If we are going to stamp a color of legality on the two (2)
conducted on February 21 and 23, 2001 are not within the ambit of strikes [day-] walk out/strike of respondents without filing a notice of strike, in
as defined in the Labor Code, since they were legitimate exercises of their effect we are giving license to all the unions in the country to paralyze
right to peaceably assemble and petition the government for redress of the operations of their companies/employers every time they wish to
grievances. Mainly relying on the doctrine laid down in the case hold a demonstration in front of any government agency. While we
of Philippine Blooming Mills Employees Organization v. Philippine recognize the right of every person or a group to peaceably assemble and
Blooming Mills Co., Inc.,[34] it argues that the protest was not directed at petition the government for redress of grievances, the exercise of such right
Toyota but towards the Government (DOLE and BLR). It explains that the is governed by existing laws, rules and regulations.
protest is not a strike as contemplated in the Labor Code. The Union points

Page 55 of 117
Although the respondent union admittedly made earnest 30 days, in case of collective bargaining deadlock,
representations with the company to hold a mass protest before the BLR, should be fully observed; and
together with their officers and members, the denial of the request by the g. The 7-day waiting period or strike ban reckoned
management should have been heeded and ended their insistence to hold after the submission of the strike vote report to the
the planned mass demonstration. Verily, the violation of the company rule
NCMB-DOLE should be fully observed in all cases.
cannot be dismissed as mere absences of two days as being suggested by
the union [are but] concerted actions detrimental to Petitioner Toyota’s
interest. (Emphasis supplied.) a. Valid and Factual Grounds
The law recognizes only two (2) grounds in support of a
3. Lawful Purpose valid strike, viz:
The right to strike is not absolute. It comes into being and
is safeguarded by law only if the acts intended to render material (i) Unfair Labor Practice (ULP or Political
aid or protection to a labor union arise from a lawful ground, Strike); or
reason or motive. But if the motive which had impelled, prompted, (ii) Collective bargaining deadlock (economic
moved or led members of a labor union or organization to stage a strike),
strike, even if they had acted in good faith in staging it, be
unlawful, illegitimate, unjust, unreasonable or trivial, the strike Collective bargaining deadlock refers to a situation
may be declared illegal [Filcon Manufacturing v. Lakas where there is failure in the collective bargaining negotiations
Manggagawa]. One example of a strike conducted for unlawful between the SEBA and the employer resulting in an impasse or
purposes is a strike to compel the dismissal of an employee stalemate [San Miguel Corporation v. NLRC]. This happens when,
[Insurefco Paper Pulp v. Sugar Refining] or a union-recognition despite their efforts at bargaining in good faith, the parties have
strike, which is calculated to compel the employer to recognize a failed to resolve the issues and it appears that there are no other
union as the employees’ bargaining representative to work out a definite options or plans in sight to break the standoff. Simply
CBA despite the union’s doubtful majority status. stated, there is a deadlock when there is a complete blocking or
stoppage in the negotiation resulting from the action of equal and
opposing forces [Capitol Medical Center Alliance of Concerned
Filcon Manufacturing v. Lakas Manggagawa
Employees v. Laguesma].
Held; As borne out by the record, respondent SWAT was
certified as the sole and (sic) bargaining agent of all the rank-and-file No other grounds may be invoked in a notice of strike. A
employees of complainant Filcon Manufacturing Corporation whose strike not based on any of the two (2) grounds discussed above is
Collective Bargaining Agreement expired last 15 January 1990 and was not illegal [San Miguel Corporation v. NLRC]. Therefore, the
renewed due to the filing of Petition for Certification Election by three (3) following are not valid grounds:
unions, namely: 1) Lakas Manggagawa sa Filcon-Lakas Manggagawa
Labor Center; 2) Bisig ng Manggagawa; and 3) Kampli-Katipunan, which (i) Violation of CBAs (GM>VA);
up to present is still pending before the Bureau of Labor Relations. In its (ii) Inter-union and internal union disputes
Collective Bargaining Agreement, it provides that the provisions contained
(BLR/MedArb);
therein shall be in full force and effect until a new one has been entered into
and one of the provisions therein reads that there shall be no strike nor lock- (iii) Issues already brought before grievance
out. This provision must be given due respect by all the parties concerned. machinery or voluntary arbitration (GM>VA)
Thus, it is evident, therefore, that the contract bar rule applies, thereby, (iv) Issues brought to compulsory arbitration
rendering the strike staged by respondent LMF-LMLC illegal.  (NLRC/LA);
(v) Issues already assumed by the DOLE Secretary
4. Procedural Requisites for a Valid Strike or certified by him to the NLRC for
Because of its potential adverse consequences to the compulsory arbitration (DOLE
striking workers, the employer and the community, a strike enjoys Sec./NLRC/LA)
recognition and respect only when it complies with the conditions (vi) Legislated wage orders (GM>VA); and
laid down by law [Libongcogon v. Phimco Industries] and (vii) Labor standard cases (LA).
pertinent prevailing jurisprudence. Following are the procedural
but mandatory requisites for a valid and legal strike: In G&T Transport v. Infante, it was held that a valid
strike therefore presupposes the existence of a labor dispute. The
a. It must be based on a valid and factual ground; strike undertaken by respondents took the form of a sit-down
b. A notice of strike must be filed with the NCMB- strike, or more aptly termed as a sympathetic strike, where the
DOLE; striking employees have no demands or grievances of their own,
c. A notice must be served to the NCMB-DOLE at but they strike for the purpose of directly or indirectly aiding
least twenty-four (24) hours prior to the taking of others, without direct relation to the advancement of the interest of
the strike vote by secret balloting, informing said the strikers.31 It is indubitable that an illegal strike in the form of a
office of the decision to conduct a strike vote, and sit-down strike occurred in petitioner's premises, as a show of
the date, place, and time thereof and asking it to sympathy to the two employees who were dismissed by petitioner.
supervise the taking of the strike vote; Apart from the allegations in its complaint for illegal strike filed
d. A strike vote must be taken where a majority of the before the Labor Arbiter, petitioner presented the affidavits and
members of the union obtained by secret ballot in a testimonies of their other employees which confirm the
meeting called for the purpose must approve it; participation of respondents in the illegal strike. Petitioner has
e. A strike vote report must be submitted to the sufficiently established that respondents remained in the work
NCMB-DOLE at least seven (7) days before the premises in the guise of waiting for orders from management to
intended date of the strike; resume operations when, in fact, they were actively participating in
f. Except in cases of union busting, observance of the the illegal strike.
cooling-off period of 15 days from the filing of the
notice of strike, in case of ULP of the employer, or Doctrine of Privilege of Good Faith Error

Page 56 of 117
In a situation where ULP is alleged, it is not essential that The NCMB has the authority to convert a notice of strike
the unfair labor practice act has, in fact, been committed; it suffices filed by the union into a preventive mediation case if it finds that
that the striking workers are shown to have acted honestly on an the real issues raised therein are non-strikeable in character.
impression that the company has committed such unfair labor Once a notice of strike/lockout is converted into a
practice and the surrounding circumstances could warrant such a preventive mediation case, it will be dropped from the docket of
belief in good faith. notices of strikes/lockouts. Once dropped therefrom, a
strike/lockout can no longer be legally staged based on the same
b. Notice of Strike notice. The conversion has the effect of dismissing the notice.
Article 278(c) of the Labor Code provides:
Philippine Airlines v. Secretary of Labor
Article 278. xxx Held: The NCMB had declared the notice of strike as
(c) In cases of bargaining deadlocks, the duly “appropriate for preventive mediation.”  The effect of that declaration
certified or recognized bargaining agent may file a notice of (which PALEA did not ask to be reconsidered or set aside) was to drop the
strike or the employer may file a notice of lockout with the case from the docket of notice of strikes, as provided in Rule 41 of the
Ministry at least 30 days before the intended date thereof. In NCMB Rules, as if there was no notice of strike.  During the pendency of
cases of unfair labor practice, the period of notice shall be 15 preventive mediation proceedings no strike could be legally declared.  The
days and in the absence of a duly certified or recognized Secretary must have thought so too, that is why he failed to act, for a period
bargaining agent, the notice of strike may be filed by any of seven (7) days, on PAL'S petition for him to assume jurisdiction over the
legitimate labor organization in behalf of its members. However, labor dispute.  The strike which the union mounted, while preventive
in case of dismissal from employment of union officers duly mediation proceedings were ongoing, was aptly described by the petitioner
elected in accordance with the union constitution and by-laws, as "an ambush" (p. 21. Rollo).
which may constitute union busting where the existence of the Since the strike was illegal, the company has a right to take
union is threatened, the 15-day cooling-off period shall not disciplinary action against the union officers who participated in it, and
apply and the union may take action immediately. against any union members who committed illegal acts during the strike.
xxx
Withdrawal of Notice
A notice of strike refers to the notification filed by a duly If the union or the employer withdraws the Notice of
registered union, after service of notice to the other party Strike or Lockout, the process goes back to square one. But it
concerned, with the NCMB-DOLE, informing the latter of its would not amount to a waiver of their right to strike based on that
intention to go on strike because of the alleged commission by the ground.
employer of ULP or because of a deadlock in the collective
bargaining negotiations. d. Strike Vote
The notice should state, among others, the names and It is a requirement that no labor organization shall
addresses of the employer and the union involved, the nature of the declare a strike without the necessary strike vote first having been
industry to which the employer belongs, the number of union obtained and reported to the NCMB-DOLE. A decision to declare a
members and workers in the bargaining unit, and such other strike must be approved by a majority of the total union
relevant data as may facilitate the settlement of the dispute, such as membership in the bargaining unit concerned, obtained by secret
a brief statement or enumeration of all pending labor disputes ballot in meetings or referenda called for that purpose. This process
involving the same parties. is called “strike vote balloting.”
The duration of the validity of the majority approval of a
c. Service of 24-Hour Prior Notice to NCMB- strike is for the whole duration of the dispute based on substantially
DOLE the same grounds considered when the strike vote was taken.
The third requisite requires that a 24-hour notice be
served to the NCMB-DOLE prior to the taking of the strike vote by e. Strike Vote Report
secret balloting, informing it of the union’s decision to conduct a In every case of strike vote, the union is required to
strike vote, as well as the date, place, and time thereof. This furnish a report on the results of the voting to the NCMB-DOLE.
requisite was enunciated in Capitol Medical Center, Inc. v. NLRC. Its submission is meant to ensure that a strike vote was indeed
The requirement of giving notice of the conduct of a strike vote to taken and in the event that the report is false, to afford the members
the NCMB at least 24 hours before the meeting for the said purpose an opportunity to take the appropriate remedy before it is too late.
is designed to (a) inform the NCMB of the intent of the union to It is a fact, for instance, that many disastrous strikes have been
conduct a strike vote; (b) give the NCMB ample time to decide on staged based merely on the insistence of minority groups within the
whether or not there is a need to supervise the conduct of the strike union. The submission of the report gives assurance that a strike
vote to prevent any acts of violence and/or irregularities attendant vote has been taken and that, if the report concerning it is false, the
thereto; and (c) should the NCMB decide on its own initiative or majority of the members can take appropriate remedy before it is
upon the request of an interested party including the employer, to too late [NFW v. Ovejara].
supervise the strike vote, to give it ample time to prepare for the A strike vote should be reported at least seven (7) days
deployment of the requisite personnel, including peace officers if before the actual staging of the intended strike/lockout, subject to
need be.  Unless and until the NCMB is notified at least 24 hours the observance of the cooling-off periods provided under the law.
of the union’s decision to conduct a strike vote, and the date, place, The failure of the union to prove that it obtained the required strike
and time thereof, the NCMB cannot determine for itself whether to vote among its members and that the results thereof were submitted
supervise a strike vote meeting or not and insure its peaceful and to the NCMB would render the strike illegal [Pinero v. NLRC].
regular conduct.  The failure of a union to comply with the In the event the result of the strike vote report is filed
requirement of the giving of notice to the NCMB at least 24 hours within the cooling-off period, the 7-day waiting period shall be
prior to the holding of a strike vote meeting will render the counted from the day following the expiration of the cooling-off
subsequent strike staged by the union illegal. period. It must be stressed that the requirements of cooling-off
period and 7-day strike ban must both be complied with, although
Preventive Mediation the labor union may take a strike vote and report the same within

Page 57 of 117
the statutory cooling-off period. If only the filing of the strike least 7 days before the actual staging of the intended strike or
notice and the strike-vote report would be deemed mandatory, but lockout, subject to the cooling-off period provided therein.
not the waiting periods so specifically and emphatically prescribed The cooling-off period and the 7-day waiting period or
by law, the purposes (hereafter discussed) for which the filing of strike ban after the submission of the strike vote report, are meant
the strike notice and strike-vote report is required would not be to be, and should be deemed, both mandatory [CCBPI Postmix
achieved, as when a strike is declared immediately after a strike Workers Union v. NLRC]. It would indeed be self-defeating for the
notice is served, or when - as in the instant case - the strike-vote law to imperatively require the filing of a strike notice and strike
report is filed with MOLE after the strike had actually commenced. vote report without at the same time making the prescribed waiting
Such interpretation of the law ought not and cannot be periods mandatory [Coca Cola Bottlers v. NLRC].
countenanced. It would indeed be self-defeating for the law to
imperatively require the filing on a strike notice and strike-vote Waiting Period vs. Cooling-Off Period
report without at the same time making the prescribed waiting The 7-day waiting period or strike ban is a distinct and
periods mandatory [NSFW v. Ovejera]. separate requirement from the 15-day or 30-day-cooling-off period
prescribed by law. The latter cannot be substituted for the former.
f. Cooling-Off Period This is clear from the provision of Article 278(f) which states that
The cooling off periods before a strike may be conducted the 7-day requirement is subject to the cooling-off period herein
are as follows: provided [Samahang Manggagawa v. Sulpicio Lines].
The cooling-off period, on the other hand, is counted
(i) In case of bargaining deadlock, the cooling-off from the time of the filing of the notice of strike up to the intended
period is 30 days; or actual staging thereof, in case of deadlock, such period is 30
(ii) In case of unfair labor practice, the cooling-off days. The 7-day waiting period strike ban, on the other hand, is
period is 15 days. reckoned from the time the strike vote report is submitted to the
NCMB-DOLE. Consequently, a strike is illegal for failure to
In requiring the cooling-off period, the avowed intent of comply with the prescribed mandatory cooling-off period and the
the law is to provide an opportunity for mediation and conciliation 7-day waiting period or strike ban after the submission of the report
by the NCMB-DOLE. It is designed to afford the parties the on the strike/lockout vote [Union of Filipro employees v. Nestle
opportunity to amicably resolve the dispute with the assistance of Philippines].
the Conciliators-Mediators of the NCMB-DOLE [NSFW v.
Ovejara]. 5. Violence
These requirements are mandatory, and the union's If the union intended to use violence as a method of
failure to comply renders the strike illegal.  The 15 to 30-day strike from the very beginning of the strike, the union is liable.
cooling-off period is designed to afford the parties the opportunity If the union intended it to be a peaceful strike, but it
to amicably resolve the dispute with the assistance of the NCMB eventually became violent, the individual employees are liable.
conciliator/mediator, while the seven-day strike ban is intended to
give the DOLE an opportunity to verify whether the projected Malayang Samahan v. Ramos
strike really carries the imprimatur of the majority of the union Held: On the allegation of violence committed in the course of
members. the strike, it must be remembered that the Labor Arbiter and the
In Phimco Industries v. Phimco Industries Labor Association , the Commission found that "the parties are agreed that there were violent
respondents fully satisfied the legal procedural requirements; a incidents x x x resulting to injuries to both sides, the union and
management." The evidence on record show that the violence cannot be
strike notice was filed on March 9, 1995; a strike vote was reached
attributed to the striking employees alone for the company itself employed
on March 16, 1995; notification of the strike vote was filed with hired men to pacify the strikers. With violence committed on both sides, the
the DOLE on March 17, 1995; and the actual strike was launched management and the employees, such violence cannot be a ground for
only on April 25, 1995. declaring the strike as illegal.

Union-Busting 6. Some Principles on Strike


The exception to the rule on the observance of the
cooling-off period is in cases of union-busting which is considered  A strike or lockout is illegal if the issues involved
an unfair labor practice where the said 15-day cooling-off period are already subject of compulsory or voluntary
may be disregarded completely. arbitration or conciliation or the steps in grievance
To constitute union-busting under Article 278(c), there machinery are not exhausted.
must be:  A strike or lockout is illegal if unlawful means were
employed or prohibited acts or practices were
(i) A dismissal from employment of union officers committed
duly elected in accordance with the union’s
 A strike or lockout is illegal if staged in violation of
constitution and by-laws; and
the “No-Strike, No-Lockout” clause in the CBA.
(ii) The existence of the union is threatened by
This clause may be invoked by the employer only
such dismissal.
when the strike is economic in nature or one which
is conducted to force wage or other concessions
The act of union-busting by an employer is an unfair
from the employer that are not mandated to be
labor practice which may be invoked as a valid ground for a strike
granted by the law itself. It does not bar strikes
[Zamboanga Wood Products v. NLRC].
grounded on ULP [Malayang Samahan v. Ramos].
 A strike or lockout is illegal if staged in violation of
g. 7- Day Waiting Period or Strike Ban
a TRO or an injunction order issued for the purpose
After the taking of the strike vote, the union, in every
of enjoining the union and/or its members from
case, should furnish the NCMB-DOLE, the results of the voting at
committing illegal and prohibited acts in the course
of a strike, such as, among others, the act of
Page 58 of 117
obstructing the free ingress to or egress from the  In violation of company code of conduct [Toyota
company premises [Association of Independent Motor v. NLRC].
Unions v. NRLC].  As a protest in front of government offices
 A strike or lockout is illegal if conducted for  As welga ng bayan. Stoppage of work due to welga
unlawful purposes. This principle applies even if the ng bayan is in the nature of a general strike and an
strikers had acted in good faith in staging it [Filcon extended sympathy strike which are illegal since the
Manufacturing v. Lakas Manggagawa]. A strike striking employees have no labor dispute with their
staged for the purpose of unreasonably demanding employer but who, on a day they are scheduled to
the dismissal of an employee like a factory foreman work, refuse to work and instead join a welga ng
is illegal [Toyota Motor Phils. Corp Woprkers bayan [Biflex Labor Union v. Filiflex Industrial].
Association v. NLRC].  In violation of the rules on picketing, such as
 A one-day strike without complying with the 7-day commission of prohibited acts like: obstructing,
strike ban is illegal. impeding or interfering with, by force, violence,
coercion, threats or intimidation, any peaceful
7. Illegal Strikes picketing; conducting a stationary picket;
committing any act of violence, coercion or
 Without complying with the procedural but intimidation by any picketer; obstructing the free
mandatory requisites ingress to or egress from the employer’s premises
 For unlawful purpose from lawful purposes; obstructing public
 Based on non-strikeable or invalid grounds thoroughfares.
 In violation of no-strike, no-lockout clause in the
CBA. 8. Improved Offer Balloting
 Without submitting the issues to the Grievance In case of a strike, the Regional Branch of the NCMB
Machinery or Voluntary Arbitrator or failing to shall conduct a REFERENDUM by secret balloting on the
exhaust the steps provided therein improved offer of the employer on or before the 30 th day of the
 Without first having bargained collectively strike.
 While the conciliation and mediation proceeding is When at least a majority of the union members vote to
on-going at the NCMB. The disregard of such accept the improved offer, the striking workers shall immediately
proceedings is a blatant violation of the IRR, which return to work and the employer shall thereupon readmit them upon
explicitly obliges the parties to bargain collectively the signing of the agreement.
in good faith and prohibits them from impeding or
disrupting the proceedings [Filipino Pipe v. NLRC]. E. LOCKOUTS
 Based on issues already brought to the VA or CA. Lockout means the temporary refusal of an employer to
The rationale behind this prohibition is that once furnish work to its employees as a result of an industrial or labor
jurisdiction over the labor dispute has been properly dispute [Article 219(p)].
acquired by competent authority, that jurisdiction Lockout consists of shutdowns, mass retrenchment and
should not be interfered with by the application of dismissals initiated by the employer [PD 823]. It, however, may
coercive process of a strike or lockout [Telefunken take other forms such as the employer’s act of excluding
Semiconductors v. Court of Appeals]. Thus, a strike employees who are union members [Complex Electronics
conducted during the pendency of the compulsory Employees v. NLRC].
arbitration proceedings on a labor dispute certified
to the NLRC by the DOLE Secretary for Shutdown vs. Lockout
compulsory arbitration is illegal [Philippine In a shutdown, the plant ceases to operate. It is a willful
Diamond Hotel v. Manila Diamond Hotel act of the employer himself. While a lockout is a compulsory
Employees Union]. stoppage of operations as a result of a strike and walkout. In a
lockout the plant continues to operate. The employee-union
 During the pendency of a case involving the same
members who are locked out are replaced by non-union substitutes,
grounds cited in the notice of strike
but the plant continues to function.
 In defiance of an assumption or certification or
return-to-work order
1. Requisites of a Valid Lockout
 After the conversion of the notice of strike into a
To be valid, a lockout should comply with the following
preventive mediation case
requisites:
 Against the prohibition of law
 By a minority union. This is so because no labor a. It must be based on any or both of the following
dispute which will justify the conduct of a strike can grounds:
exist between the employer and a minority union.
To permit the union’s picketing activities would be (i) Unfair Labor Practice of Labor
to flaunt at the will of the majority [United Organization
Restauror’s Employees v. Torres]. (ii) Collective Bargaining Deadlock
 By an illegitimate union. Under Article 278(c), only
a LLO is entitled to file a notice of strike on behalf b. A notice of lockout should be filed with the NCMB-
of its members. Absent sowing as to the legitimate DOLE;
status of the labor organization, the strike it c. A notice must be served to the NCMB-DOLE at
conducted would have to be considered as illegal least twenty-four (24) hours prior to the taking of
[Stamford Marketing v. Julian]. the lockout vote by secret balloting, informing said
 By dismissed employees [Toyota Motor v. NLRC]. office of the decision to conduct a lockout vote, and

Page 59 of 117
the date, place, and time thereof and asking it to MSF Tire & Rubber v. Court of Appeals
supervise the taking of the lockout vote; Held: In Philippine Association of Free Labor Unions (PAFLU)
d. A lockout vote must be taken where a majority of v. Cloribel, this Court, through Justice J.B.L. Reyes, stated the "innocent
the members of the Board of Directors of the bystander" rule as follows:
corporation or association or of the partners in a The right to picket as a means of communicating the facts of a
labor dispute is a phase of the freedom of speech guaranteed by the
partnership, obtained by secret ballot in a meeting
constitution. If peacefully carried out, it can not be curtailed even in the
called for the purpose, must approve it; absence of employer-employee relationship.
e. A lockout vote report should be submitted to the The right is, however, not an absolute one. While peaceful
NCMB-DOLE at least (7) days before the intended picketing is entitled to protection as an exercise of free speech, we
date of the lockout; believe the courts are not without power to confine or localize the
f. Observance of the cooling-off period of 15 days, in sphere of communication or the demonstration to the parties to the
case of ULP of the labor organization, or 30 days, in labor dispute, including those with related interest, and to insulate
case of CBD, reckoned from the date of filing of the establishments or persons with no industrial connection or having
interest totally foreign to the context of the dispute. Thus the right may
notice of lockout; and
be regulated at the instance of third parties or "innocent bystanders" if
g. The 7-day waiting period reckoned after the it appears that the inevitable result of its exercise is to create an
submission of the lockout vote report to the NCMB- impression that a labor dispute with which they have no connection or
DOLE should be fully complied with in all cases. interest exists between them and the picketing union or constitute an
invasion of their rights. In one case decided by this Court, we upheld a
2. Reduced Offer Balloting trial court's injunction prohibiting the union from blocking the entrance to a
In case of a lockout, the Regional Branch of the NCMB feed mill located within the compound of a flour mill with which the union
shall conduct a REFERENDUM by secret balloting on the reduced had a dispute. Although sustained on a different ground, no connection was
found between the two mills owned by two different corporations other
offer of the union or or before the 30th day of the lockout.
than their being situated in the same premises. It is to be noted that in the
When at least a majority of the board of directors or instances cited, peaceful picketing has not been totally banned but merely
trustees or the partners holding the controlling interest in the case regulated. And in one American case, a picket by a labor union in front of a
of partnership, vote to accept the reduced offer, the workers shall motion picture theater with which the union had a labor dispute was
immediately return to work and the employer shall thereupon enjoined by the court from being extended in front of the main entrance of
readmit them upon the signing of the agreement. the building housing the theater wherein other stores operated by third
persons were located. (Emphasis added)
F. INJUNCTIONS Thus, an "innocent bystander," who seeks to enjoin a labor
strike, must satisfy the court that aside from the grounds specified in Rule
As a general rule, strikes or lockouts that are validly
58 of the Rules of Court, it is entirely different from, without any
declared enjoy the protection of the law and cannot be enjoined connection whatsoever to, either party to the dispute and, therefore, its
unless illegal acts are committed or threatened to be committed in interests are totally foreign to the context thereof. For instance, in PAFLU
the course thereof. This policy applies even if the strike appears to v. Cloribel, supra, this Court held that Wellington and Galang were entirely
be illegal in nature. The rationale for this policy is the protection separate entities, different  from, and without any connection whatsoever to,
extended to the right to strike under the Constitution and the law. It the Metropolitan Bank and Trust Company, against whom the strike was
is basically treated as a weapon that the law guarantees to directed, other than the incidental fact that they are the bank's landlord and
employees for the advancement of their interest and for their co-lessee housed in the same building, respectively. Similarly, in Liwayway
Publications, Inc. v. Permanent Concrete Workers Union, this Court ruled
protection [Caltex Refinery Employees Association v. Lucero].
that Liwayway was an "innocent bystander" and thus entitled to enjoin the
However, in some cases, injunctions issued to enjoin the union's strike because Liwayway's only connection with the employer
conduct of the strike itself and not only the commission of illegal company was the fact that both were situated in the same premises.
or prohibited acts in the course thereof, were held to be valid. In In the case at bar, petitioner cannot be said not to have such
IBM v. NLRC, it was held that it is the legal duty and obligation of connection to the dispute. As correctly observed by the appellate court:
the NLRC to enjoin a partial strike staged in violation of the law. Coming now to the case before us, we find that the "negotiation,
Failure to promptly issue an injunction by the NLRC was held contract of sale, and the post transaction" between Philtread, as vendor, and
therein to be an abuse of discretion. Siam Tyre, as vendee, reveals a legal relation between them which, in the
interest of petitioner, we cannot ignore. To be sure, the transaction between
Philtread and Siam Tyre, was not a simple sale whereby Philtread ceased to
1. When Equity and Justice Requires have any proprietary rights over its sold assets. On the contrary, Philtread
In Bulletin Publishing v. Sanchez, an injunction was remains as 20% owner of private respondent and 60% owner of Sucat Land
allowed against the strike which was staged to compel the Corporation which was likewise incorporated in accordance with the terms
employer to ignore the law. The reason is that when trade unionism of the Memorandum of Agreement with Siam Tyre, and which now owns
and strikes are used in violation of the law, misuse thereof can be the land were subject plant is located. This, together with the fact that
subject of judicial intervention. private respondent uses the same plant or factory; similar or substantially
the same working conditions; same machinery, tools, and equipment; and
manufacture the same products as Philtread, lead us to safely conclude that
2. Innocent Bystander Rule
private respondent's personality is so closely linked to Philtread as to bar its
In situations where the picket affects not only the entitlement to an injunctive writ. Stated differently, given its close links
employer but also the business operations of other establishments with Philtread as to bar its entitlement to an injunctive writ. Stated
owned by third parties, an injunction may be secured by the latter differently, given its close links with Philtread, we find no clear and
from the regular courts to enjoin the picket. Picketing strikers unmistakable right on the part of private respondent to entitle it to the writ
cannot prevent employees of other companies from using the same of preliminary injunction it prayed for below.
premises being picketed. A picketing labor union has no right to ....
prevent employees of another company which is not their We stress that that in so ruling, we have not touched on the issue
of . . . whether or not private respondent is a mere dummy or continuation
employer, from getting in and out of its rented premises; otherwise,
of Philtread. . . .
it will be held liable for damages for its acts against an innocent Although, as petitioner contends, the corporate fiction may be
bystander [Liwayway Publications v. Permanent Concrete Workers disregarded where it is used to defeat public convenience, justify wrong,
Union]. protect fraud, defend crime, or where the corporation is used as a mere
alter-ego or business conduit, it is not these standards but those of the

Page 60 of 117
"innocent bystander" rule which govern whether or not petitioner is entitled arbitrarily. The interests of both the employers and employees are
to an injunctive writ. Since petitioner is not an "innocent bystander", the intended to be protected and not one of them is given undue
trial court's order, dated July 2, 1996, is a patent nullity, the trial court preference [Philtread v. Confesor].
having no jurisdiction to issue the writ of injunction. No motion for Both parties may request the Secretary of DOLE to
reconsideration need be filed where the order is null and void.
assume jurisdiction over the labor dispute or after a conference is
called by the DOLE Secretary on the propriety of the issuance, the
3. Industry Indispensable to the National Interest Secretary may motu proprio or upon a request or petition by either
of the parties to the labor dispute assume jurisdiction.
a. Hospitals, Clinics, or Medical Institutions Having the two (2) options above, the DOLE Secretary
The second paragraph of Article 278(g) provides: may do the following variations thereof:

(g) xxx
a. Assume jurisdiction over a labor dispute and at the
In line with the national concern for and the highest
same time certify it to the NLRC for compulsory
respect accorded to the right of patients to life and health, strikes
and lockouts in hospitals, clinics and similar medical institutions arbitration [NAFLU v. NLRC].
shall, to every extent possible, be avoided, and all serious b. Initially assume jurisdiction over a labor dispute and
efforts, not only by labor and management but government as later, on a different date, certify the same labor
well, be exhausted to substantially minimize, if not prevent, dispute to the NLRC for compulsory arbitration
their adverse effects on such life and health, through the [NAFLU v. Sulpicio Lines].
exercise, however legitimate, by labor of its right to strike and
by management to lockout. In labor disputes adversely affecting
Notwithstanding the power granted to the DOLE
the continued operation of such hospitals, clinics or medical
institutions, it shall be the duty of the striking union or locking-
Secretary to assume jurisdiction over national interest labor
out employer to provide and maintain an effective skeletal disputes or to certify them to the NLRC for compulsory arbitration,
workforce of medical and other health personnel, whose the President of the Philippines shall not be precluded from doing
movement and services shall be unhampered and unrestricted, as any of the following:
are necessary to insure the proper and adequate protection of the
life and health of its patients, most especially emergency cases, a. To determine the industries that, in his opinion, are
for the duration of the strike or lockout. In such cases, therefore, indispensable to the national interest;
the Secretary of Labor and Employment may immediately
b. To intervene at any time and assume jurisdiction
assume, within twenty four (24) hours from knowledge of the
occurrence of such a strike or lockout, jurisdiction over the same
over any such labor dispute in order to settle or
or certify it to the Commission for compulsory arbitration. For terminate it [Article 278(g)].
this purpose, the contending parties are strictly enjoined to
comply with such orders, prohibitions. Unlike his alter ego, the DOLE Secretary, the President,
xxx while possessed of the power of assumption, has no similar power
to “certify the labor dispute to the Commission for Compulsory
b. Banking Institutions arbitration,” this power being granted solely to the DOLE
Section 22 of Republic Act No. 8791 provides: Secretary. His role is confined to assuming jurisdiction thereover,
thus: “The President of the Philippines shall not be precluded from
Section 22. Strikes and Lockouts. - The banking determining the industries that, in his opinion, are indispensable to
industry is hereby declared as indispensable to the national the national interest, and from intervening at any time and
interest and, notwithstanding the provisions of any law to the
assuming jurisdiction over any such labor dispute in order to settle
contrary, any strike or lockout involving banks, if unsettled after
seven (7) calendar days shall be reported by the Bangko Sentral
or terminate the same.
to the secretary of Labor who may assume jurisdiction over the The DOLE Secretary or the NLRC, as the case may be,
dispute or decide it or certify the same to the National Labor may seek assistance of law enforcement agencies to seek assistance
Relations Commission for compulsory arbitration. However, the of law enforcement agencies to ensure compliance with the
President of the Philippines may at any time intervene and assumption or certification orders as well as with such orders
assume jurisdiction over such labor dispute in order to settle or which may forthwith be issued to enforce them.
terminate the same. (6-E)
1. Industries Indispensable to National Interest
G. ASSUMPTION OF JURISDICTION BY THE DOLE The determination of specific industries indispensable to
SECRETARY the national interest is left to the discretion of the DOLE Secretary.
Article 278(g) provides that when in the opinion of the Section 16 to Rule XXII, Book V of the Omnibus Rules provides:
DOLE Secretary, the labor dispute causes or will likely to cause a
strike or lockout in an industry indispensable to the national Section 16. Industries Indispensable to the National
interest, he is empowered to do either of the 2 things: Interest. - For the guidance of the workers and employers in the
filing of petition for assumption of jurisdiction, the following
a. He may assume jurisdiction over the labor dispute industries/services are hereby recognized as deemed
and decide it for himself; or indispensable to the national interest:
b. He may certify it to the NLRC for compulsory a. Hospital Sector;
b. Electric Power Industry;
arbitration, in which case, it will be the NLRC
c. Water Supply Services, to exclude small water
which shall hear and decide. supply services such as Bottling and Refilling Refilling Stations;
d. Air Traffic Control; and
The foregoing article clearly does not interfere with the e. Such other industries as maybe recommended by
workers' right to strike but merely regulates it, when in the exercise the National Tripartite Industrial Peace Council (NTIPC).
of such right, national interests will be affected. The rights granted
by the Constitution are not absolute. They are still subject to The above enumeration is not exclusive as other
control and limitation to ensure that they are not exercised companies or firms or industries may be considered indispensable

Page 61 of 117
to the national interest based on the appreciated and discretion of of discretion must be so patent and gross as to amount to an evasion of
the DOLE Secretary, or as may be recommended by the TIPC, or positive duty or to a virtual refusal to perform a duty enjoined by law, or to
because they have already been judicially adjudged as being act at all in contemplation of law, as where the power is exercised in an
indispensable to the national interest. arbitrary and despotic manner by reason of passion or personal hostility.

Philtread v. Confesor 2. Strikes in Hospitals, Clinics and Medical


Held: The Labor Code vests upon the Secretary of Labor the Institutions
discretion to determine what industries are indispensable to national The second paragraph of Article 278(g) provides:
interest. Thus, upon the determination of the Secretary of Labor that such
industry is indispensable to the national interest, it will assume jurisdiction In line with the national concern for and the highest
over the labor dispute of said industry. The assumption of jurisdiction is in respect accorded to the right of patients to life and health, strikes
the nature of police power measure. This is done for the promotion of the and lockouts in hospitals, clinics and similar medical institutions
common good considering that a prolonged strike or lockout can be shall, to every extent possible, be avoided, and all serious
inimical to the national economy. The Secretary of Labor acts to maintain efforts, not only by labor and management but government as
industrial peace. Thus, his certification for compulsory arbitration is not well, be exhausted to substantially minimize, if not prevent,
intended to impede the workers' right to strike but to obtain a speedy their adverse effects on such life and health, through the
settlement of the dispute. This is well-articulated in International exercise, however legitimate, by labor of its right to strike and
Pharmaceuticals, Inc. vs. Secretary of Labor, in this wise: by management to lockout. In labor disputes adversely affecting
Plainly, Article 263 (g) of the Labor Code was meant to make the continued operation of such hospitals, clinics or medical
both the Secretary (or the various regional directors) and the labor arbiters institutions, it shall be the duty of the striking union or locking-
share jurisdiction, subject to certain conditions. Otherwise, the Secretary out employer to provide and maintain an effective skeletal
would not be able to effectively and efficiently dispose of the primary workforce of medical and other health personnel, whose
dispute. To hold the contrary may even lead to the absurd and undesirable movement and services shall be unhampered and unrestricted, as
result wherein the Secretary and the labor arbiter concerned may have are necessary to insure the proper and adequate protection of the
diametrically opposed rulings. As we have said, "(i)t is fundamental that a life and health of its patients, most especially emergency cases,
statute is to be read in a manner that would breathe life into it, rather than for the duration of the strike or lockout. In such cases, therefore,
defeat it.8 the Secretary of Labor and Employment may immediately
On the second issue raised by the petitioners, We find that the assume, within twenty four (24) hours from knowledge of the
Secretary of Labor did not act with grave abuse of discretion in issuing the occurrence of such a strike or lockout, jurisdiction over the same
certification for compulsory arbitration. It had been determined by the or certify it to the Commission for compulsory arbitration. For
Labor Arbiter in NLRC-NCR Case No. 00-05-04156-94 that the work this purpose, the contending parties are strictly enjoined to
slowdowns conducted by the petitioner amounted to illegal strikes. It was comply with such orders, prohibitions and/or injunctions as are
shown that every time the respondent company failed to accede to the issued by the Secretary of Labor and Employment or the
petitioner's demands, production always declined. This resulted to the Commission, under pain of immediate disciplinary action,
significant drops in the figures of tires made, cured, and warehoused. including dismissal or loss of employment status or payment by
However, when the demand of the petitioner union for the restoration of the locking-out employer of backwages, damages and other
overtime work was allowed, production improved. The work slowdowns, affirmative relief, even criminal prosecution against either or
which were in effect, strikes on installment basis, were apparently a pattern both of them.
of manipulating production depending on whether the petitioner union's
demands were met. These strikes, however, had greatly affected the 3. Effect on Strike or Lockout
respondent company that on November 11, 1994, it had indefinitely ceased The assumption of jurisdiction or certification to the
operations because of tremendous financial losses.
NLRC of a labor dispute has the following effects:
We do not agree with the petitioners that the respondent
company is not indispensable to national interest considering that the tire
industry has already been liberalized. Philtread supplies 22% of the tire a. On the intended or impending strike or
products in the country. Moreover, it employs about 700 people. As lockout
observed by the Secretary of Labor, viz.: Upon assumption/certification, the intended or
The Company is one of the tire manufacturers in the country impending strike or lockout is automatically enjoined,
employing more or less 700 workers. Any work disruption thereat, as a notwithstanding the filing of any motion for reconsideration of the
result of a labor dispute will certainly prejudice the employment and assumption/certification order or the non-resolution of any such
livelihood of its workers and their dependents. Furthermore, the labor
motion which may have been duly submitted to the Office of the
dispute may lead to the possible closure of the Company and loss of
employment to hundreds of its workers. This will definitely aggravate the DOLE Secretary [NLRC Rules of Procedure].
already worsening unemployment situation in the country and discourage
foreign and domestic investors from further investing in the country. There b. On the actual strike or lockout
is no doubt, therefore, that the labor dispute in the Country is imbued with If a work stoppage has already been taken place at the
national interest. time of the assumption/certification, all striking or locked-out
At this point in time when all government efforts are geared employees shall immediately return to work and the employer shall
towards economic recovery and development by encouraging both foreign immediately resume operations and readmit all workers under the
and domestic investments to generate employment, we cannot afford to
same terms and conditions prevailing before the strike or lockout
derail the same as a result of a labor dispute considering that there are
alternative dispute resolution machineries available to address labor [NLRC Rules of Procedure].
problems of this nature.9
The intervention of the Secretary of Labor was therefore c. On cases already filed or may be filed
necessary to settle the labor dispute which had lingered and which had All cases between the same parties, except where the
affected both respondent company and petitioner union. Had it not been so, assumption/certification order specifies otherwise, including the
the deadlock will remain and the situation will remain uncertain. Thus, it issues submitted for arbitration which already filed or may be filed
cannot be deemed that the Secretary of Labor had acted with grave abuse of and are relevant to or are proper incidents of the certified case, are
discretion in issuing the assailed order as she had a well-founded basis in
considered subsumed or absorbed by the assumed/certified case
issuing the assailed order. It is significant at this point to point out that
grave abuse of discretion implies capricious and whimsical exercise of [NLRC Rules of Procedure].
judgment. Thus, an act may be considered as committed in grave abuse of
discretion when the same was performed in a capricious or whimsical d. On other pending cases
exercise of judgment which is equivalent to lack of jurisdiction. The abuse
Page 62 of 117
The parties to an assumed/certified case, under pain of return-to-work order in national interest cases. but such departure
contempt, are required to inform their counsels and the DOLE from the norm may only be justified in case of existence of certain
Secretary/NLRC Division concerned, as the case may be, of all special circumstances that render actual reinstatement
pending cases that are related or incident to the assumed/certified impracticable or otherwise not conducive to attaining the purposes
case before it [NLRC Rules of Procedure]. of the law [Manila Diamond Hotel Employees’ Union v. Court of
Appeals]. This is best exemplified in the case of University of Sto.
Submission of National Interest Dispute to Voluntary Tomas v. NLRC, where, by reason of the special circumstance at
Arbitration the time the teachers were ordered to return to work by reason of
Before or at any stage of the compulsory arbitration the DOLE Secretary’s certification of the labor dispute to the
process, the parties may, by mutual agreement, decide to bring the NLRC for compulsory arbitration, they could not be given back
matter for resolution before an accredited Voluntary Arbitrator or their academic assignments since the return-to-work order of the
Panel of Voluntary Arbitrators of their own choice, in which case, DOLE Secretary was issued in the middle of the first semester of
the notice is deemed automatically withdrawn and dropped from the academic year. The NLRC, to which the labor dispute was
the docket [NLRC Rules of Procedure]. certified, was therefore faced with a situation where the striking
teachers were entitled to a return-to-work order, but the university
4. Return-To-Work Order could not immediately reinstate them since it would be
It is clear that under Article 278(g), that the moment the impracticable and detrimental to the students to change teachers at
DOLE Secretary assumes jurisdiction over a labor dispute the point. Thus:
involving national interest or certifies it to the NLRC for
compulsory arbitration, such assumption or certification has the It was error for the NLRC to order the alternative
effect of automatically enjoining the intended or impending strike. remedies of payroll reinstatement or actual
Simple stated, assumption of jurisdiction over a labor dispute, or reinstatement.  However, the order did not amount to grave
abuse of discretion.  Such error is merely an error of judgment
the certification of the same to the NLRC for compulsory
which is not correctible by a special civil action
arbitration, always co-exists with an order for workers to return to for certiorari.  The NLRC was only trying its best to work out a
work immediately and for employers to readmit all of them under satisfactory ad hoc solution to a festering and serious
the same terms and conditions prevailing before the strike or problem.  In the light of our rulings on the impropriety of the
lockout [University of San Agustin Employees Union v. Court of substantially equivalent academic assignments and the need to
Appeals]. defer the changes of teachers until the end of the first semester,
the payroll reinstatements will actually minimize the petitioner's
a. Nature of Return-to-Work Order problems in the payment of full backwages.
xxx
It is also important to emphasize that the return-to-work
Although we pronounce that the dismissed faculty
order not so much confers a right as it imposes a duty; and while as members must be actually reinstated while the labor dispute is
a right it may be waived, it must be discharged as a duty even being resolved, we have to take into account the fact that at this
against the worker's will.  Returning to work in this situation is not time, the first semester for schoolyear 1990-1991 is about to
a matter of option or voluntariness but of obligation.  The worker end.  To change the faculty members around the time of final
must return to his job together with his co-workers so the examinations would adversely affect and prejudice the students
operations of the company can be resumed and it can continue whose welfare and interest we consider to be of primordial
serving the public and promoting its interest.  That is the real importance and for whom both the University and the faculty
union must subordinate their claims and desires.  This Court
reason such return can be compelled.  So imperative is the order in
therefore resolves that the actual reinstatement of the non-
fact that it is not even considered violative of the right against reinstated faculty members, pending resolution of the labor
involuntary servitude. The worker can of course give up his work, controversy before the NLRC, may take effect at the start of the
thus severing his ties with the company, if he does not want to obey second semester of the schoolyear 1990-1991 but not
the order; but the order must be obeyed if he wants to retain his later.  With this arrangement, the petitioner's reasoning that it
work even if his inclination is to strike [Asian Transmission v. will be violating contracts with the faculty members who took
NLRC]. over the dismissed professors’ teaching loads becomes moot
The assumption/certification order may be served at any considering that, as it alleges in its petition, it operates on
a semestral basis.
time of the day or night [Telefunken Semiconductors Employees’
union v. Secretary of Labor].
University of Immaculate Concepcion v. Secretary of Labor
Held: With respect to the Secretary’s Order allowing payroll
b. All Striking or Locked out Employees reinstatement instead of actual reinstatement for the individual respondents
The phrase "all striking or locked out employees" and herein, an amendment to the previous Orders issued by her office, the same
"readmit all workers" does not distinguish or qualify and is usually not allowed.  Article 263(g) of the Labor Code aforementioned
emphatically is a catch all-embracing enumeration of who should states that all workers must immediately return to work and all employers
be returned to work. Where the law does not distinguish, courts must readmit all of them under the same terms and conditions prevailing
should not distinguish [PLDT v. Manggagawa ng Komunikasyon]. before the strike or lockout.  The phrase “under the same terms and
conditions” makes it clear that the norm is actual reinstatement.  This is
consistent with the idea that any work stoppage or slowdown in that
c. Under the Same Terms and Conditions
particular industry can be detrimental to the national interest.
Prevailing Before the Strike In ordering payroll reinstatement in lieu of actual reinstatement,
As a general rule, the concept of return-to-work under then Acting Secretary of Labor Jose S. Brillantes said:
Article 278(g) contemplates actual reinstatement and not payroll Anent the Union’s Motion, we find that superseding
reinstatement. This is in accordance with the intent and spirit of circumstances would not warrant the physical reinstatement of the twelve
this Article [NUWHRAIN-APL-IUF v. Court of Appeals]. (12) terminated employees.  Hence, they are hereby ordered placed under
As with most rules, however, the actual reinstatement payroll reinstatement until the validity of their termination is finally
rule in Article 278(g) is subject to exceptions [Ibid]. Consequently, resolved.
As an exception to the rule, payroll reinstatement must rest on
there are a number of cases where, instead of actual reinstatement,
special circumstances that render actual reinstatement impracticable or
payroll reinstatement was ordered by the Court to implement the otherwise not conducive to attaining the purposes of the law.

Page 63 of 117
The “superseding circumstances” mentioned by the Acting Union]. The defiant strikers could then be validly replaced
Secretary of Labor no doubt refer to the final decision of the panel of [Marcopper Mining v. Brillantes].
arbitrators as to the confidential nature of the positions of the twelve private The refusal to acknowledge receipt of the
respondents, thereby rendering their actual and physical reinstatement assumption/certification orders and other processes is an apparent
impracticable and more likely to exacerbate the situation.  The payroll
attempt to frustrate the ends of justice, hence, invalid. The union
reinstatement in lieu of actual reinstatement ordered in these cases,
therefore, appears justified as an exception to the rule until the validity of
cannot be allowed to thwart the efficacy of the said orders issued in
their termination is finally resolved.  This Court sees no grave abuse of the national interest through the simple expediency of refusing to
discretion on the part of the Acting Secretary of Labor in ordering the acknowledge the receipt thereof [Navale v. Court of Appeals].
same.  Furthermore, the issue has not been raised by any party in this case.
Period of Defiance, Not Material
d. Issue Legality of Strike, Immaterial The length of time within which the assumption or
The brazen disregard of the return-to-work order would certification order or return-to-work order was defied by the
render the strike illegal [Union of Filipro Employees v. Nestle]. strikers is not significant in determining their liability therefor. In
Where the return-to-work order is issued pending the determination University of San Agustin Employees’ Union v. Court of Appeals,
of the legality of the strike, it is not correct to say that it may be the period of defiance was less than 9 hours.
enforced only if the strike is legal and may be disregarded if the There is no practice of giving 2 hours to strikers within
strike is illegal. Precisely, the Supreme Court in Asian which to return to work, there is no law or jurisprudence
Transmission Corporation v. NLRC, the purpose of the return-to- recognizing this practice [University of San Agustin Employees’
work order is to maintain the status quo while the determination is Union v. Court of Appeals].
being made. Otherwise, workers who contend that the strike is
legal can refuse to return to their work and use a standstill in the b. As to Employer
company operations while retaining the positions they refuse to In case of non-compliance by the employer with the
discharge or allow management to fill. Worse, they will also claim RWTO issued, he may be held liable to pay backwages, damages
payment for work not done on the ground that they are still legally and other positive or affirmative reliefs, even criminal prosecution
employed although actually engaged in activities inimical to their against him.
employer’s interest.
H. PROHIBITED ACTIVITIES
e. Effect of Compliance of RTWO
The act of the strikers in voluntarily returning to work Article 279. [264] Prohibited activities. - (a) No labor
does not result in the waiver of their original demands. Such act of organization or employer shall declare a strike or lockout without first
returning to work only means that they desisted from the strike having bargained collectively in accordance with Title VII of this Book
or without first having filed the notice required in the preceding Article
which desistance is a personal act of the strikers and cannot be used
or without the necessary strike or lockout vote first having been
against the union and interpreted as a waiver by it of its original obtained and reported to the Ministry.
demands for which the strike was adopted as a weapon [Bisaya No strike or lockout shall be declared after assumption of
Land Transportation v. CIR]. jurisdiction by the President or the Minister or after certification or
In the same breadth, a return-to-work order does not submission of the dispute to compulsory or voluntary arbitration or
generally have the effect of rendering as moot and academic the during the pendency of cases involving the same grounds for the strike
issue of the illegality of the strike [Insurefco Paper v. Insular or lockout.
Sugar]. However, according to Trans-Asia Shipping Lines v. Court Any worker whose employment has been terminated as a
consequence of any unlawful lockout shall be entitled to reinstatement
of Appeals, an employer may be considered to have waived its right
with full backwages. Any union officer who knowingly participates in
to proceed against the striking employees for alleged commission an illegal strike and any worker or union officer who knowingly
of illegal acts during the strike when, during a conference before participates in the commission of illegal acts during a strike may be
the Chairman of the NLRC, it agreed to reinstate them and comply declared to have lost his employment status: Provided, That mere
fully with the RTWO issued by the DOLE Secretary. participation of a worker in a lawful strike shall not constitute
sufficient ground for termination of his employment, even if a
f. Entitlement to Pay replacement had been hired by the employer during such lawful strike.
The return-to-work order should benefit only those (b) No person shall obstruct, impede, or interfere with by
force, violence, coercion, threats or intimidation, any peaceful
workers who complied therewith and, regardless of the outcome of
picketing by employees during any labor controversy or in the exercise
the compulsory arbitration proceedings, are entitled to be paid for of the right to self-organization or collective bargaining, or shall aid or
work they have actually performed.  Conversely, those workers abet such obstruction or interference.
who refused to obey the said order and instead waged the (c) No employer shall use or employ any strike-breaker, nor
restrained strike are not entitled to be paid for work not done or to shall any person be employed as a strike-breaker.
reinstatement to the positions they have abandoned by their refusal (d) No public official or employee, including officers and
to return thereto as ordered [Asian Transmission v. NLRC]. personnel of the New Armed Forces of the Philippines or the
Integrated National Police, or armed person, shall bring in, introduce
or escort in any manner, any individual who seeks to replace strikers in
5. Effect of Defiance of Return-To-Work Order
entering or leaving the premises of a strike area, or work in place of the
strikers. The police force shall keep out of the picket lines unless actual
a. As to the Employees violence or other criminal acts occur therein: Provided, That nothing
Any worker or union officer who defies a return-to-work herein shall be interpreted to prevent any public officer from taking
order may be subject to immediate disciplinary action, including any measure necessary to maintain peace and order, protect life and
dismissal or loss of employment status and even to criminal property, and/or enforce the law and legal orders.
prosecution. (e) No person engaged in picketing shall commit any act of
violence, coercion or intimidation or obstruct the ingress to or egress
From the moment a worker defies a return-to-work order,
from the employer’s premises for lawful purposes, or obstruct public
he is deemed to have abandoned his job [Philippine Airlines v.
thoroughfares.
Brillantes]. By so defying, the workers have forfeited their right to
be readmitted to work [Steel Corporation v. SCP Employees
Page 64 of 117
1. Test of Legality of Strike picketing, sit-down strikes, sympathy strikes or any other form of
In Toyota v. NLRC, the Court noted authority on labor interference and/or interruptions with any of the normal operations of the
law, Ludwig Teller, lists six (6) categories of an illegal strike, viz: HOTEL during the life of this Agreement.
The facts are clear that the strike arose out of a bargaining
deadlock in the CBA negotiations with the Hotel. The concerted action is
(i) [when it] is contrary to a specific prohibition of
an economic strike upon which the afore-quoted "no strike/work stoppage
law, such as strike by employees performing and lockout" prohibition is squarely applicable and legally binding.19
governmental functions; or Third, the Union officers and members' concerted action to
(ii) [when it] violates a specific requirement of shave their heads and crop their hair not only violated the Hotel's Grooming
law[, such as Article 263 of the Labor Code on Standards but also violated the Union's duty and responsibility to bargain in
the requisites of a valid strike]; or good faith. By shaving their heads and cropping their hair, the Union
(iii) [when it] is declared for an unlawful purpose, officers and members violated then Section 6, Rule XIII of the
such as inducing the employer to commit an Implementing Rules of Book V of the Labor Code. 20 This rule prohibits the
commission of any act which will disrupt or impede the early settlement of
unfair labor practice against non-union
the labor disputes that are under conciliation. Since the bargaining deadlock
employees; or is being conciliated by the NCMB, the Union's action to have their officers
(iv) [when it] employs unlawful means in the and members' heads shaved was manifestly calculated to antagonize and
pursuit of its objective, such as a widespread embarrass the Hotel management and in doing so effectively disrupted the
terrorism of non-strikers [for example, operations of the Hotel and violated their duty to bargain collectively in
prohibited acts under Art. 279(e) of the Labor good faith.
Code]; or Fourth, the Union failed to observe the mandatory 30-day
(v) [when it] is declared in violation of an existing cooling-off period and the seven-day strike ban before it conducted the
strike on January 18, 2002. The NLRC correctly held that the Union failed
injunction[, such as injunction, prohibition, or
to observe the mandatory periods before conducting or holding a strike.
order issued by the DOLE Secretary and the Records reveal that the Union filed its Notice of Strike on the ground of
NLRC under Art. 263 of the Labor Code]; or bargaining deadlock on December 20, 2001. The 30-day cooling-off period
(vi) [when it] is contrary to an existing agreement, should have been until January 19, 2002. On top of that, the strike vote was
such as a no-strike clause or conclusive held on January 14, 2002 and was submitted to the NCMB only on January
arbitration clause. 18, 2002; therefore, the 7-day strike ban should have prevented them from
holding a strike until January 25, 2002. The concerted action committed by
the Union on January 18, 2002 which resulted in the disruption of the
NUWHRAIN-APL-IUF Dusit v. Court of Appeals
Hotel's operations clearly violated the above-stated mandatory periods.
Held: With the foregoing parameters as guide and the following
Last, the Union committed illegal acts in the conduct of its
grounds as basis, we hold that the Union is liable for conducting an illegal
strike. The NLRC ruled that the strike was illegal since, as shown by the
strike for the following reasons:
pictures21 presented by the Hotel, the Union officers and members formed
First, the Union's violation of the Hotel's Grooming Standards
human barricades and obstructed the driveway of the Hotel. There is no
was clearly a deliberate and concerted action to undermine the authority of
merit in the Union's argument that it was not its members but the Hotel's
and to embarrass the Hotel and was, therefore, not a protected action. The
security guards and the police officers who blocked the driveway, as it can
appearances of the Hotel employees directly reflect the character and well-
be seen that the guards and/or police officers were just trying to secure the
being of the Hotel, being a five-star hotel that provides service to top-notch
entrance to the Hotel. The pictures clearly demonstrate the tense and highly
clients. Being bald or having cropped hair per se does not evoke negative or
explosive situation brought about by the strikers' presence in the Hotel's
unpleasant feelings. The reality that a substantial number of employees
driveway.
assigned to the food and beverage outlets of the Hotel with full heads of
hair suddenly decided to come to work bald-headed or with cropped hair,
however, suggests that something is amiss and insinuates a sense that 2. Employment of Strike-Breaker
something out of the ordinary is afoot. Obviously, the Hotel does not need "Strike-breaker" means any person who obstructs,
to advertise its labor problems with its clients. It can be gleaned from the impedes, or interferes with by force, violence, coercion, threats, or
records before us that the Union officers and members deliberately and in intimidation any peaceful picketing affecting wages, hours or
apparent concert shaved their heads or cropped their hair. This was shown conditions of work or in the exercise of the right of self-
by the fact that after coming to work on January 18, 2002, some Union
organization or collective bargaining [Article 219(r)].
members even had their heads shaved or their hair cropped at the Union
office in the Hotel's basement. Clearly, the decision to violate the company
rule on grooming was designed and calculated to place the Hotel 3. Role of PNP/AFP Personnel
management on its heels and to force it to agree to the Union's proposals.
In view of the Union's collaborative effort to violate the Hotel's REVISED PHILIPPINE NATIONAL POLICE
Grooming Standards, it succeeded in forcing the Hotel to choose between OPERATIONAL PROCEDURES
allowing its inappropriately hair styled employees to continue working, to RULE 24. RULES ON LABOR DISPUTES
the detriment of its reputation, or to refuse them work, even if it had to 24.1 General Policy and Guidelines
cease operations in affected departments or service units, which in either a. The involvement of PNP personnel during strikes,
way would disrupt the operations of the Hotel. This Court is of the opinion, lockouts and labor disputes in general shall be limited to the
therefore, that the act of the Union was not merely an expression of their maintenance of peace and order, enforcement of laws, and
grievance or displeasure but, indeed, a calibrated and calculated act implementation of legal orders of the duly constituted authorities.
designed to inflict serious damage to the Hotel's finances or its reputation. b. The PNP shall only render assistance to labor disputes
Thus, we hold that the Union's concerted violation of the Hotel's Grooming upon written request addressed to the Regional Director/ District
Standards which resulted in the temporary cessation and disruption of the Director concerned. In case of actual violence, the police can respond
Hotel's operations is an unprotected act and should be considered as an without the written request.
illegal strike. c. Insofar as practicable, no PNP personnel shall be allowed
Second, the Union's concerted action which disrupted the Hotel's to render police assistance in connection with a strike or lockout if
operations clearly violated the CBA's "No Strike, No Lockout" provision, there is question or complaint as regards his relationship by affinity or
which reads: consanguinity to any official/leader of the parties in the controversy or
ARTICLE XXII - NO STRIKE/WORK STOPPAGE AND if he has financial or pecuniary interest therein.
LOCKOUT d. PNP personnel detailed as peace-keeping force in strike or
SECTION 1. No Strikes lockout areas shall be in prescribed uniform at all times.
The Union agrees that there shall be no strikes, walkouts, e. They shall exercise maximum tolerance and when called
stoppage or slow-down of work, boycott, refusal to handle accounts, for by the situation or when all other peaceful and non-violent means

Page 65 of 117
have been exhausted, police officers may employ such means as may be a compulsory arbitration, the same may be resolved by the
necessary and reasonable to prevent or repel an aggression. Secretary or the Commission, respectively [IPI v. Secretary of
f. The matter of determining whether a strike, picket or Labor and Associated Labor Unions].
lockout is legal or not should be left to Department of Labor and Other remedies against prohibited acts are:
Employment (DOLE) and its appropriate agencies. PNP personnel
should not interfere in a strike, picket or lockout, except as herein
provided.
a. Filing a case for ULP or Prohibited before the Labor
g. No personal escort shall be provided to any of the parties Arbiter; or
to the controversy unless upon written request from DOLE. Whenever b. Filing a civil case for damages under the Civil Code
escorts are to be provided, the other party shall be informed or a criminal case under the Labor Code before the
accordingly. All escorts shall be in prescribed uniform at all times. regular courts; or
h. During the pendency of a strike/lockout, the police c. File an injunction under the Labor Code before the
personnel concerned are prohibited from socializing with any of the NLRC.
parties involved in the controversy.
i. Liaison shall be established and maintained with the
representatives of DOLE, management and the union in the
6. Liability of Union Officers and Ordinary
strike/lockout area for the purpose of maintaining peace and order, as Workers/Members
well as to maintain a continuing peaceful dialogue between the parties
to the strike/ lockout. a. Illegal Strike
j. The peace-keeping detail shall not be stationed in the
picket line (or confrontation line) but should be stationed in such (i) Union Officers
manner that their presence may deter the commission of criminal acts The mere finding or declaration of illegality of the strike
or any untoward incident from either side. The members of the
will result in the termination of all union officers who knowingly
peacekeeping detail shall stay outside a 50-meter radius from the
picket line. In cases wherein the 50-meter radius includes a public participated in the illegal strike [Lapanday Workers Union v.
thoroughfare, they may station themselves in such public thoroughfare NLRC]. Unlike ordinary members, it is not required, for purposes
for the purpose of ensuring the free flow of traffic. of termination, that the officers should be proven to have
xxx committed illegal acts during the strike in order to be held liable
therefor [Phimco Industris v. Phimco Industries Labor Union].
4. Hiring of Scabs Otherwise stated, the services of a participating union officer may
A scab is a person who works despite an ongoing strike. be terminated not only when he actually commits an illegal act
Scabs are usually individuals who were not employed by the during a strike, but also if he knowingly participates in the conduct
company prior to the trade union dispute, but rather hired after or and staging of an illegal strike. This is the most logical
during the strike to keep the organization running. consequence for knowingly participating in an illegal strike
The hiring of scabs is not illegal per se. As a general rule, [Abaria v. NLRC].
the hiring of replacements for the strikers during a strike is not an For purposes of identifying the union officers, the
unfair labor practice act of an employer. He is entitled to do it in certifications as to the names of the union officers issued by the
his effort to carry on the business. Such hiring may even be done Chief of the Labor Organization Division of the Bureau of Labor
on a permanent basis in the case of an economic strike. And in the Relations (BLR), being public records, enjoy the presumption of
event that the strikers decide to resume their work, the employer is regularity and deserve weight and probative value. Thus, in the
not duty-bound to dismiss said permanent replacements absence of a clear and convincing evidence that said certifications
[Consolidated Labor v. Marsman]. are flawed, they should be taken on their face value [Coca-Cola
But in an ULP strike, such replacements may not be Bottlers v. NLRC].
permanently employed. The employer is duty-bound to discharge Note that the verb "participates" is preceded by the
them when the strikers are reinstated to their former positions adverb "knowingly." This reflects the intent of the legislature to
[Insular Life Employees v. Insular Life]. require "knowledge" as a condition sine qua non before a union
officer can be dismissed from employment for participating in an
5. Jurisdiction Over Prohibited Acts illegal strike. The provision is worded in such a way as to make it
Article 224(a)(5) of the Labor Code provides: very difficult for employers to circumvent the law by arbitrarily
dismissing employees in the guise of exercising management
Article 224. [217] Jurisdiction of the Labor Arbiters prerogative. This is but one aspect of the State’s constitutional and
and the Commission. (a) Except as otherwise provided under statutory mandate to protect the rights of employees to self-
this Code, the Labor Arbiters shall have original and exclusive organization [Club Filipino v. Bautista].
jurisdiction to hear and decide, within thirty (30) calendar days
after the submission of the case by the parties for decision
(ii) Ordinary Union Members
without extension, even in the absence of stenographic notes, the
following cases involving all workers, whether agricultural or
The fate of ordinary union members is different. Mere
non-agricultural: participation in an illegal strike is not a sufficient ground to
xxx terminate their employment. The mere finding or declaration of
(5) Cases arising from any violation of Article 279 of illegality of a strike will not result in their termination. For a union
this Code, including questions involving the legality of strikes member to suffer the consequence of loss of employment, it must
and lockouts; and be shown by substantial evidence that he has knowingly
xxx participated in the commission of illegal acts during the strike
[Fadriquelan v. Monterey Foods]. Obviously, the Labor Code
In general, the Labor Arbiter in the appropriate protects ordinary union members who participated in such an
Arbitration Branch of the NLRC has the power to determine illegal strike from losing their jobs provided that they did not
questions involving the legality or illegality of a strike or lockout commit illegal acts in the course thereof [G&S Transport v.
upon the filing of a proper complaint and after due hearing. Infante].
Where the matter of legality or illegality of a strike is
raised in the dispute which the Secretary assumed jurisdiction or in

Page 66 of 117
Absent any showing that the employees are union
officers, they cannot be dismissed based sole on the declaration of c. Payment of Backwages
the illegality of the strike [Gold City Integrated v. NLRC]. Generally, strikers are not entitled to backwages. The
following are the exceptions:
b. Illegal Acts
As far as liability for commission of illegal acts during a. If ordered by the labor court in case of a legal strike
the strike is concerned, the issue of legality or illegality of the only (not if illegal strike);
strike is irrelevant. As long as the union officer or member b. If striker is not guilty of illegal strike and is not in
commits an illegal act in the course of the strike, be it legal or pari delicto with the employer;
illegal, his employment can be validly terminated [Toyota Motor c. If strikers voluntarily abandon the legal strike to
Phils. Corp. Workers Association v. NLRC]. Mere substantial return to work but are illegally locked out,
evidence is required to hold strikers guilty of commission of illegal suspended or dismissed by the employer;
acts [Phimco Industries v. Phimco Industries Labor Association]. d. If employee is not reinstated despite RTWO;
Liability for illegal acts should be determined on an e. If ULP termination (by reason of discrimination).
individual basis. For this purpose, the individual identity of the
union members who participated in the commission of illegal acts PMR Officers’ Association v. CIR
may be proved thru affidavits and photographs [Ibid]. Simply Held: The basic facts are stated in the opinion of the three
referring to them as “strikers,” or “complainants in this case” is not judges who denied the backpay claim:jgc:chanrobles.com.ph
enough to justify their dismissal [G&S Transport v. Infante]. "On February 25, 1954, this court issued an order setting the
Only members who are identified as having participated hearing of the case on March 1, 1954. At the hearing, Atty. Cipriano Cid,
then counsel for the petitioner, was asked several times by the trial Court
in the commission of illegal acts are liable. Those who did not
and by counsel of Compañia Maritima whether he wanted the strikers to be
participate should be blamed therefor [Phimco Industries v. ordered back to work, and his reply was: ‘We have not asked that yet. (p.
Phimco Industries Labor Association]. 15-19 hearing on March 1, 1954). And when counsel for PSNCO directly
However, a different conclusion would be called for if asked him if he wanted the strikers to go back to work, his answer in effect
the existence of force or other prohibited activities is pervasive and was, if ordered by the Court. . . ." (p. 5, Annex F-1 to Petition.)
widespread, consistently and deliberately resorted to as a matter of It is clear from the above that the petitioner union never
policy by the union. In which case, the strike becomes illegal even demanded the privilege to have its members reinstated to their positions
if the strike is justified as to its ends [Shell Oil Workers v. Shell immediately, but that they left this matter of their return to the discretion of
the court. The court, on the other hand, did not order the return of the
Oil].
strikers; it did so only in its decision after the hearing and termination of the
case.
7. Liability of Employers In Case of Strikes Under the circumstances as above indicated it is apparent that
the strikers never expressed a desire or willingness to return back to work,
a. Liability for Reinstatement (Illegal Strike) leaving that to the court’s discretion. The denial of backpay to the strikers is
If the employee participated in prohibited acts, the clearly justified, in accordance with previous decisions of this Court.
employer may not reinstate the dismissed employee. However, if
the employee did not participate in the prohibited acts, the Philippine Diamond v. Manila Diamond Hotel
employer must reinstate, Employees
Held: This Court must thus hearken to its policy that "when
b. Separation Pay employees voluntarily go on strike, even if in protest against unfair labor
If the employee participated in prohibited acts, the practices," no backwages during the strike is awarded.
employer, even if as a financial assistance, the employer is not In Cromwell Commercial Employees and Laborers Union
(PTUC) v. Court of Industrial Relations, this Court made a distinction
liable for payment of separation pay.
between two types of employees involved in a ULP: those who are
The alternative relief for union members who were
discriminatorily dismissed for union activities, and those who voluntarily
dismissed for having participated in an illegal strike is the payment go on strike even if it is in protest of an ULP. Discriminatorily dismissed
of separation pay in lieu of reinstatement under the following employees were ordered entitled to backpay from the date of the act of
circumstances: (a) when reinstatement can no longer be effected in discrimination, that is, from the day of their discharge, whereas employees
view of the passage of a long period of time or because of the who struck as a voluntary act of protest against what they considered a ULP
realities of the situation; (b) reinstatement is inimical to the of their employer were held generally not entitled to backpay.
employer’s interest; (c) reinstatement is no longer feasible; (d) Jurisprudential law, however, recognizes several exceptions to
the "no backwages rule," to wit: when the employees were illegally locked
reinstatement does not serve the best interests of the parties
to thus compel them to stage a strike; when the employer is guilty of the
involved; (e) the employer is prejudiced by the workers’ continued grossest form of ULP; when the employer committed discrimination in the
employment; (f) facts that make execution unjust or inequitable rehiring of strikers refusing to readmit those against whom there were
have supervened; or (g) strained relations between the employer pending criminal cases while admitting nonstrikers who were also
and employee. criminally charged in court; or when the workers who staged a voluntary
ULP strike offered to return to work unconditionally but the employer
Waiver refused to reinstate them. Not any of these or analogous instances is,
The grant of separation benefits is not condonation while however, present in the instant case.
Respondent urges this Court to apply the exceptional rule
the determination of illegality of strike was still pending because it
enunciated in Philippine Marine Officers’ Guild v. Compañia Maritima and
was paid only to comply with the employer’s legal obligation similar cases where the employees unconditionally offered to return to
[Interphil Lab Employees v. Interphil]. work, it arguing that there was such an offer on its part to return to work but
However, in Citizens Labor Union v. Standard Vacuum the Hotel screened the returning strikers and refused to readmit those whom
Oil, it was held that the act of voluntary reinstatement of workers it found to have perpetrated prohibited acts during the strike.
constitutes a waiver of the illegality of the strike. The difference It must be stressed, however, that for the exception in Philippine
between the two cases is that in the latter, the act of the employer is Marine Officers’ Guild to apply, it is required that the strike must be legal.65
voluntary, whereas in the former the act of the employer was done Reinstatement without backwages of striking members of
respondent who did not commit illegal acts would thus suffice under the
in compliance of its legal obligation.
circumstances of the case. If reinstatement is no longer possible, given the
Page 67 of 117
lapse of considerable time from the occurrence of the strike, the award of
separation pay of one (1) month salary for each year of service, in lieu of No limitation in the law was placed upon the power
reinstatement, is in order. of the DOLE to determine the existence of an employer-
employee relationship.  No procedure was laid down where the
d. Payment of Attorney’s Fees DOLE would only make a preliminary finding, that the power
was primarily held by the NLRC.  The law did not say that the
Strikers who are illegally dismissed are entitled to
DOLE would first seek the NLRC’s determination of the
Attorney’s Fees. existence of an employer-employee relationship, or that should
the existence of the employer-employee relationship be
VII disputed, the DOLE would refer the matter to the NLRC.  The
JURISDICTION DOLE must have the power to determine whether or not an
employer-employee relationship exists, and from there to decide
whether or not to issue compliance orders in accordance with
A. PRELIMINARY CONSIDERATIONS ON Art. 128(b) of the Labor Code, as amended by RA 7730.
JURISDICTION AND REMEDIES The DOLE, in determining the existence of an
employer-employee relationship, has a ready set of guidelines to
1. Existence of Employer-Employee Relationship follow, the same guide the courts themselves use.  The elements
The existence of employer-employee relationship to determine the existence of an employment relationship are:
between the parties-litigants, or a reasonable causal connection to (1) the selection and engagement of the employee; (2) the
such relationship is a jurisdictional pre-requisite for the exercise of payment of wages; (3) the power of dismissal; (4) the
employer’s power to control the employee’s conduct. The use of
jurisdiction over a labor dispute by the Labor Arbiters [Uy v.
this test is not solely limited to the NLRC. The DOLE Secretary,
Bueno] or any other labor tribunals. or his or her representatives, can utilize the same test, even in
Even if there is employer-employee relationship, if the the course of inspection, making use of the same evidence that
cause of action did not arise out of or was not incurred in the would have been presented before the NLRC.
connection with the employer-employee relationship, Labor The determination of the existence of an employer-
Arbiters have no jurisdiction thereover [Pondoc v. NLRC]. This is employee relationship by the DOLE must be respected.  The
so because not every dispute between an employer and employee expanded visitorial and enforcement power of the DOLE
involves matters that only labor tribunals like the Labor Arbiters granted by RA 7730 would be rendered nugatory if the alleged
employer could, by the simple expedient of disputing the
and the NLRC can resolve in the exercise of their adjudicatory or
employer-employee relationship, force the referral of the matter
quasi-judicial power. Actions between employers and employees to the NLRC.  The Court issued the declaration that at least a
where the employer-employee relationship is merely incidental are prima facie showing of the absence of an employer-employee
within the exclusive original jurisdiction of the regular courts relationship be made to oust the DOLE of jurisdiction.  But it is
[Villamaria, Jr. v. Court of Appeals]. precisely the DOLE that will be faced with that evidence, and it
is the DOLE that will weigh it, to see if the same does
Exception of OFW Cases successfully refute the existence of an employer-employee
In cases filed by OFWs, the Labor Arbiters may exercise relationship.
If the DOLE makes a finding that there is an existing
jurisdiction even absent the employment relationship. In Santiago
employer-employee relationship, it takes cognizance of the
v. CF Sharp Crew Management, Inc., it was held that a seafarer matter, to the exclusion of the NLRC.  The DOLE would have
who has already signed a POEA-approved employment contract no jurisdiction only if the employer-employee relationship has
but was not deployed overseas and, therefore, there is no employer- already been terminated, or it appears, upon review, that no
employee relationship, may file his monetary claims case with the employer-employee relationship existed in the first place.
Labor Arbiter. This is because the jurisdiction of the Labor The Court, in limiting the power of the DOLE, gave
Arbiters is not limited to claims arising from employer-employee the rationale that such limitation would eliminate the prospect of
relationships. Under Section 10 of RA 8042, the Labor Arbiter may competing conclusions between the DOLE and the NLRC.  The
prospect of competing conclusions could just as well have been
exercise jurisdiction over the claims of OFWs arising out of an
eliminated by according respect to the DOLE findings, to the
employer-employee relationship or by virtue of any law or contract exclusion of the NLRC, and this We believe is the more prudent
involving Filipino workers for overseas deployment, including course of action to take.
claims for actual, moral, exemplary and other forms of damage. This is not to say that the determination by the
DOLE is beyond question or review.  Suffice it to say, there are
2. Reasonable Causal Connection Rule judicial remedies such as a petition for certiorari under Rule 65
The Reasonable Causal Connection Rule is a rule to that may be availed of, should a party wish to dispute the
determine jurisdiction between labor courts or regular courts. findings of the DOLE.
It must also be remembered that the power of the
Under this rule, if there is a reasonable causal connection between
DOLE to determine the existence of an employer-employee
the claim asserted and the employer-employee relations, then the relationship need not necessarily result in an affirmative
case is within the jurisdiction of the labor courts [Dai-ichi finding.  The DOLE may well make the determination that no
Electronics v. Villarama, Jr]. In the absence of such nexus, it is the employer-employee relationship exists, thus divesting itself of
regular courts that have jurisdiction [San Miguel Corporation v. jurisdiction over the case.  It must not be precluded from being
Etcuban]. able to reach its own conclusions, not by the parties, and
certainly not by this Court.
3. Power to Determine Employment Relationship Under Art. 128(b) of the Labor Code, as amended by
RA 7730, the DOLE is fully empowered to make a
Under labor laws, it is not only the Labor Arbiters and
determination as to the existence of an employer-employee
the NLRC that are vested with the power to determine the relationship in the exercise of its visitorial and enforcement
existence of employer-employee relationship. power, subject to judicial review, not review by the NLRC.
There is a view that despite Art. 128(b) of the Labor
a. DOLE Secretary and the DOLE Regional Code, as amended by RA 7730, there is still a threshold amount
Directors set by Arts. 129 and 217 of the Labor Code when money claims
In People’s Broadcasting Service v. Secretary, it was are involved, i.e., that if it is for PhP 5,000 and below, the
held: jurisdiction is with the regional director of the DOLE, under Art.

Page 68 of 117
129, and if the amount involved exceeds PhP 5,000, the existence of the employer-employee relationship. The SSC also has
jurisdiction is with the labor arbiter, under Art. 217.  The view that power.
states that despite the wording of Art. 128(b), this would only
apply in the course of regular inspections undertaken by the
B. LABOR ARBITER
DOLE, as differentiated from cases under Arts. 129 and 217,
which originate from complaints.  There are several cases, The Labor Arbiter is an official in the Regional
however, where the Court has ruled that Art. 128(b) has been Arbitration Branch of the NLRC who hears and decides cases
amended to expand the powers of the DOLE Secretary and his falling under his original and exclusive jurisdiction as provided by
duly authorized representatives by RA 7730.   In these cases, the law.
Court resolved that the DOLE had the jurisdiction, despite the Besides their adjudicatory power to hear and decide cases
amount of the money claims involved.  Furthermore, in these over which they have jurisdiction, the Labor Arbiters have (1)
cases, the inspection held by the DOLE regional director was contempt power; and (2) power to conduct ocular inspection.
prompted specifically by a complaint.  Therefore, the initiation
However, the Labor Arbiter has no more injunctive power. Only
of a case through a complaint does not divest the DOLE
Secretary or his duly authorized representative of jurisdiction the NLRC has that power.
under Art. 128(b). The jurisdiction conferred upon the Labor Arbiters is
To recapitulate, if a complaint is brought before the both original and exclusive. This means that as a general rule, no
DOLE to give effect to the labor standards provisions of the other officers or tribunals can take cognizance of, or hear and
Labor Code or other labor legislation, and there is a finding by decide, any of the cases therein enumerated.
the DOLE that there is an existing employer-employee However, the following are the exceptions to the exercise
relationship, the DOLE exercises jurisdiction to the exclusion of of original and exclusive jurisdiction of Labor Arbiters:
the NLRC.  If the DOLE finds that there is no employer-
employee relationship, the jurisdiction is properly with the
NLRC.  If a complaint is filed with the DOLE, and it is a. When the DOLE Secretary or the President
accompanied by a claim for reinstatement, the jurisdiction is exercises his power under Article 278(g) of the
properly with the Labor Arbiter, under Art. 217(3) of the Labor Labor Code to assume jurisdiction over national
Code, which provides that the Labor Arbiter has original and interest cases and decide them himself.
exclusive jurisdiction over those cases involving wages, rates of b. When the NLRC exercises its power of compulsory
pay, hours of work, and other terms and conditions of arbitration over similar national interest cases that
employment, if accompanied by a claim for reinstatement.  If a are certified to it by the DOLE Secretary pursuant to
complaint is filed with the NLRC, and there is still an existing
the exercise by the latter of his certification power
employer-employee relationship, the jurisdiction is properly
with the DOLE.  The findings of the DOLE, however, may still under the same Article.
be questioned through a petition for certiorari under Rule 65 of c. When cases arise from the interpretation or
the Rules of Court. implementation of collective bargaining agreements
In the present case, the finding of the DOLE and from the interpretation of enforcement of
Regional Director that there was an employer-employee company personnel policies which shall be disposed
relationship has been subjected to review by this Court, with the of by the Labor Arbiter by referring the same to the
finding being that there was no employer-employee relationship grievance machinery and voluntary arbitration, as
between petitioner and private respondent, based on the
may be provided in said agreements.
evidence presented.  Private respondent presented self-serving
allegations as well as self-defeating evidence. The findings of d. When the parties agree to submit the case to
the Regional Director were not based on substantial evidence, voluntary arbitration before a Voluntary Arbitrator
and private respondent failed to prove the existence of an or panel of Voluntary Arbitrators who, under
employer-employee relationship.  The DOLE had no jurisdiction Articles 274 and 275, are also possessed of original
over the case, as there was no employer-employee relationship and exclusive jurisdiction to hear and decide cases
present.  Thus, the dismissal of the complaint against petitioner mutually submitted to them by the parties for
is proper. arbitration and adjudication.

b. Med-Arbiters 1. Jurisdiction of the Labor Arbiters


As the authority to determine the employer-employee
relationship is necessary and indispensable in the exercise of
Article 224. [217] Jurisdiction of the Labor Arbiters and the
jurisdiction by the med-arbiter, his finding thereon may only be Commission. (a) Except as otherwise provided under this Code, the
reviewed and reversed by the Secretary of Labor who exercises Labor Arbiters shall have original and exclusive jurisdiction to hear
appellate jurisdiction. It is absurd to suggest that the med-arbiter and decide, within thirty (30) calendar days after the submission of the
and Secretary of Labor cannot make their own independent finding case by the parties for decision without extension, even in the absence
as to the existence of such relationship and must have to rely and of stenographic notes, the following cases involving all workers,
wait for such a determination by the labor arbiter or NLRC in a whether agricultural or non-agricultural:
separate proceeding.  For then, given a situation where there is no (1) Unfair labor practice cases;
(2) Termination disputes;
separate complaint filed with the labor arbiter, the med-arbiter
(3) If accompanied with a claim for reinstatement, those
and/or the Secretary of Labor can never decide a certification cases that workers may file involving wages, rates of pay, hours of
election case or any labor-management dispute properly brought work and other terms and conditions of employment;
before them as they have no authority to determine the existence of (4) Claims for actual, moral, exemplary and other forms of
an employer-employee relationship.  Such a proposition is, to say damages arising from the employer-employee relations;
the least, anomalous [M.Y. San Biscuits v. Laguesma]. (5) Cases arising from any violation of Article 264 of this
Code, including questions involving the legality of strikes and lockouts;
and
c. Social Security Commission (SSC)
(6) Except claims for Employees Compensation, Social
The SSC is also vested with this power. In Republic v.
Security, Medicare and maternity benefits, all other claims arising
Asiapro Cooperative, involving the issue of coverage of owner- from employer-employee relations, including those of persons in
members of respondent cooperative under the Social Security domestic or household service, involving an amount exceeding five
System (SSS), it was held that it is not only the Labor Arbiter or
the NLRC that has the exclusive jurisdiction to determine the
Page 69 of 117
thousand pesos (P5,000.00) regardless of whether accompanied with a that their cases still fell under the grievance machinery. According to
claim for reinstatement. petitioner, without having exhausted said machinery, the private
xxx respondents filed their action before the NLRC, in a clear act of forum-
(c) Cases arising from the interpretation or implementation shopping. However, it is worth pointing out that private respondents went
of collective bargaining agreements and those arising from the to the NLRC only after the labor arbiter dismissed their original complaint
interpretation or enforcement of company personnel policies shall be for illegal dismissal. Under these circumstances private respondents had to
disposed of by the Labor Arbiter by referring the same to the find another avenue for redress. We agree with the NLRC that it was
grievance machinery and voluntary arbitration as may be provided in petitioner who failed to show proof that it took steps to convene the
said agreements. grievance machinery after the labor arbiter first dismissed the complaints
for illegal dismissal and directed the parties to avail of the grievance
a. Jurisdiction over ULP Cases procedure under Article VII of the existing CBA. They could not now be
faulted for attempting to find an impartial forum, after petitioner failed to
The Labor Arbiters have jurisdiction only on the civil
listen to them and after the intercession of the labor arbiter proved futile.
aspect of ULP which may include claims for actual, moral, The NLRC had aptly concluded in part that private respondents had already
exemplary and other forms of damages, attorney’s fees and other exhausted the remedies under the grievance procedure. It erred only in
affirmative reliefs. It must be noted that recovery of civil liability finding that their cause of action was ripe for arbitration.
in the administrative proceeding before the LA bars recovery under In the case of Maneja vs. NLRC, we held that the dismissal case
the Civil Code. does not fall within the phrase “grievances arising from the interpretation or
implementation of the collective bargaining agreement and those
b. Jurisdiction over Termination Disputes arising from the interpretation or enforcement of company personnel
policies.” In Maneja, the hotel employee was dismissed without hearing.
The validity of the exercise of jurisdiction by Labor
We ruled that her dismissal was unjustified, and her right to due process
Arbiters over illegal dismissal cases is not dependent on the kind or was violated, absent the twin requirements of notice and hearing. We also
nature of the ground cited in support of the dismissal; hence, held that the labor arbiter had original and exclusive jurisdiction over the
whether the dismissal is for just cause or authorized cause, it is of termination case, and that it was error to give the voluntary arbitrator
no consequence [C. Alcantara & Sons v. Court of Appeals]. jurisdiction over the illegal dismissal case.
In case of conflict of jurisdiction between Labor Arbiter In Vivero vs. CA,  private respondents attempted to justify the
and the Voluntary Arbitrator over termination cases, the former’s jurisdiction of the voluntary arbitrator over a termination dispute alleging
jurisdiction shall prevail for the following reasons: that the issue involved the interpretation and implementation of the
grievance procedure in the CBA. There, we held that since what was
challenged was the legality of the employee’s dismissal for lack of cause
(i) Termination of employment is not a grievable and lack of due process, the case was primarily a termination dispute. The
issue that must be submitted to the grievance issue of whether there was proper interpretation and implementation of the
machinery or voluntary arbitration for CBA provisions came into play only because the grievance procedure in the
adjudication [Navarro III v. Damasco]. The CBA was not observed, after he sought his union’s assistance. Since the
jurisdiction thereover remains within the real issue then was whether there was a valid termination, there was no
original and exclusive ambit of the Labor reason to invoke the need to interpret nor question an implementation of
any CBA provision.
Arbiter and not of the Voluntary Arbitrator
One significant fact in the present petition also needs stressing.
[Maneja v. NLRC].
Pursuant to Article 260 of the Labor Code, the parties to a CBA shall name
(ii) Even if the CBA provides that termination or designate their respective representatives to the grievance machinery and
disputes are grievable, the same is merely if the grievance is unsettled in that level, it shall automatically be referred
discretionary on the part of the parties thereto to the voluntary arbitrators designated in advance by the parties to a CBA.
[San Miguel Corporation v. NLRC]. Consequently only disputes involving the union and the company shall be
(iii) Once there is actual termination, jurisdiction is referred to the grievance machinery or voluntary arbitrators. In these
conferred upon Labor Arbiters by operation of termination cases of private respondents, the union had no participation, it
having failed to object to the dismissal of the employees concerned by the
law [Atlas Farms v. NLRC].
petitioner. It is obvious that arbitration without the union’s active
(iv) Interpretation of CBA and enforcement of
participation on behalf of the dismissed employees would be pointless, or
company personnel policies are merely even prejudicial to their cause.
corollary to an illegal dismissal case [Maneja Coming to the merits of the petition, the NLRC found that
v. NLRC]. petitioner did not comply with the requirements of a valid dismissal. For a
(v) Article 224 is deemed written into the CBA dismissal to be valid, the employer must show that: (1) the employee was
being an intrinsic part thereof [Landtex accorded due process, and (2) the dismissal must be for any of the valid
Industries v. Court of Appeals]. causes provided for by law. No evidence was shown that private
respondents refused, as alleged, to receive the notices requiring them to
show cause why no disciplinary action should be taken against them.
In other words, a Voluntary Arbitrator will only have Without proof of notice, private respondents who were subsequently
jurisdiction over illegal dismissal cases when there is express dismissed without hearing were also deprived of a chance to air their side at
agreement of the parties in the CBA, i.e., the employer and the the level of the grievance machinery. Given the fact of dismissal, it can be
bargaining agent, to submit the termination case to voluntary said that the cases were effectively removed from the jurisdiction of the
arbitration. Absent the express mutual agreement of the parties, the voluntary arbitrator, thus placing them within the jurisdiction of the labor
Voluntary Arbitrator cannot acquire jurisdiction over termination arbiter. Where the dispute is just in the interpretation, implementation or
cases [Maneja v. NLRC]. enforcement stage, it may be referred to the grievance machinery set up in
the CBA, or brought to voluntary arbitration. But, where there was already
The express agreement must be stated in the CBA or, in
actual termination, with alleged violation of the employee’s rights, it is
its absence, there must be enough evidence on record unmistakably already cognizable by the labor arbiter.
showing that the parties have agreed to resort to voluntary In sum, we conclude that the labor arbiter and then the NLRC
arbitration [University of the Immaculate Conception v. NLRC]. had jurisdiction over the cases involving private respondents’ dismissal,
and no error was committed by the appellate court in upholding their
Atlas Farms v. NLRC assumption of jurisdiction.
Held: Records show, however, that private respondents sought
without success to avail of the grievance procedure in their CBA. On this c. Jurisdiction over Money Claims Cases
point, petitioner maintains that by so doing, private respondents recognized

Page 70 of 117
Money claims falling within the original and exclusive this case, therefore, arose out of or in connection with his employment
jurisdiction of the Labor Arbiters may be classified as follows: relationship with petitioner.

(i) Any money claim, regardless of amount, Labor Arbiter vs. Voluntary Arbitrator
accompanied with a claim for reinstatement; or The original and exclusive jurisdiction of the Labor
(ii) Any money claim, regardless of whether Arbiters under Article 224(c), over cases for money claims is
accompanied with a claim for reinstatement, limited only to those arising from statutes or contracts other than a
exceeding the amount of P5,000.00. CBA. The voluntary Arbitrators, under Article 274, have original
and exclusive jurisdiction over money claims “arising from the
Money claims must arise out of employer-employee interpretation or implementation of the CBA and, those arising
relationship [San Miguel Corporation v. NLRC]. If not, jurisdiction from the interpretation or enforcement of company personnel
is with the regular courts [Lapanday Agricultural v. Court of policies.”
Appeals]. San Jose v. NLRC ruled that it was correct for the NLRC
The money claim in Item (i) above presupposes that it to hold that the Labor Arbiter has no jurisdiction to hear and decide
proceeds from a termination case, it being accompanied with a the employee’s money claims (underpayment of retirement
claim for reinstatement. Hence, it falls within the jurisdiction of the benefits), as the controversy between the parties involved an issue
Labor Arbiter since it is principally a termination dispute. “arising from the interpretation or implementation” of a provision
The money claim in item (ii) above does not necessarily of the CBA. The Voluntary Arbitrator has original and exclusive
arise from or involve a termination case btu because the amount jurisdiction over this controversy under Article 274.
exceeds P5,000.00, it falls within the jurisdiction of the Labor
Arbiter. If the amount does not exceed P5,000.00, it is the Regional Jurisdiction over Contested Cases under the Exception
Director of the DOLE or his duly authorized hearing officers who Clause in Article 128(b)
have jurisdiction to take cognizance thereof [Article 129]. Article 128(b) provides:
The award of statutory benefits even if not prayed for is
valid [Oasis Academy v. DOLE]. Article 128. Visitorial and Enforcement Power. xxx
xxx
(b) Notwithstanding the provisions of Articles 129
San Miguel Corporation v. NLRC
and 21789 of this Code to the contrary, and in cases where the
Held: While paragraph 3 above refers to "all money claims of
relationship of employer-employee still exists, the Secretary of
workers," it is not necessary to suppose that the entire universe of money
Labor and Employment or his duly authorized representatives
claims that might be asserted by workers against their employers has been
shall have the power to issue compliance orders to give effect to
absorbed into the original and exclusive jurisdiction of Labor Arbiters. In
the labor standards provisions of this Code and other labor
the first place, paragraph 3 should be read not in isolation from but rather
legislation based on the findings of labor employment and
within the context formed by paragraph 1 related to unfair labor practices),
enforcement officers or industrial safety engineers made in the
paragraph 2 (relating to claims concerning terms and conditions of
course of inspection. The Secretary or his duly authorized
employment), paragraph 4 (claims relating to household services, a
representatives shall issue writs of execution to the appropriate
particular species of employer-employee relations), and paragraph 5
authority for the enforcement of their orders, except in cases
(relating to certain activities prohibited to employees or to
where the employer contests the findings of the labor
employers).<äre||anº•1àw>  It is evident that there is a unifying element
employment and enforcement officer and raises issues
which runs through paragraphs 1 to 5 and that is, that they all refer to cases
supported by documentary proofs which were not
or disputes arising out of or in connection with an employer-employee
considered in the course of inspection.
relationship. This is, in other words, a situation where the rule of noscitur a
xxx
sociis may be usefully invoked in clarifying the scope of paragraph 3, and
any other paragraph of Article 217 of the Labor Code, as amended. We
reach the above conclusion from an examination of the terms themselves of The above highlighted portion of Article 128(b)
Article 217, as last amended by B.P. Blg. 227, and even though earlier providing for the exception grants jurisdiction to Labor Arbiters
versions of Article 217 of the Labor Code expressly brought within the over contested cases falling thereunder.
jurisdiction of the Labor Arbiters and the NLRC "cases arising from In interpreting the afore-quoted provision of the
employer employee relations," 6 which clause was not expressly carried exception clause, three elements must concur to divest the Regional
over, in printer's ink, in Article 217 as it exists today. For it cannot be Directors or their representatives of jurisdiction thereunder, to wit:
presumed that money claims of workers which do not arise out of or in
connection with their employer-employee relationship, and which would
therefore fall within the general jurisdiction of the regular courts of justice, (i) That the employer contests the findings of the
were intended by the legislative authority to be taken away from the labor inspector and raises issues thereon;
jurisdiction of the courts and lodged with Labor Arbiters on an exclusive (ii) That in order to resolve such issues, there is a
basis. The Court, therefore, believes and so holds that the money claims of need to examine evidentiary matters; and
workers" referred to in paragraph 3 of Article 217 embraces money claims (iii) That such matters are not verifiable in the
which arise out of or in connection with the employer-employee normal course of inspection [Ex-Bataan
relationship, or some aspect or incident of such relationship. Put a little Veterans v. Laguesma].
differently, that money claims of workers which now fall within the
original and exclusive jurisdiction of Labor Arbiters are those money
claims which have some reasonable causal connection with the employer- Reluctantly, if the said elements are present and therefore
employee relationship. the labor standards case is covered by said exception clause, then
Applying the foregoing reading to the present case, we note that the Regional Director will have to endorse the case to the Labor
petitioner's Innovation Program is an employee incentive scheme offered Arbiters of the NLRC [Ex-Bataan Veterans v. Laguesma].
and open only to employees of petitioner Corporation, more specifically to
employees below the rank of manager. Without the existing employer- d. Jurisdiction over Claims for Damages
employee relationship between the parties here, there would have been no It is now well settled that claims for damages as well as
occasion to consider the petitioner's Innovation Program or the submission
attorney’s fees in labor cases are cognizable by the Labor Arbiters,
by Mr. Vega of his proposal concerning beer grande; without that
relationship, private respondent Vega's suit against petitioner Corporation to the exclusion of all other courts. Rulings to the contrary are
would never have arisen. The money claim of private respondent Vega in deemed abandoned or modified accordingly [Primero v. IAC]. No

Page 71 of 117
matter how designated, for as long as the action primarily involves
an employer-employee relationship, the labor court has jurisdiction 4. Jurisdiction Over Execution and Enforcement of
over any damage claims [Rodriguez, Jr. v. Aguilar, Sr.]. Decisions of Voluntary Arbitrators
It is now well settled that money claims of workers Article 276 prescribes the procedures that Voluntary
provided by law over which the labor arbiter has original and Arbitrators should follow in adjudicating cases filed before them.
exclusive jurisdiction are comprehensive enough to include claims Once a decision has been rendered in a case and subsequently
for moral damages of a dismissed employee against his employer. becomes final and executory, it may be enforced through the writ
The Labor Arbiter has jurisdiction to award to the dismissed of execution issued by the same Voluntary Arbitrator who rendered
employee not only the reliefs specifically provided by labor laws, it, addressed to and requiring certain public officers to execute the
but also moral and the forms of damages governed by the Civil final decision, order or award.
Code. Moral damages would be recoverable, for example, where In situations, however, where the VA who rendered the
the dismissal of the employee was not only effected without decision is absent or incapacitated for any reason, Article 276
authorized cause and/or due process - for which relief is granted by grants jurisdiction to any Labor Arbiter in the region where the
the Labor Code — but was attended by bad faith or fraud, or winning party resides, to take cognizance of a motion for the
constituted an act oppressive to labor, or was done in a manner issuance of the writ of execution filed by such party and
contrary to morals, good customs or public policy — for which the accordingly issue such writ addressed to and requiring the public
obtainable relief is determined by ‘the Civil Code [Suario v. BPI]. officers to execute the final decision, order or award of the VA.

e. Jurisdiction over Legality of Strikes and 5. Jurisdiction Over Cases of Overseas Filipino
Lockouts Workers
In general, the Labor Arbiter has the power to determine RA 8042 confers original and exclusive jurisdiction upon
questions involving the legality or illegality of a strike or lockout Labor Arbiters, to hear and decide all claims arising from
upon the filing of a proper complaint and after due proceedings. employment relationship or by virtue of any law or contract
The employer, in case of a strike, or the union, in case of involving OFWs, including claims for actual, moral, exemplary
a lockout, may file the proper petition with the Labor Arbiter to and other forms of damages.
seek a declaration of the illegality thereof. It shall be the duty of the If there is a CBA between the foreign employer and the
Labor Arbiter concerned to act on the case immediately and bargaining union of the OFWs, the jurisdiction over monetary
dispose of the same, subject only to the requirements of due claims of OFWs is vested in the Voluntary Arbitrator and not in the
process. Labor Arbiter [Ace Navigation v. Fernandez].

2. Jurisdiction Over Cases Involving Legislated PNB v. Cabansag


Wage Increases and Wage Distortions Held: More specifically, Section 10 of RA 8042 reads in part:
In establishments where there are no CBAs or certified “SECTION 10. Money Claims. — Notwithstanding any
SEBAs, the Labor Arbiters have jurisdiction to hear and decide provision of law to the contrary, the Labor Arbiters of the National Labor
wage distortion cases after the parties and the NCMB failed to Relations Commission (NLRC) shall have the original and exclusive
jurisdiction to hear and decide, within ninety (90) calendar days after the
correct the distortion [Article 124].
filing of the complaint, the claims arising out of an employer-employee
In establishments where there are existing CBAs or relationship or by virtue of any law or contract involving Filipino workers
SEBAs, RA 6727 vests upon the VA the jurisdiction to hear and for overseas deployment including claims for actual, moral, exemplary and
decide wage distortion cases, after the grievance procedure in the other forms of damages.
CBA failed to settle the same [Ibid]. x x x                          x x x                   x x x”
Based on the foregoing provisions, labor arbiters clearly
3. Jurisdiction Over Enforcement or Annulment of have original and exclusive jurisdiction over claims arising from employer-
Compromise Agreements employee relations, including termination disputes involving all workers,
among whom are overseas Filipino workers (OFW).[15]
We are not unmindful of the fact that respondent was directly
Article 233. [227] Compromise Agreements. Any compromise hired, while on a tourist status in Singapore, by the PNB branch in that city
settlement, including those involving labor standard laws, voluntarily state. Prior to employing respondent, petitioner had to obtain an
agreed upon by the parties with the assistance of the Bureau or the employment pass for her from the Singapore Ministry of Manpower.
regional office of the Department of Labor, shall be final and binding Securing the pass was a regulatory requirement pursuant to the immigration
upon the parties. The National Labor Relations Commission or any regulations of that country.
court, shall not assume jurisdiction over issues involved therein except Similarly, the Philippine government requires non-Filipinos
in case of non-compliance thereof or if there is prima facie evidence working in the country to first obtain a local work permit in order to be
that the settlement was obtained through fraud, misrepresentation, or legally employed here. That permit, however, does not automatically mean
coercion. that the non-citizen is thereby bound by local laws only, as averred by
petitioner. It does not at all imply a waiver of one’s national laws on labor.
It is clear from the foregoing provision that although the Absent any clear and convincing evidence to the contrary, such permit
compromise agreement may have been entered into by the parties simply means that its holder has a legal status as a worker in the issuing
before the BLR or the DOLE Regional office, it is the Labor country.
Arbiter who has jurisdiction to take cognizance of the following Noteworthy is the fact that respondent likewise applied for and
secured an Overseas Employment Certificate from the POEA through the
issues related thereto, to the exclusion of the BLR and the DOLE
Philippine Embassy in Singapore. The Certificate, issued on March 8, 1999,
Regional Directors: declared her a bona fide contract worker for Singapore. Under Philippine
law, this document authorized her working status in a foreign country and
a. To enforce the compromise agreement in case of entitled her to all benefits and processes under our statutes. Thus, even
non-compliance therewith by any of the parties assuming arguendo that she was considered at the start of her employment
thereto; or as a “direct hire” governed by and subject to the laws, common practices
b. To nullify it if there is prima facie evidence that the and customs prevailing in Singapore[17] she subsequently became a contract
settlement was obtained through fraud, worker or an OFW who was covered by Philippine labor laws and policies
misrepresentation, or coercion.

Page 72 of 117
upon certification by the POEA. At the time her employment was illegally reduced to a mere right of action in favor of the person whom it
terminated, she already possessed the POEA employment Certificate. favors and must be enforced, as are all ordinary actions, by the
Moreover, petitioner admits that it is a Philippine corporation institution of a complaint in a regular form [IBM v. Nestle
doing business through a branch office in Singapore. Significantly, Philippines].
respondent’s employment by the Singapore branch office had to be
approved by Benjamin P. Palma Gil, the president of the bank whose
principal offices were in Manila. This circumstance militates against
7. Jurisdiction Over Other Cases
petitioner’s contention that respondent was “locally hired”; and totally In accordance with well-entrenched jurisprudence, the
“governed by and subject to the laws, common practices and customs” of issues, claims or cases of the following fall under the jurisdiction of
Singapore, not of the Philippines. Instead, with more reason does this fact the Labor Arbiters:
reinforce the presumption that respondent falls under the legal definition
of migrant worker, in this case one deployed in Singapore. Hence, a. Employees in government-owned and/or
petitioner cannot escape the application of Philippine laws or the controlled corporations without Original
jurisdiction of the NLRC and the labor arbiter.
Charters;
In any event, we recall the following policy pronouncement of
the Court in Royal Crown Internationale v. NLRC:
The hiring and firing of employees of GOCCs without
“x x x. Whether employed locally or overseas, all Filipino original charters are covered by the Labor Code, and therefore, the
workers enjoy the protective mantle of Philippine labor and social Labor Arbiters have jurisdiction over illegal dismissal and other
legislation, contract stipulations to the contrary notwithstanding. This cases that may be filed under this law; while those with original
pronouncement is in keeping with the basic public policy of the State to charters are basically governed by the Civil Service Law, rules and
afford protection to labor, promote full employment, ensure equal work regulations, and therefore, jurisdiction on any of the cases that may
opportunities regardless of sex, race or creed, and regulate the relations be initiated under this law is vested in the Civil Service
between workers and employers. For the State assures the basic rights of all
Commission (CSC) [Zamboanga City Water District v. Buat].
workers to self-organization, collective bargaining, security of tenure, and
just and humane conditions of work [Article 3 of the Labor Code of the
Philippines; See also Section 18, Article II and Section 3, Article XIII, 1987 b. Alien parties;
Constitution]. This ruling is likewise rendered imperative by Article 17 of A basic policy of contract is to protect the expectations of
the Civil Code which states that laws ‘which have for their object public the parties. Such party expectations are protected by giving effect
order, public policy and good customs shall not be rendered ineffective by to the parties’ own choice of the applicable law. The choice of law
laws or judgments promulgated, or by determination or conventions agreed must, however, bear some relationship to the parties or their
upon in a foreign country.’” transaction [Asia International v. Mondejar]. A manning agency,
for instance, cannot be faulted for complying with the applicable
6. Issuance of Writ of Execution foreign law. By so complying, it has discharged its monetary
Pertinent portions of Sections 4 (a) and 6, Rule III, of the obligation to the employee [Omanfil International v. NLRC].
NLRC Manual on Execution of Judgment, provide as follows:
c. Priests and ministers;
The fact that a case involves as parties thereto the church
Section 4. Issuance of a Writ: - Execution shall issue and its religious minister does not ipso facto give the case a
upon an order, resolution or decision that finally disposes of the religious significance. If what is involved is a labor case, the
actions or proceedings and after the counsel of record and the relationship of the church, as employer, and the priest or minister,
parties have been duly furnished with the copies of the same in as employee is a purely secular matter not related to the practice of
accordance with the NLRC Rules of Procedure, provided:
faith, worship, or doctrines of the church. Hence, Labor Arbiters
a) The Commission or Labor Arbiter shall, motu
proprio or upon motion of any interested party, issue a writ of may validly exercise jurisdiction over the labor case.
execution on a judgment only within five (5) years from the date The religious minister in Austria v. NLRC was not
it becomes final and executory, x x x excommunicated or expelled from the membership of the church
xxx  xxx  xxx but was terminated from employment based on the just causes
Section 6. Execution by Independent Action. - A provided in Article 297. Indeed, the matter of terminating an
judgment after the lapse of five (5) years from the date it employee which is purely secular in nature is different from the
becomes final and executory and before it is barred by ecclesiastical act of expelling a member from the religious
prescription, may only be enforced by an independent action.
congregation. As such, the State, through the Labor Arbiter and the
NLRC, has the right to take cognizance of the case to determine
Similarly, Section 6, Rule 39 of the Rules of Court,
whether the church, as employer, rightfully exercised its
which can be applied in a suppletory manner, provides:
management prerogative to dismiss the religious minister as its
employee.
Sec. 6. Execution by motion or by independent
action. - A final and executory judgment or order may be d. Employees of cooperatives;
executed on motion within five (5) years from the date of its
The Labor Arbiter has jurisdiction only over monetary
entry. After the lapse of such time, and before it is barred by the
statute of limitations, a judgment may be enforced by action. claims and illegal dismissal cases involving employees of
The revived judgment may also be enforced by motion within cooperatives btu not the claims or termination of membership of
five years from the date of its entry and, thereafter, by action members thereof. Cooperatives organized under RA 6938 (The
before it is barred by the statute of limitations. Cooperative Code of the Philippines) are composed of members;
hence, issues on the termination of their membership with the
 A judgment may be executed on motion within five cooperative do not fall within the jurisdiction of the Labor Arbiter
years from the date of its entry or from the date it becomes final but with the Cooperative Development Authority (CDA).
and executory. After the lapse of such time, and before it is barred
by the statute of limitations, a judgment may be enforced by action. e. Counter-claims of employers against
If the prevailing party fails to have the decision enforced by a mere employees.
motion after the lapse of five years from the date of its entry (or In Banez v. Valdevilla, it was held that the jurisdiction of
from the date it becomes final and executory), the said judgment is the Labor Arbiters and the NLRC is comprehensive enough to

Page 73 of 117
include claims for all forms of damages “arising from the Held: Here, following Wesleyan, and several other cases, the
employer-employee relations.” By this clause, Article 224 should NLRC was correct in ruling that jurisdiction is not conferred by estoppel or
apply with equal force to the claim of an employer for actual agreement of the parties, but by law, following Republic Act No. 8799 in
damages against its dismissed employee, where the basis for the relation to Presidential Decree No. 902-A. It is the Regional Trial Courts
that exercise exclusive jurisdiction over all controversies in the election or
claim arises from or is necessarily connected with the fact of
appointment of directors, trustees, officers or managers of corporations,
termination, and should be entered as a counter-claim in the illegal partnerships or associations. Thus, since petitioner is questioning the
dismissal case. This is in accord with paragraph 6 of Article 224(a), validity of her dismissal as the President and CEO of respondent PNB Life
which covers “all other claims, arising from employer-employee Insurance Co., the determination of her rights and the corporation's liability
relations.” arising from her dismissal is an intra-corporate dispute subject to the
jurisdiction of the regular courts.
8. Cases Over Which Labor Arbiters Have NO
Jurisdiction Matling Industrial v. Coros
The following issues or cases do not fall under the Held: Coros was a regular employee of Matling. And his
jurisdiction of Labor Arbiters: complaint for illegal dismissal is within the jurisdiction of the LA.
The illegal dismissal of an officer or other employee of a private
a. Claims for Damages Arising from Breach of employer is properly cognizable by the LA. This is pursuant to Article 217
(a) 2 of the Labor Code, as amended. Where the complaint for illegal
Non-Compete Clause and Other Post-
dismissal concerns a corporate officer, however, the controversy falls
Employment Prohibitions
under the jurisdiction of the Securities and Exchange Commission (SEC)
In case of violation of the non-compete clause and now with RTC effective August 8, 2000 pursuant to RA8799 or SRC. 
similar post-employment bans or prohibitions, the employer can On Coros’ being a VP for Finance and Admin Section 25 of
assert his claim for damages against the erring employee with the the Corporation Code provides:
regular courts and not with the labor courts, such breach being civil Section 25. Corporate officers, quorum.--Immediately after their
in nature [Dai-Chi Electronics v. Villarama]. election, the directors of a corporation must formally organize by the
election of a president, who shall be a director, a treasurer who may or
may not be a director, a secretary who shall be a resident and citizen of the
b. Employer’s Claims for Cash Advances, Car,
Philippines, and such other officers as may be provided for in the by-
Appliance and Other Personal loans of laws. Any two (2) or more positions may be held concurrently by the same
Employees person, except that no one shall act as president and secretary or as
With respect to resolving issues involving loans availed president and treasurer at the same time.
of by employees from their employers, it has been the consistent Conformably with Section 25 RCCP, a position must be
ruling of the Supreme Court that the Labor Arbiters have no expressly mentioned in the By-Laws in order to be considered as a
jurisdiction thereover but the regular courts. corporate office. Thus, the creation of an office pursuant to or under a
By-Law enabling provision is not enough to make a position a
corporate office.
c. Dismissal of Directors and Corporate
Thus, it was held in Easycall Communications Phils., Inc. v.
Officers King: An "office" is created by the charter of the corporation and the officer
A corporate officer’s dismissal is always a corporate act, is elected by the directors or stockholders. On the other hand, an employee
or an intra-corporate controversy, and the nature is not altered by occupies no office and generally is employed not by the action of the
the reason or wisdom with which the Board of Directors may have directors or stockholders but by the managing officer of the corporation
in taking such action. Also, an intra-corporate controversy is one who also determines the compensation to be paid to such employee.
which arises between a stockholder and the corporation. There is In this case, respondent was appointed vice president for
no distinction, qualification, nor any exemption whatsoever. The nationwide expansion by Malonzo, petitioner’s general manager, not
by the board of directors of petitioner.
provision is broad and covers all kinds of controversies between
SEC OPINION on the interpretation of Sec 25 dated
stockholders and corporations [Tabang v. NLRC]. November 25, 1993
The president, vice-president, secretary and treasurer are Corporate officers are those enumerated in the by-laws. They are
commonly regarded as the principal or executive officers of a the exclusive Officers of the corporation and the Board has no power to
corporation, and they are usually designated as the officers of the create other Offices without amending first the corporate By-
corporation. However, other officers are sometimes created by laws. However, the Board may create appointive positions other than
the charter or by-laws of a corporation, or the board of directors the positions of corporate Officers, but the persons occupying such
may be empowered under the by-laws of a corporation to create positions are not considered as corporate officers within the meaning of
Section 25 of the Corporation Code
additional offices as may be necessary. This Court expounded that
The Board of Directors of Matling could not validly delegate the
an "office" is created by the charter of the corporation and the power to create a corporate office to the President, in light of Section 25 of
officer is elected by the directors or stockholders, while the Corporation Code requiring the Board of Directors itself to elect the
an "employee" usually occupies no office and generally is corporate officers.
employed not by action of the directors or stockholders but by the Did Coros status as a director/stockholder of the
managing officer of the corporation who also determines the Corporation convert his dismissal into an intracorporate dispute to fall
compensation to be paid to such employee. From the foregoing, under the jurisdiction of the SEC (now RTC)? NO
that the creation of the position is under the corporation's charter or In an intra-corporate controversy, the Court considers two
elements, namely: (a) the status or relationship of the parties; and (b)
by-laws, and that the election of the officer is by the directors or
the nature of the question that is the subject of their controversy
stockholders must concur in order for an individual to be Not every conflict between a corporation and its
considered a corporate officer, as against an ordinary employee or stockholders involves corporate matters that only the SEC can resolve
officer. It is only when the officer claiming to have been illegally in the exercise of its adjudicatory or quasi-judicial powers. If, for
dismissed is classified as such corporate officer that the issue is example, a person leases an apartment owned by a corporation of which he
deemed an intra-corporate dispute which falls within the is a stockholder, there should be no question that a complaint for his
jurisdiction of the trial courts [Wesleyan University v. Maglaya]. ejectment for non-payment of rentals would still come under the
jurisdiction of the regular courts
The fact that the parties involved in the controversy are all
Tan v. PNB Life stockholders or that the parties involved are the stockholders and the

Page 74 of 117
corporation does not necessarily place the dispute within the ambit of the a. The management of the Cooperative shall be vested in a
jurisdiction of SEC. General Manager who shall be appointed by the Board and who shall be
Obviously enough, the respondent was not appointed as Vice responsible to the Board for performance of his duties as set forth in a
President for Finance and Administration because of his being a position description adopted by the Board, in conformance with guidelines
stockholder or Director of Matling. He had started working for Matling on established by the National Electrification Administration. It is incumbent
September 8, 1966, and had been employed continuously for 33 years until upon the Manager to keep the Board fully informed of all aspects of the
his termination on April 17, 2000, first as a bookkeeper, and his climb in operations and activities of the Cooperative. The appointment and dismissal
1987 to his last position as Vice President for Finance and Administration of the General Manager shall require approval of NEA.
had been gradual but steady b. No member of the board may hold or apply for the position of
Coros promotion to the position of Vice President for General Manager while serving as a Director or within twelve months
Finance and Administration in 1987 was by virtue of the length of following his resignation or the termination of his tenure.45
quality service he had rendered as an employee of Matling. His Evidently, the functions of the office of the General
subsequent acquisition of the status of Director/stockholder had no Manager, i.e., management of the Cooperative and to keep the Board fully
relation to his promotion. Besides, his status of Director/stockholder was informed of all aspects of the operations and activities of the Cooperative
unaffected by his dismissal from employment as Vice President for Finance are specifically laid down under BATELEC I's By-laws itself. It is therefore
and Administration. beyond cavil that Ellao's position as General Manager is a cooperative
office. Accordingly, his complaint for illegal dismissal partakes of the
nature of an intra-cooperative controversy; it involves a dispute between a
Nacpil v. Intercontinental Broadcasting
cooperative officer on one hand, and the Board of Directors, on the other.
Held: The Court has consistently held that there are two
On this score, the Court's pronouncement in Celso F. Pascual,
elements to be considered in determining whether the SEC has jurisdiction
Sr. and Serafin Terencio v. Caniogan Credit and Development
over the controversy, to wit: (1) the status or relationship of the parties; and
Cooperative,46 finds suitable application:
(2) the nature of the question that is the subject of their controversy.
Petitioners clarify that they do not take issue on the power of the
Even assuming that Nacpil was in fact appointed by the General
Board of Directors to remove them. Rather, they dispute the "manner,
Manager, such appointment was subsequently approved by the Board of
cause[,] and legality" of their removal from their respective offices as
Directors of the IBC. That the position of comptroller is not expressly
General Manager and Collection Manager. Even so, we hold that an
mentioned among the officers of the IBC in the By-Laws is of no moment,
officer's dismissal is a matter that comes with the conduct and management
because the IBC’s Board of Directors is empowered under Section 25 of the
of the affairs of a cooperative and/or an intra-cooperative controversy, and
Corporation Code and under the corporation’s By-Laws to appoint such
that nature is not altered by reason or wisdom that the Board of Directors
other officers as it may deem necessary. The By-Laws of the IBC
may have in taking such action. Accordingly, the case a quo is not a labor
categorically provides:
dispute requiring the expertise of the Labor Arbiter or of the National Labor
XII. OFFICERS
Relations Commission. It is an intra-cooperative dispute that is within the
The officers of the corporation shall consist of a President, a
jurisdiction of the Regional Trial Court xxx.47
Vice-President, a Secretary-Treasurer, a General Manager, and
As such, the CA committed no reversible error when it ordered
such other officers as the Board of Directors may from time to time does
the dismissal of Ellao's Complaint for illegal dismissal without prejudice to
fit to provide for. Said officers shall be elected by a majority vote of the
the latter's filing of his complaint at the proper forum. Considering that the
Board of Directors and shall have such powers and duties as shall
Labor Arbiter and the NLRC were without ample jurisdiction to take
hereinafter provide.
cognizance of Ellao's Complaint, the labor tribunals' rulings therein made
The Court has held that in most cases, the “by-laws may and
are resultantly void. There is therefore no need to discuss the issue on
usually do provide for such other officers,” and that where a corporate
illegal dismissal and monetary claims at this point.
office is not specifically indicated in the roster of corporate offices in the
by-laws of a corporation, the Board of directors may also be empowered
under the by-laws to create additional officers as may be necessary. Nacpil v. Intercontinental Broadcasting
In the present case, since Nacpil’s appointment was approved Held: The Court has consistently held that there are two
unanimously by the corporation’s Board of Directors, he is therefore elements to be considered in determining whether the SEC has jurisdiction
considered a corporate officer. His claim of illegal dismissal is a over the controversy, to wit: (1) the status or relationship of the parties; and
controversy that falls under the jurisdiction of the SEC as (2) the nature of the question that is the subject of their controversy.
contemplated by Section 5 of P.D. 902-A. The rule is that dismissal or Even assuming that Nacpil was in fact appointed by the General
nonappointment of a corporate officer is clearly an intra-corporate Manager, such appointment was subsequently approved by the Board of
matter, and jurisdiction over the case properly belongs to the SEC, not Directors of the IBC. That the position of comptroller is not expressly
to the NLRC. mentioned among the officers of the IBC in the By-Laws is of no moment,
As to Nacpil’s argument that the nature of his functions is because the IBC’s Board of Directors is empowered under Section 25 of the
recommendatory, thereby making him a mere managerial officer, the Court Corporation Code and under the corporation’s By-Laws to appoint such
has previously held that the relationship of a person to a corporation, other officers as it may deem necessary. The By-Laws of the IBC
whether as officer or agent or employee, is not determined by the nature of categorically provides:
the services performed, but instead by the incidents of the relationship as XII. OFFICERS
they exist. The officers of the corporation shall consist of a President, a
It is likewise of no consequence that petitioner’s complaint for Vice-President, a Secretary-Treasurer, a General Manager, and
illegal dismissal includes money claims, for such claims are part of the such other officers as the Board of Directors may from time to time does
perquisites of his position in, and therefore linked with his relations with, fit to provide for. Said officers shall be elected by a majority vote of the
the corporation. The inclusion of such money claims does not convert the Board of Directors and shall have such powers and duties as shall
issue into a simple labor problem. Clearly, the issues raised by Nacpil hereinafter provide.
against the IBC are matters that come within the area of corporate affairs The Court has held that in most cases, the “by-laws may and
and management and constitute a corporate controversy in contemplation of usually do provide for such other officers,” and that where a corporate
the Corporation Code. office is not specifically indicated in the roster of corporate offices in the
by-laws of a corporation, the Board of directors may also be empowered
under the by-laws to create additional officers as may be necessary.
Ellao v. Batangas I Electric Cooperative
In the present case, since Nacpil’s appointment was approved
Held: Here, the position of General Manager is expressly
unanimously by the corporation’s Board of Directors, he is therefore
provided for under Article VI, Section 10 of BATELEC I's By-laws,
considered a corporate officer. His claim of illegal dismissal is a
enumerating the cooperative offices as follows:
controversy that falls under the jurisdiction of the SEC as
ARTICLE VI- OFFICERS
contemplated by Section 5 of P.D. 902-A. The rule is that dismissal or
xxxx
nonappointment of a corporate officer is clearly an intra-corporate
SECTION 10. General Manager
matter, and jurisdiction over the case properly belongs to the SEC, not
to the NLRC.
Page 75 of 117
As to Nacpil’s argument that the nature of his functions is the discharge of its proprietary functions, it impliedly divested
recommendatory, thereby making him a mere managerial officer, the Court itself of its sovereign immunity from suit.
has previously held that the relationship of a person to a corporation, But these considerations notwithstanding, we hold
whether as officer or agent or employee, is not determined by the nature of that the complaint against the petitioners in the court below must
the services performed, but instead by the incidents of the relationship as still be dismissed. While suable, the petitioners are nevertheless
they exist. not liable. It is obvious that the claim for damages cannot be
It is likewise of no consequence that petitioner’s complaint for allowed on the strength of the evidence before us, which we
illegal dismissal includes money claims, for such claims are part of the have carefully examined.
perquisites of his position in, and therefore linked with his relations with, The dismissal of the private respondent was decided
the corporation. The inclusion of such money claims does not convert the upon only after a thorough investigation where it was
issue into a simple labor problem. Clearly, the issues raised by Nacpil established beyond doubt that he had polluted the soup stock
against the IBC are matters that come within the area of corporate affairs with urine.
and management and constitute a corporate controversy in contemplation of
the Corporation Code. Conversely, if the contract was entered into in the
discharge of its governmental functions, the sovereign state cannot
d. Labor Cases Involving Entities Immune from be deemed to have waived its immunity from suit. Such is the case
Suit of JUSMAG v. NLRC, holding thus:
In this jurisdiction, we recognize and adopt the generally
accepted principles of international law as part of the law of the Prescinding from this premise, we need not
land. Immunity of State from suit is one of these universally determine whether JUSMAG controls the employment
recognized principles. In international law, conditions of the private respondent.
We also hold that there appears to be no basis for
“immunity” is commonly understood as the exemption of the state
public respondent to rule that JUSMAG is estoppel from
and its organs from the judicial jurisdiction of another state. [ This is denying the existence of employer-employee relationship with
anchored on the principle of the sovereign equality of states under private respondent. On the contrary, in its Opposition before the
which one state cannot assert jurisdiction over another in violation public respondent, JUSMAG consistently contended that the
of the maxim par in parem non habet imperium (an equal has no (74) SASP, including private respondent, working in JUSMAG,
power over an equal) [JUSMAG v. NLRC]. are employees of the Armed Forces of the Philippines. This can
In USA v. Guinto, the Supreme Court ruled: be gleaned from: (1) the Military Assistance Agreement, supra,
(2) the exchange of notes between our Government, thru
Department of Foreign Affairs, and the Unites States, thru the
The traditional rule of immunity exempts a State
US Embassy to the Philippines, and (3) the Agreement on May
from being sued in the courts of another State without its
21, 1991, supra, between the Armed Forces of the Philippines
consent or waiver. This rule is a necessary consequence of the
and JUSMAG.
principles of independence and equality of States. However, the
rules of International Law are not petrified; they are constantly
developing and evolving. And because the activities of states e. Doctrine of Forum Non Conveniens
have multiplied, it has been necessary to distinguish them — This doctrine is an international law principle which has
between sovereign and governmental acts (jure imperii) and been applied to labor cases. The following are the requisites for its
private, commercial and proprietary acts (jure gestionis). The applicability:
result is that State immunity now extends only to acts  jure
imperii. (i) That the Philippine court is one to which the
The restrictive application of State immunity is
parties may conveniently resort;
proper only when the proceedings arise out of commercial
transactions of the foreign sovereign, its commercial activities or (ii) That the Philippine court is in a position to
economic affairs. Stated differently, a State may be said to have make an intelligent decision as to the law and
descended to the level of an individual and can thus be deemed the facts; and
to have tacitly given its consent to be sued only when it enters (iii) That the Philippine court has or is likely to
into business contracts. It does not apply where the contract have power to enforce its decision [Bank of
relates to the exercise of its sovereign functions. America v. Court of Appeals].
. From these circumstances, the Court can assume
that the restaurant services offered at the John Hay Air Station
partake of the nature of a business enterprise undertaken by the Manila Hotel v. NLRC
United States government in its proprietary capacity. Such Held: Under the rule, a Philippine court or agency may assume
services are not extended to the American servicemen for free as jurisdiction over the case if it chooses to do so provided: 
a perquisite of membership in the Armed Forces of the United 1. that the Philippine court is one to which the parties may conveniently
States. Neither does it appear that they are exclusively offered to resort to; 
these servicemen; on the contrary, it is well known that they are 2. that the Philippine court is in a position to make an intelligent
available to the general public as well, including the tourists in decision as to the law and the facts; and 
Baguio City, many of whom make it a point to visit John Hay 3. that the Philippine court has or is likely to have power to enforce its
for this reason. All persons availing themselves of this facility decision
pay for the privilege like all other customers as in ordinary LA is not a forum to which the parties may conveniently resort
restaurants. Although the prices are concededly reasonable and to
relatively low, such services are undoubtedly operated for profit, The Palace Hotel and Manila Hotel are not nationals of the
as a commercial and not a governmental activity. Philippines. Neither are they "doing business in the Philippines." Likewise,
The consequence of this finding is that the petitioners the main witnesses, are non-residents of the Philippines. The only factor
cannot invoke the doctrine of state immunity to justify the tying this case to the Philippines is that Santos was a Filipino Citizen
dismissal of the damage suit against them by Genove. Such LA is not in a position to make an intelligent decision as to the
defense will not prosper even if it be established that they were law.
acting as agents of the United States when they investigated and Pursuant to lex loci contractus, an intelligent decision cannot be
later dismissed Genove. For that matter, not even the United made as to the law governing the employment contract as such was
States government itself can claim such immunity. The reason is perfected in foreign soil. 
that by entering into the employment contract with Genove in LA is not in a position to make an intelligent decision as to the
facts. 

Page 76 of 117
All acts complained of took place in Beijing, People's Republic This amount is recoverable if the action is based on a quasi delict as
of China. The NLRC was not in a position to determine whether the provided for in Article 2206 of the Civil Code, but not in the Labor Code.
Tiannamen Square incident truly adversely affected operations of the While it is true that labor arbiters and the NLRC have
Palace Hotel as to justify Santos' retrenchment. jurisdiction to award not only reliefs provided by labor laws, but also
damages governed by the Civil Code, these reliefs must still be based on an
f. Quasi-Delict or Tort Cases action that has a reasonable causal connection with the Labor Code, other
labor statutes, or collective bargaining agreements.
Damages arising from quasi-delict or tort are often
It must be noted that a worker’s loss of earning capacity and
confused with damages that may be claimed under labor laws and blacklisting are not to be equated with wages, overtime compensation or
labor agreements. Consequently, quasi-delict or tort damages are separation pay, and other labor benefits that are generally cognized in labor
asserted, though erroneously, in labor cases filed with the Labor disputes. The loss of earning capacity is a relief or claim resulting from a
Arbiters. As earlier emphasized, however, Labor Arbiters and the quasi delict or a similar cause within the realm of civil law.
NLRC have no power or authority to grant reliefs in claims that do “Claims for damages under paragraph 4 of Article 217 must
not arise from employer-employee relationship such as those have a reasonable causal connection with any of the claims provided for in
emanating from quasi-delict or tort cases per Article 2176 of the the article in order to be cognizable by the labor arbiter. Only if there is
such a connection with the other claims can the claim for damages be
Civil Code that have no reasonable causal connection to any of the
considered as arising from employer-employee relations.” In the present
claims provided in the Labor Code, other labor statutes, or case, petitioner’s claim for damages is not related to any other claim under
collective bargaining agreements. Article 217, other labor statutes, or collective bargaining agreements.
Petitioner cannot anchor her claim for damages to Article 161 of
Tolosa v. NLRC the Labor Code, which does not grant or specify a claim or relief. This
Held: We affirm the CA’s ruling that the NLRC and the labor provision is only a safety and health standard under Book IV of the same
arbiter had no jurisdiction over petitioner’s claim for damages, because that Code. The enforcement of this labor standard rests with the labor
ruling was based on a quasi delict or tort per Article 2176 of the Civil Code. secretary. Thus, claims for an employer’s violation thereof are beyond the
Time and time again, we have held that the allegations in the jurisdiction of the labor arbiter. In other words, petitioner cannot enforce
complaint determine the nature of the action and, consequently, the the labor standard provided for in Article 161 by suing for damages before
jurisdiction of the courts. After carefully examining the complaint/position the labor arbiter.
paper of petitioner, we are convinced that the allegations therein are in the It is not the NLRC but the regular courts that have jurisdiction
nature of an action based on a quasi delict or tort. It is evident that she sued over actions for damages, in which the employer-employee relation is
Pedro Garate and Mario Asis for gross negligence. merely incidental, and in which the cause of action proceeds from a
Petitioner’s complaint/position paper refers to and extensively different source of obligation such as a tort. [25] Since petitioner’s claim for
discusses the negligent acts of shipmates Garate and Asis, who had no damages is predicated on a quasi delict or tort that has no reasonable causal
employer-employee relation with Captain Tolosa. Specifically, the paper connection with any of the claims provided for in Article 217, other labor
alleges the following tortious acts: statutes, or collective bargaining agreements, jurisdiction over the action
“x x x [R]espondent Asis was the medical officer of the Vessel, lies with the regular courts[26] -- not with the NLRC or the labor arbiters.
who failed to regularly monitor Capt. Tolosa’s condition, and who needed
the USCG to prod him to take the latter’s vital signs. In fact, he failed to g. Criminal and Civil Actions Arising from
keep a medical record, like a patient’s card or folder, of Capt. Tolosa’s Violations of the Penal Provisions of the
illness.”
Labor Code
“Respondents, however, failed Capt. Tolosa because Garate
The Labor Code has expressly conferred jurisdiction over
never initiated actions to save him. x x x In fact, Garate rarely checked
personally on Capt. Tolosa’s condition, to wit:”[13] criminal and civil cases arising from violations of the Labor Code
“x x x Noticeably, the History (Annex “D”) fails to mention any with the regular courts.
instance when Garate consulted the other officers, much less Capt. Tolosa,
regarding the possibility of deviation. To save Capt. Tolosa’s life was h. Constitutionality of CBA Provisions
surely a just cause for the change in course, which the other officers would In Halaguena v. Philippine Airlines, it was pronounced
have concurred in had they been consulted by respondent Garate – which that it is not the Labor Arbiter but the regular court which has
he grossly neglected to do.
jurisdiction to rule on the constitutionality of labor contracts such
“Garate’s poor judgement, since he was the officer effectively in
command of the vessel, prevented him from undertaking these emergency
as a CBA.
measures, the neglect of which resulted in Capt. Tolosa’s untimely
demise.” i. Replevin Intertwined with a Labor Dispute
The labor arbiter himself classified petitioner’s case as “a Replevin is a possessory action, the gist of which is the
complaint for damages, blacklisting and watchlisting (pending inquiry) for right of possession in the plaintiff. The primary relief sought
gross negligence resulting in the death of complainant’s husband, Capt. therein is the return of the property in specie wrongfully detained
Virgilio Tolosa.”[ by another person. It is an ordinary statutory proceeding to
We stress that the case does not involve the adjudication of a
adjudicate rights to the title or possession of personal property. The
labor dispute, but the recovery of damages based on a quasi delict. The
jurisdiction of labor tribunals is limited to disputes arising from employer-
question of whether or not a party has the right of possession over
employee relations. the property involved and if so, whether or not the adverse party
Not every dispute between an employer and employee involves has wrongfully taken and detained said property as to require its
matters that only labor arbiters and the NLRC can resolve in the exercise of return to plaintiff, is outside the pale of competence of a labor
their adjudicatory or quasi-judicial powers. The jurisdiction of labor tribunal; it is beyond the field of specialization of Labor Arbiters
arbiters and the NLRC under Article 217 of the Labor Code is limited to [Basaya, Jr. v. Militante].
disputes arising from an employer-employee relationship which can only be
resolved by reference to the Labor Code, other labor statutes, or their
j.Cases Arriving from Violation of Training
collective bargaining agreement.
The pivotal question is whether the Labor Code has any
Agreement
relevance to the relief sought by petitioner. From her paper, it is evident The primary relief sought is for liquidated damages for
that the primary reliefs she seeks are as follows: (a) loss of earning capacity breach of a contractual obligation. The other items demanded are
denominated therein as “actual damages” or “lost income” and (b) not labor benefits demanded by workers generally taken
blacklisting. The loss she claims does not refer to the actual earnings of the cognizance of in labor disputes, such as payment of wages,
deceased, but to his earning capacity based on a life expectancy of 65 years. overtime compensation or separation pay. The items claimed are

Page 77 of 117
the natural consequences flowing from breach of an obligation, The Commission shall decide all cases within twenty (20)
intrinsically a civil dispute [Singapore Airlines v. Pano]. calendar days from receipt of the answer of the appellee.
The decision of the Commission shall be final and executory
C. NATIONAL LABOR RELATIONS COMMISSION after ten (10) calendar days from receipt thereof by the parties.
Any law enforcement agency may be deputized by the
The NLRC exercises two (2) kinds of jurisdiction:
Secretary of Labor and Employment or the Commission in the
enforcement of decisions, awards or orders.
(1) Exclusive original jurisdiction; and xxx
(2) Exclusive appellate jurisdiction.
The NLRC has exclusive appellate jurisdiction over the
1. Exclusive Original Jurisdiction following:
The NLRC exercises exclusive and original jurisdiction
over the following cases: a. All cases decided by the Labor Arbiters
[Article 224(b)].
a. Petition for injunction in ordinary labor b. Cases by decided by the DOLE Regional
disputes to enjoin or restrain any actual or Directors or hearing officers involving small
threatened commission of any or all prohibited money claims under Article 129 of the Labor
or unlawful acts or to require the performance Code.
of a particular act in any labor dispute which, c. Contempt cases decided by the Labor Arbiters.
if not restrained or performed forthwith, may
cause grave irreparable damage to any party The perfection of an appeal shall stay the execution of
[Article 218(e)]. the decision of the Labor Arbiter except execution for
b. Petition for injunction in strikes or lockouts reinstatement pending appeal [NLRC Rules of Procedure].
under Article 279 of the Labor Code.
c. Certified cases which refer to labor disputes a. Grounds for Appeal
causing or likely to cause a strike or lockout in The appeal to the NLRC may be entertained only on any
an industry indispensable to the national of the following grounds:
interest, certified to it by the Secretary of
Labor and Employment for compulsory (i) If there is prima facie evidence of abuse of
arbitration by virtue of Article 278(g) of the discretion on the part of the labor arbiter;
Labor Code [Article 278(g)]. (ii) If the decision, order or award was secured
d. Petition for extraordinary remedies from through fraud or coercion, including graft and
orders or resolutions of Labor Arbiters corruption;
(including those issued during execution (iii) If made purely on questions of law; and/or
proceedings). (iv) If serious errors in the findings of fact are
raised which, if not corrected, would cause
2. Exclusive Appellate Jurisdiction grave or irreparable damage or injury to the
appellant.
Article 229. [223] Appeal. Decisions, awards, or orders of the
Labor Arbiter are final and executory unless appealed to the
The NLRC has certiorari power. The first ground above
Commission by any or both parties within ten (10) calendar days from
receipt of such decisions, awards, or orders. Such appeal may be regarding prima facie evidence of abuse of discretion on the part of
entertained only on any of the following grounds: the Labor Arbiter is actually an exercise of certiorari power by the
(a) If there is prima facie evidence of abuse of discretion on NLRC. The case of Triad Security v. Ortega expressly recognized
the part of the Labor Arbiter; this certiorari power. Clearly, according to Auza, Jr. v. MOL
(b) If the decision, order or award was secured through Philippines, the NLRC is possessed of the power to rectify any
fraud or coercion, including graft and corruption; abuse of discretion committed by the Labor Arbiter.
(c) If made purely on questions of law; and
(d) If serious errors in the findings of facts are raised which
b.Requirements to Perfect an Appeal before
would cause grave or irreparable damage or injury to the appellant.
In case of a judgment involving a monetary award, an the NLRC
appeal by the employer may be perfected only upon the posting of a The perfection of appeal within the period and in the
cash or surety bond issued by a reputable bonding company duly manner prescribed by law is jurisdictional and non-compliance
accredited by the Commission in the amount equivalent to the with the legal requirements is fatal and has the effect of rendering
monetary award in the judgment appealed from. the judgment final and executory, hence, unappealable [Opinaldo
In any event, the decision of the Labor Arbiter reinstating a v. Ravina]. The date of receipt of decisions, resolutions, or orders
dismissed or separated employee, insofar as the reinstatement aspect is by the parties is of no moment. For purposes of appeal, the
concerned, shall immediately be executory, even pending appeal. The
reglementary period shall be counted from receipt of such
employee shall either be admitted back to work under the same terms
and conditions prevailing prior to his dismissal or separation or, at the decisions, resolutions, or orders by the counsel or representative of
option of the employer, merely reinstated in the payroll. The posting of record [NLRC Rules of Procedure].
a bond by the employer shall not stay the execution for reinstatement The date of mailing by registered mail of the appeal
provided herein. memorandum is the date of its filing [Associated Anglo-American
To discourage frivolous or dilatory appeals, the Commission Tobacco Corporation v. NLRC].
or the Labor Arbiter shall impose reasonable penalty, including fines The requisites for perfection of appeal to the NLRC are
or censures, upon the erring parties. as follows:
In all cases, the appellant shall furnish a copy of the
memorandum of appeal to the other party who shall file an answer not
later than ten (10) calendar days from receipt thereof. (i) Observance of the reglementary period;
(ii) Payment of appeal and legal research fee;
(iii) Filing of a memorandum of appeal;
Page 78 of 117
(iv) Proof of service to the other party; and [Philippine Geothermal v. NLRC], in which
(v) Posting of cash, property or surety bond, in case, the appeal may be filed the next working
case of monetary awards. day.
(ii) Reliance on erroneous notice of decision
Reglementary Period [Firestone Tire and Rubber v. Lariosa] as
The reglementary period is mandatory and not a mere when the notice expressly states “working
technicality [Calipay v. NLRC]. days” and not “calendar days.”
The reglementary period depends on where the appeal to (iii) Filing of petition or extraordinary remedies
the NLRC emanates, viz: from orders or resolutions of Labor Arbiters or
on third party claims – ten (10) calendar days.
(i) Ten (10) calendar days in case of appeals from (iv) When technical rules are disregarded under
decisions of the Labor Arbiters under Article Article 227 [City Fair Corporation v. NLRC].
229; (v) When there are some compelling reasons that
(ii) Five (5) calendar days in case of appeals in justify the allowance of the appeal despite its
contempt cases decided by Labor Arbiters; and late filing such as when it is granted in the
(iii) Five (5) calendar days in case of appeals from interest of substantial justice [Surima v.
decisions of the DOLE Regional Director NLRC].
under Article 129 (small money claims of
P5000.00 or less). Republic v. Guinmapang
Held: The general rule is that the perfection of an appeal in the
Section 4, Rule III of the New Rules of Procedure of the manner and within the period prescribed by law is, not only mandatory, but
NLRC expressly mandates that "(F)or the purpose(s) of computing jurisdictional, and failure to conform to the rules will render the judgment
the period of appeal, the same shall be counted from receipt of such sought to be reviewed final and unappealable. By way of exception,
unintended lapses are disregarded so as to give due course to appeals filed
decisions, awards, or orders by the counsel of record. "Although
beyond the reglementary period on the basis of strong and compelling
this rule explicitly contemplates an appeal before tile Labor reasons, such as serving the ends of justice and preventing a grave
Arbiter and tile NLRC, we do not see any cogent reason why tile miscarriage thereof. The purpose behind the limitation of the period of
same rule should not apply to petitions for certiorari filed with the appeal is to avoid an unreasonable delay in the administration of justice
Court of Appeals from decisions of the NLRC. This procedure is in and to put an end to controversies.
line with the established rule that notice to counsel is notice to In Chronicle Securities Corporation v. NLRC, we ruled:
party and when a party is represented by counsel, notices should In not a few instances, we relaxed the rigid application of the
be made upon the counsel of record at his given address to which rules of procedure to afford the parties the opportunity to fully ventilate
their cases on the merits. This is in line with the time honored principle that
notices of all kinds emanating from the court should be sent. It is
cases should be decided only after giving all the parties the chance to argue
to be noted also that Section 7 of the NLRC Rules of their causes and defenses. Technicality and procedural imperfections should
Procedure provides that "(A)ttorneys and other representatives of thus not serve as bases of decisions. In that way, the ends of justice would
parties shall have authority to bind their clients in all matters of be better served. For indeed, the general objective of procedure is to
procedure"' a provision which is similar to Section 23, Rule 138 of facilitate the application of justice to the rival claims of contending parties,
the Rules of Court. More importantly, Section 2, Rule 13 of the bearing always in mind that procedure is not to hinder but to promote the
1997 Rules of Civil Procedure analogously provides that if any administration of justice.
party has appeared by counsel, service upon him shall be made Indeed the prevailing trend is to accord party litigants the
amplest opportunity for the proper and just determination of their causes,
upon his counsel [Cervantes v. City Service Corporation]. 
free from the constraints of needless technicalities.
In Bello v. NLRC, citing anew Ginete v. Sunrise Manning A one-day delay in the perfection of the appeal was excused in
Agency, et al., the Court held that "the period for filing a petition Gana v. NLRC, Surigao del Norte Electric Cooperative v. NLRC, City Fair
for certiorari should be reckoned from the time the counsel of Corporation v. NLRC, Pacific Asia Overseas Shipping Corp. v. NLRC,
record received a copy of the Resolution denying the motion for and Insular Life Assurance Co., Ltd. v. NLRC.
reconsideration." We agree with the Court of Appeals that since no intent to delay
The periods provided are all calendar days and not the administration of justice could be attributed to Guinmapang, a one day
working days [RJL Martinez Fishing v. NLRC]. Consequently, delay does not justify the appeal’s denial. More importantly, the Court of
Appeals declared that Guinmapang’s appeal, on its face, appears to be
Saturdays, Sundays and legal holidays are included in reckoning
impressed with merit. The constitutional mandate to accord full protection
and computing of the reglementary period [Judy Philippines v. to labor and to safeguard the employee’s means of livelihood should be
NLRC]. given proper attention and sanction. A greater injustice may occur if said
However, certain procedural lapses may be disregarded appeal is not given due course than if the reglementary period to appeal
where there is an acceptable reason to excuse tardiness in the were strictly followed.25 In this case, we are inclined to excuse the one day
taking of the appeal [Philippine Airlines v. NLRC]. It is always delay in order to fully settle the merits of the case. This is in line with our
within the power of the court to suspend its own rules or to except policy to encourage full adjudication of the merits of an appeal.
a particular case from its operation, whenever the purposes of
justice require it [Chronicle Securities v. NLRC]. Thus, procedural Posting of Bond
rules may be waived, dispensed with, or relaxed in the interest of In case the decision of the Labor Arbiter or the Regional
substantial justice. The Court may deign to veer away from the Director involves a monetary award, an appeal by the employer
general rule if, on its face, the appeal appears to be absolutely may be perfected only upon the posting of a bond, which shall
meritorious [Tiger Construction v. Abay]. either be in the form of cash deposit or surety bond in an amount
The following are the specific instances where the rules equivalent to the monetary award, exclusive of damages and
on the reckoning of the reglementary period have not been strictly attorney’s fees.
observed: Only in case the decision of the Labor Arbiter or the
DOLE Regional Director involves a monetary award, that an
(i) 10th day (or 5th day) falling on a Saturday [Judy appeal by the employer may be perfected upon the posting of a
Philippines v. NLRC], Sunday or Holiday bond. When the judgment of the Labor Arbiter does not involve

Page 79 of 117
any monetary award, no appeal bond is necessary [Aba v. NLRC]. Rosewood v. NLRC
The Labor Arbiter’s decision or order should state the amount Held: In Globe General Services and Security Agency
awarded. If the amount of the monetary award is not contained or vs. National Labor Relations Commission, 15 the Court observed that the
fixed in the judgment, the appeal bond is not required to be posted NLRC, in actual practice, allows the reduction of the appeal bond upon
[Orozco v. Court of Appeals]. In case of conflict between the body motion of the appellant and on meritorious grounds; hence, petitioners in
that case should have filed a motion to reduce the bond within the
and the fallo of the decision, the latter should prevail [Mendoza, Jr.
reglementary period for appeal.
v. San Miguel Foods, Inc.]. That is the exact situation in the case at bar. Here, petitioner
The surety bond must be issued by a reputable bonding claims to have received the labor arbiter's Decision on April 6, 1993. 16 On
company duly accredited by the NLRC or the Supreme Court April 16, 1993, it filed, together with its memorandum on appeal 17 and
[UERM-Memorial medical Center v. NLRC]. notice of appeal, a motion to reduce the appeal bond 18 accompanied by a
The amount of such bound should be equivalent to the surety bond for fifty thousand pesos issued by prudential Guarantee and
monetary award, exclusive of damages and attorney’s fees [NLRC Assurance, Inc. 19 Ignoring petitioner's motion (to reduce bond),
Rules of Procedure]. In other words, only monetary awards (such Respondent Commission rendered its assailed Resolution dismissing the
appeal due to the late filing of the appeal bond.
as unpaid wages, backwages, separation pay, 13 th month pay, etc.)
The solicitor general argues for the affirmation of the assailed
are required to be covered by the bond. Moral and exemplary Resolution for the sole reason that the appeal bond, even if it was filed on
damages and attorney’s fees are excluded. There is no conflict time, was defective, as it was not in an amount "equivalent to the monetary
between the two provisions. Article 223 lays down the requirement award in the judgment appealed from." The Court disagrees.
that an appeal bond should be filed. The implementing rule, on the We hold that petitioner's motion to reduce the bond is a
other hand, explains how the appeal bond shall be computed. The substantial compliance with the Labor Code. This holding is consistent with
rule explicitly excludes moral and exemplary damages and the norm that letter-perfect rules must yield to the broader interest of
attorney's fees from the computation of the appeal bond. This substantial justice.
Where a decision may be made to rest on informed judgment
exclusion has been recognized by the Court in a number of cases.
rather than rigid rules, the equities of the case must be accorded their due
Hence, in Erectors vs. NLRC,  the Court nullified an NLRC order weight because labor determinations should not only be "secundum
requiring the posting of an appeal bond which, among others, rationem but also secundum caritatem." 21 A judicious reading of the
"even included in the computation the award of P400,000.00 for memorandum of appeal would have made it evident to Respondent
moral and exemplary damages." Indeed, the said implementing Commission that the recourse was meritorious. Respondent Commission
rule is a contemporaneous construction of Article 223 by the acted with grave abuse of discretion in peremptorily dismissing the appeal
NLRC pursuant to the mandate of the Labor Code; hence, it is without passing upon - in fact, ignoring - the motion to reduce the appeal
accorded great respect by this Court.  bond.
We repeat: Considering the clear merits which appear, res ipsa
The cash or surety bond required for the perfection
loquitur, in the appeal from the labor arbiter's Decision, and the petitioner's
appeal should be posted within the reglementary period [Gaudia v. substantial compliance with rules governing appeals, we hold that the
NLRC]. If a party failed to perfect his appeal by the non-payment NLRC gravely abused its discretion in dismissing said appeal and in failing
of the appeal bond within the 10-calendar day period provided by to pass upon the grounds alleged in the Motion for Reconsideration.
law, the decision of the Labor Arbiter becomes final and executory
upon the expiration the said period [Santos v. Velarde]. c. Prohibited Motions
In case the employer failed to post a bond to perfect its Rule V, Section 5 of the NLRC Rules of Procedure
appeal, the remedy of the employee is to file a motion to dismiss provides:
the appeal and not a petition for mandamus for the issuance of writ
of execution [Diaz v. Nora]. SECTION 5. PROHIBITED PLEADINGS AND
MOTIONS. – The following pleadings and motions shall not be
Motion to Reduce Appeal Bond allowed and acted upon nor elevated to the Commission:
No motion to reduce bond shall be entertained except on (a) Motion to dismiss the complaint except on the
meritorious grounds, and only upon the posting of a bond in a ground of lack of jurisdiction over the subject matter, improper
venue, res judicata, prescription and forum shopping;
reasonable amount in relation to the monetary award. The mere
(b) Motion for a bill of particulars;
filing of a motion to reduce bond without complying with the (c) Motion for new trial;
requisites in the preceding paragraphs shall not stop the running of (d) Petition for relief from judgment;
the period to perfect an appeal [NLRC Rules of Procedure]. (e) Motion to declare respondent in default;
The general rule is that the appeal bond that should be (f) Motion for reconsideration of any decision or any
posted should be equivalent to the monetary award of the Labor order of the Labor Arbiter;
Arbiter [Ramirez v. Court of Appeals]. In practice the NLRC has (g) Motion to Quash and/or Motion to Lift
allowed the reduction of the bond upon showing of meritorious Garnishment if a Petition had been filed under Rule XII;
(h) Appeal from any interlocutory order of the Labor
grounds and filed within the reglementary period for appeal.
Arbiter, such as but not limited to, an order: (1) denying a
The following are principles on reduction of bond: motion to dismiss; (2) denying a motion to inhibit; (3) denying a
motion for issuance of writ of execution; or (4) denying a
 Bond may be reduced when the decision failed motion to quash writ of execution;
to specify the exact amount of monetary award. (i) Appeal from the issuance of a certificate of
 Monetary award running into millions is not finality of decision by the Labor Arbiter;
justification to reduce bond. (j) Appeal from orders issued by the Labor Arbiter in
the course of execution proceedings; and
 Financial difficulties or financial incapacity is (k) Such other pleadings, motions and petitions of
not sufficient grounds to reduce bond. similar nature intended to circumvent above provisions. (5a,
 Full amount of bond must be posted within the RIII) (As amended by En Banc Resolution No. 02-15, Series of
reglementary period while motion to reduce 2015)
bond is pending. Alternatively, a reasonable
partial appeal bond may be paid within the d. Frivolous Appeals
reglementary period. – No appeal from an interlocutory order shall be
entertained. To discourage frivolous or dilatory appeals, including

Page 80 of 117
those taken from interlocutory orders, the Commission after reinstatement of the employee whose dismissal is declared illegal
hearing may censure or cite in contempt the erring parties and their [Mt. Carmel College v. Resuena].
counsels, or subject them to reasonable fine or penalty [Section 10, Moreover, reinstatement pending appeal does not apply
NLRC Rules]. when the dismissal is legal but reinstatement is ordered for some
Although, as a general rule, an appeal should not be reasons like equity and compassionate justice [Lansangan v.
dismissed on a ground which goes to the merits of the cause or to Amkor Technology].
the right of plaintiff or defendant to recover, yet; in exceptional In case of two successive dismissals, the order of
instances, an appellate court may order the dismissal when the reinstatement pending appeal under Article 229 issued in the first
appeal appears to be manifestly and palpably frivolous. And where, case shall apply only to the first case and should not affect the
as in the instant case, the dismissal has been ordered by the trial second dismissal [Sevilla v. NLRC].
court, it will not be disturbed in the appellate court if the latter
finds the appeal to have been interposed ostensibly for delay. It has Options of the Employer
been held that a frivolous appeal is one presenting no justiciable To implement the reinstatement aspect of a Labor
question, or one so readily recognizable as devoid of merit on the Arbiter’s decision, there are only two (2) options available to the
face of record that there is little, if any, prospect that it can ever employer, to wit:
succeed. The instant case is one such instance in which the appeal
is evidently without merit, taken manifestly for delay [Dela Cruz v. (i) Actual reinstatement. – The employee should
Blanco]. be reinstated to his position which he occupies
In Bongay v. Martinez, the contention of the private prior to his illegal dismissal under the same
respondents that the determination of whether or not their appeal terms and conditions prevailing prior to his
was filed seasonably should be left to the Commission as the latter dismissal or separation or, if no longer
has exclusive jurisdiction to rule on the question failed. That available, to a substantially-equivalent
argument is belied by Section 5, Rule IX of the Rules of the NLRC position; or
which empowers not only the Commission but also the Labor (ii) Payroll reinstatement. – The employee should
Arbiter to impose reasonable penalties, including fines and be reinstated in the payroll of the company
censures, upon a party for filing a frivolous appeal. This implies without requiring him to report back to his
that even when the appeal is still with the Labor Arbiter, and not work.
yet transmitted to the Commission, the former may already find it
frivolous and, there and then, terminate that appeal. Had it been as It is required that in case the decision of the Labor
the private respondents claim, the Labor Arbiter would not have Arbiter includes an order of reinstatement, it should contain:
been given such power.
(i) A statement that the reinstatement aspect is
e. Reinstatement Pending Appeal immediately executory; and
Reinstatement pending appeal applies to all kinds of (ii) A directive for the employer to submit a report
illegal dismissal cases, regardless of the grounds therefor [C. of compliance within 10 calendar days from
Alcantara & Sons v. Court of Appeals]. receipt of the said decision [NLRC Rules of
An order of reinstatement issued by the Labor Arbiter Procedure].
under Article 229 is self-executory even pending appeal by the
employer. This means that while perfection of an appeal shall stay From the moment an employee is ordered reinstated by
the execution of the decision of the Labor Arbiter the exception is the Labor Arbiter on the basis of the finding that his dismissal is
in respect to the execution of the reinstatement order which should illegal, up to the time that an appellate tribunal like the NLRC, CA
proceed even pending appeal by the employer. An order or award or the Supreme Court, as the case may be, reverses the said finding,
for reinstatement does not require a writ of execution. The the reinstated employee is generally entitled to his so-called
employer has no way of staying execution of immediate “reinstatement salaries/wages” and regular allowances, benefits,
reinstatement. He cannot post bond to prevent its execution incentives, and bonuses [Bago v. NLRC].
[Pioneer Texturizing Corporation v. NLRC]. In cases where the employer totally fails or refuses to
When the former position is already filled up, the reinstate the employee but nonetheless appeals from the Labor
employee ordered reinstated pending appeal should be reinstated to Arbiter’s adverse decision, there arises the issue of whether the
a substantially equivalent position [Medina v. Consolidated employee would still be entitled to his accrued salaries/wages,
Broadcasting System]. Reinstatement to a position lower in rank is allowances and other benefits in the event that the employer is
not proper [Panuncillo v. CAP Philippines]. subsequently successful in having the reinstatement order reversed
Reinstatement pending appeal is not affected by the in its appeal.
reinstated employee’s employment elsewhere [Triad Security v. Section 12, Rule XI of the NLRC Rules of Procedure
Ortega]. Such employment elsewhere also does not affect the provides:
obligation of the employer to pay reinstatement backwages.
The failure of the employee ordered reinstated pending SECTION 12. EXECUTION OF
appeal to report back to work as directed by the employer does not REINSTATEMENT PENDING APPEAL. – In case the
give the employer the right to remove him, especially when there is decision includes an order of reinstatement, and the employer
disobeys the directive under the second paragraph of Section 19
a reasonable explanation for his failure [Buenviaje v. Court of
of Rule V or refuses to reinstate the dismissed employee, the
Appeals]. Labor Arbiter shall immediately issue writ of execution, even
By way of distinction, the rule on reinstatement pending pending appeal, directing the employer to immediately reinstate
appeal applies only to the order of reinstatement issued by the the dismissed employee either physically or in the payroll, and
Labor Arbiter and to no other. This means that if the reinstatement to pay the accrued salaries as a consequence of such non-
order is issued by the NLRC on appeal, or by the Court of Appeals reinstatement in the amount specified in the decision.
or by the Supreme Court, there is a need to secure a writ of The Labor Arbiter shall motu proprio issue a
execution from the Labor Arbiter of origin to enforce the corresponding writ to satisfy the reinstatement wages as they

Page 81 of 117
accrue until actual reinstatement or reversal of the order of xxx
reinstatement. (En Banc Resolution No. 11-12, Series of 2012)
The Sheriff shall serve the writ of execution upon the The case of Garcia v. Philippine Airlines, also known as
employer or any other person required by law to obey the same. the Garcia Doctrine, downplays the "stray posture" of Genuino
If he/she disobeys the writ, such employer or person may be
and reaffirms the no-reimbursement doctrine in Roquero and
cited for contempt in accordance with Rule IX. (6a)
conforming rulings. The court explains in Garcia that pursuant to
the police power, the state may authorize an immediate
As a consequence of such disobedience, the employer
implementation, pending appeal, of a decision reinstating a
has the following liabilities [NLRC Rules of Procedure]:
dismissed or separated employee. The immediate reinstatement is
"a saving act" designed to stop a continuing threat or danger to the
(i) He shall be liable to pay the accrued salaries of
survival or even the life of the employee and his family. Moreover,
the reinstated employee as a consequence of
the social justice principles of labor law outweigh or render
such non-reinstatement in the amount specified
inapplicable the civil law doctrine of unjust enrichment.
in the decision; and
However, Garcia modified the doctrine enunciated in
(ii) He may be cited for contempt, in accordance
Roquero, to wit:
with the NLRC Rules of Procedure, for his
refusal to comply with the writ of execution
xxx
ordering the reinstatement. This remedy, After the labor arbiter's decision is reversed by a
however, is available only after the sheriff shall higher tribunal, the employee may be barred from collecting
have served the writ of execution upon the the accrued wages, if it is shown that the delay in enforcing
employer or any other person required by law the reinstatement pending appeal was without fault on the
to obey the same. part of the employer.
The test is two-fold: (1) there must be actual delay or
Roquero Doctrine the fact that the order of reinstatement pending appeal was not
executed prior to its reversal; and (2) the delay must not be due
Roquero v. Philippine Airlines declared the rule that in
to the employer's unjustified act or omission. If the delay is due
cases where an employee is ordered reinstated by the Labor Arbiter to the employer's unjustified refusal, the employer may still be
and the employer fails or refuses to obey the reinstatement order required to pay the salaries notwithstanding the reversal of the
but initiates an appeal, the employer’s success in having the Labor Arbiter's decision.
decision of the Labor Arbiter’s decision reversed on appeal will not In Genuino, there was no showing that the employer
exculpate him from the liability to pay the reinstatement wages and refused to reinstate the employee, who was the Treasury Sales
benefits of the employee, reckoned and computed from the time the Division Head, during the short span of four months or from the
employee was ordered reinstated by the Labor Arbiter until the date promulgation on May 2, 1994 of the Labor Arbiter's Decision up
to the promulgation on September 3, 1994 of the NLRC
of reversal on appeal.
Decision. Notably, the former NLRC Rules of Procedure did not
lay down a mechanism to promptly effectuate the self-executory
No Obligation to Reimburse order of reinstatement, making it difficult to establish that the
In the Roquero case, both the CA and the SC reversed the employer actually refused to comply.
order of reinstatement as they upheld the employee's dismissal. In a situation like that in International Container
Must the employee pay back the salary he received during the Terminal Services, Inc. v. NLRC where it was alleged that the
reinstatement? The Court said no. employer was willing to comply with the order and that the
employee opted not to pursue the execution of the order, the
"We reiterate the rule that technicalities have no Court upheld the self-executory nature of the reinstatement
room in labor cases where the Rules of Court are applied only in order and ruled that the salary automatically accrued from notice
a suppletory manner and only to effectuate the objectives of the of the Labor Arbiter's order of reinstatement until its ultimate
Labor Code and not to defeat them. Hence, even if the order of reversal by the NLRC. It was later discovered that the employee
reinstatement of the Labor Arbiter is reversed on appeal, it is indeed moved for the issuance of a writ but was not acted upon
obligatory on the part of the employer to reinstate and pay the by the Labor Arbiter. In that scenario where the delay was
wages of the dismissed employee during the period of appeal caused by the Labor Arbiter, it was ruled that the inaction of the
until reversal by the higher court. On the other hand, if the Labor Arbiter who failed to act upon the employee's motion for
employee has been reinstated during the appeal period and such the issuance of a writ of execution may no longer adversely
reinstatement order is reversed with finality, the employee is not affect the cause of the dismissed employee in view of the self-
required to reimburse whatever salary he received for he is executory nature of the order of reinstatement.
entitled to such, more so if he actually rendered services during The new NLRC Rules of Procedure, which took
the period." effect on January 7, 2006, now require the employer to submit a
report of compliance within 10 calendar days from receipt of the
Labor Arbiter's decision, disobedience to which clearly denotes
But a deviation occurred in Genuino v. NLRC, also a refusal to reinstate. The employee need not file a motion for
known as the Genuino Doctrine. The Court in Genuino said: the issuance of the writ of execution since the Labor
Arbiter shall thereafter  motu proprio issue the writ. With the
xxx new rules in place, there is hardly any difficulty in
If the decision of the labor arbiter is later reversed on determining the employer's intransigence in immediately
appeal upon the finding that the ground for dismissal is valid, complying with the order.
then the employer has the right to require the dismissed In the case at bar, petitioners exerted efforts to
employee on payroll reinstatement to refund the salaries s/he execute the Labor Arbiter's order of reinstatement until they
received while the case was pending appeal, or it can be were able to secure a writ of execution, albeit issued on October
deducted from the accrued benefits that the dismissed employee 5, 2000 after  the reversal by the NLRC of the Labor Arbiter's
was entitled to receive from his/her employer under existing decision. Technically, there was still actual delay which brings
laws, collective bargaining agreement provisions, and company to the question of whether the delay was due to respondent's
practices. However, if the employee was reinstated to work unjustified act or omission.
during the pendency of the appeal, then the employee is entitled It is apparent that there was inaction on the part of
to the compensation received for actual services rendered respondent to reinstate them, but whether such omission was
without need of refund.

Page 82 of 117
justified depends on the onset of the exigency of corporate and determine the disputes in the absence of any party thereto who has
rehabilitation. been summoned or served with notice to appear, conduct its
It is settled that upon appointment by the SEC of a proceedings or any part thereof in public or in private, adjourn its
rehabilitation receiver, all actions for claims before any court, hearings to any time and place, refer technical matters or accounts to
tribunal or board against the corporation shall ipso jure be an expert and to accept his report as evidence after hearing of the
suspended. As stated early on, during the pendency of parties upon due notice, direct parties to be joined in or excluded from
petitioners' complaint before the Labor Arbiter, the SEC placed the proceedings, correct, amend, or waive any error, defect or
respondent under an Interim Rehabilitation Receiver. After the irregularity whether in substance or in form, give all such directions as
Labor Arbiter rendered his decision, the SEC replaced the it may deem necessary or expedient in the determination of the dispute
Interim Rehabilitation Receiver with a Permanent Rehabilitation before it, and dismiss any matter or refrain from further hearing or
Receiver. from determining the dispute or part thereof, where it is trivial or
Case law recognizes that unless there is a restraining where further proceedings by the Commission are not necessary or
order, the implementation of the order of reinstatement is desirable; and
ministerial and mandatory. This injunction or suspension of (d) To hold any person in contempt directly or indirectly
claims by legislative fiat partakes of the nature of a restraining and impose appropriate penalties therefor in accordance with law.
order that constitutes a legal justification for respondent's non- A person guilty of misbehavior in the presence of or so near
compliance with the reinstatement order. Respondent's failure to the Chairman or any member of the Commission or any Labor Arbiter
exercise the alternative options of actual reinstatement and as to obstruct or interrupt the proceedings before the same, including
payroll reinstatement was thus justified. Such being the case, disrespect toward said officials, offensive personalities toward others,
respondent's obligation to pay the salaries pending appeal, as the or refusal to be sworn, or to answer as a witness or to subscribe an
normal effect of the non-exercise of the options, did not attach. affidavit or deposition when lawfully required to do so, may be
While reinstatement pending appeal aims to avert the summarily adjudged in direct contempt by said officials and punished
continuing threat or danger to the survival or even the life of the by fine not exceeding five hundred pesos (P500) or imprisonment not
dismissed employee and his family, it does not contemplate the exceeding five (5) days, or both, if it be the Commission or a member
period when the employer-corporation itself is similarly in thereof, or by a fine not exceeding one hundred pesos (P100) or
a judicially monitored state of being resuscitated in order to imprisonment not exceeding one (1) day, or both, if it be a Labor
survive. Arbiter.
The parallelism between a judicial order of The person adjudged in direct contempt by a Labor Arbiter
corporation rehabilitation as a justification for the non-exercise may appeal to the Commission and the execution of the judgment shall
of its options, on the one hand, and a claim of actual and be suspended pending the resolution of the appeal upon the filing by
imminent substantial losses as ground for retrenchment, on the such person of a bond on condition that he will abide by and perform
other hand, stops at the red line on the financial statements. the judgment of the Commission should the appeal be decided against
Beyond the analogous condition of financial gloom, as discussed him. Judgment of the Commission on direct contempt is immediately
by Justice Leonardo Quisumbing in his Separate Opinion, are executory and unappealable. Indirect contempt shall be dealt with by
more salient distinctions. Unlike the ground of substantial losses the Commission or Labor Arbiter in the manner prescribed under
contemplated in a retrenchment case, the state of corporate Rule 71 of the Revised Rules of Court; and
rehabilitation was judicially pre-determined by a competent (e) To enjoin or restrain any actual or threatened
court and not formulated for the first time in this case by commission of any or all prohibited or unlawful acts or to require the
respondent. performance of a particular act in any labor dispute which, if not
More importantly, there are legal effects arising from restrained or performed forthwith, may cause grave or irreparable
a judicial order placing a corporation under rehabilitation. damage to any party or render ineffectual any decision in favor of such
Respondent was, during the period material to the case, party: Provided, That no temporary or permanent injunction in any
effectively deprived of the alternative choices under Article 223 case involving or growing out of a labor dispute as defined in this Code
of the Labor Code, not only by virtue of the statutory injunction shall be issued except after hearing the testimony of witnesses, with
but also in view of the interim relinquishment of management opportunity for cross-examination, in support of the allegations of a
control to give way to the full exercise of the powers of the complaint made under oath, and testimony in opposition thereto, if
rehabilitation receiver. Had there been no need to rehabilitate, offered, and only after a finding of fact by the Commission, to the
respondent may have opted for actual physical reinstatement effect:
pending appeal to optimize the utilization of resources. Then (1) That prohibited or unlawful acts have been threatened
again, though the management may think this wise, the and will be committed unless restrained, or have been committed and
rehabilitation receiver may decide otherwise, not to mention the will be continued unless restrained, but no injunction or temporary
subsistence of the injunction on claims. restraining order shall be issued on account of any threat, prohibited
In sum, the obligation to pay the employee's salaries or unlawful act, except against the person or persons, association or
upon the employer's failure to exercise the alternative options organization making the threat or committing the prohibited or
under Article 223 of the Labor Code is not a hard and fast rule, unlawful act or actually authorizing or ratifying the same after actual
considering the inherent constraints of corporate rehabilitation. knowledge thereof;
xxx (2) That substantial and irreparable injury to complainant’s
property will follow
3. Powers of the Commission (3) That as to each item of relief to be granted, greater
injury will be inflicted upon complainant by the denial of relief than
Article 225. [218] Powers of the Commission. The will be inflicted upon defendants by the granting of relief;
Commission shall have the power and authority: (4) That complainant has no adequate remedy at law; and
(a) To promulgate rules and regulations governing the (5) That the public officers charged with the duty to protect
hearing and disposition of cases before it and its regional branches, as complainant’s property are unable or unwilling to furnish adequate
well as those pertaining to its internal functions and such rules and protection.
regulations as may be necessary to carry out the purposes of this Code; Such hearing shall be held after due and personal notice
(b) To administer oaths, summon the parties to a thereof has been served, in such manner as the Commission shall
controversy, issue subpoenas requiring the attendance and testimony of direct, to all known persons against whom relief is sought, and also to
witnesses or the production of such books, papers, contracts, records, the Chief Executive and other public officials of the province or city
statement of accounts, agreements, and others as may be material to a within which the unlawful acts have been threatened or committed,
just determination of the matter under investigation, and to testify in charged with the duty to protect complainant’s property: Provided,
any investigation or hearing conducted in pursuance of this Code; however, that if a complainant shall also allege that, unless a
(c) To conduct investigation for the determination of a temporary restraining order shall be issued without notice, a
question, matter or controversy within its jurisdiction, proceed to hear substantial and irreparable injury to complainant’s property will be

Page 83 of 117
unavoidable, such a temporary restraining order may be issued upon unfair labor practice and shall be resolved as grievances under the
testimony under oath, sufficient, if sustained, to justify the Commission Collective Bargaining Agreement. For purposes of this article, gross
in issuing a temporary injunction upon hearing after notice. Such a violations of Collective Bargaining Agreement shall mean flagrant
temporary restraining order shall be effective for no longer than and/or malicious refusal to comply with the economic provisions of
twenty (20) days and shall become void at the expiration of said twenty such agreement.
(20) days. No such temporary restraining order or temporary The Commission, its Regional Offices and the Regional
injunction shall be issued except on condition that complainant shall Directors of the Department of Labor and Employment shall not
first file an undertaking with adequate security in an amount to be entertain disputes, grievances or matters under the exclusive and
fixed by the Commission sufficient to recompense those enjoined for original jurisdiction of the Voluntary Arbitrator or panel of Voluntary
any loss, expense or damage caused by the improvident or erroneous Arbitrators and shall immediately dispose and refer the same to the
issuance of such order or injunction, including all reasonable costs, Grievance Machinery or Voluntary Arbitration provided in the
together with a reasonable attorney’s fee, and expense of defense Collective Bargaining Agreement.
against the order or against the granting of any injunctive relief sought
in the same proceeding and subsequently denied by the Commission. In Sime Darby Pilipinas, Inc. v. Deputy Administrator
The undertaking herein mentioned shall be understood to
Magsalin, it was held that a voluntary arbitrator has plenary
constitute an agreement entered into by the complainant and the surety
upon which an order may be rendered in the same suit or proceeding jurisdiction and authority to interpret an agreement to arbitrate and
against said complainant and surety, upon a hearing to assess damages, to determine the scope of his own authority when the said
of which hearing, complainant and surety shall have reasonable notice, agreement is vague — subject only, in a proper case, to
the said complainant and surety submitting themselves to the the certiorari jurisdiction of this Court.
jurisdiction of the Commission for that purpose. But nothing herein Voluntary arbitrator may award backwages upon a
contained shall deprive any party having a claim or cause of action finding of illegal dismissal, even though the issue of entitlement
under or upon such undertaking from electing to pursue his ordinary thereto is not explicitly claimed in the Submission Agreement.
remedy by suit at law or in equity: Provided, further, That the
Backwages, in general, are awarded on the ground of equity as a
reception of evidence for the application of a writ of injunction may be
delegated by the Commission to any of its Labor Arbiters who shall form of relief that restores the income lost by the terminated
conduct such hearings in such places as he may determine to be employee by reason of his illegal dismissal [7k Corporation v.
accessible to the parties and their witnesses and shall submit thereafter Albarico].
his recommendation to the Commission.
1. Interpretation or Implementation of The
A writ of preliminary injunction and a TRO are Collective Bargaining Agreement
injunctive reliefs and preservative remedies for the protection of In San Jose v. NLRC, it was held that the Labor Arbiter
substantive rights and interests. An application for the issuance of a had no jurisdiction to hear and decide petitioners money-
writ of preliminary injunction and/or TRO may be granted upon the claim underpayment of retirement benefits, as the controversy
filing of a verified application showing facts entitling the applicant between the parties involved an issue arising from the
to the relief demanded. Essential for granting the injunctive relief is interpretation or implementation of a provision of the collective
the existence of an urgent necessity for the writ in order to prevent bargaining agreement. The Voluntary Arbitrator or Panel of
serious damage. A TRO issues only if the matter is of such extreme Voluntary Arbitrators has original and exclusive jurisdiction over
urgency that grave injustice and irreparable injury will arise unless the controversy under Article 261 of the Labor Code, and not the
it is issued immediately [Brizuela v. Legaspi] Labor Arbiter.
As with all equitable remedies, injunction must be issued In the case of Maneja v. NLRC, we held that the
only at the instance of a party who possesses sufficient interest in dismissal case does not fall within the phrase "grievances arising
or title to the right or the property sought to be protected. It is from the interpretation or implementation of the collective
proper only when the applicant appears to be entitled to the relief bargaining agreement and those arising from the interpretation or
demanded in the complaint, which must aver the existence of the enforcement of company personnel policies." In Maneja, the hotel
right and the violation of the right, or whose averments must in the employee was dismissed without hearing. We ruled that her
minimum constitute a prima facieshowing of a right to the final dismissal was unjustified, and her right to due process was
relief sought. Accordingly, the conditions for the issuance of the violated, absent the twin requirements of notice and hearing. We
injunctive writ are: (a) that the right to be protected exists prima also held that the labor arbiter had original and exclusive
facie; (b) that the act sought to be enjoined is violative of that right; jurisdiction over the termination case, and that it was error to give
and (c) that there is an urgent and paramount necessity for the writ the voluntary arbitrator jurisdiction over the illegal dismissal case.
to prevent serious damage. An injunction will not issue to protect a
right not in esse, or a right which is merely contingent and may 2. Interpretation or Enforcement of Company
never arise; or to restrain an act which does not give rise to a cause Personnel Policies
of action; or to prevent the perpetration of an act prohibited by Company personnel policies are guiding principles stated
statute. Indeed, a right, to be protected by injunction, means a right in broad, long-range terms that express the philosophy or beliefs of
clearly founded on or granted by law or is enforceable as a matter an organizations top authority regarding personnel matters. They
of law [BPI v. Hontanosas]. deal with matters affecting efficiency and well-being of employees
and include, among others, the procedure in the administration of
D. VOLUNTARY ARBITRATOR wages, benefits, promotions, transfer and other personnel
movements which are usually not spelled out in the collective
Article 274. [261] Jurisdiction of Voluntary Arbitrators and agreement. The usual source of grievances, however, are the rules
Panel of Voluntary Arbitrators. The Voluntary Arbitrator or panel of and regulations governing disciplinary actions [Maneja v. NLRC].
Voluntary Arbitrators shall have original and exclusive jurisdiction to
hear and decide all unresolved grievances arising from the 3. Other Labor Disputes
interpretation or implementation of the Collective Bargaining
Agreement and those arising from the interpretation or enforcement of
Article 275. [262] Jurisdiction over other Labor Disputes. The
company personnel policies referred to in the immediately preceding
Voluntary Arbitrator or panel of Voluntary Arbitrators, upon
article. Accordingly, violations of a Collective Bargaining Agreement,
except those which are gross in character, shall no longer be treated as

Page 84 of 117
agreement of the parties, shall also hear and decide all other labor doctrine of exhaustion of administrative remedies. For this reason, an
disputes including unfair labor practices and bargaining deadlocks. appeal from administrative agencies to the CA via Rule 43 of the Rules
of Court requires exhaustion of available remedies as a condition
4. Procedure precedent to a petition under that Rule.
The requirement that administrative remedies be exhausted is
based on the doctrine that in providing for a remedy before an
Article 276. [262-A] Procedures. The Voluntary Arbitrator administrative agency, every opportunity must be given to the agency to
or panel of Voluntary Arbitrators shall have the power to hold resolve the matter and to exhaust all opportunities for a resolution under the
hearings, receive evidences and take whatever action is necessary to given remedy before bringing an action in, or resorting to, the courts of
resolve the issue or issues subject of the dispute, including efforts to justice. Where Congress has not clearly required exhaustion, sound judicial
effect a voluntary settlement between parties. discretion governs, guided by congressional intent.
All parties to the dispute shall be entitled to attend the By disallowing reconsideration of the VA's decision, Section
arbitration proceedings. The attendance of any third party or the 7, Rule XIX of DO 40-03 and Section 7 of the 2005 Procedural
exclusion of any witness from the proceedings shall be determined by Guidelines went directly against the legislative intent behind Article
the Voluntary Arbitrator or panel of Voluntary Arbitrators. Hearing 262-A of the Labor Code. These rules deny the VA the chance to
may be adjourned for cause or upon agreement by the parties. correct himself and compel the courts of justice to prematurely
Unless the parties agree otherwise, it shall be mandatory for intervene with the action of an administrative agency entrusted with
the Voluntary Arbitrator or panel of Voluntary Arbitrators to render the adjudication of controversies coming under its special knowledge,
an award or decision within twenty (20) calendar days from the date of training and specific field of expertise. In this era of clogged court
submission of the dispute to voluntary arbitration. dockets, the need for specialized administrative agencies with the special
The award or decision of the Voluntary Arbitrator or panel knowledge, experience and capability to hear and determine promptly
of Voluntary Arbitrators shall contain the facts and the law on which it disputes on technical matters or intricate questions of facts, subject to
is based. It shall be final and executory after ten (10) calendar days judicial review, is indispensable. In Industrial Enterprises, Inc. v. Court of
from receipt of the copy of the award or decision by the parties. Appeals, we ruled that relief must first be obtained in an administrative
Upon motion of any interested party, the Voluntary proceeding before a remedy will be supplied by the courts even though the
Arbitrator or panel of Voluntary Arbitrators or the Labor Arbiter in matter is within the proper jurisdiction of a court. (Emphasis supplied)
the region where the movant resides, in case of the absence or Hence, the 10-day period stated in Article 276 should be
incapacity of the Voluntary Arbitrator or panel of Voluntary understood as the period within which the party adversely affected by the
Arbitrators, for any reason, may issue a writ of execution requiring ruling of the Voluntary Arbitrators or Panel of Arbitrators may file a
either the sheriff of the Commission or regular courts or any public motion for reconsideration. Only after the resolution of the motion for
official whom the parties may designate in the submission agreement to reconsideration may the aggrieved party appeal to the CA by filing the
execute the final decision, order or award. petition for review under Rule 43 of the Rules of Court within 15 days
from notice pursuant to Section 4 of Rule 43. (Citations omitted;
The 10-day period stated in Article 276 should be emphasis and underscoring supplied)
understood as the period within which the party adversely affected The Court further noted in Guagua that despite the clarification
by the ruling of the Voluntary Arbitrators or Panel of Arbitrators made in Teng v. Pagahac in 2010, the Department of Labor and
Employment (DOLE) and NCMB have yet to revise or amend Section
may file a motion for reconsideration. Only after the resolution of
7, Rule VII of the Revised Procedural Guidelines in the Conduct of
the motion for reconsideration may the aggrieved party appeal to Voluntary Arbitration Proceedings and that such inaction has caused
the CA by filing the petition for review under Rule 43 of the Rules confusion, particularly with respect to the filing of the motion for
of Court within 15 days from notice pursuant to Section 4 of Rule reconsideration as a condition precedent to the filing of the petition for
43 [Guagua National Colleges v. Court of Appeals]. review in the CA. Thus, the Court expressly directed the DOLE and the
NCMB to cause the revision or amendment of the aforesaid section in order
Chin v. Maersk-Filipinas Crewing Inc. to allow the filing of motions for reconsideration in line with Article 276 of
Held: In the 2018 case of Guagua National Colleges vs. the Labor Code. Unfortunately, no revision has yet been made in this
CA (Guagua), the Court acknowledged the variance in its rulings and regard. Consequently, the DOLE and the NCMB are again reminded to
categorically declared that the correct period to appeal the decision or cause the revision or amendment of Section 7, Rule VII of the Revised
award of the Voluntary Arbitrator or Panel of Arbitrators to the CA via a Procedural Guidelines in the Conduct of Voluntary Arbitration Proceedings
petition for review under Rule 43 of the Rules of Court is the fifteen (15)- insofar as it prohibits the filing of a motion for reconsideration, if they have
day period set forth in Section 4 thereof reckoned from notice or receipt of not done so.
the VA's resolution on the motion for reconsideration, and that the ten (10)- In view of the foregoing, petitioner in this case had fifteen (15)
day period provided in Article 276 of the Labor Code refers to the period days from receipt of the Resolution denying his motion for reconsideration
within which an aggrieved party may file said motion for reconsideration, to file his petition for review with the CA. Having received a copy of the
to wit: VA's October 29, 2018 Resolution on November 22, 2018, petitioner
Given the variable rulings of the Court, what should now be the therefore had until December 7, 2018 to file his petition. As the records
period to be followed in appealing the decisions or awards of the Voluntary show that the petition was filed on December 4, 2018, albeit through a
Arbitrators or Panel of Arbitrators? private courier, it was therefore timely filed and the CA erred in dismissing
In the 2010 ruling in Teng v. Pagahac, the Court clarified that it outright. To rule otherwise would be clearly antithetical to the tenets of
the 10-day period set in Article 276 of the Labor Code gave the aggrieved fair play, not to mention the undue prejudice to petitioner's rights.[30] Thus,
parties the opportunity to file their motion for reconsideration, which was in light of the fact that the CA dismissed the petition for review outright
more in keeping with the principle of exhaustion of administrative based solely on procedural grounds, a remand of the case for a resolution
remedies, holding thusly: on the merits is warranted.
In the exercise of its power to promulgate implementing rules
and regulations, an implementing agency, such as the Department of Labor, Unicrafts Industries v. Court of Appeals
is restricted from going beyond the terms of the law it seeks to implement; Held: While under the law decisions of voluntary arbitrators are
it should neither modify nor improve the law. The agency formulating the accorded finality, the same may still be subject to review, such as here
rules and guidelines cannot exceed the statutory authority granted to it by where there was a violation of to due process such as in this case. 
the legislature. The proceedings before the Voluntary Arbitrator are non-
By allowing a 10-day period, the obvious intent of Congress litigious in nature, and they are nit governed by technical rules applicable to
in amending Article 263 to Article 262-A is to provide an opportunity court or judicial proceedings but they must, at all times, comply with the
for the party adversely affected by the VA's decision to seek recourse requirement of due process making reference to Rule VI, Section 6 of the
via a motion for reconsideration or a petition for review under Rule 43 Procedural Guidelines in the Conduct of Voluntary Arbitration Proceeding
of the Rules of Court filed with the CA. Indeed, a motion for which provides:
reconsideration is the more appropriate remedy in line with the

Page 85 of 117
SECTION 6. Arbitration Hearing. — In the conduct of hearing, Despite our sympathy for the workers’ plight, however, we find
the arbitrator shall provide the parties adequate opportunities to be heard. no legal support for their opposition to the conclusion and findings of the
He shall control the proceedings and see to it that proper decorum is voluntary arbitrator and the Court of Appeals.
observed. He must render a ruling of the issue/s raised in the course of the On record, there is no showing that the abolition of the
proceedings. He must treat all significant aspects of the proceedings as petitioners positions was capricious or whimsical. The appellate court, as
confidential in nature unless confidentiality is waived by the parties.  well as the voluntary arbitrator, based their decisions on applicable law and
Further, While the intendment of our laws is to favor the the evidence. As confirmed by the appellate court, the voluntary arbitrator
employee, it in no way implies that the employer is not entitled to due also found that petitioners reinstatement had become not only inappropriate
process, hence, the finding of the Voluntary Arbitrator, however, was but also impossible.
premature and null and void, and there is a need to remand the case to the Regrettably, petitioners now raise questions the determination of
Voluntary Arbitrator to allow Unicraft to present evidence. As such, the which would require the Court to look into the evidence adduced by the
Court of Appeals committed grave abuse of discretion amounting to lack of parties. This cannot be done in a petition for review on certiorari. It is
jurisdiction when it ordered the immediate execution of the Voluntary outside its purview under Rule 45 of the 1997 Rules of Court. Factual
Arbitrator's decision.  findings of labor officials who are deemed to have acquired expertise in
matters within their respective jurisdiction are generally accorded not only
respect but even finality, and bind us when supported by substantial
Abalos v. Philex Mining
evidence.
Held: A basic tenet in our rules of procedure is that an award
that is final and executory cannot be amended or modified anymore.
Nothing is more settled in law than that once a judgment attains finality it E. DOLE REGIONAL DIRECTORS
thereby becomes immutable and unalterable. It may no longer be The DOLE Regional Directors are the duly authorized
modified in any respect, even if the modification is meant to correct what is representatives of the DOLE Secretary referred to in Article 128 of
perceived to be an erroneous conclusion of fact or law, and regardless of the Labor Code which grants to them both visitorial and
whether the modification is attempted to be made by the court rendering it enforcement powers. They are in charge of the administration and
or by the highest court of the land.
enforcement of labor standards within their respective territorial
However, this rule is subject to exceptions as stated in the case
of David vs. CA, 316 SCRA 710 (1999), cited by respondent:
jurisdiction [Atilano v. De la Cruz].
One exception is that where facts and/or events transpire after a The DOLE Regional Directors have original and
decision has become executory, which facts and/or events present a exclusive jurisdiction over the following cases:
supervening cause or reason which renders the final and executory
decision no longer enforceable.  a. Visitorial (inspection) cases under Article 37 of the
Under the law, the court may modify or alter a judgment Labor Code referring to the inspection of the
even after the same has become executory whenever circumstances premises, books of accounts and records of any
transpire rendering its execution unjust and inequitable, as where
person or entity covered by the Title I, Book I,
certain facts and circumstances justifying or requiring such
modification or alteration transpired after the judgment has become
Labor Code.
final and executory. b. Visitorial (inspection) and enforcement cases under
The fact that the decision has become final does not preclude a Article 128, (either routine or initiated through a
modification or an alteration thereof because even with the finality of complaint).
judgment, when its execution becomes impossible or unjust, it may be c. Visitorial cases under Article 289, involving
modified or altered to harmonize the same with justice and the facts. examination of books of accounts of independent
In Deltaventures Resources Inc. vs. Cabato, 327 SCRA 521 unions, local chapters/chartered locals and workers’
(2000), we held that “jurisdiction once acquired is not lost upon the
associations.
instance of the parties but continues until the case is terminated.” The
power of a voluntary arbitrator to issue a writ of execution carries with d. Occupational safety and health violations.
it the power to inquire into the correctness of its execution and to e. Small money claims cases arising from labor
consider whatever supervening events transpire during execution. standards violations in an amount not exceeding
Therefore, we are in agreement with the appellate court that a voluntary P5000.00 and not accompanied with a claim for
arbitrator has jurisdiction to amend the mode of executing an award if reinstatement under Article 129.
and when the case merits such amendment. f. Cases related to private recruitment and placement
However, we find respondent’s reliance on the doctrine of agencies for local employment.
“strained relations” misplaced. In Mercury Drug Corporation vs.
g. Cases submitted for voluntary arbitration in their
Quijano, we stated that said doctrine is inapplicable to a situation where
the employee has no say in the operation of the employer’s business.  capacity as Ex-Officio Voluntary Arbitrators under
Petitioners herein are part of the rank-and-file workforce; they D.O. No. 83-07
are cooks, miners, helpers and mechanics of the respondent. h. Request for SEBA certification when made in an
As held also in the Mercury Drug case: unorganized establishment with only one (1)
To protect labor’s security of tenure, we emphasize that the legitimate union.
doctrine of strained relations should be strictly applied so as not to
deprive an illegally dismissed employee of his right to reinstatement. 1. Small Money Claims Cases
Every labor dispute almost always results in strained relations and the
The DOLE Regional Director has original jurisdiction
phrase cannot be given an overarching interpretation, otherwise an
unjustly dismissed employee can never be reinstated. over small money claims cases arising from labor standards
Considering the circumstances in the present case, we find violations in the amount not exceeding P5000.00 and not
that the only issue to be resolved is whether the supervening events are accompanied with a claim for reinstatement under Article 129 of
grave enough to warrant a modification in the execution of the the Labor Code.
judgment.  Article 129 contemplates the recovery of wages and other
Both the voluntary arbitrator and the Court of Appeals found monetary claims and benefits, including legal interest, owing to an
that reinstatement is no longer possible due to the fact that respondent has employee arising from employer-employee relationship provided
been continuously suffering business losses and reducing the number of its
the claim does not exceed P5000.00
employees pending litigation, and so the positions held by petitioners were
abolished as a cost-cutting measure.  The following requisites for the valid exercise of
jurisdiction over small money claims must all concur, to wit:

Page 86 of 117
a. The claim is presented by an employee or domestic
worker or kasambahay; a. Article 37 treats of the visitorial power of the DOLE
b. The claimant, no longer being employed, does not Secretary and the DOLE Regional Directors in
seek reinstatement; and relation to recruitment and placement of workers for
c. The aggregate money claim of the employee does both local and overseas employment.
not exceed P5000.00. b. Article 289 treats of the visitorial power of the
DOLE Secretary and the DOLE Regional Directors
In the absence of any of the aforesaid three (3) requisites, to inquire into the financial activities of legitimate
the Labor Arbiters have original and exclusive jurisdiction over all labor organizations.
claims arising from employer-employee relations, other than claims
for employees’ compensation, social security, PhilHealth and Article 129 vs. Article 128
maternity benefits.
Article 129 Article 128
2. Visitorial and Enforcement Powers Adjudication powers Visitorial and enforcement powers
Article 128 basically enunciates the three kinds of power Power to hear and decide any Inspection of establishments and
claim for recovery of wages, the issuance of orders to comply
which the DOLE Secretary and/or the Regional Directors, his duly
simple (small) money claims, and with labor standards, wage orders,
authorized representatives, may exercise in connection with the other benefits of employees, and other labor laws and
administration and enforcement of the labor standards provisions of domestic worker, or kasambahay, regulations
the Labor Code and of any labor law, wage order or rules and arising from a severed employer-
regulations issued pursuant thereto. employee relationship
No employer-employee Employment relationship is
What is being inspected in the exercise of the visitorial
relationship required
and enforcement powers granted to the DOLE Secretary or the Appealable to the NLRC Appealable to the DOLE
DOLE Regional Directors under Article 128 is the employer- Secretary
establishment and not the employees thereof. Consequently, in case
of a finding of violation of the labor standards, the awards granted 3. Appeals from Decisions of DOLE Regional
in the inspection case are not confined to employees who signed Directors
the complaint inspection but are equally applicable to all those who Decisions in the following cases relevant and related to
were employed by the establishment concerned at the time the labor relations are appealable to the BLR Director:
complaint was filed, even if they were not signatories [Maternity
Children’s Hospital v. Secretary of Labor]. a. Visitorial cases under Article 289, involving
examination of books of accounts of independent
Grant of Another Visitorial Power Under Articles 37 unions, local chapters/chartered locals and workers’
and 289 associations;
Besides the visitorial power granted under Article 128, b. Union registration-related cases such as:
another visitorial power is granted to the DOLE Secretary and the
DOLE Regional Directors under Article 37 and 289 of the Labor (i) Denial of applications for union registration
Code, to wit: of independent unions, local chapters and
workers’ associations;
Article 37. Visitorial Power. The Secretary of Labor (ii) Revocation or cancellation of registration
or his duly authorized representatives may, at any time, inspect of said unions
the premises, books of accounts and records of any person or
entity covered by this Title, require it to submit reports regularly
on prescribed forms, and act on violation of any provisions of
c. Notice of merger, consolidation, affiliation and
this Title. change of name of said unions and or petition for
denial thereof;
Article 289. [274] Visitorial Power. The Secretary of d. CBA-related cases, such as:
Labor and Employment or his duly authorized representative is
hereby empowered to inquire into financial activities of (i) Applications for registration of single-
legitimate labor organizations upon the filing of a complaint enterprise CBAs or petition for
under oath and duly supported by the written consent of at least
deregistration thereof;
twenty percent (20%) of the total membership of the labor
organization concerned and to examine their books of accounts (ii) Petition for denial of registration of
and other records to determine compliance or non-compliance single-enterprise CBAs or denial of
with the law and to prosecute any violations of the law and the petition thereof.
union constitution and by-laws: Provided, That such inquiry or
examination shall not be conducted during the sixty (60) days Decisions of the DOLE Regional Directors in the
freedom period nor within the thirty (30) days immediately following cases which are not related to labor relations are
preceding the date of election of union officials. appealable to the DOLE Secretary and not to the BLR Director:

Article 128 should not be confused with Articles 37 and a. Visitorial (inspection) cases under Article 37;
289 because the purpose and object of the DOLE Secretary’s b. Visitorial (inspection) and enforcement cases under
exercise of his visitorial power provided thereunder are completely Article 128, (either routine or initiated through a
distinct from each other. complaint);
While Article 128 dwells on the visitorial and c. Occupational safety and health violations;
enforcement powers of the DOLE Secretary to inquire into the d. Cases related to private recruitment and placement
employer’s compliance with labor standards prescribed under labor agencies (PRPAs) for local employment, such as:
laws and social legislations, the purposes of the other articles are
different, thus: (i) Application for license or denial thereof;

Page 87 of 117
(ii) Complaints for suspension or Article 37. Visitorial Power. The Secretary of Labor or his
cancellation of license by reason of duly authorized representatives may, at any time, inspect the premises,
administrative offenses; books of accounts and records of any person or entity covered by this
(iii) Complaints for illegal recruitment; and Title, require it to submit reports regularly on prescribed forms, and
act on violation of any provisions of this Title.
(iv) Petition for closure of agency.

Additionally, their decisions on small money claims Article 289. [274] Visitorial Power. The Secretary of Labor
cases arising from labor standards violations in an amount not and Employment or his duly authorized representative is hereby
empowered to inquire into financial activities of legitimate labor
exceeding P5000.00 and not accompanied with a claim for
organizations upon the filing of a complaint under oath and duly
reinstatement under Article 129 are appealable to the NLRC. supported by the written consent of at least twenty percent (20%) of
the total membership of the labor organization concerned and to
F. DOLE SECRETARY examine their books of accounts and other records to determine
compliance or non-compliance with the law and to prosecute any
1. Visitorial and Enforcement Powers violations of the law and the union constitution and by-laws: Provided,
That such inquiry or examination shall not be conducted during the
sixty (60) days freedom period nor within the thirty (30) days
Article 128. Visitorial and Enforcement Power. (a) The
immediately preceding the date of election of union officials.
Secretary of Labor and Employment or his duly authorized
representatives, including labor regulation officers, shall have access to
or night whenever work is being undertaken therein, and the right to As a summary, the DOLE Secretary has the following
copy therefrom, to question any employee and investigate any fact, enforcement powers:
condition or matter which may be necessary to determine violations or
which may aid in the enforcement of this Code and of any labor law, a. To issue compliance orders to give effect to the
wage order or rules and regulations issued pursuant thereto. labor standards provisions of the Labor Code and
(b) Notwithstanding the provisions of Articles 129 and 21789
other labor legislation [Article 128(b)];
of this Code to the contrary, and in cases where the relationship of
employer-employee still exists, the Secretary of Labor and b. To issue writs of execution to the appropriate
Employment or his duly authorized representatives shall have the authority for the enforcement of their orders, except
power to issue compliance orders to give effect to the labor standards in cases where the employer contests the findings of
provisions of this Code and other labor legislation based on the the labor and employment officers and raises issues
findings of labor employment and enforcement officers or industrial supported by documentary proofs which were not
safety engineers made in the course of inspection. The Secretary or his considered in the course of inspection [Ibid].
duly authorized representatives shall issue writs of execution to the
c. To order stoppage of work or suspension of
appropriate authority for the enforcement of their orders, except in
operations of any unit or department of an
cases where the employer contests the findings of the labor
employment and enforcement officer and raises issues supported by establishment when non-compliance with the law or
documentary proofs which were not considered in the course of implementing rules and regulations poses grave and
inspection. imminent danger to the health and safety of workers
An order issued by the duly authorized representative of the in the workplace [Article 128(c)];
Secretary of Labor and Employment under this Article may be d. To require employers to keep and maintain
appealed to the latter. In case said order involves a monetary award, employment records as may be necessary in aid of
an appeal by the employer may be perfected only upon the posting of a
his visitorial and enforcement powers under the
cash or surety bond issued by a reputable bonding company duly
accredited by the Secretary of Labor and Employment in the amount
Labor Code [Article 128(f)].
equivalent to the monetary award in the order appealed from.
(c) The Secretary of Labor and Employment may likewise 2. Power to Suspend Effects of Termination
order stoppage of work or suspension of operations of any unit or The DOLE Secretary may suspend the effects of
department of an establishment when non-compliance with the law or termination pending resolution of the dispute in the event of a
implementing rules and regulations poses grave and imminent danger prima facie finding by the appropriate official of the DOLE before
to the health and safety of workers in the workplace. Within twenty- whom the dispute is pending that:
four hours, a hearing shall be conducted to determine whether an
order for the stoppage of work or suspension of operations shall be
lifted or not. In case the violation is attributable to the fault of the
a. The termination may cause a serious labor dispute;
employer, he shall pay the employees concerned their salaries or wages and/or
during the period of such stoppage of work or suspension of operation. b. The termination is in implementation of a mass lay-
(d) It shall be unlawful for any person or entity to obstruct, off [Article 292(b)].
impede, delay or otherwise render ineffective the orders of the
Secretary of Labor and Employment or his duly authorized The Labor Arbiters and the Voluntary Arbitrators or
representatives issued pursuant to the authority granted under this panel of Voluntary Arbitrators, as the case may be, are the
Article, and no inferior court or entity shall issue temporary or
“appropriate officials” referred to in Article 292(b) who may make
permanent injunction or restraining order or otherwise assume
jurisdiction over any case involving the enforcement orders issued in
the preliminary determination of the existence of prima facie
accordance with this Article. evidence that the termination will cause a serious labor dispute or
(e) Any government employee found guilty of violation of, or is being made in implementation of a mass lay-off. Such prima
abuse of authority, under this Article shall, after appropriate facie finding will then become the basis for the issuance by the
administrative investigation, be subject to summary dismissal from the DOLE Secretary of his order suspending the effects of termination
service. which, as earlier emphasized, would mean the immediate
(f) The Secretary of Labor and Employment may, by reinstatement of the terminated employees pending the final
appropriate regulations, require employers to keep and maintain such
resolution of their termination case.
employment records as may be necessary in aid of his visitorial and
enforcement powers under this Code.
Suspension of Termination vs. Assumption or
Certification

Page 88 of 117
Suspension of Termination Assumption or Certification workers’ associations operating in more than one
Involves only the issue of Applicable to all labor disputes, region;
termination of employment which irrespective of the grounds, d. All cases decided by the POEA in its original and
may cause serious labor dispute or provided it will cause strikes or
exclusive jurisdiction.
mass lay-off lockouts in industries
indispensable to national interest
Requires preliminary Does not require preliminary 4. Contempt Powers
determination of the existence of prima facie determination. In fact,
prima facie evidence prior notice and hearing is not Article 231. [225] Contempt Powers of the Secretary. In the
required exercise of his powers under this Code, the Secretary of Labor may
Serious labor dispute may or may Labor dispute will cause or likely
hold any person in direct or indirect contempt and impose the
not involve a strike lockout to cause a strike or lockout
appropriate penalties therefor.
Irrespective of the nature of the Only in industries indispensable
business of the employer to national interest
Remedy is immediate Remedy is automatic return to 5. Petition for Certiorari under Rule 65
reinstatement pending resolution work of the strikers or locked-out It has long been settled that the remedy of an aggrieved
of the termination case employees or enjoining the strike party in a decision or resolution of the Secretary of Labor is to
or lockout.
timely file a motion for reconsideration as a precondition for any
further or subsequent remedy, and then seasonably file a special
The case of University of Sto. Tomas v. NLRC and UST
civil action for certiorari under Rule 65 of the 1997 Rules on Civil
Faculty Union illustrates the situation where the DOLE Secretary
Procedure; it is a condition sine qua non to afford an opportunity
ordered both the suspension of the effects of termination and the
for the correction of the error or mistake complained of [Philtranco
return to work of employees pursuant to a certification order. In
Service v. Philtranco Workers Union].
this case, all the sixteen (16) officers and directors of the faculty
Accordingly, the Secretary of Labor’s Decision is a
union were terminated on the grounds of grave misconduct, serious
proper subject of certiorari. In fact, the Court said that there is no
disrespect to a superior and conduct unbecoming a faculty member.
distinction: when the Secretary of Labor assumes jurisdiction over
As a result of said dismissal, some faculty members staged mass
a labor case in an industry indispensable to national interest "he
leaves of absence for several days, disrupting classes in all levels at
exercises great breadth of discretion" in finding a solution to the
the university., the faculty union filed a complaint for illegal
parties’ dispute. "[T]he authority of the Secretary of Labor to
dismissal and unfair labor practice with the Labor Arbiter who, on
assume jurisdiction over a labor dispute causing or likely to cause a
a prima facie showing that the termination was causing a serious
strike or lockout in an industry indispensable to national interest
labor dispute, certified the matter to the DOLE Secretary for a
includes and extends to all questions and controversies arising
possible suspension of the effects of termination. On this basis,
therefrom. This power may not be the subject of appeal [Ibid].
Secretary Franklin Drilon issued an order suspending the effects of
the termination of the union officers and directors and directing the
G. BUREAU OF LABOR RELATIONS
university to accept them back to work under the same terms and
conditions prevailing prior to their dismissal. Later, on the asis of a
1. Administrative Functions
petition for assumption or certification filed by the university,
The BLR has to exercise its administrative functions over
Drilon modified said order by certifying the labor dispute to the
the following:
NLRC for compulsory arbitration pursuant to Article 278(g). He
accordingly ordered the university to readmit all its faculty
a. Applications for registration of federations, national
members, including the 16 union officers and directors, under the
unions or workers’ associations operating in more
same terms and conditions prevailing prior to the dispute.
than one region.
Based on the foregoing, it may be said that suspension of
b. Notice for change of name of a registered labor
the effects of termination has the same effect as assumption or
organization.
certification as far as the reinstatement of the affected employees is
c. Notice of merger or consolidation of federations or
concerned.
national unions
d. Keeping of registry of legitimate labor organizations
3. Exclusive Appellate Jurisdiction
e. Petition for the conduct of election of officers of
The DOLE Secretary has appellate jurisdiction over the
federations, national, or industry unions and trade
following cases:
union centers.
f. Request for examination of books of accounts of
a. Orders issued by the Regional Director under:
federations or national unions and trade union
centers (Article 289)
(i) Article 128
g. Petition for cancellation or application for voluntary
(ii) Occupational safety and health violations;
dissolution of federations, national or industry
(iii) Complaints against private recruitment and
unions and trade union centers.
placement agencies (PRPAs) for local
h. Registration of multi-employer collective bargaining
employment.
agreements.
b. Order of the Med Arbiter granting the petition for
NOTE: All issues pertaining to the (1) validity of the
Certification Election (in an organized
petitioning union’s certificate of registration or its legal personality
establishment) and denying the petition for
as a labor organization, (2) validity of registration and execution of
certification election (either in organized or
CBA shall be heard and resolved by the Regional Director in an
unorganized establishment);
independent petition or cancellation of its registration and not by
c. All cases decided by the BLR in its original and
the Mediator-Arbiter in the petition or certification election, unless
exclusive jurisdiction including denial of application
the petitioning union is not listed in the Department’s roster of
for union registration or cancellation of union
registration of federations, national unions or
Page 89 of 117
legitimate labor organizations, or an existing collective bargaining Other related labor relations dispute refers to any conflict
agreement is not registered with the Department. between a labor union and the employer or any individual, entity or
group that is not a labor union or workers’ association. More
2. Exclusive Original Jurisdiction specifically, it may refer to any of the following:

Article 232. [226] Bureau of Labor Relations. The Bureau of a. Any conflict between:
Labor Relations and the Labor Relations Divisions in the regional
offices of the Department of Labor shall have original and exclusive (i) A labor union and an employer, or
authority to act, at their own initiative or upon request of either or (ii) A labor union and a group that is not a
both parties, on all inter-union and intra-union conflicts, and all
labor organization; or
disputes, grievances or problems arising from or affecting labor-
management relations in all workplaces, whether agricultural or non-
(iii) A labor union and an individual who is
agricultural, except those arising from the implementation or not a member of such union.
interpretation of collective bargaining agreements which shall be the
subject of grievance procedure and/or voluntary arbitration. b. Cancellation of registration of unions and workers’
The Bureau shall have fifteen (15) working days to act on associations filed by individuals other than their
labor cases before it, subject to extension by agreement of the parties. members, or by a group that is not a labor
organization; and
The BLR has exclusive original jurisdiction over: c. A petition for interpleader involving labor relations.

a. Intra/inter-union disputes of federations, national or


industry unions, trade union centers, its officers or The Mediator-Arbiter has the following administrative
member organizations. functions:
b. Petitions for cancellation of registration of
federations, national or industry unions, trade union a. Grant or deny a Petition for Certification Election
centers. b. Conduct preliminary conference and hearing to
determine: (i) the bargaining unit to be represented;
Labor Relations Division (LRD) refers to the following (ii) contending labor unions; (iii) possibility of a
units in the DOLE Regional Office: (1) Labor Organization and consent election; (iv) existence of any of the bars to
CBA Registration Unit; and (2) Med-Arbitration Unit. certification election under Section 3 of this Rule;
The BLR and the LRDs in the Regional Offices have and (v) such other matters as may be relevant for the
concurrent jurisdiction over the cases described in Article 232. This final disposition of the case.
concurrent character is stressed not only in the provision of this
article but in Article 219 of the Labor Code which defines the term NOTE: The Med-Arbiter can rule on all issues pertaining
“Bureau” as referring to both the BLR and/or the LRD in the to the existence of employer-employee relationship raised before
regional offices. the Med-Arbiter during the hearing and in the pleadings
For purposes of clarifying the issue of jurisdiction over
cases mentioned in Article 232, there is a need to cite first the 4. Appeals from Decisions of Med-Arbiters
following labor officials who exercise such jurisdiction, to wit: Decisions in cases falling under the original and
exclusive jurisdiction of the Med-Arbiters are appealable as
a. Mediator-Arbiters (Med-Arbiters); follows:
b. DOLE Regional Directors; and
c. BLR Director a. Inter-union disputes (representation or certification
election conflicts) – DOLE Secretary
3. Mediator-Arbiter
The Mediator-Arbiter refers to an officer in the Regional (i) Request for SEBA Certification when
Office or in the BLR authorized to hear and decide representation made in an unorganized establishment
cases, inter-union or intraunion disputes and other related labor with only one or more than 1 legitimate
relations disputes, except cancellation of union registration cases. union or in an organized establishment –
Thus, the cases falling under the original and exclusive DOLE Secretary
jurisdiction of the Mediator-Arbiter are as follows: (ii) Petition for certification election, consent
election, run-off election or re-run
a. Inter-union disputes (representation or certification election – DOLE Secretary
election conflicts), such as
b. Intra-union disputes – BLR Director
(i) Request for SEBA certification when made c. Other related labor relations disputes – BLR
in an unorganized establishment with only Director
one or more than one (1) legitimate union d. Injunction cases – BLR Director
or in an organized establishment, or e. Contempt Cases – BLR Director
(ii) Petition for certification election, consent
election, run-off election or re-run election; 5. BLR Director
The following cases fall under the original jurisdiction of
b. Intra-union disputes; the BLR Director:
c. Other related labor relations disputes;
d. Contempt cases. a. Complaints and petitions involving the application
for registration, revocation or cancellation of
registration of federations, national unions, industry

Page 90 of 117
unions, trade union centers and their local two or more conflicting parties for the purpose of reconciling their
chapters/chartered locals, affiliates and member differences or persuading them into adjusting or settling their
organizations; dispute. The Conciliator or Mediator normally does not make or
b. Request for examination of books of accounts of render any decision, his role being confined to the functions afore-
said labor organizations under Article 289; described.
c. Intra-union disputes involving said labor
organizations; Conciliator-Mediator
d. Notice of merger, consolidation, affiliation and A Conciliator-Mediator refers to an officer of the NCMB
change of name of said unions and or petition for whose principal function is to assist in the settlement and
denial thereof; disposition of labor-management disputes through conciliation and
e. Registration of multi-employer CBAs or petitions preventive mediation, including the promotion and encouragement
for deregistration thereof; of voluntary approaches to labor disputes prevention and settlement
f. Contempt cases. [D.O. No. 40-03].

6. Appeals from Decisions of BLR Director 2. Preventive Mediation


Decisions in the cases falling under the original and Preventive mediation, as a remedy, is not found in the
exclusive jurisdiction of the BLR Director are all appealable to the Labor Code. But under the law which created the NCMB, it is
DOLE Secretary, to wit: expressly stated that one of its functions is to provide preventive
mediation to disputing parties [E.O. No. 126]. It covers potential
a. Complaints and petitions involving the application labor disputes that are the subject of a formal or informal request
for registration, revocation, or cancellation of for conciliation and mediation assistance sought by either or both
registration of federations, national unions, industry parties or upon the initiative of the NCMB to avoid the occurrence
unions, trade union centers and their local of actual labor disputes and in order to remedy, contain or prevent
chapters/chartered locals, affiliates and member its degeneration into a full-blown dispute through amicable
organizations; settlement [NCMB Manual of Procedures].
b. Request for examination of books of accounts of Preventive mediation proceeding may be intiated in two
said labor organizations under Article 289 of the (2) ways:
Labor Code;
c. Intra-union disputes involving said labor a. By filing a notice or request of preventive
organizations; mediation, as distinguished from a notice of
d. Notice of merger, consolidation, affiliation and strike/lockout; or
change of name of said unions and or petition for b. By conversion of the notice of strike/lockout into a
denial thereof; preventive mediation case.
e. Registration of multi-employer CBAs or petitions
for deregistration thereof; and Procedurally, the filing of the notice of preventive
f. Contempt cases. suspension is the first step to submit a case for mediation. It is only
after this step that a submission agreement may be entered into by
The Secretary of Labor and Employment has no the parties concerned [Insular Hotel Employees Union v.
jurisdiction over decisions of the Bureau of Labor Relations Waterfront Insular Hotel Davao]. Moreover, just like in notices of
rendered in the exercise of its appellate power to review the strike or lockout, only certified SEBAs may file a notice or request
decision of the Regional Director in a petition to cancel the union’s for preventive mediation in cases of bargaining deadlocks and
certificate of registration, said decisions being final and unfair labor practices, the only two (2) grounds that may be
unappealable [Abbott v. Abbot Union]. invoked in support of a strike or lockout [Ibid].
As distinguished from a notice of strike/lockout, “notice
H. NATIONAL CONCILIATION AND MEDIATION of preventive mediation” refers to the notification filed by either an
BOARD (NCMB) employer or a duly registered labor union with the NCMB-DOLE
The NCMB is an agency attached to the DOLE informing the latter of its desire to submit the issues between them
principally in-charge of the settlement of labor disputes through for preventive mediation and conciliation. The issues that may be
conciliation, mediation and voluntary arbitration. It is charged with submitted for preventive mediation may either be strikeable or non-
the promotion of voluntary approaches to labor dispute prevention strikeable.
and settlement. It also administers the voluntary arbitration In case of strikeable issues, the parties may mutually
program; maintains/updates a list of voluntary arbitrators; compiles agree that the same be treated or converted into a preventive
arbitration awards and decisions; and provides counselling and mediation case, in which event, no strike or lockout may be legally
preventive mediation assistance particularly in the administration and validly mounted based on the same issues since their
of collective agreements. It is with the NCMB that Notices of conversion into a preventive mediation case has the effect of
Strike or Lockout are filed. dismissing the notice of strike/lockout and removing it from the
The NCMB is not a quasi-judicial agency. Its decisions, docket of notices of strike/lockout.
not having been rendered in the exercise of its quasi-judicial In case of non-strikeable issues raised in a notice of strike
functions, cannot be elevated to the Court of Appeals under Rule or notice of lockout, the NCMB may, motu proprio, convert the
43 of the Rules of Court [Tabigue v. International Copra Export same into a preventive mediation case or, alternatively, refer said
Corporation]. issues to voluntary arbitration, if they are in the nature of
unresolved grievances or to the Med-Arbiter, if they involve
1. Conciliation and Mediation representation or inter-union disputes.
Both terms refer to a process whereby a neutral third
person usually called Conciliator (in case of conciliation) or 3. Authority to Convert a Notice of Strike/Lockout
Mediator (in case of mediation), intervenes in a dispute involving Into a Preventive Mediation Case

Page 91 of 117
The NCMB has the authority to convert a notice of the product of the imagination or a matter of speculation, and that the
strike/lockout filed by the union/employer into a preventive pleading is filed in good faith. The court or tribunal may order the
mediation case under any of the following circumstances: correction of the pleading if verification is lacking or act on the pleading
although it is not verified, if the attending circumstances are such that strict
compliance with the rules may be dispensed with in order that the ends of
a. When the issues raised in the notice of
justice may thereby be served. 
strike/lockout are not strikeable in character.
b. When the party which filed the notice of
strike/lockout voluntarily asks for the conversion. Huntington v. NLRC
c. When both parties to a labor dispute mutually agree Held: The NLRC is a quasi-judicial agency, hence, initiatory
pleadings filed before it should be accompanied by a certificate of non-
to have it subjected to preventive mediation
forum-shopping.
proceeding. Nevertheless, in Loyola v. Court of Appeals, 17 we held that
substantial compliance with the requirement of certificate of non-forum
Such authority is in pursuance of the NCMB’s duty to shopping is sufficient. Here, we find that the certification of non-forum
exert all efforts at mediation and conciliation to enable the parties shopping was not filed simultaneously with the initiatory pleading. But we
to settle their dispute amicably and in line with the State policy of held that the filing of the certification within the reglementary period of
favoring voluntary modes of settling labor disputes [NCMB filing the initiatory pleading was substantial compliance. The fact that the
Circular requires strict compliance merely underscores its mandatory nature
Primer].
that it cannot be dispensed with or its requirements altogether disregarded,
but it does not thereby interdict substantial compliance with its provisions
I. PROCEDURE under justifiable circumstances.
Additionally, Supreme Court Administrative Circular No. 04-94,
1. Technical Rules now Section 5, Rule 7 of the Rules of Civil Procedure, must be construed
and applied to achieve its purpose. The Supreme Court promulgated the
Article 227. [221] Technical Rules not Binding and Prior Circular to promote and facilitate the orderly administration of justice. It
Resort to Amicable Settlement. In any proceeding before the should not be interpreted with such absolute literalness as to subvert its own
Commission or any of the Labor Arbiters, the rules of evidence ultimate and legitimate objective which is the goal of all rules of procedure
prevailing in courts of law or equity shall not be controlling and it is - that is, to achieve substantial justice as expeditiously as possible.
the spirit and intention of this Code that the Commission and its Meanwhile, in the case of Melo v. Court of Appeals,  we said that
members and the Labor Arbiters shall use every and all reasonable in those cases where we excused non-compliance with the requirements of
means to ascertain the facts in each case speedily and objectively, Supreme Court Administrative Circular No. 04-94, there were special
without regard to technicalities of law or procedure, all in the interest circumstances or compelling reasons that made the strict application of said
of due process. In any proceeding before the Commission or any Labor Circular clearly unjustified. The rule is crystal clear and plainly
Arbiter, the parties may be represented by legal counsel but it shall be unambiguous that the certification is a mandatory part of an initiatory
the duty of the Chairman, any Presiding Commissioner or pleading, i.e., the complaint, and its omission may be excused only upon
Commissioner or any Labor Arbiter to exercise complete control of the manifest equitable grounds proving substantial compliance therewith.
proceedings at all stages. In the present case, the respondents reasoned that they failed to
Any provision of law to the contrary notwithstanding, the comply with the Circular because the complaint form supplied by the Labor
Labor Arbiter shall exert all efforts towards the amicable settlement of Arbiter did not contain the required undertaking. They simply filled up the
a labor dispute within his jurisdiction on or before the first hearing. blanks therein. Hence, we agree with the Court of Appeals’ conclusion that
The same rule shall apply to the Commission in the exercise of its respondents should not be faulted for not having the certification of non-
original jurisdiction. forum shopping in their complaint.
The strict application of the Circular in the instant case, in our
view, would be contrary to the goals of the Rules of Civil Procedure – that
It is well settled that the application of technical rules of is, "just, speedy and inexpensive disposition of every action and
procedure may be relaxed to serve the demands of substantial proceeding."23 Technical rules of procedure in labor cases are not to be
justice, particularly in labor cases. Again, while as a rule, the Court strictly applied if the result would be detrimental to the working-man. Thus,
strictly adheres to the rules of procedure, it may take exception to the NLRC did not err in ordering that the corrections be made at the
such general rule when a strict implementation of the rules would Arbitration Branch, since the NLRC has also the power to order corrections
cause substantial injustice to the parties [Reyes v. Global Beer]. in case of irregularities in the proceedings before it.
Anent petitioners’ contention that the Labor Arbiter did not
acquire jurisdiction over the case for failure to include the certificate of
Millenium Erectors v. Magallanes non-forum shopping in the complaint, this contention finds no support in
Held: The NLRC did not err in treating respondent’s motion for law and in jurisprudence. Supreme Court Administrative Circular No. 04-
reconsideration as an appeal, the presence of some procedural flaws 94 is mandatory but not jurisdictional, as jurisdiction over the subject or
including the lack of verification and proof of service notwithstanding. nature of the cause of action is conferred by law
In labor cases, rules of procedure should not be applied in a
very rigid and technical sense.  They are merely tools designed to
facilitate the attainment of justice, and where their strict application would 2. Appearances and Attorney’s Fees
result in the frustration rather than promotion of substantial justice,
technicalities must be avoided. Technicalities should not be permitted to Article 228. [222] Appearances and Fees. 172 (a) Non-
stand in the way of equitably and completely resolving the rights and lawyers may appear before the Commission or any Labor Arbiter
obligations of the parties. Where the ends of substantial justice shall be only:
better served, the application of technical rules of procedure may be 1. If they represent themselves; or
relaxed. 2. If they represent their organization or members thereof.
Respecting the lack of verification, Pacquing v. Coca-Cola (2) No attorney’s fees, negotiation fees or similar charges of
Philippines, Inc. instructs: any kind arising from any collective bargaining agreement shall be
As to the defective verification in the appeal memorandum imposed on any individual member of the contracting union: Provided,
before the NLRC, the same liberality applies.  After all, the However, that attorney’s fees may be charged against union funds in
requirement regarding verification of a pleading is formal, not an amount to be agreed upon by the parties. Any contract, agreement,
jurisdictional. Such requirement is simply a condition affecting the form of or arrangement of any sort to the contrary shall be null and void.
pleading, the non-compliance of which does not necessarily render the
pleading fatally defective. Verification is simply intended to secure an  Article 228 of the Labor Code requiring an individual
assurance that the allegations in the pleading are true and correct and not
written authorization as a prerequisite to wage deductions seeks to
Page 92 of 117
protect the employee against unwarranted practices that would
diminish his compensation without his knowledge and 5. Illegal Recrutiment Cases
consent.  However, for all intents and purposes, the deductions The prescriptive period of simple illegal recruitment
required of the petitioner and the employees do not run counter to cases is five (5) years [RA 8042].
the express mandate of the law since the same are not unwarranted The prescriptive period of illegal recruitment cases
or without their knowledge and consent. Also, the deductions for involving economic sabotage is twenty (20) years [Ibid].
the union service fee in question are authorized by law and do not
require individual check-off authorizations [RCPI v. Secretary of 6. Actions Involving Union Funds
Labor]. Actions involving union funds must be brought within 3
years from the date of submission of audited financial statements or
J. PRESCRIPTION OF ACTIONS from the date they should have been submitted as required by law,
Filing a case with the grievance machinery tolls the whichever comes first.
running of the prescriptive period.
VIII
1. Money Claims TERMINATION OF EMPLOYMENT BY
The prescriptive period of money claims (like separation EMPLOYER
pay) and benefits arising from employer-employee relationship is
three (3) years under Article 306 of the Labor Code, reckoned from A. SECURITY OF TENURE
the time the cause of action accrued; otherwise, they shall be
forever barred [IRR]. Article 294. Security of Tenure. In cases of regular
Money claims under Article 306 include those arising employment, the employer shall not terminate the services of an
from: employee except for a just cause or when authorized by this Title. An
employee who is unjustly dismissed from work shall be entitled to
a. Law reinstatement without loss of seniority rights and other privileges and
b. CBA to his full backwages, inclusive of allowances, and to his other benefits
c. Incremental proceeds from tuition increases or their monetary equivalent computed from the time his compensation
was withheld from him up to the time of his actual reinstatement.
d. Overseas employment of OFWs.

A cause of action for money claims accrues upon the Termination Law is the implementation of security
categorical denial of a claim. The three-year prescriptive period is tenure which is a policy translated by the 1987 Constitution in
not applicable to execution of final judgment which should be done Article 13 in Section 3 which states:
within 5 years.
The State shall afford full protection to labor, local
and overseas, organized and unorganized, and promote full
2. Illegal Dismissal employment and equality of employment opportunities for all.
The 3-year prescriptive period in Article 291 solely It shall guarantee the rights of all workers to self-
applies to money claims but not to illegal dismissal cases which are organization, collective bargaining and negotiations, and
not in the nature of money claims. The prescriptive period of illegal peaceful concerted activities, including the right to strike in
dismissal cases is 4 years under Article 1146 of the Civil Code. accordance with law. They shall be entitled to security of tenure,
humane conditions of work, and a living wage. They shall also
Article 291 covers claims for overtime pay, holiday pay, service
participate in policy and decision-making processes affecting
incentive leave pay, bonuses, salary differentials, and illegal their rights and benefits as may be provided by law.
deductions by an employer.  It also covers money claims arising
from seafarer contracts [Arriola v. Pilipino Star Ngayon]. B. JUST CAUSES
Settled is the rule that when one is arbitrarily and
unjustly deprived of his job or means of livelihood, the action Article 297.Termination by Employer. An employer may
instituted to contest the legality of one’s dismissal from terminate an employment for any of the following causes:
employment constitutes, in essence, an action predicated upon an (a) Serious misconduct or willful disobedience by the
injury to the rights of the plaintiff, as contemplated under Article employee of the lawful orders of his employer or representative in
1146 of the New Civil Code, which must be brought within four connection with his work;
years [Montero v. Times Transportation]. (b) Gross and habitual neglect by the employee of his duties;
(c) Fraud or willful breach by the employee of the trust
reposed in him by his employer or duly authorized representative;
3. Unfair Labor Practice (ULP)
(d) Commission of a crime or offense by the employee
The prescriptive period for criminal complaints involving against the person of his employer or any immediate member of his
ULPs is one (1) year from the time the acts complained of were family or his duly authorized representatives; and
committed; otherwise, they shall be forever barred [Article 305]. (e) Other causes analogous to the foregoing.
However, before a criminal case for ULP can be filed, it
is a pre-requisite that a labor case for ULP involving the same set Dismissal is cessation of an employment due to a just
of facts should first be initiated with the competent labor court. It is cause, attributable to the employee. Termination is cessation of an
only upon a finding of guilt in the labor case and after the decision employment due to an authorized cause, which is not the fault of
herein has become final and executory that the one-year the employee. Dismissal does not merit separation pay;
prescriptive period to prosecute the criminal aspect of ULP starts to termination carries with it separation pay.
run. Tenure is presumed. The burden of proving that the
termination was for a valid or authorized cause shall rest on the
4. Offenses Under The Labor Code employer.
The prescriptive period of all criminal offenses penalized The just causes in the Labor Code are found in the
under the Labor Code and its IRR is three (3) years from the time following provisions thereof:
of commission thereof [Article 305].

Page 93 of 117
(1) Article 297 [282] - (Termination by the guidelines set by the Department of Labor and Employment.
Employer) which provides for the following Any decision taken by the employer shall be without prejudice
grounds: to the right of the worker to contest the validity or legality of his
dismissal by filing a complaint with the regional branch of the
National Labor Relations Commission. The burden of proving
(a) Serious misconduct or willful disobedience that the termination was for a valid or authorized cause shall rest
by the employee of the lawful orders of his on the employer. The Secretary of the Department of Labor and
employer or representative in connection Employment may suspend the effects of the termination pending
with his work; resolution of the dispute in the event of a prima facie finding by
(b) Gross and habitual neglect by the employee the appropriate official of the Department of Labor and
of his duties; Employment before whom such dispute is pending that the
(c) Fraud or willful breach by the employee of termination may cause a serious labor dispute or is in
implementation of a mass lay-off.
the trust reposed in him by his employer or
duly authorized representative;
1. Serious Misconduct
(d) Commission of a crime or offense by the
For serious misconduct or improper behavior to be a just
employee against the person of his employer
cause for dismissal, the following requisites must concur:
or any immediate member of his family or
his duly authorized representatives; and
(a) It must be serious;
(e) Other causes analogous to the foregoing.
(b) It must relate to the performance of the employee’s
duties;
(2) Article 279(a) [264(a)] - (Prohibited
(c) It must show that he has become unfit to continue
Activities) which provides for the termination of
working for the employer; and
the following:
(d) It must have been performed with wrongful intent.
(a) Union officers who knowingly participate in
The 2014 case of Imasen v. Alcon added the 4th requisite
an illegal strike and therefore deemed to
above which, it said, is “equally important and required.” All the
have lost their employment status.
above requisites must concur [Coca-Cola Bottlers v. KMMC-
(b) Any employee, union officer or ordinary
FFW].
member who knowingly participates in the
commission of illegal acts during a strike
(irrespective of whether the strike is legal or Colegio de San Juan de Letran v. Meris
Held: The fact that eight students were made beneficiaries of
illegal), is also deemed to have lost his
such increase does not justify the irregular alteration since the rule is, the
employment status. rating of the pupil should be based on his scholastic record, even if the
same is non-tested or qualitative in nature, as in the case at bar.
(3) Article 278(g) [263(g)] - (National Interest Respondent’s prerogative to give her students the grade that they deserve is
Cases) where strikers who violate orders, not incoherent with having a fair and reasonable basis therefor.
prohibitions and/or injunctions as are issued by the To our mind, the acts of the respondent in altering the grades in
DOLE Secretary or the NLRC, may be imposed the Clean Records even after the same were already reviewed and approved
immediate disciplinary action, including dismissal by the subject coordinators; of effecting the alterations and erasures without
placing her initials thereon; of not informing the subject coordinators of
or loss of employment status.
such alterations and erasures; of allowing the discrepancies to last without
(4) Article 259(e) [248(e)] - (Union Security any effort to reconcile the same to avoid any doubts on the grading system
Clause) where violation of the union security of petitioner; of refusing to accept the memo informing her of the aforesaid
agreement in the CBA may result in termination of tampering and snubbing any explanation relevant thereto, are all acts of
employment. Under this clause, the bargaining transgression of school rules, regulations and policies. Truly, then,
union can demand from the employer the respondent had committed a misconduct, serious enough to warrant her
dismissal of an employee who commits a breach dismissal from employment under paragraph (a) of Article 282 of the Labor
of union security arrangement, such as failure to Code, as well as Section 94(b), Article XVII of the Manual of Regulations
for Private Schools, which provides that the employment of a teacher may
join the union or to maintain his membership in
be terminated for negligence in keeping school or student records, or
good standing therein. The same union can also tampering with or falsification of the same.
demand the dismissal of a member who commits
an act of disloyalty against it, such as when the
Settled Principles
member organizes a rival union.
 Serious misconduct implies that it must be of such grave
The Secretary of Labor has extraordinary, express
and aggravated character and not merely trivial or
power to intervene in a labor dispute to enjoin a potential
unimportant.
termination
 Simple or minor misconduct would not justify the
Article 292 [b] of the Labor Code provides:
termination of the services of an employee.
Subject to the constitutional right of workers to  Possession or use of shabu or other drugs is a valid ground
security of tenure and their right to be protected against to terminate employment.
dismissal except for a just and authorized cause and without  Immorality, as a general rule, is not a just ground to
prejudice to the requirement of notice under Article 283 of this terminate employment. The exception is when such
Code, employer shall furnish the worker whose employment is immoral conduct is prejudicial or detrimental to the
sought to be terminated a written notice containing a statement interest of the employer
of the causes for termination and shall afford the latter ample
opportunity to be heard and to defend himself with the
 Immoral act committed beyond office hours is a valid
assistance of his representative if he so desires in accordance ground to terminate employment.
with company rules and regulations promulgated pursuant to

Page 94 of 117
 Sexual intercourse inside company premises constitutes (b) Sufficiently known to the employee; and
serious misconduct. (c) In connection with the duties for which the
 The act of a 30-year old lady teacher in falling in love employee has been engaged to discharge.
with a 16-year old student is not immoral.
 Fighting is a ground for termination but only the An order which is not based on a rule, regulation or
instigator or aggressor and not the victim who was policy which does not satisfy the foregoing three (3) requisites is
constrained to defend himself should be dismissed. not lawful and thus may not be invoked as basis for terminating an
 Challenging superiors to a fight is a just cause for employee on the ground of insubordination. Needlessly, absent any
termination. of the foregoing elements would make the refusal of the employee
 Assaulting another employee is a just cause for to comply with the rule, regulation, or policy justified and not
termination. constitutive of “willful disobedience” as would warrant the
 Utterance of obscene, insulting or offensive words imposition of the penalty of dismissal for such refusal.
constitutes serious misconduct.
Settled Principles
 Gambling within company premises is a serious
misconduct.
 Making false allegations in complaint does not constitute
 Rendering service to business rival is a just cause to
insubordination.
terminate employment.
 Failure to answer memo to explain constitutes willful
 Selling products of a competitor is a just cause for
disobedience.
termination.
 Another notice is required in case of termination on the ground
 Organizing a credit union by employees in a bank is a of failure to answer memo to explain.
serious misconduct.  Refusal to undergo random drug testing constitutes both
 Deceiving a customer for personal gain is a just cause for serious misconduct and insubordination.
termination.  Refusal to render overtime to meet production deadline
 Contracting work in competition with employer constitutes insubordination.
constitutes serious misconduct.  Refusal to comply with a lawful transfer constitutes
 Intoxication which interferes with the employee’s work insubordination.
constitutes serious misconduct.
 The act of a teacher in pressuring a colleague to change 3. Gross and Habitual Neglect of Duties
the failing grade of a student is serious misconduct. The following are the requisites:
 Sexual harassment is a just ground to dismiss.
 Sleeping while on duty is a ground for termination. (a) There must be negligence which is gross and/or
 Dismissal is too harsh a penalty for eating while at work. habitual in character; and
 Pilferage or theft of company-owned property is a just (b) It must be work-related as would make him unfit to
cause to terminate. work for his employer.
 Theft of funds or property not owned by employer is not a
ground to terminate. Settled Principles
 Act of falsification is a valid ground to terminate
employment.  Simple negligence is not sufficient to terminate employment.
 Punching-in of time cards of other employees is a just  The negligence must be gross in character which means absence
cause for termination. of that diligence that an ordinarily prudent man would use in his
own affairs.
2. Insubordination or Willful Disobedience of  As a general rule, negligence must be both gross and habitual
Lawful Orders to be a valid ground to dismiss.
One of the fundamental duties of an employer is to obey  Habituality may be disregarded if negligence is gross or the
all reasonable rules, orders and instructions of the employer. In damage or loss is substantial. “Habitual negligence” implies
order to validly invoke this ground, the following requisites must repeated failure to perform one’s duties for a period of time,
be complied with, to wit: depending upon the circumstances.
 Actual damage, loss or injury is not an essential requisite.
(a) The employee’s assailed conduct must have been  Gross negligence may result to loss of trust and confidence.
willful or intentional, the willfulness being  Absences, if authorized, cannot be cited as a ground to terminate
characterized by a wrongful and perverse attitude; employment.
and  Tardiness or absenteeism, if not habitual, cannot be cited as a
(b) The order violated must be based on a reasonable ground to terminate employment.
and lawful company rule, regulation or policy and  Tardiness or absenteeism, if habitual, may be cited as a ground to
made known to the employee and must pertain to terminate employment.
the duties for which he has been engaged to  Tardiness or absenteeism, if habitual, may be tantamount to
discharge. serious misconduct.
 Absences or tardiness due to emergency, ailment or fortuitous
Requisites for validity of company rules and event are justified and may not be cited as just cause to terminate
regulations employment.
As far as the second requisite for insubordination or  Unsatisfactory or poor performance, inefficiency and
willful disobedience is concerned, it is required that there should incompetence are considered just causes for dismissal only if
exist a rule, regulation or policy upon which the order is based they amount to gross and habitual neglect of duties.
which must be:
(a) Lawful and reasonable; 4. Abandonment of Work

Page 95 of 117
Abandonment is a form of neglect of duty, hence, a just prescriptive period within which to institute his action for illegal
cause for termination of employment under Article 297 of the dismissal.
Labor Code. To constitute abandonment, two (2) elements must  Filing of a case to pre-empt investigation of the administrative
concur, namely: case is tantamount to abandonment.
(a) The employee must have failed to report for work or  When what is prayed for in the complaint is separation pay
must have been absent without valid or justifiable and not reinstatement, the filing of complaint does not negate
reason; and abandonment.
(b) There must have been a clear intention on the part of  It is abandonment when what is prayed for in the complaint is
the employee to sever the employer-employee separation pay and it was only in the position paper that
relationship manifested by some overt act. reinstatement was prayed for.
 Employment in another firm coinciding with the filing of
No hearing is required to validly dismiss an employee for complaint does not indicate abandonment.
abandonment [Intertranz Container v. Bautista].  Offer of reinstatement by employer during proceedings before
Labor Arbiter and refusal by employee does not indicate
a. Due Process abandonment but more of a symptom of strained relations
Due process in abandonment cases consists only of the between the parties.
service of two (2) notices to the employee, viz:  An employee may be absolved from the charge of abandonment
of work but adjudged guilty of AWOL. These two grounds are
i. First notice directing the employee to explain why separate and distinct from each other.
he should not be declared as having abandoned his  An employee who failed to report for work after the expiration of
job; and the duly approved leave of absence is considered to have
ii. Second notice to inform him of the employer’s abandoned his job.
decision to dismiss him on the ground of  An employee who failed to comply with the order for his
abandonment. reinstatement is deemed to have abandoned his work.
 An employee who, after being transferred to a new assignment,
Batangas Laguna Tayabas Bus Company v. NLRC did not report for work anymore is deemed to have abandoned
Held: The contention of the petitioner that the private his job.
respondents abandoned their position is also not acceptable. An employee  An employee who deliberately absented from work without leave
who forthwith takes steps to protest his lay-off cannot by any logic be said or permission from his employer for the purpose of looking for a
to have abandoned his work. job elsewhere is deemed to have abandoned his work.
For abandonment to constitute a valid cause for termination of
 Imprisonment or detention by military does not constitute
employment, there must be a deliberate, unjustified refusal of the employee
to resume his employment.[4] This refusal must be clearly established. As
abandonment.
we stressed in a recent case, [5] mere absence is not sufficient; it must be  Absence to evade arrest is not a valid justification. To do so
accompanied by overt acts unerringly pointing to the fact that the employee would be to place an imprimatur on the employee’s attempt to
simply does not want to work anymore. In the case at bar, the affidavit of derail the normal course of the administration of justice.
Eduardo Azucena, BLTBCo operations manager,[6] besides being hearsay,
lacks credibility in light of the subsequent acts of the private respondents in 5. Fraud
complaining about their separation. Fraud is separate and distinct from the other ground
A worker who joins a strike does so precisely to assert or
provided in the same paragraph, that is, loss of trust and confidence
improve the terms and conditions of his employment. If his purpose is to
abandon his work, he would not go to the trouble of joining a strike.
[Sanden Aircon v. Rosales].
The following are the requisites of this ground:
b. Settled Principles
(a) The employee has committed fraud, an intentional
deception and used dishonest methods for personal
 Mere absence is not enough to constitute abandonment.
gain or to damage he employer; and
 Clear intention to sever employment relationship is necessary.
(b) The fraud is work-related and rendered him unfit for
 Due process in abandonment cases consists only of the service
work for his employer.
of 2 notices to the employee, viz.:
Falsification constitutes not only serious misconduct but
i. First notice directing the employee to explain why he
fraud under the Labor Code [San Miguel Corp v. NLRC].
should not be declared as having abandoned his job; and
ii.Second notice to inform him of the employer’s decision
Settled Principles
to dismiss him on the ground of abandonment.
 Failure to deposit collection constitutes fraud.
 No hearing is required to validly dismiss an employee for
 Lack of damage or losses is not necessary in fraud cases. The fact
abandonment.
that the employer did not suffer losses from the dishonesty of the
 Notices in abandonment cases must be sent to employee’s last
dismissed employee because of its timely discovery does not
known address per record of the company. The employer need
excuse the latter from any culpability.
not look for the employee’s current whereabouts.
 Lack of misappropriation or shortage is immaterial in case of
 Immediate filing of a complaint for illegal dismissal praying for
unauthorized encashment of personal checks by teller and
reinstatement negates abandonment.
cashier.
 Lapse of time between dismissal and filing of a case is not a
 Restitution does not have absolutory effect.
material indication of abandonment. Hence, lapse of 2 years and
5 months or 20 months or 9 months or 8 months before filing the
6. Willful Breach of Trust and Confidence
complaint for illegal dismissal is not an indication of
For the doctrine of loss of trust and confidence to apply,
abandonment. Under the law, the employee has a 4-year
the following requisites must concur:

Page 96 of 117
 There must be “some basis” for the loss of trust and
(a) The employee holds a position of trust and confidence which means that there is reasonable ground to
confidence; believe, if not to entertain the moral conviction, that the
(b) There exists an act justifying the loss of trust and concerned employee is responsible for the misconduct and
confidence, which means that the act that betrays that the nature of his participation therein rendered him
the employer’s trust must be real, i.e., founded on absolutely unworthy of trust and confidence demanded by his
clearly established facts; position.
(c) The employee’s breach of the trust must be willful,  Dismissal due to feng shui mismatch is not a valid ground to
i.e., it was done intentionally, knowingly and lose trust and confidence.
purposely, without justifiable excuse; and  Command responsibility of managerial employees is a ground
(d) The act must be in relation to his work which would to dismiss.
render him unfit to perform it.  Confidential employee may be dismissed for loss of trust and
confidence.
a. Guidelines  Grant of promotions and bonuses negates loss of trust and
As a safeguard against employers who indiscriminately confidence.
use “loss of trust and confidence” to justify arbitrary dismissal of  Long years of service, absence of derogatory record and small
employees, the Court, in addition to the above elements, imposes amount involved are deemed inconsequential insofar as loss of
the following guidelines for the doctrine to apply: trust and confidence is concerned.
(a) The loss of confidence must not be simulated;
 Dropping of criminal charges or acquittal in a criminal case
(b) It should not be used as a subterfuge for causes
arising from the same act does not affect the validity of
which are illegal, improper or unjustified;
dismissal based on loss of trust and confidence.
(c) It may not be arbitrarily asserted in the face of
overwhelming evidence to the contrary; and
7. Commission of Crime or Offense
(d) It must be genuine, not a mere afterthought, to
The following are the requisites for the valid invocation
justify earlier action taken in bad faith.
of this ground:
b. Settled Principles
(a) A crime or offense was committed by the employee;
(b) It was committed against any of the following
 Employee’s position must be reposed with trust and persons:
confidence. (i) His employer;
 “Position of trust and confidence” is one where a person is (ii) Any immediate member of his employer’s
entrusted with confidence on delicate matters, or with the family; or
custody, handling, or care and protection of the employer’s (iii) His employer’s duly authorized
property. representative.
 Two (2) classes of positions of trust. The first class consists of
managerial employees or those who, by the nature of their Because of its gravity, work-relation is not necessary.
position, are entrusted with confidential and delicate matters Neither is it necessary to show that the commission of the criminal
and from whom greater fidelity to duty is correspondingly act would render the employee unfit to perform his work for the
expected. They refer to those vested with the powers or employer.
prerogatives to lay down and execute management policies The conviction of an employee in a criminal case is not
and/or to hire, transfer suspend, lay-off, recall, discharge, indispensable to warrant his dismissal by his employer. If there is
assign or discipline employees or to effectively recommend sufficient evidence to show that the employee has been guilty of a
such managerial actions. Their primary duty consists of the breach of trust, or that his employer has ample reason to distrust
management of the establishment in which they are employed him, it cannot justly deny to the employer the authority to dismiss
or of a department or a subdivision thereof. such employee. All that is incumbent upon the Court of Industrial
 The second class consists of fiduciary rank-and-file Relations (now National Labor Relations Commission) to
employees who, though rank-and-file, are routinely charged determine is whether the proposed dismissal is for just cause xxx. It
with the custody, handling or care and protection of the is not necessary for said court to find that an employee has been
employer's money or property, or entrusted with confidence guilty of a crime beyond reasonable doubt in order to authorize his
on delicate matters, and are thus classified as occupying dismissal [Concepcion v. Minex Import Corporation].
positions of trust and confidence. Included under this class are In Philippine Long Distance Telephone Co.(BLTB Co.)
“cashiers, auditors, property custodians, or those who, in the vs. NLRC, the Court held that the acquittal of the employee from
normal and routine exercise of their functions, regularly the criminal prosecution for a crime committed against the interest
handle significant amounts of [the employer’s] money or of the employer did not automatically eliminate loss of confidence
property.” as a basis for administrative action against the employee; and that
 Rules on termination of managerial and supervisory in cases where the acts of misconduct amounted to a crime, a
employees different from those applicable to rank- and-file dismissal might still be properly ordered notwithstanding that the
employees. Thus, with respect to rank-and-file personnel, loss employee was not criminally prosecuted or was acquitted after a
of trust and confidence as a ground for valid dismissal criminal prosecution.
requires proof of involvement in the alleged events in question
and that mere uncorroborated assertions and accusations by 8. Other Causes
the employer will not be sufficient. But as regards a
managerial employee, the mere existence of a basis for a. Union Officers who knowingly participate in
believing that he has breached the trust of his employer would an Illegal Strike
suffice for his dismissal. Article 279(a) provides:

Page 97 of 117
No labor organization or employer shall declare a In terminating the employment of an employee by enforcing the
strike or lockout without first having bargained collectively in union security clause, the employer needs only to determine and prove that:
accordance with Title VII of this Book or without first having (1) the union security clause is applicable; (2) the union is requesting for
filed the notice required in the preceding Article or without the the enforcement of the union security provision in the CBA; and (3) there is
necessary strike or lockout vote first having been obtained and sufficient evidence to support the decision of the union to expel the
reported to the Ministry. employee from the union. These requisites constitute just cause for
No strike or lockout shall be declared after terminating an employee based on the union security provision of the CBA.
assumption of jurisdiction by the President or the Minister or [26]

after certification or submission of the dispute to compulsory or There is no question that in the present case, the CBA between
voluntary arbitration or during the pendency of cases involving GMC and IBM-Local 31 included a maintenance of membership and closed
the same grounds for the strike or lockout. shop clause as can be gleaned from Sections 3 and 6 of Article II. IBM-
Any worker whose employment has been terminated Local 31, by written request, can ask GMC to terminate the employment of
as a consequence of any unlawful lockout shall be entitled to the employee/worker who failed to maintain its good standing as a union
reinstatement with full backwages. Any union officer who member.
knowingly participates in an illegal strike and any worker or It is similarly undisputed that IBM-Local 31, through Gabiana,
union officer who knowingly participates in the commission of the IBM Regional Director for Visayas and Mindanao, twice requested
illegal acts during a strike may be declared to have lost his GMC, in the letters dated March 10 and 19, 1992, to terminate the
employment status: Provided, That mere participation of a employment of Casio, et al. as a necessary consequence of their expulsion
worker in a lawful strike shall not constitute sufficient ground from the union.
for termination of his employment, even if a replacement had It is the third requisite - that there is sufficient evidence to
been hired by the employer during such lawful strike. support the decision of IBM-Local 31 to expel Casio, et al. - which appears
to be lacking in this case.
Jackbilt Industries v. Jackbilt Employees Workers Union The provisions of the CBA are clear enough. The termination
Held: The principle of conclusiveness of judgment, embodied in of employment on the basis of the closed shop provision of the CBA is well
Section 47(c), Rule 39 of the Rules of Court,[24]  holds that the parties to a recognized in law and in jurisprudence.
case are bound by the findings in a previous judgment with respect to There is no valid ground to refuse to terminate. On the other
matters actually raised and adjudged therein.[25] hand as pointed out in the union's strongly demanding letter dated March
Article 264(e) of the Labor Code prohibits any person engaged 19, 1992, the company could be sued for unfair labor practice. While
in picketing from obstructing the free ingress to and egress from the we would have wanted not to accommodate the union's request, we are
employer's premises. Since respondent was found in the July 17, 1998 left with no other option. The terms of the CBA should be respected. To
decision of the NLRC to have prevented the free entry into and exit of refuse to enforce the CBA would result in the breakdown of industrial
vehicles from petitioner's compound, respondent's officers and employees peace and the end of harmonious relations between the union and
clearly committed illegal acts in the course of the March 9, 1998 strike management. The company would face the collective anger and enmity of
The use of unlawful means in the course of a strike renders such its employees who are union members.
strike illegal.[26] Therefore, pursuant to the principle of conclusiveness of In the light of the union's very insistent demand, verbal and in
judgment, the March 9, 1998 strike was ipso facto illegal. The filing of a writing and to avoid the union accusation of "coddling" you, and
petition to declare the strike illegal was thus unnecessary. considering the explicitly mandatory language of the closed shop provision
Consequently, we uphold the legality of the dismissal of of the CBA, the company is constrained to terminate your employment, to
respondent's officers and employees. Article 264 of the Labor give you ample time to look and find another employment, and/or exert
Code[27] further provides that an employer may terminate employees found efforts to become again a member of good standing of your union, effective
to have committed illegal acts in the course of a strike. [28] Petitioner clearly April 24, 1992.
had the legal right to terminate respondent's officers and employees In the meantime, to prevent serious danger to the life and
property of the company and of its employees, we are placing you under
preventive suspension beginning today.
b.
Employees who knowingly violate the union
It is apparent from the aforequoted letter that GMC terminated
security clause stipulated in the CBA the employment of Casio, et al. relying upon the Resolution dated February
Another cause for termination is dismissal from 29, 1992 of Pino, et al. expelling Casio, et al. from IBM-Local 31;
employment due to the enforcement of the union security clause in Gabiana's Letters dated March 10 and 19, 1992 demanding that GMC
the CBA [Alabang Country Club v. NLRC]. terminate the employment of Casio, et al. on the basis of the closed shop
clause in the CBA; and the threat of being sued by IBM-Local 31 for unfair
labor practice. The letter made no mention at all of the evidence supporting
General Milling Corp. v. Casio
the decision of IBM-Local 31 to expel Casio, et al. from the union. GMC
Held: Union security clauses are recognized and explicitly
never alleged nor attempted to prove that the company actually looked into
allowed under Article 248(e) of the Labor Code, which provides that:
the evidence of IBM-Local 31 for expelling Casio, et al. and made a
Art. 248. Unfair Labor Practices of Employers. x x x
determination on the sufficiency thereof. Without such a determination,
(e) To discriminate in regard to wages, hours of work, and other
GMC cannot claim that it had terminated the employment of Casio, et al.
terms and conditions of employment in order to encourage or discourage
for just cause.
membership in any labor organization. Nothing in this Code or in any
The failure of GMC to make a determination of the sufficiency
other law shall stop the parties from requiring membership in a
of evidence supporting the decision of IBM-Local 31 to expel Casio, et al.
recognized collective bargaining agent as a condition for employment,
is a direct consequence of the non-observance by GMC of procedural due
except those employees who are already members of another union at
process in the dismissal of employees.
the time of the signing of the collective bargaining
As a defense, GMC contends that is an employer, its only duty
agreement.  (Emphasis supplied.)
was to ascertain that IBM-Local 31 accorded Casio, et al. due process; and,
It is State policy to promote unionism to enable workers to
it is the finding of the company that IBM-Local 31 did give Casio, et al. the
negotiate with management on an even playing field and with more
opportunity to answer the charges against them, but they refused to avail
persuasiveness than if they were to individually and separately bargain with
themselves of such opportunity.
the employer. For this reason, the law has allowed stipulations for "union
This argument is without basis.
shop" and "closed shop" as means of encouraging workers to join and
The Court has stressed time and again that allegations must be
support the union of their choice in the protection of their rights and
proven by sufficient evidence because mere allegation is definitely not
interest vis-à-vis the employer.[24]
evidence.[28] Once more, in Great Southern Maritime Services Corporation.
Moreover, a stipulation in the CBA authorizing the dismissal of
v. Acuña,[29] the Court declared:
employees are of equal import as the statutory provisions on dismissal
Time and again we have ruled that in illegal dismissal cases like
under the Labor Code, since "a CBA is the law between the company and
the present one, the onus of proving that the employee was not dismissed or
the union and compliance therewith is mandated by the express policy to
if dismissed, that the dismissal was not illegal, rests on the employer and
give protection to labor."[25]

Page 98 of 117
failure to discharge the same would mean that the dismissal is not justified Department of Labor and Employment within the sixty-day period before
and therefore illegal. Thus, petitioners must not only rely on the the expiration of a collective bargaining agreement, the Med-Arbiter shall
weakness of respondents' evidence but must stand on the merits of automatically order an election by secret ballot when the verified petition is
their own defense. A party alleging a critical fact must support his supported by the written consent of at least twenty-five percent (25%) of all
allegation with substantial evidence for any decision based on the employees in the bargaining unit to ascertain the will of the employees
unsubstantiated allegation cannot stand as it will offend due process. in the appropriate bargaining unit. To have a valid election, at least a
majority of all eligible voters in the unit must have cast their votes. The
PICOP Resources v. Taneca labor union receiving the majority of the valid votes cast shall be certified
as the exclusive bargaining agent of all the workers in the unit. When an
Held: However, in terminating the employment of an employee
election which provides for three or more choices results in no choice
by enforcing the union security clause, the employer needs to determine
receiving a majority of the valid votes cast, a run-off election shall be
and prove that: (1) the union security clause is applicable; (2) the union is
conducted between the labor unions receiving the two highest number of
requesting for the enforcement of the union security provision in the CBA;
votes: Provided, That the total number of votes for all contending unions is
and (3) there is sufficient evidence to support the decision of the union to
at least fifty per cent (50%) of the number of votes cast.
expel the employee from the union. These requisites constitute just cause
At the expiration of the freedom period, the employer shall
for terminating an employee based on the union security provision of the
continue to recognize the majority status of the incumbent bargaining
CBA.[
agent where no petition for certification election is filed.[19]
As to the first requisite, there is no question that the CBA
Applying the same provision, it can be said that while it is
between PRI and respondents included a union security clause, specifically,
incumbent for the employer to continue to recognize the majority status of
a maintenance of membership as stipulated in Sections 6 of Article II,
the incumbent bargaining agent even after the expiration of the freedom
Union Security and Check-Off.  Following the same provision, PRI, upon
period, they could only do so when no petition for certification election was
written request from the Union, can indeed terminate the employment of
filed. The reason is, with a pending petition for certification, any such
the employee who failed to maintain its good standing as a union member.
agreement entered into by management with a labor organization is fraught
Secondly, it is likewise undisputed that NAMAPRI-SPFL, in
with the risk that such a labor union may not be chosen thereafter as the
two (2) occasions demanded from PRI, in their letters dated May 16 and 23,
collective bargaining representative.[20] The provision for status quo  is
2000, to terminate the employment of respondents due to their acts of
conditioned on the fact that no certification election was filed during the
disloyalty to the Union.
freedom period.  Any other view would render nugatory the clear statutory
However, as to the third requisite, we find that there is no
policy to favor certification election as the means of ascertaining the true
sufficient evidence to support the decision of PRI to terminate the
expression of the will of the workers as to which labor organization would
employment of the respondents.
represent them.[21]
PRI alleged that respondents were terminated from employment
In the instant case, four (4) petitions were filed as early as May
based on the alleged acts of disloyalty they committed when they signed an
12, 2000. In fact, a petition for certification election was already ordered by
authorization for the Federation of Free Workers (FFW) to file a Petition
the Med-Arbiter of DOLE Caraga Region on August 23, 2000. [22] 
for Certification Election among all rank-and-file employees of PRI.  It
Therefore, following Article 256, at the expiration of the freedom period,
contends that the acts of respondents are a violation of the Union Security
PRI's obligation to recognize NAMAPRI-SPFL as the incumbent
Clause, as provided in their Collective Bargaining Agreement.
bargaining agent does not hold true when petitions for certification election
We are unconvinced.
were filed, as in this case.
We are in consonance with the Court of Appeals when it held
Moreover, the last sentence of Article 253 which provides for
that the mere signing of the authorization in support of the Petition for
automatic renewal pertains only to the economic provisions of the CBA,
Certification Election of FFW on March 19, 20 and 21, or before the
and does not include representational aspect of the CBA. An existing CBA
"freedom period," is not sufficient ground to terminate the employment of
cannot constitute a bar to a filing of a petition for certification election.
respondents inasmuch as the petition itself was actually filed during the
When there is a representational issue, the status quo provision in so far as
freedom period.  Nothing in the records would show that respondents failed
the need to await the creation of a new agreement will not apply.
to maintain their membership in good standing in the Union. Respondents
Otherwise, it will create an absurd situation where the union members will
did not resign or withdraw their membership from the Union to which they
be forced to maintain membership by virtue of the union security clause
belong. Respondents continued to pay their union dues and never joined the
existing under the CBA and, thereafter, support another union when filing a
FFW.
petition for certification election. If we apply it, there will always be an
Significantly, petitioner's act of dismissing respondents stemmed
issue of disloyalty whenever the employees exercise their right to self-
from the latter's act of signing an authorization letter to file a petition for
organization. The holding of a certification election is a statutory policy
certification election as they signed it outside the freedom period. However,
that should not be circumvented,[23] or compromised.
we are constrained to believe that an "authorization letter to file a petition
Time and again, we have ruled that we adhere to the policy of
for certification election" is different from an actual "Petition for
enhancing the welfare of the workers. Their freedom to choose who should
Certification Election."  Likewise, as per records, it was clear that the actual
be their bargaining representative is of paramount importance. The fact that
Petition for Certification Election of FFW was filed only on May 18, 2000.
there already exists a bargaining representative in the unit concerned is of
[17]
 Thus, it was within the ambit of the freedom period which commenced
no moment as long as the petition for certification election was filed within
from March 21, 2000 until May 21, 2000. Strictly speaking, what is
the freedom period. What is imperative is that by such a petition for
prohibited is the filing of a petition for certification election outside the 60-
certification election the employees are given the opportunity to make
day freedom period.[18] This is not the situation in this case. If at all, the
known of who shall have the right to represent them thereafter. Not only
signing of the authorization to file a certification election was merely
some, but all of them should have the right to do so. What is equally
preparatory to the filing of the petition for certification election, or an
important is that everyone be given a democratic space in the bargaining
exercise of respondents' right to self-organization.
unit concerned.[24]
Moreover, PRI anchored their decision to terminate respondents'
We will emphasize anew that the power to dismiss is a normal
employment on Article 253 of the Labor Code which states that "it shall be
prerogative of the employer. This, however, is not without limitations. The
the duty of both parties to keep the status quo and to continue in full
employer is bound to exercise caution in terminating the services of his
force and effect the terms and conditions of the existing agreement
employees especially so when it is made upon the request of a labor union
during the 60-day period and/or until a new agreement is reached by the
pursuant to the Collective Bargaining Agreement. Dismissals must not be
parties."  It claimed that they are still bound by the Union Security Clause
arbitrary and capricious. Due process must be observed in dismissing an
of the CBA even after the expiration of the CBA; hence, the need to
employee, because it affects not only his position but also his means of
terminate the employment of respondents.
livelihood. Employers should, therefore, respect and protect the rights of
Petitioner's reliance on Article 253 is misplaced.
their employees, which include the right to labor.
The provision of Article 256 of the Labor Code is particularly
An employee who is illegally dismissed is entitled to the twin
enlightening.  It reads:
reliefs of full backwages and reinstatement. If reinstatement is not viable,
Article 256. Representation issue in organized establishments. -
separation pay is awarded to the employee. In awarding separation pay to
In organized establishments, when a verified petition questioning the
an illegally dismissed employee, in lieu of reinstatement, the amount to be
majority status of the incumbent bargaining agent is filed before the

Page 99 of 117
awarded shall be equivalent to one month salary for every year of service. 1. Service of First Written Notice (show-cause
Under Republic Act No. 6715, employees who are illegally dismissed are notice).
entitled to full backwages, inclusive of allowances and other benefits, or The first written notice to be served on the employee
their monetary equivalent, computed from the time their actual should:
compensation was withheld from them up to the time of their actual
reinstatement.  But if reinstatement is no longer possible, the backwages
shall be computed from the time of their illegal termination up to the
(a) Contain the specific causes or grounds for
finality of the decision.  Moreover, respondents, having been compelled to termination against him;
litigate in order to seek redress for their illegal dismissal, are entitled to the (b) Contain a directive that the employee is given the
award of attorney's fees equivalent to 10% of the total monetary award. opportunity to submit his written explanation with in
the reasonable period of five (5) calendar days from
c. National Interest Cases receipt of the notice;
Where strikers who violate orders, prohibitions and/or
injunctions as are issued by the DOLE Secretary or the NLRC, (i) To enable him to prepare adequately for his
dismissal or loss of employment status may be imposed. defense;
(ii) To study the accusation against him;
d. Failure to comply with drug test (iii) To consult a union official or lawyer;
(iv) To gather data and evidence; and
Plantation Bay v. Dubrico (v) To decide on the defenses he will raise
Held: As reflected in the above matrix, the confirmatory test against the complainant.
results were released earlier than those of the drug test, thereby casting
doubts on the veracity of the confirmatory results. (c) Contain a detailed narration of the facts and
Indeed, how can the presence of shabu be confirmed when the circumstances that will serve as a basis for the
results of the initial screening were not yet out? Plantation Bay's arguments charge against the employee. This is required in
that it should not be made liable thereof and that the doubt arising from the order to enable him to intelligently prepare his
time of the conduct of the drug and confirmatory tests was the result of the
explanation and defenses. A general description of
big volume of printouts being handled by Martell do not thus lie. It was
Plantation Bay's responsibility to ensure that the tests would be properly the charge will not suffice.
administered, the results thereof being the bases in terminating the (d) Specifically mention which company rules, if any,
employees' services. are violated and/or which among the grounds under
Time and again, we have ruled that where there is no showing Article 297 is being charged against the employee.
of a clear, valid and legal cause for termination of employment, the law
considers the case a matter of illegal dismissal. The burden is on the Remedy if employee refuses to receive notice: service by
employer to prove that the termination of employment was for a valid
registered mail to last known address
and legal cause. For an employee's dismissal to be valid, (a) the dismissal
In Nueva Ecija Electric Coop v. NLRC, it was held:
must be for a valid cause and (b) the employee must be afforded due
process.
In fine, as petitioners failed to indubitably prove that That private respondent refused to receive the
respondents were guilty of drug use in contravention of its drug-free memorandum is to us, too self-serving a claim on the part of
workplace policy amounting to serious misconduct, respondents are petitioner in the absence of any showing of the signature or
deemed to have been illegally dismissed. initial of the proper serving office. Moreover, petitioner could
have easily remedied the situation by the expediency of sending
the memorandum to private respondent by registered mail at his
e.Other Just Causes under Prevailing last known address as usually contained in the Personal Data
Jurisprudence Sheet or any personal file containing his last known address.
In addition to the just causes mentioned in the Labor
Code, the following are also found to be just causes: Employee is given a reasonable period to answer
charges
a. Violation of Company Rules and Regulations or In R.B. Michael Press v. Galit, the Supreme Court
Code of Conduct or Code of Discipline reiterated the rule enunciated in King of Kings Transport v.
[Sampaguita Auto Transport v. NLRC]. Mamac, that the reasonable period within which an employee being
b. Theft of property owned by a co-employee [John cited administratively should submit his written explanation is five
Hancock Life Insurance v. Davis], as distinguished (5) calendar days from receipt of the first notice to give him an
from company-owned property, which is considered opportunity to study the accusation against him, consult a union
serious misconduct. official or lawyer, or gather data and evidence, and decide on the
c. Incompetence, inefficiency or ineptitude [Reyes- defenses he will raise against the complaint.
Rayel v. Philippine Luen Thai Holdings].
d. Failure to Attain Work Quota [Aliling v. Feliciano]. CBA Grievance proceedings, held after employee had
e. Failure to comply with weight standards of been dismissed, do not take the place of the required procedure
employer [Yrasuegui v. Philippine Airlines]. demanded by the Labor Code before effecting dismissal
f. Attitude Problem [Reyes-Rayel v Philippine Luen In Standard Electric Manufacturing v. Standard Electric
Thai Holdings]. Employees Union, the Court held:

C. JUST CAUSE TERMINATION DUE PROCESS Further, we cannot subscribe to the petitioner's
The Supreme Court has standardized procedural due contention that the due process requirement relative to the
process in just cause termination in the 2007 case of King of Kings dismissal of respondent Javier was duly complied with when he
Transport v. Mamac. It proclaimed the following steps should be was allowed to explain his side during the grievance machinery
conferences. Indeed, in the case at bar, the petitioner did not
complied with:
conduct any investigation whatsoever prior to his termination,
despite being informed of respondent Javier's predicament by
the latter's siblings, his Union and his counsel. The meetings

Page 100 of 117


held pursuant to the grievance machinery provisions of the rule or practice requires it, or when similar
collective bargaining agreement were only done after  his circumstances justify it.
dismissal had already taken effect on February 5, 1996. Clearly, (c) the "ample opportunity to be heard" standard in the
well-meaning these conferences might be, they cannot cure an
Labor Code prevails over the "hearing or conference"
otherwise unlawful termination.
requirement in the implementing rules and
regulations:
2. Conduct of hearing.
(i) under Article 292(b) of the Labor Code, the
After serving the first notice above, the employer should
employer is required to afford the employee
schedule and conduct a hearing or conference wherein the
“ample opportunity to be heard and to defend
employee will be given the opportunity to:
himself with the assistance of his
representative if he so desires”; while –
(a) Explain and clarify his defenses to the charge/s against
(ii) Under Section 2(d), Rule I, Book VI of the
him;
IRR, the employee is required to afford the
(b) Present evidence in support of his defenses; and
employee a “hearing or conference during
(c) Rebut the evidence presented against him by the
which the employee concerned, with the
management.
assistance of counsel, if he so desires, is
given opportunity to respond to the charge,
During the hearing or conference, the employee should
present his evidence or rebut the evidence
be given the chance to defend himself personally, with the
presented against him.”
assistance of a representative or counsel of his choice. Moreover,
this conference or hearing could be used by the parties as an
If the employee admits his responsibility for the act he
opportunity to come to an amicable settlement.
was accused of, a formal hearing is no longer necessary, as held in
Bernardo v. NLRC.
Perez Doctrine: New Guiding Principle on the Hearing
The unilateral confession made by an alleged co-
Requirement
conspirator cannot be the basis for terminating an employee. Such
The above 2007 King of Kings concept of hearing as part
confession must be corroborated by other competent and
of due process has been significantly changed in 2009 in the en
convincing evidence. Absent any such corroborative evidence, the
banc case of Perez v. Philippine Telegraph and Telephone
confession must be received with considerable caution [Century
Company. It enunciates the new guiding principles on the hearing
Textile Mills, Inc. v. NLRC]. Likewise, it was held that the act of
aspect of procedural thus, a formal hearing or conference is no
the employer in making “prior consultation” with the union of
longer mandatory. It becomes mandatory only under any of the
which the dismissed employee is a member, is not sufficient
following circumstances:
compliance with due process.
a. When requested by the employee in writing; or
It is important to stress that the rights of an employee
b. When substantial evidentiary disputes exist; or whose services are sought to be terminated to be informed
c. When a company rule or practice requires it; or beforehand of his proposed dismissal (or suspension) as well as
d. When similar circumstances justify it. of the reasons therefor, and to be afforded an adequate
opportunity to defend himself from the charges levelled against
Thus, the Supreme Court held: him, are rights personal to the employee. Those rights were not
satisfied by petitioner Corporation's obtaining the consent of or
We note a marked difference in the standards of due consulting with the labor union; such consultation or consent
process to be followed as prescribed in the Labor Code and its was not a substitute for actual observance of those rights of
implementing rules. The Labor Code, on one hand, provides that private respondent Calangi. The employee can waive those
an employer must provide the employee ample opportunity to rights, if he so chooses, but the union cannot waive them for
be heard and to defend himself  with the assistance of his him. That the private respondent simply 'kept silent" all the
representative if he so desires. x x x while, is not adequate to show an effective waiver of his rights.
The omnibus rules implementing the Labor Code, on Notice and opportunity to be heard must be accorded by an
the other hand, require a hearing and conference during which employer even though the employee does not affirmatively
the employee concerned is given the opportunity to respond to demand them.
the charge, present his evidence or rebut the evidence presented
against him. NOTA BENE: It is surprising that the SC does not quote
Therefore, while the phrase "ample opportunity to be from a long line of decisions starting from Rabago v. NLRC
heard" may in fact include an actual hearing, it is not limited to a (1991); Rase v. NLRC (1994); Libres v. NLRC (1999). Those cases,
formal hearing only. In other words, the existence of an actual,
and many more thereafter, held that personal confrontation and
formal "trial-type" hearing, although preferred, is not absolutely
necessary to satisfy the employee's right to be heard. cross examination cannot be invoked as a matter of right in
procedural due process of employee dismissal cases. The right to
In sum, the following are the guiding principles in cross-examine belongs to the accused only in criminal prosecutions
connection with the hearing requirement in dismissal cases: by the “people of the state. In Manggagawa ng Komunikasyon v.
NLRC (1992), the Court however conceded that “actual adversarial
(a) "ample opportunity to be heard" means any proceedings may be necessary for clarification purposes or when
meaningful opportunity (verbal or written) given to there is need to propound searching questions to unclear
the employee to answer the charges against him and witnesses.” That has to do with the duty of the trier of facts, usually
submit evidence in support of his defense, whether in the labor arbiter, to ascertain whether or not there is “substantial
a hearing, conference or some other fair, just and evidence” supporting a claim.
reasonable way.
(b) a formal hearing or conference becomes mandatory Technol Eight Philippines v. NLRC
only when requested by the employee in writing or Held: The labor arbiter ruled that Technol failed to afford
Amular procedural due process, since he was not able to present his side
substantial evidentiary disputes exist or a company
Page 101 of 117
regarding the incident; at the time he was called to a hearing, he had already adversarial proceedings become necessary only for clarification, or
filed the illegal dismissal complaint. The NLRC, on the other hand, held when there is a need to propound searching questions to witnesses
that the memorandum terminating Amular's employment was a mere who give vague testimonies. This is not an inherent right, and in
formality, an afterthought designed to evade company liability since company investigations, summary proceedings may be conducted
Amular had already filed an illegal dismissal case against Technol.
[Tuazon v. Wenphil].
We disagree with these conclusions. The notice of preventive
suspension/notice of discharge served on Amular and Ducay required them
to explain within forty-eight (48) hours why no disciplinary action should If employee refuses to answer/participate, investigation
be taken against them for their involvement in the mauling incident. should still proceed
Amular submitted two written statements: the first received by the company In Hagonoy Rural Bank v. NLRC, the Court held that
on May 19, 2002 and the other received on May 20, 2002. On June 8, 2002, petitioners simply kept silent from the time they were suspended
Technol management sent Amular a memorandum informing him of an until they were formally dismissed is not adequate to constitute a
administrative hearing on June 14, 2002 at 10:00 a.m., regarding the waiver of their rights. Notice and hearing must be accorded by an
charges against him. At the bottom left hand corner of the memorandum,
employer, even though the employee does not affirmatively
the following notation appears: "accept the copy of notice but refused to
receive, he will study first." A day before the administrative hearing or on
demand it.
June 13, 2002, Amular filed the complaint for illegal
suspension/dismissal and did not appear at the administrative hearing. On 3. Service of Second Written Notice (Notice of
July 4, 2002, the company sent Amular a notice of dismissal. Termination)
What we see in the records belie Amular's claim of denial of After determining that termination of employment is
procedural due process. He chose not to present his side at the justified, the employer shall serve the employees a written notice of
administrative hearing. In fact, he avoided the investigation into the charges termination indicating that:
against him by filing his illegal dismissal complaint ahead of the scheduled
investigation. Under these facts, he was given the opportunity to be heard
and he cannot now come to us protesting that he was denied this (a) All circumstances involving the charge/s
opportunity. To belabor a point the Court has repeatedly made in employee against the employee have been considered;
dismissal cases, the essence of due process is simply an opportunity to be and
heard; it is the denial of this opportunity that constitutes violation of due (b) Grounds have been established to justify the
process of law. severance of his employment.

When hearing is not required D. AUTHORIZED CAUSES


The authorized causes provided in the Labor Code may
a. Termination of project, seasonal, causal or fixed-term generally be classified into two (2), namely:
employment.
b. Termination of probationary employment on the ground (1) Business-related causes. – Referring to the grounds
of failure of the probationary employee to qualify as a specifically mentioned in Article 298, to wit:
regular employee in accordance with reasonable
standards made known to him at the start of the (a) Installation of labor-saving device;
employment. (b) Redundancy
c. Termination due to abandonment of work. (c) Retrenchment;
d. Termination due to authorized causes under Article 298. (d) Closure or cessation of business operations
In such cases, there are no allegations which the NOT due to serious business losses or financial
employees should refute and defend themselves from by reverses; and
way of a hearing. (e) Closure or cessation of business operations.
e. Termination due to disease under Article 299.
f. Termination by the employee (resignation) under Article (2) Health-related causes. – Referring to disease under
300. Article 299.
g. Termination after 6 months of bona-fide suspension of
operation under Article 301. For purposes of satisfying There are certain requisites that are common to the five
due process, what is required is simply hat notices (5) grounds in Article 298. To simplify the discussion, the
provided under Article 298 be served to both the following five (5) common requisites are applicable to the said
affected employees and the DOLE at lease one (1) grounds:
month before the termination becomes effective.
h. Termination due to retirement under Article 302. 1. There is good faith in effecting the termination;
i. Termination due to expiration of tenure made 2. The termination is a matter of last resort, there being
coterminous with lease. no other option available to the employer after
j. Termination due to closure or stoppage of work by resorting to cost-cutting measures;
government authorities when non-compliance with the 3. Two (2) separate written notices are served on both
law or implementing rules and regulations poses grave the affected employee and the DOLE at least one (1)
and imminent danger to the health and safety of workers month prior to the intended date of termination;
in the workplace. 4. Separation pay is paid to the affected employee, to
k. Termination of employee who has admitted his guilt for wit:
the offense charged.
(a) If based on (1) installation of labor-saving
Confrontation of witnesses, not a matter of right in device, or (2) redundancy. – One (1) month pay
company investigations or at least one (1) month pay for every year of
Confrontation of witnesses is required only in adversarial service, whichever is higher, a fraction of at
criminal prosecutions, and not in company investigations for the least six (6) months shall be considered as one
administrative liability of the employee. Additionally, actual (1) year.

Page 102 of 117


(b) If based on (1) retrenchment, or (2) closure NOT Redundancy results from installation of labor-saving
due serious business losses or financial reverses. device. The installation of labor-saving device will result in making
– One (1) month pay or at lease one-half (1/2) the positions being held by employees who will be adversely
month pay for every year of service, whichever affected thereby redundant and unnecessary [Soriano v. NLRC].
is higher, a fraction of at least six (6) months
shall be considered as one (1) whole year. 2. Redundancy
(c) If closure is due to serious business losses or In addition to the five (5) common requisites earlier
financial reverses, NO separation pay is required mentioned, any of the following factors must be present in order
to be paid. for redundancy to be a valid ground to terminate employment:
(d) In case the CBA or company policy provides for
a higher separation pay, the same must be a. Where the services of employees are in excess of
followed instead of he one provided in Article what is reasonably demanded by the actual
298. requirements of the enterprise [Nippon Housing v.
Leynes].
5. Fair and reasonable criteria in ascertaining what b. Where the position is superfluous because of a
positions are to be affected by the termination, such number of factors, such as over-hiring of workers,
as, but not limited to: nature of work; status of decreased volume of business, dropping of a
employment (whether casual, temporary, or regular); particular product line or service activity previously
experience; efficiency; seniority; dependability; manufactured or undertaken by the enterprise or
adaptability; flexibility; trainability; job performance; phasing out of service activity priorly undertaken by
discipline; and attitude towards work. Failure to the business.
follow fair and reasonable criteria in selecting who to c. Where there is duplication of work. Indeed, in any
terminate would render the termination invalid. well-organized business enterprise, it would be
surprising to find duplication of work and two (2) or
Each of the five grounds has its own unique requisite/s more people doing the work of one person [Caltex
that distinguishes it from the others. For instance, the requisite for v. NLRC].
extreme business losses or financial reverses is distinctively d. Where it is validly resorted to as a cost-cutting
applicable to retrenchment in order for termination based on this measure and to streamline operations so as to make
ground to be valid and legal. Termination due to redundancy does them more viable [Maya Farms Employees
not require existence of losses or financial reverses to validate it. Organization v. NLRC].
While losses or reverses may be considered as a major factor in
cases of closure or cessation of business operations, but their Time and again, it has been ruled that employer has no
relevance is only in relation to the determination of whether the legal obligation to keep more employees than are necessary for the
employer is liable for separation pay or not. Consequently, if the operation of its business [Morales v. Metrobank]. Thus, the
closure or cessation of business operations is due to serious employer has the prerogative to implement reorganization and
business losses or financial reverses, the employer is not liable to redundancy and to adopt such measures as will promote greater
pay any separation pay [North Davao Mining v. NLRC]. efficiency, reduce overhead costs and enhance prospects of
economic gains, albeit always within the framework of existing
1. Installation of Labor-Saving Device laws [Smart Comms v. Astorga]. However, if there is no proof that
the essential requisites for a valid redundancy program as a ground
Article 298. Closure of Establishment and Reduction of for the termination of the employee are present, the termination
Personnel. The employer may also terminate the employment of any should be declared illegal [Lamber Pawnbrokers v. Binamira].
employee due to the installation of labor-saving devices, redundancy,
retrenchment to prevent losses or the closing or cessation of operation 3. Retrenchment
of the establishment or undertaking unless the closing is for the
Retrenchment has been defined as the termination of
purpose of circumventing the provisions of this Title, by serving a
written notice on the workers and the Ministry of Labor and
employment initiated by the employer through no fault of the
Employment at least one (1) month before the intended date thereof. In employees and without prejudice to the latter, resorted by
case of termination due to the installation of labor-saving devices or management during periods of business recession, industrial
redundancy, the worker affected thereby shall be entitled to a depression, or seasonal fluctuations; or during lulls occasioned by
separation pay equivalent to at least his one (1) month pay or to at least lack of work or orders, shortage of materials or considerable
one (1) month pay for every year of service, whichever is higher. In reduction in the volume of the employer’s business, conversion of
case of retrenchment to prevent losses and in cases of closures or the plant for a new production program or the introduction of new
cessation of operations of establishment or undertaking not due to
methods or more efficient machinery, or of automation [Anabe v.
serious business losses or financial reverses, the separation pay shall be
equivalent to one (1) month pay or at least one-half (1/2) month pay for
Asian Construction].
every year of service, whichever is higher. A fraction of at least six (6) In addition to the five (5) common requisites mentioned
months shall be considered one (1) whole year. earlier, the unique requisite for this ground is that there should be
proof of actual losses or possible imminent losses that would
In addition to the five (5) common requisites above, the justify termination of employment. This is the most singular
unique requisite for this ground is that the purpose for such distinctive requisite of retrenchment. This, in fact, is the only
installation of labor-saving device/s must be valid, such as to save statutory ground in Article 298 which requires this kind of proof.
on cost, enhance efficiency and other justifiable economic reasons. As stressed earlier, the grounds of installation of labor-saving
The installation of these devices is a management device and redundancy do not impose this requirement. The other
prerogative and the courts will not interfere with its exercise in the ground of closure or cessation of business operations may be
absence of abuse of discretion, arbitrariness, or malice on the part resorted to with or without losses [Precision Electronics v. NLRC].
of management [Magnolia Dairy Products v. NLRC].
Standards to determine validity of losses

Page 103 of 117


The general standards in terms of which the act of an Closure or cessation of business is the complete or partial
employer in retrenching or reducing the number of its employees cessation of the operations and/or shutdown of the establishment of
must be appraised are as follows: the employer. It is carried out to either stave off the financial ruin
or promote business interest of the employer [Eastridge Gold Club
a. The losses expected should be substantial and not v. Eastridge Labor Union]. Closure involves two (2) situations:
merely de minimis or insubstantial and (a) When NOT due to serious business losses or
inconsequential in extent. financial reverses; or
b. The substantial loss apprehended must be (b) When not due to serious business losses or financial
reasonably imminent, as such imminence can be reverses.
perceived objectively and in good faith by the
employer. It is only in the first that payment of separation pay is
c. Retrenchment must be reasonably necessary and required. No such requirement is imposed in the second [North
likely to effectively prevent the expected losses. The Davao Mining v. NLRC].
employer should have taken other measures prior or Employer may close its business whether its suffering
parallel to retrenchment to forestall losses, i.e., cut from business losses or not; court cannot order employer to
other costs other than labor costs. continue its business [Penafrancia Tours v. Sarmienta].
d. The alleged losses, if already realized, and the
expected imminent losses sought to be forestalled, Retrenchment v. Closure of business
must be proved by sufficient and convincing In a number of cases. Retrenchment has been confused
evidence through presentation of externally audited with closure of the entire business establishment or department,
financial statements. division or outlet thereof. While the two are often used
interchangeably and are interrelated, they are actually two separate
If the above standards are present, the wisdom to retrench and independent authorized causes for termination of employment.
cannot be questioned [NDC v. NLRC]. Termination of an employment may be predicated on one without
need of resorting to the other.
Retrenchment to prevent losses, meaning Closure of business, on one hand, is the reversal of
In its ordinary connotation, this phrase means that fortune of the employer whereby there is a complete cessation of
retrenchment must be undertaken by the employer before the losses business operations and/or an actual locking-up of the doors of the
anticipated are actually sustained or realized. The Supreme Court, establishment, usually due to financial losses. Closure of business
in a plethora of cases, has thus interpreted it to mean that the as an authorized cause for termination of employment aims to
employer need not keep all his employees until after its losses shall prevent further financial drain upon an employer who cannot pay
have materialized [TPI Philippines v. Cajucom]. This is never the anymore his employees since business has already stopped. On the
intention of the lawmaker. If such an intent were expressly written other hand, retrenchment is a reduction of personnel usually due to
into the law, that law may well be vulnerable to constitutional poor financial returns so as to cut down on costs of operations in
attack as unduly taking property from one man to be given to terms of salaries and wages to prevent bankruptcy of the company.
another [Asian Alcohol v. NLRC]. It is sometimes also referred to as down-sizing. It is an authorizes
cause for termination of employment which the law accords an
Redundancy v. Retrenchment employer who is not making good in its operations in order to cut
Retrenchment and redundancy are two different back on expenses for salaries and wages by laying off some
concepts; they are not synonymous; thus, they should not be used employees. The purpose of retrenchment is to save a financially
interchangeably [Arabit v. Jardine Pacific]. ailing business establishment from eventually collapsing [Sanoh
Redundancy exists when the services of an employee are Fulon Phils v. Bernardo].
in excess of what is required by an enterprise. Retrenchment, on Unlike retrenchment, closure or cessation of business, as
the other hand, is resorted to primarily to avoid or minimize an authorized cause of termination of employment, need not
business losses. Thus, a “Redundancy Program,” while depend for its validity on evidence of actual or imminent reversal
denominated as such, is more precisely termed “retrenchment” if it of the employer’s fortune. Article 298 authorizes termination of
was primarily intended to prevent serious business losses [Atlantic employment due to business closure regardless of the underlying
Gulf v. NLRC]. reasons and motivations therefor, be it financial losses or not
Redundancy does not need to be always triggered by a [Eastridge Golf Club v. Eastridge Labor Union].
decline in the business. Primarily, employers resort to redundancy
when the functions of an employee have already become MPCEU v. Manila Polo Club summarized:
superfluous, duplicitous or in excess of what the business requires.
Thus, even if a business is doing well, an employer can still validly a. Closure or cessation of operations of establishment or
dismiss an employee from service due to redundancy if that undertaking may either be partial or total.
employee’s position has already become in excess of what the b. Closure or cessation of operations of establishment or
employer’s enterprise requires [Andrada v. NLRC]. undertaking may or may not be due to serious
In terms of monetary consequence, the employer stands business losses or financial reverses. However, in both
to pay more separation pay if it denominates the personnel instances, proof must be shown that: (1) it was done in
reduction program it is implementing as redundancy and not good faith to advance the employer's interest and not
retrenchment. Under Article 298, redundancy would require the for the purpose of defeating or circumventing the
employer to pay its employees a separation pay equivalent to at rights of employees under the law or a valid
least their one (1) month pay or to at least one (1) month pay for agreement; and (2) a written notice on the affected
every year of service, whichever is higher; while retrenchment employees and the DOLE is served at least one month
would only entail half of this amount. before the intended date of termination of
employment.
4. Closure or Cessation of Business Operations

Page 104 of 117


c. The employer can lawfully close shop even if not due his health as well as to the health of his co-employees.” This
to serious business losses or financial reverses but ground may not, however, be solely confined to these kinds of
separation pay, which is equivalent to at least one diseases. Deoferio enunciates that the phrase “prejudicial to his
month pay as provided for by Article 283 of the Labor health as well as to the health of his co-employees” should be
Code, as amended, must be given to all the affected liberally construed to mean “prejudicial to his health or to the
employees. health of his co-employees.” It is clear, therefore, that the intent of
d. If the closure or cessation of operations of the law is to allow the termination of an employee if he suffers a
establishment or undertaking is due to serious disease and his continued employment will either be prejudicial:
business losses or financial reverses, the employer
must prove such allegation in order to avoid the (i) To his own health; or
payment of separation pay.  Otherwise, the affected (ii) To the health of his co-employees.
employees are entitled to separation pay.
e. The burden of proving compliance with all the above- Consistent with this construction, this provision has been
stated falls upon the employer. applied in resolving illegal dismissal cases due to non-contagious
diseases such as stroke, heart attack, osteoarthritis, and eye
5. Disease cataract, among others.

Article 299. Disease as Ground for Termination. An employer 3rd Substantive Element
may terminate the services of an employee who has been found to be The third element on presentation of a medical certificate
suffering from any disease and whose continued employment is issued by a competent public health authority substantiates the
prohibited by law or is prejudicial to his health as well as to the health contention that the employee has indeed been suffering from a
of his co-employees: Provided, That he is paid separation pay
disease that:
equivalent to at least one (1) month salary or to one-half (1/2) month
salary for every year of service, whichever is greater, a fraction of at
least six (6) months being considered as one (1) whole year. (i) Is prejudicial to his health as well as to the health of
his co-employees; and
In the case of Deoferio v. Intel Technology Philippines, (ii) Cannot be cured within a period of six months even
Inc., the requisites that must be complied with before termination with proper medical treatment.
of employment due to disease may be justified were specifically
divided into two, namely: Without the medical certificate, there can be no
authorized cause for the employee’s dismissal. The absence of this
a. Substantive requisites; and element thus renders he dismissal void and illegal. Deoforio
b. Procedural requisites. instructs that this 3rd element is not merely a procedural
requirement but a substantive one. That certification from a
a. Substantive Requisites competent public health authority is precisely the substantial
Based on the Labor Code and its Implementing Rules, evidence required by law to prove the existence of the disease
the following three (3) substantive elements, according to Deoferio itself, its non-curability within a period of 6 months even with
v. Intel Technology may be drawn therefrom, to wit: proper medical treatment, and the prejudice it would cause to the
health of the sick employee and to those of his co-employees.
(i) An employee has been found to be suffering from The company’s own physician engaged by the employer
any disease; as its employee or hired on a retainer fee basis to whom sick
(ii) His continued employment is: workers are referred for consultation or treatment, is not the
“competent public health authority: referred to in the law. Hence, a
medical certificate issued by the company’s own physician is not
 Prohibited by law; or
an acceptable certificate for purposes of terminating an
 Prejudicial o his health as well as to the health
employment based on Article 299 [Cebu Royal Plant v. Deputy
of his co-employees; and
Minister].
As to who should procure the medical certificate, the
(iii) A competent public health authority issues a
Court ruled in Tan v. NLRC, that it devolves upon the employer the
medical certificate that the disease of such nature or
obligation to obtain a medical certificate from a competent public
at such a stage that it cannot be cured within a
health authority that the employee’s disease is at such stage or of
period of six (6) months even with proper medical
such nature that it cannot be cured within 6 months even with
treatment.
proper medical treatment. It is the employer and not the employee,
who has the burden of proof to justify that the termination was
1st Substantive Element
supported by said certificate. Clearly, it is only where there is such
The fact alone that an employee is suffering from a
prior certification that the employee could be validly terminated
disease is not generally a sufficient ground to terminate his
from his job. The burden of proving the existence of the medical
employment. That a person has a disease does not per se entitle the
certificate required under the law is upon the employer, not the
employer to terminate his or her services. Termination is the last
employee [ATCI Overseas Corporation v. Court of Appeals].
resort. Even if the disease is a contagious one, like pulmonary
tuberculosis, mere sufferance thereof by an employee does not ipso
b. Procedural Requisites
facto make him a sure candidate for dismissal [Tan v. NLRC].
Deoferio pronounced the rule that due process in
termination due to disease is similar to due process for just cause
2nd Substantive Element
termination. Thus, the employer must furnish the employee two (2)
Contagious or communicable diseases or infecions, like
written notices, namely:
sexually transmitted diseases or infections, tuberculosis, hepatitis
A, malaria, and among others, are bext examples of diseases which
would render an employee’s “continued employment prejudicial to
Page 105 of 117
(i) The notice to apprise the employee of the ground for due process if notice requirement is not complied with at least a
which his dismissal is sought; and month prior to the effectivity of the termination. Hence, the
(ii) The notice informing the employee of his dismissal, dismissal, if properly and validly effected for authorized cause,
to be issued after the employee has been given would still be declared legal but the employer shall be held liable to
reasonable opportunity to answer and be heard on pay for indemnity in the form of nominal damages in the stiffer
his defense. amount of P50,000, per Jaka Food Processing v. Pacot.

Under this present rule, the employee should be given F. EFFECT OF LEGAL DISMISSAL WITHOUT DUE
reasonable opportunity to answer and to be heard on his defense. PROCESS
Although sufferance of disease is not to be equated with The substantive (just or authorized cause) and procedural
commission of a wrongful act which is the principal requisite of due process requirements in termination of employment can only
just cause termination, the ailing employee needs to be given be better understood and appreciated by looking at them through
“reasonable opportunity to answer and to be heard on his defense” the prism of the four (4) standard situations provided in the Labor
before he could validly be dismissed on the ground of disease. Code and enunciated in pertinent jurisprudence. Thus, the dismissal
Consequently, once the substantive requisites for termination due is:
to disease are complied with, the employer should comply the other
equally important procedural requisite as prescribed in Deoferio. (1) LEGAL if it was done with both substantive and
Per Deoferio, the 2nd required notice informing the procedural due process.
employee of his dismissal should be issued “after the employee has (2) ILLEGAL if it was done without substantive due
been given reasonable opportunity to answer and to be heard on his process although procedural process was observed.
defense.” This requirement, in effect, dictates that before an (3) ILLEGAL if it was done without both substantive
employee may be terminated due to disease, he must first be given and procedural due process.
a show-cause notice that would afford him “a reasonable (4) LEGAL if it was done with substantive due process
opportunity to answer” the charge of his being terminable by but without procedural due process.
reason of his suffering a disease, and secondly, for him to be
afforded a hearing on his defense. 1. Belated Due Process Rule
Prior to 1989, the rule was that a dismissal or termination
Employee has the right to present countervailing is illegal if the employee was not given procedural due process. In
medical certificates the 1989 case of Wenphil Corp v. NLRC (Feb, 8, 1989), the Court
The employee has the right to present countervailing reversed this long-standing rule and held that the dismissed
evidence in the form of medical certificates to prove that his employee, although not given any notice and hearing, was not
dismissal due to disease is not proper and therefore illegal [Fuji entitled to reinstatement and backwages because the dismissal was
Television Network v. Espiritu]. for a just cause, i.e., grave miscoundct and insubordination, a just
ground for termination under Article 297. The employee here had a
E. AUTHORIZED CAUSE TERMINATION DUE violent temper and caused trouble during office hours and defying
PROCESS superiors who tried to pacify him. The Court concluded that
Due process in authorized cause termination is classified reinstating the employee and awarding backwages may encourage
into two (2), as follows: him to do even worse and will render a mockery of the rules of
discipline that employees are required to observe.
1. Termination due to Business-Related Causes, such However, the employer committed an infraction of the
as: installation of labor-saving device, redundancy, second requirement; thus, it must be imposed a sanction for its
retrenchment and closure of business or failure to give a formal notice and conduct an investigation as
establishment. required by law before dismissing the employee from employment.
2. Termination due to Health-Related Causes Considering the circumstances of this case, the employer must
(Disease). indemnify the employee. The measure of this award depends of the
facts of each case and the graviy of the omission committed by the
1. Due Process in Termination Due to Business- employer.
Related Authorized Causes The rule thus evolved: where the employer had a valid
Procedural due process in termination due to any of the reason to dismiss an employee but did not follow the due process
authorized causes of installation of labor-saving device, requirement, the dismissal may be upheld but the employer will be
redundancy, retrenchment and closure of business or establishment penalized to pay an indemnity to the employee.
is deemed complied with upon the separate and simultaneous
service of a written notice of the intended termination to both: 2. Ineffectual Dismissal Rule
In 2000, the rule on the extent of the sanction was
a. The employee to be terminated; and changed in the en banc decision in Serrano v. NLRC. The Court
b. The appropriate DOLE Regional Office held that the vilation by the employer of the notice requirement in
termination for just or authorized causes was not a denial of due
At least one (1) month before the intended date of the process that will nullify the termination. However, the dismissal is
termination specifying the ground/s therefor and the undertaking to declared ineffectual and the employer must pay the full backwages
pay the separation pay required under Article 298 of the Labor from the time of termination until it is judicially declared that the
Code or the employment contract or the CBA, whichever is higher. dismissal was for a just cause or authorized cause. The Court, in
To iterate, hearing is required. effect, re-examined the Wenphil doctrine. Hence, instead of
penalty, Serrano now required payment of full backwages from the
One-month period is mandatory time of dismissal until the time the Court finds the dismissal was
The observance of the period of 1 month mentioned in for a just or authorized cause. Serrano thus confronted the practice
Article 298 is mandatory. There is deprivation of right to statutory

Page 106 of 117


of employers to dismiss now and pay later by imposing full employer bears the burden of proof to prove that the termination
backwages as penalty. was for a valid or authorized cause. However, before the employer
must bear the burden of proving that the dismissal was legal, the
3. Statutory Due Process Rule employee must first establish by substantial evidence the fact of
About 4 years after Serrano, the Supreme Court, in the his dismissal from service. Logically, if there is no dismissal, then
2004 en banc decision in Agabon v. NLRC, abandoned Serrano and there can be no question as to the legality or illegality thereof
reverted to the Wehnphil doctrine. It thus ruled that where the [Ledesma, Jr. v. NLRC].
dismissal is for a just cause, as in the instant case, the lack of
statutory due process should not nullify the dismissal, or render it 2. Quitclaims
illegal, or ineffectual. However, the employer should indemnify the Quitclaims are valid provided that these are voluntarily
employee for the violation of his right to statutory due process. signed by the employee involved, the consideration is reasonable,
Such indemnity or sanction, however, must be stiffer than that and it is not against the law or public policy [More Maritime
imposed in Wenphil. Consequently, the sanction imposed upon the Agencies v. NLRC].
employer in this case was in the form of nominal damages in the However, quitclaims entered into by union officers and
higher amount of P30,000. The imposition of this form of damages some members do not bind those who did not sign it [Liana’s
would serve to deter employers from future violations of the Supermarket v. NLRC].
statutory due processs rights of employees. At the very least., it
provides a vindication or recognition of his fundamental right 3. Reliefs for Illegal Dismissal
granted to the latter under the Labor Code and its Implementing
Rules. Article 294. [279] Security of Tenure. In cases of regular
employment, the employer shall not terminate the services of an
4. Contractual Due Process Rule employee except for a just cause or when authorized by this Title. An
A new doctrine of contractual due process was employee who is unjustly dismissed from work shall be entitled to
reinstatement without loss of seniority rights and other privileges and
pronounced in the 2013 en banc decision in Abbott Laboratories v.
to his full backwages, inclusive of allowances, and to his other benefits
Alcaraz. It was held here that in a situation where there is an or their monetary equivalent computed from the time his compensation
existing company policy enunciating the procedural due process was withheld from him up to the time of his actual reinstatement
that must be observed in termination of employment, compliance
alone with the statutory due process, would not suffice. Under Article 294, an illegally dismissed employee is
Additionally, there must be compliance too with the company- entitled to the following reliefs:
prescribed due process procedure or the so-called contractual due
process. Otherwise, the same consequence as in Agabon case will a. Reinstatement without loss of seniority rights and
ensue, that is, the termination shall be considered legal and valid other privileges;
but for lack of contractual due process, the employer will be b. Full backwages, inclusive of allowances; and
penalized with the payment of indemnity in the form of nominal c. Other benefits or their monetary equivalent.
damages in the same amount of P30,000 as awarded in Agabon.
The following reliefs that are awarded in illegal dismissal
5. Indemnity in the Form of Nominal Damages cases are missing in Article 279:
Termination for a just cause or authorized cause but
without affording the employee procedural due process should no a. Award of separation pay in lieu of reinstatement.
longer be considered illegal or ineffectual [Per Serrano v. NLRC] b. Award of penalty in the form of nominal damages in
but legal. Consequently, the employee will not be ordered case of termination due to just or authorized cause
reinstated but will be awarded an indemnity in the form of nominal but without observance of procedural due process.
damages, the amount of which will depend on whether the c. Reliefs to illegally dismissed employee whose
termination is grounded on just cause or authorized cause, thus: employment is for a fixed period. The proper relief
is only the payment of the employee’s salaries
a. If based on just cause – P30,000.00 per Agabon corresponding to the unexpired portion of the
Doctrine employment contract.
b. If based on authorized cause – P50,000.00 per Jaka d. Award of damages and attorney’s fees.
Doctrine. e. Award of financial assistance in cases where the
employee’s dismissal is declared legal but because
The measure of penalty or indemnity is no longer full of long years of service, and other considerations,
backwages but nominal damages. financial assistance is awarded.
f. Imposition of legal interest on separation pay,
G. ILLEGAL DISMISSAL backwages and other monetary awards.
Under the Labor Code, the ordinary and proper recourse
of an illegally dismissed employee is to file a complaint for illegal a. Reinstatement
dismissal with the labor arbiter [Philippine Airlines v. NLRC]. Reinstatement, in its generally accepted sense, refers to a
restoration to a state from which one has been removed or
1. Burden and Degree of Proof separated, it is the return to the position from which he was
It is a basic rule in evidence, however, that the burden of removed [San Miguel Brewery v. Santos].
proof is on the part of the party who makes the allegations – ei
incumbit probatio, qui dicit, non qui negat. If he claims a right Reinstatement in Article 229 vs. 294
granted by law, he must prove his claim by competent evidence,
relying on the strength of his own evidence and not upon the
Article 229 Article 294
weakness of that of his opponent. While this Court is not
Finality No finality yet. Already final and
unmindful of the rule that in cases of illegal dismissal, the Subject of an appeal executory

Page 107 of 117


Employer’s option Employer has the No option but to  To prevent further delay in the execution of the
to reinstate option to actual actually reinstate to decision to the prejudice of private respondent.
reinstatement or former position or to
payroll reinstatement. a substantially  Other circumstances such as (a) when
equivalent position reinstatement is inimical to the employer’s
Duty of Labor Ministerial for the Not ministerial. interest; (b) reinstatement does not serve the
Arbiter to LA to implement his Requires motion for best interests of the parties involved; (c) the
implement order reinstatement order. the issuance of writ employer is prejudiced by the workers’
Reinstatement is self- of execution before
executory LA can implement continued employment; or (d) that it will not
the reinstatement serve any prudent purpose as when
order supervening facts transpired which made
Necessity of Not necessary Indispensable to execution unjust or inequitable.
issuance of Writ of effect reinstatement
Execution
Computation
Per prevailing jurisprudence, the following are the
b. Separation Pay in Lieu of Reinstatement
components of separation pay in lieu of reinstatement:
This remedy is not found in the Labor Code but is
granted in case reinstatement is no longer possible or feasible, such
(i) The amount equivalent to at least one (1)
as when any of the following circumstances exists:
month salary or to one (1) month salary for
every year of service, whichever is higher, a
(i) Where the continued relationship between the
fraction of at least six (6) months being
employer and the employee is no longer viable
considered as one (1) whole year.
due to the strained relations and antagonism
(ii) Allowances that the employee has been
between them (Doctrine of Strained Relations).
receiving on a regular basis
(ii) When reinstatement proves impossible,
impracticable, not feasible or unwarranted for
The salary rate prevailing at the end of the period of
varied reasons and thus hardly in the best
putative service should be the basis for computation which refers to
interest of the parties such as:
the period of imputed service for which the employee is entitled to
backwages.
 Where the employee has already been
replaced permanently as when his position has
Period Covered
already been taken over by a regular employee
The period for computation shall be from start of
and there is no substantially equivalent
employment up to the date of finality of decision except:
position to which he may be reinstated.
 Where the dismissed employee’s position is
(i) When the employer has ceased its operation
no longer available at the time of
earlier, in which case, the same should be
reinstatement for reasons not attributable to
computed up to the date of closure.
the fault of the employer.
(ii) When the employee is a fixed term employee,
 When there has been long lapse or passage of in which case, it shall only be up to the
time that the employee was out of employer’s unexpired portion of the fixed-term contract.
employ from the date of the dismissal to the (iii) When the employee reached retirement or
final resolution of the case or because of the retirement, in which case, it shall only be up to
realities of the situation. retirement age or date of retrenchment
 By reason of the injury suffered by the (iv) When the employee dies during the pendency
employee. of the case, in which case, it shall only be up to
 The employee has already reached retirement the time of the death of such employee.
age under a Retirement Plan.
 When the illegally dismissed employees are Principles
over-age or beyond the compulsory retirement
age and their reinstatement would unjustly  Award of separation pay and backwages are not inconsistent
prejudice their employer. with each other. Hence, both may be awarded to an illegally
dismissed employee. The payment of separation pay is in
(iii) Where the employee decides not to be addition to payment of backwages.
reinstated as when he does not pray for  Reinstatement cannot be granted when what is prayed for by
reinstatement in his complaint or position paper employee is separation pay in lieu thereof.
but asked for separation pay instead.  Award of separation pay in lieu of reinstatement is not proper
(iv) When reinstatement is rendered moot and if there is no finding of illegal dismissal.
academic due to supervening events, such as:
 Separation pay, as a substitute remedy, is only proper for
reinstatement but not for backwages.
 Death of the illegally dismissed employee.
 Employer has no option to choose between reinstatement and
 Declaration of insolvency of the employer by separation pay in lieu thereof. Reinstatement is still the
the court. preference.
 Fire which gutted the employer’s establishment  Grant of separation pay in lieu of reinstatement converts the
and resulted in its total destruction. award of reinstatement into a monetary award; hence, legal
 In case the establishment where the employee interest may be imposed thereon.
is to be reinstated has closed or ceased
operation.

Page 108 of 117


 An employee is not entitled to separation pay when he or she  Employers offer to reinstate does not forestall payment of full
resigns voluntarily, unless it is a company practice or provided backwages.
in the CBA [Hanford Philippines v. Joseph].
Limited Backwages
c. Backwages There are also instances where backwages were not
Backwages is a relief that restores the income that was awarded in full but merely limited for the reason of good faith on
lost by reason of the illegal dismissal. It refers to the compensation the part of the employer.
which an employee would have earned had he not been unjustly
dismissed. On the other hand, “unpaid wages” refer to No Backwages
compensation for services already rendered but withheld by the Under the following situations, reinstatement of an
employer. illegally dismissed employee is granted without the accompanying
In 1996, the Supreme Court changed the rule on the backwages:
reckoning of backwages. It announced a new doctrine in the case of
Bustamante v. NLRC, which is now known as the Bustamante (i) When the dismissal is deemed too harsh a
doctrine. Under this rule, the term “full backwages” should mean penalty;
exactly that, i.e., without deducting from backwages the earnings (ii) When the employer acted in good faith; or
derived elsewhere by the concerned employee during the period of (iii) Where there is no evidence that the employer
his illegal dismissal. dismissed the employee.
The components of backwages are as follows:
Thus, the backwages will not be granted in full but
(i) Salaries or wages computed on the basis of the limited to 1 year, 2 years or 5 years.
wage rate level at the time of the illegal
dismissal and not in accordance with the latest, As to the first situation, one illustrative case is ALU-
current wage level of the employee’s position. TUCP v. NLRC, where reinstatement with no backwages was
(ii) Allowances and other benefits regularly ordered because the penalty imposed on the employee for
granted to and received by the employee committing theft of company property was reduced to suspension
should be made part of backwages. due to mitigating circumstances. The justification was that the
entire period when the employee was out of job because of his
This is computed from the time compensation was dismissal should already be considered as the period of his
withheld up to the time of the employee’s reinstatement (actual or suspension; hence, he should no longer be entitled to backwages
payroll) except: for the same period.
Another case is Yupangco v. NLRC, where, after finding
(i) If reinstatement is not possible (e.g. if what is that the employee was illegally dismissed but at the same time
awarded is separation pay in lieu of guilty of misconduct, it was ruled that there was no grave abuse of
reinstatement) – only up to the date of finality discretion in the resolution of the NLRC which meted only the
of decision. penalty of suspension without backwages.
(ii) If employer ceased operations – only up to the In Pepsi-Cola v. NLRC, where the employee filed a leave
date of closure. of absence for one day after he suffered stomach ache and upon the
(iii) If fixed-term/probationary employee – only up advice of his doctor, he took a rest for 25 days without prior leave.
to the unexpired portion of the fixed-term When he reported back for work, he was told he had been
contract or probationary employment contract. dismissed for being absent without leave. It was held that while he
(iv) If employee was confined in prison – only up was at fault, he could not be dismissed. He was thus ordered
to the date of confinement in prison. reinstated but he was denied backwages.
(v) If employee reached retirement or
retrenchment – only up to retirement age or d. Damages
date of retrenchment. The employee is entitled to moral damages when the
(vi) If employee becomes physically or mentally employer acted a) in bad faith or fraud; b) in a manner oppressive
incapacitated – only up to the date of to labor; or c) in a manner contrary to morals, good customs, or
incapacity. public policy [Montinola v. Philippine Airlines].
(vii) If employee dies during the pendency of the Bad faith "implies a conscious and intentional design to
case – only up to the time of death. do a wrongful act for a dishonest purpose or moral obliquity."
Cathay Pacific Airways v. Spouses Vazquez established that bad
Settled Principles faith must be proven through clear and convincing evidence. This
is because "[b]adfaith and fraud . . . are serious accusations that can
 Any amount received during payroll reinstatement is be so conveniently and casually invoked, and that is why they are
deductible from backwages. never presumed. They amount to mere slogans or mudslinging
 Salary increases during period of unemployment are not unless convincingly substantiated by whoever is alleging them."76
included as component in the computation of backwages. Here, there was clear and convincing evidence of bad faith adduced
 Dismissed employee’s ability to earn is irrelevant in the award in the lower tribunals.
of backwages.
 Employee is entitled to backwages even if it is not included in e. Attorney’s Fees
his prayer or even if the LA or NLRC failed to award To recapitulate, both the Labor Code and the Civil Code
backwages. provide that attorney's fees may be recovered in the following
 Backwages excludes period of valid suspension but includes instances, namely, (i) in cases involving the unlawful withholding
period of preventive suspension. of wages;[38] (ii) where the defendant's act or omission has
compelled the plaintiff to litigate with third persons or the plaintiff

Page 109 of 117


incurred expenses to protect his interest;[39] (iii) in actions for the employer failed to complete the hearing or investigation within
recovery of wages of household helpers, laborers and skilled said period due to justifiable grounds. No extension thereof can
workers;[40] (iv) in actions for indemnity under workmen's be made based on whimsical, capricious or unreasonable
compensation and employer's liability laws;[41] and (v) in cases grounds.
where the court deems it just and equitable that attorney's fees and  Preventive suspension lasting longer than 30 days, without the
expenses of litigation should be recovered [Alva v. High Capacity benefit of valid extension, amounts to constructive dismissal.
Security Force].  Indefinite preventive suspension amounts to constructive
In a catena of cases, the Court awarded attorney's fees in dismissal.
favor of illegally dismissed employees who were compelled to file
an action for the recovery of their lawful wages, which were IX
withheld by the employer without any valid and legal basis.[43] A TERMINATION OF EMPLOYMENT BY
plain showing that the lawful wages were not paid without EMPLOYEE
justification was sufficient to warrant an award of attorney's fees
[Kaisahan v. Manila Water]. Article 300. [285] Termination by Employee. (a) An employee
may terminate without just cause the employee-employer relationship
H. PREVENTIVE SUSPENSION by serving a written notice on the employer at least one (1) month in
Preventive suspension may be legally imposed against an advance. The employer upon whom no such notice was served may
errant employee only while he is undergoing an investigation for hold the employee liable for damages.
certain serious offenses. Consequently, its purpose is to prevent (b) An employee may put an end to the relationship without
him from causing harm or injury to the company as well as to his serving any notice on the employer for any of the following just causes:
1. Serious insult by the employer or his representative on the
fellow employees, hence, his actual presence in the workplace
honor and person of the employee;
would not be desirable for the meaningful conduct of the 2. Inhuman and unbearable treatment accorded the
investigation of his case. Its imposition is thus justified only in employee by the employer or his representative;
cases where the employee’s continued presence in the company 3. Commission of a crime or offense by the employer or his
premises during the investigation poses a serious and imminent representative against the person of the employee or any of the
threat to the life or property of the employer or of the immediate members of his family; and
employee’s co-workers. Without this threat, preventive 4. Other causes analogous to any of the foregoing.
suspension is not proper.
A. VOLUNTARY RESIGNATION
Settled Principles Resignation is the voluntary act of an employee who
finds herself in a situation where she believes that personal reasons
 Preventive suspension, by itself, does not signify that the cannot be sacrificed in favor of the exigency of the service and that
company has already adjudged the employee guilty of the she has no other choice but to disassociate herself from
charges for which she was asked to answer and explain. employment. Employees resign for various reasons. A big salary is
 Preventive suspension is not a penalty. This is different from certainly no hindrance to a voluntary cessation of employment.
PUNITIVE SUSPENSION which is imposed as a penalty less Human resource studies reveal that various factors (in and out of
harsh than dismissal. the workplace) affect an employee’s employment decision [Globe
 Preventive suspension is neither equivalent nor tantamount to Telecom v. Crisologo].
dismissal.
 If the basis of the preventive suspension is the employee’s 1. Requisites
absences and tardiness, the imposition of preventive suspension In case of voluntary resignation without just cause, the
on him is not justified as his presence in the company premises following requisites must concur:
does not pose any such serious or imminent threat to the life or
property of the employer or of the employee’s co-workers (a) The resigning employee should tender a written (not
simply “by incurring repeated absences and tardiness.” verbal) notice of the termination (commonly known
 Preventive suspension does not mean that due process may be as “resignation letter”);
disregarded. (b)Service of such notice to the employer at least one
 Preventive suspension should only be for a maximum period of (1) month in advance; and
thirty (30) days. After the lapse of the 30-day period, the (c) Written acceptance by the employer of the
employer is required to reinstate the worker to his former resignation.
position or to a substantially equivalent position. For the
construction industry, preventive suspension should only be for Written acceptance of resignation is necessary to make it
a maximum of 15 days. binding and effective. A duly accepted resignation effectively
 During the 30-day preventive suspension, the worker is not terminates the employer-employee relationship [BMG Records v.
entitled to his wages and other benefits. However, if the Aparecio].
employer decides, for a justifiable reason, to extend the period
of preventive suspension beyond said 30- day period, he is Phimco v. NLRC
obligated to pay the wages and other benefits due the worker Held: As regards the validity of the dismissal of Carpio,
during said period of extension. In such a case, the worker is not petitioner claims that Carpio received a copy of the company’s Handbook
bound to reimburse the amount paid to him during the extension on 26 October 1990 which clearly states that if an employee resigns from
work he has to serve an advance written notice at least thirty (30) days
if the employer decides to dismiss him after the completion of
before he leaves the service with a proviso that a shorter notification would
the investigation. be acceptable only if authorized by the department head concerned and that
 Extension of period must be justified. During the 30-day period he should continue to render service during the interim unless he has
of preventive suspension, the employer is expected to conduct secured authority to go on leave. Moreover, any resignation shall be
and finish the investigation of the employee’s administrative effected only upon its proper acceptance by management. The penalty
case. The period of thirty (30) days may only be extended if the prescribed for the violation thereof is dismissal. Accordingly, PHIMCO

Page 110 of 117


maintains that since Carpio violated the rules and regulations of the employer, the consequent effect of resignation is severance of the
company with respect to his leaving the company which resulted in the contract of employment [Philippines Today v. NLRC].
disruption of its operations, then his termination was for a just cause based A resigned employee who desires to take his job back has
particularly on willful disobedience under the law. Corollarily, PHIMCO to re-apply therefor and he shall have the status of a stranger who
was not obligated to give separation pay to Carpio.
cannot unilaterally demand an appointment. He cannot arrogate
We find the penalty of dismissal imposed on Carpio for non-
observance of the rules and regulations provided in the Handbook of
unto himself the same position which he earlier decided to leave.
PHIMCO, particularly Rules 7, 7.1 and 7.2 concerning the resignation of To allow him to do so would be to deprive the employer of his
the employee, too harsh. There is no dispute that Carpio failed to comply basic right to choose whom to employ. Such is tantamount to
with company rules and regulations regarding his resignation. Nonetheless, undue oppression of the employer. It has been held that an
it has to be noted and emphasized that he did not outrightly disregard the employer is free to regulate, according to his own discretion and
same. Looking back at the antecedents, it was on 14 August 1991 that he judgment, all aspects of employment including hiring. The law, in
tendered his resignation to take effect fifteen (15) days later or on 30 protecting the rights of the laborer, impels neither the oppression
August 1991. Before that period expired he still reported for work.
nor self-destruction of the employer [Ibid].
Significantly, the fact that his letter of resignation was only acted upon after
he had left for United States opens the avenues for speculations and
suspicions. While he continued to work to await the acceptance of his Intertrod v. NLRC
resignation, he was not even informed of the status thereof or that he had to Held: Resignations, once accepted and being the sole act of the
stay for fifteen (15) more days. The rule could have been easily pointed out employee, may not be withdrawn without the consent of the employer. In
or relayed to him by Mr. Lopez to whom he handed his letter of resignation the instant case, the Master had already accepted the resignation and,
and who was staying with him in the same office. But management waited although the private respondent was being required to serve the thirty (30)
until after he had left for the United States. Evidently, there was bad faith in days notice provided in the contract, his resignation was already approved.
the manner his resignation was resolved. Private respondent cannot claim that his resignation ceased to be effective
Petitioner also avers that Carpio’s termination was for a just because he was not immediately discharged in Port Pylos, Greece, for he
cause under the law grounded on his willful disobedience to comply with could no longer unilaterally withdraw such resignation. When he later
the company’s rules and regulation. But to constitute willful disobedience signified his intention of continuing his work, it was already up to the
the employee’s conduct must be willful or intentional, the willfulness being petitioners to accept his withdrawal of his resignation. The mere fact that
characterized by a wrongful and perverse attitude and the order violated they did not accept such withdrawal did not constitute illegal dismissal for
must have been reasonable, lawful, made known to the employee and must acceptance of the withdrawal of the resignation was their (petitioners’) sole
pertain to the duties which he has been engaged to discharge. 10 In the prerogative.
instant case, we find absent any intentional or willful conduct on the part of Once an employee resigns and his resignation is accepted, he no
Carpio to disregard the rules regarding voluntary resignation. On the longer has any right to the job. If the employee later changes his mind, he
contrary, there was earnest and sincere effort on the part of Carpio to must ask for approval of the withdrawal of his resignation from his
comply. employer, as if he were re-applying for the job. It will then be up to the
In cases of voluntary resignation, the employee finds himself in employer to determine whether or not his service would be continued. If the
a situation where he believes that personal reasons cannot be sacrificed in employer accepts said withdrawal, the employee retains his job. If the
favor of the exigency of the service and he has no other choice but to employer does not, as in this case, the employee cannot claim illegal
disassociate himself from his employment; 11 hence, the law affords the dismissal for the employer has the right to determine who his employees
employee the right to resign regardless of whether the company has found will be. To say that an employee who has resigned is illegally dismissed, is
an able and competent replacement and whether the operation of the to encroach upon the right of employers to hire persons who will be of
company would be affected provided he serves a written notice on the service to them.
employer at least one (1) month in advance. The rule of requiring an
employee to stay or complete the 30-day period prior to the effectivity of 4. Separation Pay
his resignation becomes discretionary on the part of management as an The general rule is that an employee who voluntarily
employee who intends to resign may be allowed a shorter period before his
resigns from employment is not entitled to separation pay unless,
resignation becomes effective. In the instant case, the noncompliance with
the period should not be used by management as a subterfuge to avoid the however. there is a stipulation for payment of such in the
payment of separation pay.  employment contract or Collective Bargaining Agreement (CBA),
or payment of the amount is sanctioned by established employer
2. Liability for Damages practice or policy [Travelaire & Tours v. NLRC].
The failure by the resigning employee to comply with the
legal requirement of service of a written notice (resignation letter) B. INVOLUNTARY RESIGNATION
at least 30 days from its effectivity does not result in making his The following are the just causes that may justify the
resignation void but only in making him liable for damages termination by the employee of the employment relationship
[Serrano v. NLRC]. without need to comply with the 30-day prior written notice
requirement:
3. Withdrawal of Resignation
Resignations, once accepted, may not be withdrawn (a) Serious insult by the employer or his representative
without the consent of the employer. If the employer accepts the on the honor and person of the employee;
withdrawal, the employee retains his job. If the employer does not, (b) Inhuman and unbearable treatment accorded the
the employee cannot claim illegal dismissal. To say that an employee by the employer or his representative;
employee who has resigned is illegally dismissed, is to encroach (c) Commission of a crime or offense by the employer
upon the right of employers to hire persons who will be of service or his representative against the person of the
to them. Obviously, this is a recognition of the contractual nature employee or any of the immediate members of his
of employment which requires mutuality of consent between the family; and
parties. An employment contract is consensual and voluntary. (d) Other causes analogous to any of the foregoing.
Hence, if the employee "finds-himself in a situation where he
believes that personal reasons cannot be sacrificed in favor of the C. CONSTRUCTIVE DISMISSAL
exigency of the service, then he has no other choice but to Constructive dismissal contemplates any of the following
disassociate himself from his employment". If accepted by the situations:

Page 111 of 117


a. An involuntary resignation resorted to when On the other hand, respondent employer argues that petitioner’s
continued employment is rendered impossible, preventive suspension for one day can hardly be considered indefinite,
unreasonable or unlikely; given the fact that petitioner immediately resigned one day after the
b. A demotion in rank and/or a diminution in pay; or suspension.
We find that there was no act of discrimination committed
c. A clear discrimination, insensibility or disdain by
against petitioner that would render her employment unbearable.
an employer which becomes unbearable to the Preventive suspension may be legally imposed against an
employee that it could foreclose any choice by him employee whose alleged violation is the subject of an investigation. The
except to forego his continued employment. purpose of his suspension is to prevent him from causing harm or injury to
the company as well as to his fellow employees.
The employer has to prove that such managerial actions The pertinent rules dealing with preventive suspension are found
do not constitute constructive dismissal [Blue Dairy Corp. v. in Section 8 and Section 9 of Rule XXIII, Book V of the Omnibus Rules
NLRC]. Implementing the Labor Code, as amended by Department Order No. 9,
Series of 1997, which read as follows:
The test of constructive dismissal is whether a reasonable
Section 8. Preventive suspension. The employer may place the
person in the employee’s position would have felt compelled to worker concerned under preventive suspension only if his continued
give up his position under the circumstances. It is an act amounting employment poses a serious and imminent threat to the life or property of
to dismissal but made to appear as if it were not. In fact, the the employer or of his co-workers.
employee who is constructively dismissed may be allowed to keep Section 9. Period of suspension. No preventive suspension shall
on coming to work. Constructive dismissal is, therefore, a last longer than thirty (30) days. The employer shall thereafter reinstate the
DISMISSAL IN DISGUISE. The law recognizes and resolves this worker in his former or in a substantially equivalent position or the
situation in favor of the employees in order to protect their rights employer may extend the period of suspension provided that during the
period of extension, he pays the wages and other benefits due to the worker.
and interests from the coercive acts of the employer [Ang v. San
In such case, the worker shall not be bound to reimburse the amount paid to
Joaquin]. him during the extension if the employer decides, after completion of the
hearing, to dismiss the worker.
Singa Ship Management v. NLRC When preventive suspension exceeds the maximum period
Held: Corollarily, the contention of petitioners that respondent allowed without reinstating the employee either by actual or payroll
Sangil voluntarily quit and was not illegally dismissed is without merit. We reinstatement or when preventive suspension is for indefinite period, only
defer to the findings of the NLRC - then will constructive dismissal set in.
Since complainant is not the aggressor, and since he figured a While no period was mentioned in the show-cause
head injury, he is then afraid to go back to the ship and to mix with his memorandum, it was wrong for petitioner to infer that her suspension was
aggressor. This apprehension or fear is normal to an ordinary prudent for an indefinite period. It must be pointed out that the inclusion of the
individual and is tantamount to self-preservation. Therefore, his decision to phrase "during the course of investigation" would lead to a reasonable and
leave the ship "Crown Odyssey" is not voluntary. He did not leave the ship logical presumption that said suspension in fact has a duration which could
out of his own free will but his departure was precipitated by fear. Without very well be not more than 30 days as mandated by law. And, as the Court
that incident on 20 July 1990 he would have no reason not to go back to the of Appeals correctly observed, the suspension has been rendered moot by
ship but with that incident, he fears for his life and limb. petitioner’s resignation tendered a day after the suspension was made
In People's Security, Inc. v. NLRC we said that "constructive effective.
dismissal exists when there is a quitting because continued employment is Petitioner contests the grounds for her suspension as she denies
rendered impossible, unreasonable or unlikely x x x x" Then in Philippine posing a danger on the lives of the officers or employees of respondent or
Advertising Counselors, Inc. v. NLRC we held that "[c]onstructive of their properties. Petitioner adds that she was not in a position to bind
dismissal, however, does not always involve such kinds of diminution; an respondent to any contract, therefore, she could not and would not be able
act of clear discrimination, insensibility, or disdain by an employer may to sabotage the operations of respondent. Upon the other hand, respondent
become so unbearable on the part of the employee that it could foreclose asserts that preventive suspension was necessary in order to protect the
any choice by him except to forego his continued employment." assets and operations of the company pending investigation of the alleged
In the instant case, respondent Sangil quit because he feared for infractions committed by the employee concerned.
his life and his fear was well founded. He already figured in an incident Respondent is correct. Indeed, as sales manager, petitioner had
with the much taller, bigger and heavier Zakkas who had been intimidating the power and authority to enter into contracts that would bind respondent,
him. He already suffered a cut in his head. Earlier, Zakkas threatened to regardless of whether these contracts would prove to be beneficial or
pour hot coffee on his head. Indeed, the intense undercurrent between the prejudicial to the interest of respondent. Respondent has every right to
Filipinos and the Greeks that could erupt into violence at the slightest protect its assets and operations pending investigation of petitioner.
provocation was apparent as manifested by the writings in blood on the Neither could we consider the acts of disconnection of computer
wall of the word "magkaisa," as witnessed by another Greek steward on the and internet access privileges as harassment. Respondent clearly explained
day of the incident. And, Sangil could not get any protection from the that the cessation of her internet and network privileges were but a
Greek ship captain, not even the slightest assurance of safety from him. In consequence of the investigation against her and not for the purpose of
fine, the decision of respondent Sangil to leave the ship was not voluntary harassment. The Court of Appeals gave merit to respondent’s explanation
at all but was impelled by a legitimate desire for self-preservation. He did and held, thus:
not leave the ship out of his own free will, whim or caprice but was moved x x x while her suspension, cessation of internet privileges, and
by fear for his life. Without that incident on 20 July 1990 where he was exclusion from local network access were but a consequence of the
pushed by the Greek Zakkas, respondent Sangil would not have had any investigation against her, and were intended to prevent her from having
reason not to return to his ship. After all, he must have worked so hard to further access to the company’s network-based documents and forms.
get on board "Crown Odyssey." The acts respondent complains about are just measures enforced
by respondent to protect itself while the investigation was ongoing.
Petitioner claims that Longstaff forced her to resign by baiting
Mandapat v. Add Force Personnel Services her with the promise of separation pay; but respondent maintains that there
Held: Petitioner reiterates that she was constructively dismissed.
was nothing illegal in giving petitioner the option to either resign or be
She harps on the alleged pattern of harassment committed by respondent as
separated for a just cause.
tantamount to constructive dismissal, such as, illegally placing her under
We agree with the Court of Appeals that there was no coercion
preventive suspension, the disconnection of her internet account, and the
employed on petitioner. The appellate court made the following
pressure exerted by respondent to force her to resign.
observation:
Petitioner claims that the preventive suspension meted upon her
Unfortunately, however, before the investigation could proceed
is illegal for being indefinite, as the duration of her suspension was not
to the second step of the termination process into a hearing or conference,
stated in the company’s memorandum.
Mandapat chose to resign from her job. Mandapat’s bare allegation that she

Page 112 of 117


was coerced into resigning can hardly be given credence in the absence of which is forced on the person to do as the lesser evil; and (4) that it
clear evidence proving the same. No doubt, Mandapat read the writing on produces a well-grounded fear from the fact that the person from whom it
the wall, knew that she would be fired for her transgressions, and beat the comes has the necessary means or ability to inflict the threatened injury to
company to it by resigning. Indeed, by the disrespectful tenor of her his person or property x x x.
memorandum, Mandapat practically indicated that she was no longer After a careful review of the records, the Court finds that the
interested in continuing cordial relations, much less gainful employment above-stated requisites are absent, and that the NLRC has exhaustively
with Add Force. discussed that Doble was not coerced into submitting a resignation letter,
Mere allegations of threat or force do not constitute evidence to thus:
support a finding of forced resignation. In order for intimidation to vitiate "[c]omplainant has been employed with Respondent-ABB for
consent, the following requisites must concur: (1) that the intimidation nineteen (19) years. He is holding one of the top positions in the company
caused the consent to be given; (2) that the threatened act be unjust or and answerable only to the President, herein Respondent-Desai. He is a
unlawful; (3) that the threat be real or serious, there being evident highly educated man. It is improbable that a man of his stature may be
disproportion between the evil and the resistance which all men can offer, pressured into doing something that he does not want to do. Being a man of
leading to the choice of doing the act which is forced on the person to do as high educational attainment and qualifications, he is expected to know the
the lesser evil; and (4) that it produces a well-grounded fear from the fact import of everything he executes. His claim that he was forced to resign by
that the person from whom it comes has the necessary means or ability to HR Miranda is unbelievable. The Complainant is the Vice-President and
inflict the threatened injury to his person or property. Local Division Manager of the Power System Division of the
None of these requisites was proven by petitioner. No demand Respondent-ABB, while HR Miranda is the Country HR Manager. The
was made on petitioner to resign. At most, she was merely given the option latter docs not outrank the former. It is likewise unbelievable that the
to either resign or face disciplinary investigation, which respondent had HR Manager would prevent the Complainant from leaving the
every right to conduct in light of the numerous infractions committed by premises of the company nor prevent him from taking his lunch
petitioner. There is nothing irregular in providing an option to petitioner. wherever he wants to take it. HR Miranda simply docs not have that
Ultimately, the final decision on whether to resign or face disciplinary power and she cannot possibly do that to a high-ranking officer who
action rests on petitioner alone. has served the company for nineteen (19) years. The event of 13 March
All told, the instances of harassment alleged by petitioner appear 2012 is undoubtedly stressful to the Complainant as the top management
to be more apparent than real. We find no reason to disturb the conclusion had already expressed displeasure with his performance. But such degree of
of the Court of Appeals that petitioner resigned and was not constructively tension is expected in a corporation environment where the primordial
dismissed. consideration is to earn profit. As stated in the sworn statement of HR
Miranda, the Complainant was given the option to resign by
Respondent-Desai. Her statement that the Complainant negotiated for
Consolidated Foods v. NLRC
a higher benefit is more attuned with what actually transpired on 13
Held: We find that petitioners acts of conducting audits and
March 2012. The retirement plan for Respondent-ABB only gives a
investigation on the alleged irregularities committed by private respondent
retiree 75% of his monthly pay for every year of service. The
and in reassigning him to another place of work pending the results of the
Complainant was able to get a higher rate equivalent to one (1) month
investigation were based on valid and legitimate grounds. As such, these
salary for every year of service.
acts of management cannot amount to constructive dismissal. It is worthy to
The Complainant prepared his resignation letter in his own
note that petitioners gave Baron every opportunity to raise his defense and
office. His first letter was not accepted by HR Miranda because it gave the
fully explain the discrepancies in the funds in his possession. In fact private
impression that he was being directed or ordered to resign. HR Miranda
respondent informed petitioners that he would be returning for work on 5
made it clear to him that he is not being ordered to resign as it is his own
March 1991 after his sick leave. But instead of doing so, he filed a
decision whether to resign or not. The Complainant submitted another
complaint for constructive dismissal before the Labor Arbiter.
resignation letter which was accepted by Respondent-ABB through its
By leaving his job without submitting the required final
Country HR Manager. Thereafter, the Complainant no longer reported for
explanation on the alleged irregularities, private respondent deprived
work as his resignation was effective immediately. It was ten (10) days
himself of the opportunity to face his accusers and prove his innocence of
after he submitted his resignation letter that he again met with HR Miranda
the charges hurled against him.
to get his retirement benefits. The meeting took place outside the company
premises. If, indeed, the resignation of the Complainant was
Dangan v. NLRC involuntary, he could have easily sought legal counsel or advice right
Held: Instead of terminating the petitioner’s employment, the after he left the company premises on 13 March 2012. Instead, he
private respondent merely reassigned her to other temporary positions waited for his clearance to be processed and his check prepared. He
before giving her a permanent job as secretary in the Technical Training cannot claim that he was still under duress from March 14 to 22, 2012.
Department located at Bicutan, Taguig, Metro Manila. The acts of the The Complainant waited to be given his benefits first, and three (3)
company negate the petitioner’s claim of discrimination and illegal days thereafter filed his complaint before this Office. This is hardly the
constructive dismissal. It tried its best to continue using her services. mindset of a person who is not in control of his life.
Needless to say, no reinstatement can be effected because the petitioner has On the other hand, the Court disagrees with the findings of the
not been dismissed. Neither may she be reinstated to her former position Labor Arbiter that Doble's resignation was not voluntary based on the
which no longer exists. Ad imposible nemo tenetur. Insofar as this aspect of following events, to wit: (1) on March 2, 2012, Doble's Performance and
the petition is concerned, the respondent NLRC committed no abuse of Development Approval rating in 2011 is unsatisfactory; (2) there are no
discretion in dismissing the petitioner’s appeal. prior circumstances that may show his intention to resign; (3) on March 13,
2012, Desai raised the option for him to resign, after explaining that due to
the extent of losses and level of discontent among the ranks of the PS
Doble v. ABB, Inc. Division, the Global and Regional management have demanded for a
Held: Since Doble claims to have been forced to submit a change in leadership; (4) from the circumstances surrounding his
resignation letter, it is incumbent upon him to prove with clear and resignation, the option to resign did not originate from Doble but from
convincing evidence that his resignation was not voluntary, but was Desai, whose actuations was not a mere suggestion but a directive or order
actually a case of constructive dismissal, i.e., a product of coercion or that was effected on the same day of March 13, 2012; (5) HR Manager
intimidation.37 Coercion exists when there is a reasonable or well-grounded Miranda's affidavit clearly show that Doble underwent pressure to resign
fear of an imminent evil upon a person or his property or upon the person or because starting 11:00 a.m. until 6:00 p.m. of even date, the option to resign
property of his spouse, descendants or ascendants.38 The requisites for was reiterated and repeated until he handed a revised resignation letter; and
intimidation to vitiate one's consent are stated in St. Michael Academy v. (6) Doble was not given the opportunity or option to stay in the service.
NLRC, thus: Even if the option to resign originated from the employer, what
... (1) that the intimidation caused the consent to be given; (2) is important for resignation to be deemed voluntary is that the employee's
that the threatened act be unjust or unlawful; (3) that the threat be real or intent to relinquish must concur with the overt act of relinquishment. There
serious, there being evident disproportion between the evil and the can be no doubt as to the drastic and shocking nature of the abrupt decision
resistance which all men can offer, leading to the choice of doing the act of ABB, Inc. to let Doble resign on March 13, 2012 after almost 19 years of

Page 113 of 117


dedicated and satisfactory service, on account of the extent of losses, the undertaking for a period not exceeding six (6) months, or the
level of discontent among the ranks of PS Division, and the ABB, Inc. fulfillment by the employee of a military or civic duty shall not
Global and Regional management's demand for a change in leadership. It terminate employment. In all such cases, the employer shall reinstate
bears emphasis, however, that between the start of the conference at around the employee to his former position without loss of seniority rights if he
11:00 a.m. and about eight (8) hours later in the evening when he left the indicates his desire to resume his work not later than one (1) month
company premises, Doble negotiated for a higher separation pay, i.e., from from the resumption of operations of his employer or from his relief
75% of the monthly salary for every year of service allowed under the from the military or civic duty.
company retirement plan up to double that amount, or 1.5 month's pay for
every year of service. In fact, Doble tendered a resignation letter only after Article 301 of the Labor Code allows the bona
being offered a better separation benefit of 1-month pay for every year of
fide suspension of operations for a period not exceeding six (6)
service, and even submitted a separate letter expressing his intent to buy his
service vehicle. After considering the acts of Doble before and after his months.  During the suspension, an employee is not deemed
resignation, the Court is convinced of Doble's clear intention to sever his terminated.  As a matter of fact, the employee is entitled to be
employment with ABB, Inc. reinstated once the employer resumes operations within the 6-
Doble claimed that while inside the conference room at about month period.  However, Article 301 is silent with respect to the
2:00 p.m. of March 13, 2012, "he was aware that respondents were actually rights of the employee if the suspension of operations lasts for
terminating his services illegally and without due process, that the letter of more than 6 months [Manila Mining Corp. Employees v. Manila
resignation he was being made to prepare was only a 'palusot' (to borrow Mining Corp.].
the word of Cong. Fariñas) of respondents (ABB, Inc. and Desai)." Despite
being aware of the illegality of his dismissal, Doble submitted a resignation
letter and a letter of intent to purchase his service vehicle, allowed Miranda Manila Mining Corp. Employees v. Manila Mining
to process his resignation papers, met her outside company premises on Held: MMC subscribes to the view that for purposes of
March 23, 2012 to sign a waiver and quitclaim and to receive his separation determining employer responsibility, an employment should likewise not be
benefits. In view of the lapse of considerable period between his resignation deemed terminated, should the suspension of operation go beyond six (6)
until the execution of a quitclaim and receipt of his separation benefits months as long as the continued suspension is due, as in this case, to a
about ten (10) days later, the Court is inclined to rule that the filing of his cause beyond the control of the employer.
complaint for illegal dismissal on March 26, 2012 is a mere afterthought, if We disagree.
not a mere pretention. As correctly elucidated upon by the Court of Appeals:
We observe that MMC was forced by the circumstances, hence,
it resorted to a temporary suspension of its mining and milling operations. 
Cases where there was constructive dismissal It is clear that MMC had no choice.  It would be well to reiterate at this
juncture that the reason for such suspension cannot be attributed to DENR-
 Denying to the workers entry to their work area and placing EMB. It is thus, evident, that the MMC declared temporary suspension of
them on shifts “not by weeks but almost by month” by reducing operations to avert further losses.
their workweek to three days. The decision to suspend operation ultimately lies with the
 Reducing employee’s functions employer, who in its desire to avert possible financial losses, declares, as
 Blackmailing an employee to resign here, suspension of operations
Article 283 of the Labor Code applies to MMC and it provides:
 Delaying when the employee can start working or asking the
ARTICLE 283.Closure of establishment and reduction of
employee to wait for a long period of time before rehiring personnel. - The employer may also terminate the employment of any
 Barring the employees from entering the premises whenever employee due to the installation of labor-saving devices, redundancy,
they would report for work in the morning without any retrenchment to prevent losses or the closing or cessation of operation of
justifiable reason, and they were made to wait for a certain the establishment or undertaking unless the closing is for the purpose of
employee who would arrive in the office at around noon, after circumventing the provisions of this Title, by serving a written notice on the
they had waited for a long time and had left. workers and the Ministry of Labor and Employment at least one (1) month
 Sending to an employee a notice of indefinite suspension which before the intended date thereof. In case of termination due to the
installation of labor-saving devices or redundancy, the worker affected
is tantamount to dismissal.
thereby shall be entitled to a separation pay equivalent to at least his one (1)
 Imposing indefinite preventive suspension without actually month pay or to at least one (1) month pay for every year of service,
conducting any investigation. whichever is higher. In case of retrenchment to prevent losses and in cases
 Changing the employee’s status from regular to casual of closures or cessation of operations of establishment or undertaking not
constitutes constructive dismissal. due to serious business losses or financial reverses, the separation pay shall
 Preventing the employee from reporting for work by ordering be equivalent to one (1) month pay or at least one-half (1/2) month pay for
the guards not to let her in. This is clear notice of dismissal. every year of service, whichever is higher. A fraction of at least six (6)
months shall be considered one (1) whole year.
Said provision is emphatic that an employee, who was dismissed
Illegal Dismissal vs. Constructive Dismissal
due to cessation of business operation, is entitled to the separation pay
In illegal dismissal, the employer openly shows his equivalent to one (1) month pay or at least one-half (1/2) month pay for
intention to dismiss the employee. In fact, the employer, in every year of service, whichever is higher.  And it is jurisprudential that
compliance with due process, asks the employee to explain why he separation pay should also be paid to employees even if the closure or
should not be dismissed for committing a wrongful act and he is cessation of operations is not due to losses.
given due process prior to terminating him. The Court is not impressed with the claim that actual severe
In contrast, in constructive dismissal, the employer will financial losses exempt MMC from paying separation benefits to
never indicate that he is terminating the employee. He will even complainants.  In the first place, MMC did not appeal the decision of the
Court of Appeals which affirmed the NLRC's award of separation pay to
allow the employee to report to his work every day. But he will do
complainants.  MMC's failure had the effect of making the awards final so
any of the three (3) acts mentioned above that indicates his that MMC could no longer seek any other affirmative relief.  In the second
intention to get rid of the services of the employee. This is the place, the non-issuance of a permit forced MMC to permanently cease its
reason why it is called “dismissal in disguise.” business operations, as confirmed by the Court of Appeals.  Under Article
283, the employer can lawfully close shop anytime as long as cessation of
D. WHEN EMPLOYMENT NOT DEEMED TERMINATED or withdrawal from business operations is bona fide in character and not
impelled by a motive to defeat or circumvent the tenurial rights of
employees, and as long as he pays his employees their termination pay in
Article 301. [286] When Employment not Deemed
the amount corresponding to their length of service. [26]  The cessation of
Terminated. The bona fide suspension of the operation of a business or

Page 114 of 117


operations, in the case at bar is of such nature.  It was proven that MMC b. Employees of retail, service and agricultural
stopped its operations precisely due to failure to secure permit to operate a establishments or operations regularly employing not
tailings pond. Separation pay must nonetheless be given to the separated more than ten (10) employees. These terms are
employees. defined as follows:

Floating Status (i) “Retail establishment” is one principally


At the outset, it bears reiterating that although placing an engaged in the sale of goods to end-users for
employee like a security guard on “floating” status (or sometimes personal or household use. It shall lose its
called temporary “off-detail” status) is considered a temporary retail character qualified for exemption if it is
retrenchment measure, the Supreme Court, in Exocet v. Serrano, engaged in both retail and wholesale of goods.
recognized the fact that there is similarly no provision in the Labor (ii) “Service establishment” is one principally
Code which treats of a temporary retrenchment or lay-off. Neither engaged in the sale of service to individuals
is there any provision which provides for its requisites or its for their own or household use and is
duration. generally recognized as such.

E. RETIREMENT 2. Options
Retirement and dismissal are entirely different from each The two (2) types of retirement under the law (Article
other. Retirement is the result of a bilateral act of the parties, a 302 [287] of the Labor Code) are:
voluntary agreement between the employer and the employees
whereby the latter after reaching a certain age agrees and/or a. Optional retirement upon reaching the age of
consents to severe his employment with the former. On the other sixty (60) years (50 if underground/surface
hand, dismissal refers to the unilateral act of the employer in mine employees).
terminating services of an employee with or without cause. In fine, b. Compulsory retirement upon reaching the age
in the case of dismissal, it is only the employer who decides when of sixty-five (65) years (60 if
to terminate the services of an employee. Moreover, concomitant underground/surface mine employees).
with the provisions on retirement in a Labor Agreement is a
stipulation regarding retirement benefits pertaining to a retired It is the employee who exercises the option under No. 1
employee. Here again, the retirement benefits are subject to above. At age 65, there is no more option of the employee to speak
stipulation by the parties unlike in dismissals were separation pay of. He has to retire as this age is considered compulsory retirement
is fixed by law in cases of dismissals without just cause. Evident, age.
therefore, from the foregoing is that retirements which are agreed Article 302 provides for two types of retirement; (a)
upon by the employer and the employee in their collective compulsory and (b) optional. The first takes place at age 65, while
bargaining agreement are not dismissals . . . . To further fortify the the second is primarily determined by the collective bargaining
aforesaid conclusion, it is noteworthy that even the New Labor agreement or other employment contract or employer’s retirement
Code recognizes this distinction when it treats retirement form plan. In the absence of any provision on optional retirement in a
service under a separate title from that of dismissal or termination collective bargaining agreement, other employment contract or
of employment, aside from expressly recognizing the right of the employer’s retirement plan, an employee may optionally retire
employer to retire any employee who has reached the retirement upon reaching the age of 60 years or more, but not beyond 65
age established in the collective bargaining agreement or other years, provided he has served at least five years in the
applicable employment contract and the latter to receive such establishment concerned. That prerogative is exclusively lodged in
retirement benefits as he may have earned under existing laws and the employee [Capili v. NLRC].
any collective bargaining or other agreement [Pantranco v. NLRC]. In almost all countries today, early retirement, i.e., before
age 60, is considered a reward for services rendered since it enables
1. Coverage an employee to reap the fruits of his labor — particularly
The following employees are eligible to avail of retirement benefits, whether lump-sum or otherwise — at an earlier
retirement benefits under Article 302 [287] of the Labor Code: age, when said employee, in presumably better physical and mental
condition, can enjoy them better and longer. As a matter of fact,
a. All employees in the private sector, regardless of one of the advantages of early retirement is that the corresponding
their position, designation or status and irrespective retirement benefits, usually consisting of a substantial cash
of the method by which their wages are paid; windfall, can early on be put to productive and profitable uses by
b. Part-time employees; way of income — generating investments, thereby affording a
c. Employees of service and other job contractors; more significant measure of financial security and independence
d. Domestic workers/kasambahays or persons in the for the retiree who, up till then, had to contend with life’s
personal service of another; vicissitudes within the parameters of his fortnightly or weekly
e. Underground mine workers; wages. Thus we are now seeing many CBAs with such early
f. Employees of government-owned and/or controlled retirement provisions. And the same cannot be considered a
corporations organized under the Corporation Code diminution of employment benefits. Incidentally, we call attention
(without original charters). to Republic Act No. 7641, known as "The Retirement Pay Law,"
which went into effect on January 7, 1993 [Pantranco v. NLRC]. 
Article 302 [287], as amended, does not apply to the
following employees: Laya v. Court of Appeals
Held: The CA concluded that the petitioner had agreed to be
a. Employees of the national government and its political bound by the retirement plan of PVB when he accepted the letter of
subdivisions, including government-owned and/or appointment as its Chief Legal Counsel.
controlled corporations, if they are covered by the We disagree with the conclusion. We declare that based on the
Civil Service Law and its regulations. clear circumstances herein the CA erred in so concluding.

Page 115 of 117


The petitioner's letter of appointment pertinently stated: Member of the Plan on the date he becomes a regular permanent Employee,
3. As a Senior Officer of the Bank, you are entitled to the provided he is less than 55 years old as of such date.
following executive benefits: Section 3. Continuation/Termination of Membership.
Car Plan limit of P700,000.00, without equity on your part; a Membership in the Plan shall be concurrent with employment with the
gasoline subsidy of 300 liters per month and subject further to The Car Plan Bank, and shall cease automatically upon termination of the Member's
Policy of the Bank. service with the Bank for any reason whatsoever.74 (Bold underscoring
Membership in a professional organization in relation to your supplied for emphasis)
profession and/or assigned functions in the Bank. Having thus automatically become a member of the retirement
Membership in the Provident Fund Program/Retirement plan through his acceptance of employment as Chief Legal Officer of
Program. PVB,75 the petitioner could not withdraw from the plan except upon his
Entitlement to any and all other basic and fringe benefits termination from employment.
enjoyed by the officers; core of the Bank relative to Insurance covers, It is also notable that the retirement plan had been in existence
Healthcare Insurance, vacation and sick leaves, among others. since January 1, 1996, or more than five years prior to the petitioner's
Obviously, the mere mention of the retirement plan in the letter employment by PVB. The plan was established solely by the PVB, and
of appointment did not sufficiently inform the petitioner of the contents or approved by its president. As such, the plan was in the nature of a contract
details of the retirement program. To construe from the petitioner's of adhesion, in respect to which the petitioner was reduced to mere
acceptance of his appointment that he had acquiesced to be retired earlier submission by accepting his employment, and automatically became a
than the compulsory age of 65 years would, therefore, not be warranted. member of the plan. With the plan being a contract of adhesion, to consider
This is because retirement should be the result of the bilateral act of both him to have voluntarily and freely given his consent to the terms thereof as
the employer and the employee based on their voluntary agreement that the to warrant his being compulsorily retired at the age of 60 years is factually
employee agrees to sever his employment upon reaching a certain age. unwarranted.
That the petitioner might be well aware of the existence of the In view of the foregoing, the Court disagrees with the view
retirement program at the time of his engagement did not suffice. His tendered by Justice Leonen to the effect that the petitioner, because of his
implied knowledge, regardless of duration, did not equate to the voluntary legal expertise and educational attainment, could not now validly claim that
acceptance required by law in granting an early retirement age option to the he was not informed of the provisions of the retirement program. The
employee. The law demanded more than a passive acquiescence on the part pertinent rule on retirement plans does not presume consent or
of the employee, considering that his early retirement age option involved acquiescence from the high educational attainment or legal knowledge of
conceding the constitutional right to security of tenure. the employee. In fact, the rule provides that the acquiescence by the
In Cercado v. Uniprom, Inc., we have underscored the character employee cannot be lightly inferred from his acceptance of employment.
of the employee's consent in agreeing to the early retirement policy of the Moreover, it was incumbent upon PVB to prove that the
employer, viz.: petitioner had been fully apprised of the terms of the retirement program at
Acceptance by the employees of an early retirement age option the time of his acceptance of the offer of employment. PVB did not
must be explicit, voluntary, free, and uncompelled. While an employer may discharge its burden, for the petitioner's appointment letter apparently
unilaterally retire an employee earlier than the legally permissible ages enumerated only the minimum benefits that he would enjoy during his
under the Labor Code, this prerogative must be exercised pursuant to a employment by PVB, and contained no indication of PVB having given
mutually instituted early retirement plan. In other words, only the him a copy of the program itself in order to fully apprise him of the
implementation and execution of the option may be unilateral, but not the contents and details thereof. Nonetheless, even assuming that he
adoption and institution of the retirement plan containing such option. For subsequently obtained information about the program in the course of his
the option to be valid, the retirement plan containing it must be voluntarily employment, he still could not opt to simply withdraw from the program
assented to by the employees or at least by a majority of them through a due to his membership therein being automatic for the regular employees of
bargaining representative. PVB.
Furthermore, the petitioner's membership in the retirement plan To stress, company retirement plans must not only comply with
could not be justifiably attributed to his signing of the letter of appointment the standards set by the prevailing labor laws but must also be accepted by
that only listed the minimum benefits provided to PVB's employees. the employees as commensurate to their faithful services to the employer
Indeed, in Cercado, we have declared that the employee's consent to the within the requisite period.80 Although the employer could be free to
retirement plan that came into being two years after the hiring could not be impose a retirement age lower than 65 years for as long its employees
inferred from her signature on the personnel action forms accepting the consented, the retirement of the employee whose intent to retire was not
terms of her job description, and compliance with the company policies, clearly established, or whose retirement was involuntary is to be treated as a
rules and regulations, to wit: discharge.
We also cannot subscribe to respondent's submission that
petitioner's consent to the retirement plan may be inferred from her 3. Amount of Retirement Pay
signature in the personnel action forms containing the phrase: "Employee
In the absence of a retirement plan or agreement
hereby expressly acknowledges receipt of and undertakes to abide by tile
provisions of his/her Job Description, Company Code of Conduct and such providing for retirement benefits of employees in the
other policies, guidelines, rules and regulations the company may establishment, an employee, upon reaching the optional or
prescribe." compulsory retirement age specified in Article 287, shall be
It should be noted that the personnel action forms relate to the entitled to retirement pay equivalent to at least one-half (½) month
increase in petitioner's salary at various periodic intervals. To conclude that salary for every year of service, a fraction of at least six (6) months
her acceptance of the salary increases was also, simultaneously, a being considered as one (1) whole year.
concurrence to the retirement plan would be tantamount to compelling her For purposes of determining the minimum retirement pay
to agree to the latter. Moreover, voluntary and equivocal acceptance by an
due an employee under Article 287, the term “one- half month
employee of an early retirement age option in a retirement plan necessarily
connotes that her consent specifically refers to the plan or that she has at salary” shall include all of the following:
least read the same when she affixed her conformity thereto.
A perusal of PVB's retirement plan shows that under its Article (i) Fifteen (15) days salary of the employee based on
III all the regular employees of PVB were automatically admitted into his latest salary rate.
membership, thus: (ii) The cash equivalent of five (5) days of service
ARTICLE III incentive leave;
MEMBERSHIP IN THE PLAN (iii)One-twelfth (1/12) of the 13th month pay due the
Section 1. Eligibility at Effective Date. Any Employee of the
employee; and
Bank as of January 1, 1996 shall automatically be a Member of the Plan as
of such date. (iv) All other benefits that the employer and employee
Section 2. Eligibility after Effective Date. Any person who may agree upon that should be included in the
becomes an Employee after January 1, 1996 shall automatically become a computation of the employee’s retirement pay.

Page 116 of 117


“One-half (½) month salary” means 22.5 days.
“One-half [½] month salary” is equivalent to “22.5
days” arrived at after adding 15 days plus 2.5 days representing
one-twelfth [1/12] of the 13th month pay plus 5 days of service
incentive leave [Grace Christian High School v. Lavandera].

Page 117 of 117

You might also like